Sie sind auf Seite 1von 468
VOLUME |! Power-System-Control and Stability P. M. ANDERSON A. A. FOUAD THE IOWA STATE UNIVERSITY PRESS, AMES IOWA, USA P M. Anderson is Program Manager for Research in Power System Planning, Security, and Control with the Electric Power Research Institute, Palo Alto, California. Prior to 1975 he was Professor of Electrical Engineering at lowa State University whese he received the Ph.D. degree in 1961 His industrial experience includes six years with the lowa Public Service Company and a university leave for research with the Pacific Gas and Electrie Company, San Francisco. He is author of numerous journal articles and the book Analysis of Faulted Power Systems A.A. Fouad is Professor of Electrical Engineering at lowa State University. He te- ceived the B.Sc. degree from Cairo University and the PhD degree from lowa State University in 1956 and has extensive overseas experience in Egypt, Brazil, and the Philippines He is active in the Power Engineering Society of the Institute of Elec- trical and Electronics Engineers where he serves as a member of the Power System Engineering Committee, System Control Subcommittee, and the working Group on Dy- namic System Performance © 1977 The lowa State University Press Ames, lowa 50010. All rights reserved ‘Composed and printed by Science Press, Ephrata, Pa 17522 t edition, 1977 Library of Congress Cataloging in Publication Data Anderson Paul 1926- Power system control and stability Includes bibliographical references 1 Blectric power systems 2 System analysis, 1. Fouad, Abdel-Aziz A joint author I Title TKIOOSAT 6213 76-2602 ISBN 0-8158-1245-3 To Our Families nee; Contents Preface. xi PartI Introduction Chapter 1. Power System Stability Li Introduction 3 12 Requirements of a Reliable Electrical Power Service 3 13 Statement of the Problem 4 14 Effect of an Impact upon System Components 8 1.5 Methods of Simulation 10 Problems M Chapter 2, The Elementary Mathematical Model 21 Swing Equation B 22 Units 1S 23 Mechanical Torque 16 24 Electrical Torque 20 25 Power-Angle Curve of a Synchronous Machine a 26 Natural Frequencies of Oscillation of a Synchronous Machine 4 2.7 System of One Machine against an Infinite Bus— The Classical Model 26 2.8 Equal Area Criterion 31 29 Classical Model of a Multimachine System 35 210 Classical Stability Study of a Nine-Bus System 37 2.1L Shortcomings of the Classical Model 45 212 Block Diagram of One Machine 47 Problems 48 References. 52 Chapter 3. System Response to Small Disturbances 3.1 Introduction 53 3.2 Types of Problems Studied 54 3.3 The Unregulated Synchronous Machine 55. 3.4 Modes of Oscillation of an Unregulated Multimachine System 59 3.5 Regulated Synchronous Machine 66 3.6 Distribution of Power Impacts 6 Problems 80 References 80 Part Contents The Electromagnetic Torque Chapter 4 The Synchronous Machine 44 42 43 44 4s 46 47 48 49 410 aul 412 413 414 4s 416 Introduction Park's Transformation Flux Linkage Equations Voltage Equations Formulation of State-Space Equations Current Formulation Per Unit Conversion ‘Normalizing the Voltage Equations Normalizing the Torque Equations Torque and Power Equivalent Cireuit of a Synchronous Machine The Flux Linkage State-Space Model Load Equations Subtransient and Transient Inductances and Time Constants Simplified Models of the Synchronous Machine Turbine Generator Dynamic Models Problems : : References Chapter 5. The Simulation of Synchronous Machines 51 5.2 53 54 55 56 57 58 59 510 Introduction Steady-State Equations and Phasor Diagrams Machine Connected to an Infinite Bus through a Transmission Line Machine Connected to an Infinite Bus with Local Load at Machine Terminal Determining Steady-State Conditions Examples : Initial Conditions for a Multimachine System Determination of Machine Parameters from Manufacturers’ Data Analog Computer Simulation of the Synchronous Machine Digital Simulation of Synchronous Machines Problems References Chapter 6. Linear Models of the Synchronous Machine 61 62 63 64 65 66 67 Introduction Linearization of the Generator State-Space Current Model Linearization of the Load Equation for the One-Machine Problem Linearization of the Flux Linkage Model Simplified Linear Model Block Diagrams State-Space Representation of Simplified Model Problems References Chapter 7 Excitation Systems a 72 Simplified View of Excitation Control Control Configurations 83 83 85 38 ot on 92 99 103 405 107 109 4 122 127 143 146 148 150 150 153 154 157 159 165 166 170 184 206 206 208 209 213 217 222 231 231 232 232 233, 235 Contents 7.3 Typical Excitation Configurations 7.4 Excitation Control System Definitions 75 Voltage Regulator 76 Exciter Buildup 77 Excitation System Response 7.8 State-Space Description of the Excitation System 79 Computer Representation of Excitation Systems 7.10 Typical System Constants 7AL The Effect of Excitation on Generator Performance Problems References Chapter 8 Effect of Excitation on Stability 81 Introduction 82 Effect of Excitation on Generator Power Limits 83. Eflect of the Excitation System on Transient Stability ., 84 _ Effect of Excitation on Dynamic Stability 8.5 Root-Locus Analysis of a Regulated Machine Connected to an Infinite Bus 86 Approximate System Representation 8.7 Supplementary Stabilizing Signals 8.8 Linear Analysis of the Stabilized Generator 8.9 Analog Computer Studies 8.10 Digital Computer Transient Stability Studies 811 Some General Comments on the Effect of Excitation on Stability Problems References Chapter 9. Multimachine Systems with Constant Impedance Loads 9.1 Introduction 9.2 Statement of the Problem 9.3 Matrix Representation of a Passive Network 94 Converting Machine Coordinates to System Reference 95 Relation Between Machine Currents and Voltages. 9.6 System Order 9.7 Machines Represented by Classical Methods 9.8 Linearized Model for the Network 9.9 Hybrid Formulation 910 Network Equations with Flux Linkage Model 911 Total System Equations 9.12 Multimachine System Study Problems References Appendix A. Trigonometric Identities for Three-Phase Systems Appendix B_ Some Computer Methods for Solving Differential Equations Appendix C. Normalization. Appendix D— Typicat System Data Appendix E. Excitation Control System Definitions Index 236 243 250 254 268 285 292 299 304 304 307 309 311 315 321 327 333 338 344 347 353 363 365 366 368 368 369 373 374 377 378 381 386 388 390 392 396 397 398, 400 414 424 451 459 copter | A Power System Stability a 1.1 tatreduction Since the industtial revolution man’s demand for and consumption of energy has increased steadily The invention of the induction motor by Nikola Tesla in 1888 si naled the growing importance of electrical energy in the industrial world as well as its use for artificial lighting. A major portion of the energy needs of a modern society is supplied in the form of electrical energy Industrially developed societies need an ever- and the demand on the North American continent has been doubling every ten yeass Very complex power systems have been built to satisfy this increasing demand The trend in electric power production is toward an interconnected network of transmission lines linking generators and loads into large integrated systems, some of which span en- tire continents Indeed, in the United States and Canada, generators located thousands of miles apart operate in parallel This vast enterprise of supplying electrical energy presents many engineering prob- lems that provide the engineer with a variety of challenges The planning, construction, and operation of such systems become exceedingly complex. Some of the problems stimulate the enginect's managerial talents; others tax his knowledge and experience in system design. The entire design must be predicated on automatic control and not on the slow response of human operators To be able to predict the performance of such complex systems, the engineer is forced to seek ever mote powerful tools of analysis and synthesis This book is concerned with some aspects of the design problem, particularly the dynamic performance, of interconnected power systems, Characteristics of the various components of a power system during normal operating conditions and during dis- turbances will be examined, and effects on the overall system performance will be analyzed, Emphasis will be given to the transient behavior in which the system is de- scribed mathematically by ordinary differential equations 1.2. Requirements of a Reliable Electrical Power Service Successful operation of a power system depends largely on the engineer's ability to provide reliable and uninterrupted service to the loads The reliability of the power supply implies much more than merely being available. Ideally, the loads must be fed at constant voltage and frequency at all times. In practical terms this means that both voltage and frequency must be held within close tolerances so that the consumer's 3 4 Chapter 1 equipment may operate satistactorily. For example, a drop in voltage of 10-15% or a reduction of the system frequency of only a few hertz may lead to stalling of the motor loads on the system. Thus it can be accurately stated that the power system operator must maintain a very high standard of continuous electrical service The first requirement of reliable service is to keep the synchronous generators running in parallel and with adequate capacity to meet the load demand. If at any time generator loses synchronism with the rest of the system, significant voltage and current fluctuations may occur and transmission lines may be automatically tripped by theit relays at undesired locations. Ifa generator is separated from the system, it must be re- synchronized and then loaded, assuming it has not been damaged and its prime mover has not been shut down due to the disturbance that caused the loss of synchronism. Synchronous machines do not easily fall out of step under normal conditions If a machine tends to speed up or slow down, synchronizing forees tend to keep it in step Conditions do arise, however, in which operation is such that the synchronizing forces for one or more machines may not be adequate, and small impacts in the system may cause these machines to lose synchronism A major shock to the system may also lead to a loss of synchronism for one or more machines, A second requitement of reliable electrical service is to maintain the integrity of the power network. The high-voltage transmission system connects the generating stations and the Joad centers Interruptions in this network may hinder the flow of power to the load. This usually requires a study of large geographical areas since almost all power systems are interconnected with neighboring systems Economic power as well as emergency power may flow over interconnecting tie lines to help maintain continuity of service Therefore, successful operation of the system means that these lines must re~ main in setvice if firm power is to be exchanged between the areas of the system While it is frequently convenient to talk about the power sysiem in the “steady state,” such a state never exists in the true sense. Random changes in load are taking place at all times, with subsequent adjustments of generation. Furthermore, major changes do take place at times, e.g., a fault on the network, failure in a piece of equip- ment, sudden application of a major load such as a stee! mill, or loss of a line or gen- erating unit, We may look at any of these as a change from one equilibrium state to another It might be tempting to say that successful operation requires only that the new state be a “stable” state (whatever that means) For example, if a generator is lost, the remaining connected generators must be capable of meeting the load demand; or if a line is lost, the power it was carrying must be obtainable from another source, Unfortunately, this view is erroneous in one important aspect: it neglects the dynamics of the transition from one equilibrium state to another. Synchronism frequently may be lost in that transition period, or growing oscillations may occur over a transmission line, eventually leading to its tripping These problems must be studied by the power sys- tem engineer and fall under the heading “power system stability.” 1.3. Statement of the joblem The stability problem is concerned with the behavior of the synchronous machines after they have been perturbed. If the perturbation does not involve any net change in power, the machines should return to their original state. If an unbalance between the supply and demand is created by a change in load, in generation, or in network condi- tions, a new operating state js necessary. In any case aif interconnected synchronous machines should remain in synchronism if the system is stable; ie , they should all re- main operating in parallel and at the same speed Power System Stability 5 The transient following a system perturbation js oscillatory in nature; but if the sys- tem is stable, these oscillations will be damped toward a new quiescent operating con- dition These oscillations, however, are reflected as fluctuations in the power flow over the transmission lines If a certain line connecting two groups of machines undergoes excessive power fluctuations, it may be tripped out by its protective equipment thereby disconnecting the two groups of machines. This problem is termed the stability of the lie line, even though in reality it reflects the stability of the two groups of machines A statement declaring a power system to be “stable” is rather ambiguous unless the conditions under which this stability has been examined are clearly stated. This in- cludes the operating conditions as well as the type of perturbation given to the system, The same thing can be said about tie-line stability, Since we are concerned here with the tripping of the line, the power fluctuation that can be tolerated depends on the initial operating condition of the system, including the line loading and the nature of the impacts to which it is subjected. These questions have become vitally important with the advent of large-scale interconnections. In fact, a severe (but improbable) distur- bance can always be found that will cause instability. Therefore, the disturbances for which the system should be designed to maintain stability must be deliberately selected 1.3.1 Primitive definition of stability Having introduced the term “stability,” we now propose a simple nonmathematical definition of the term that will be satisfactory for elementary problems Later, we will provide a more rigorous mathematical definition The problem of interest is one where a power system operating under a steady load condition is perturbed, causing the readjustment of the voltage angles of the syn- chronous machines Jf such an occurrence creates an unbalance between the system generation and load, it results in the establishment of a new steady-state operating con- dition, with the subsequent adjustment of the voltage angles. The perturbation could be a majos disturbance such as the loss of a generator, a fault or the loss of a Tine, or a combination of such events. It could also be a small load or random load changes occurring under normal operating conditions. Adjustment to the new operating condition is called the transient period The sys- tem behavior during this time is called the dynamic system performance, which is of concern in defining system stability The main criterion for stability is that the syn- chronous machines maintain synchronism at the end of the transient period Definition If the oscillatory response of a power system during the transient period following a disturbance is damped and the system setiles in a finite time to a new steady operating condition, we say the system is stable If the system is not stable, it is considered unstable This primitive definition of stability requires that the system oscillations be damped This condition is sometimes called asymptotic stability and means that the system con- tains inherent forces that tend to reduce oscillations. This is a desirable feature in many systems and is considered necessary for power systems The definition also excludes continuous oscillation fiom the family of stable sys- tems, although oscillators are stable in a mathematical sense. The reason is practical since a continually oscillating system would be undesirable for both the supplier and the user of electric power Hence the definition describes a practical specification for an ac- ceptable operating condition ‘ Chapter 1 1.3.2. Other stal ty problems While the stability of synchronous machines and tie lines is the most important and common problem, other stability problems may exist, particularly in power systems having appreciable capacitances In such cases arrangements must be made to avoid excessive voltages during light load conditions, to avoid damage to equipment, and to prevent self-excitation of machines Some of these problems are discussed in Volume 2, while others are beyond the scope of this book 13.3. Stability of synchronous machines Distinction should be made between sudden and major changes, which we shall call large impacts, and smaller and more normal random impacts A fault on the high- voltage transmission network or the loss of a major generating unit are examples of large impacts. If one of these large impacts occurs, the synchronous machines may lose synchronism This problem is referred to in the literature as the transient stability problem Without detailed discussion, some general comments are in order. First, these impacts have a finite probability of occurring Those that the system should be de- signed to withstand must therefore be selected a priori’ Second, the ability of the sys- tem to survive a certain disturbance depends on its precise operating condition at the time of the occurrence A change in the system loading, generation schedule, network interconnections, or type of circuit protection may give completely different results in a stability study for the same disturbance Thus the transient stability study is a very specific one, from which the engineer concludes that under given system conditions and for a given impact the synchronous machines will or will not remain in synchronism Stability depends strongly upon the magnitude and location of the disturbance and to a lesser extent upon the initial state or operating condition of the system Let us now consider a situation where there are no major shocks or impacts, but rather a random occurrence of small changes in system loading Here we would expect the system operator to have scheduled enough machine capacity to handle the load. We would also expect each synchronous machine to be operating on the stable portion of its power-angle curve, ie, the portion in which the power inereases with incteased angle In the dynamics of the transition from one operating point to another, to adjust for load changes, the stability of the machines will be determined by many factors, including the power-angle curve It is sometimes incorrect to consider a single power-angle curve, since modern exciters will change the operating curve during the petiod under study The problem of studying the stability of synchronous machines under the condition of small load changes has been called “steady-state” stability A more recent and certainly more appropriate name is dynamic stability In contrast to transient stability, dynamic stability tends to be a property of the state of the system Transient stability and dynamic stability are both questions that must be answered to the satisfaction of the engineer tor successful planning and operation of the system This attitude is adopted in spite of the fact that an artificial separation between the two problems has been made in the past. This was simply a convenience to accommo- date the different approximations and assumptions made in the mathematical treat- 1 Inthe United States the regional committees of the National Electric Reliability Council (NERC) specify the contingencies against which the system must be proven stable Power System Stability 7 ments of the two problems n support of this viewpoint the following points are pertinent First, the availability of high-speed digital computers and modern modeling tech- niques makes it possible to represent any component of the power system in almost any degree of complexity required or desired Thus questionable simplifications or assump- tions are no longer needed and are often not justified Second, and perhaps more important, in a large interconnected system the full eflect of a disturbance is felt at the remote parts some time after its occurrence, perhaps a few seconds Thus different parts of the interconnected system will respond to lo- calized disturbances at different times Whether they will act to aid stability is difficult to predict beforehand The problem is aggravated if the initial disturbance causes other disturbances in neighboring areas due to power swings As these conditions spread, a chain reaction may result and large-scale interruptions of service may occur However, in a farge interconnected system, the effect of an impact must be studied over a telatively long period, usually several seconds and in some cases a few minutes. Per- formance of dynamic stability studies for such long periods will require the simulation of system components often neglected in the so-called transient stability studies. 134° Tie-tine oscillations ‘As random power impacts occur during the normal operation of a system, this added power must be supplied by the generators, The portion supplied by the different generators under different conditions depends upon electrical proximity to the posi of impact, energy stored in the rotating masses, governor characteristics, and other factors The machines therefore are never truly at steady state except when at standstill Each machine is in continuous oscillation with respect to the others due to the effect of these random stimuli, These oscillations are reflected in the flow of power in the trans- mission lines, If the power in 4”) line is monitored, periodic oscillations are observed to be superimposed on the steady flow Normally, these oscillations are not large and hence not objectionable. The situation in a tie line is different in one sense since it connects one group of machines to another. These two groups are in continuous oscillation with respect to each other, and this is reflected in the power fiow over the tie line. The situation may be further complicated by the fact that each machine group in turn is connected to other groups Thus the tie line under study may in effect be connecting two huge systems In this case the smallest oscillatory adjustments in the large systems are reflected as sizable power oscillations in the tie line The question then becomes, To what degree can these oscillations be tolerated? The above problem is entirely different from that of maintaining a scheduled power interchange over the tie line; control equipment can be provided to perform this function These controllers are usually too slow to interfere with the dynamic oseilla- tions mentioned above To alter these oscillations, the dynamic response of the com- ponents of the overall interconnected system must be considered. The problem is not only in the tie line itself but also in the two systems it connects and in the sensitivity of control in these systems The electrical strength (admittance) or capacity of the tie cannot be divorced from this problem. For example, a 40-MW oscillation on a 400-MW tie is a much less scrious problem than the same oscillation on a 100-MW tie. The oscillation frequency has an effect on the damping characteristics of prime movers, 8 Chapter | exciters, ete Therefore, there is a minimum size of tie that can be effectively made from the viewpoint of stability 1.4 Effect of an Impact upon System Components In this section a survey of the effect of impacts is made to estimate the elements that should be considered in a stability study. A convenient starting point is to relate an im- pact to a change in power somewhere in the network. Our “test” stimulus will be a change in power, and we will use the point of impact as our reference point The follow- ing effects, in whole or in part, may be felt The system frequency will change be- cause, until the input power is adjusted by the machine governors, the power change will go to or come from the energy in the rotating masses The change in frequency will affect the loads, especially the motor loads. A common tule of thumb used among power system engineers is that a decrease in frequency results in a load decrease of equal percentage; ie, load regulation is 100% The network bus voltages will be affected to a lesser degree unless the change in power is accompanied by a change in reactive power Tine, Times Fig 11 Response of # four-machine system during a transient: (a) stable system (b) unstable system Power System Stability 9 1.4.1. Loss of synchronism Any unbalance between the generation and load initiates a transient that causes the rotors of the synchronous machines to “swing” because net accelerating (or decelerat- ing) torques are exerted on these rotors If these net torques are sufficiently large to cause some of the rotors to swing far enough so that one or more machines “slip a pole,” synchronism is lost. To assure stability, a new equilibrium state must be reached before any of the machines experience this condition. Loss of synchronism can also happen in stages, ¢ g., if the initial transient causes an electrical link in the transmission network to be interrupted during the swing This creates another transient, which when superimposed on the first may cause synchronism to be lost, . Let us now consider a severe impact initiated by a sizable generation unbalance, say excess generation, The major portion of the excess energy will be converted into kinetic energy. Thus most of the machine rotor angular velocities will increase. A lesser part will be consumed in the loads and through various losses in the system However, an appreciable increase in machine speeds may not necessarily mean that synchronism will be lost The important factor here is the angle difference between machines, where the rotor angle is measured with respect to a synchronously rotating reference. This is illustrated in Figure 1 1 in which the rotor angles of the machines in a hypothetical four-machine system are plotted against time during a transient In case (a) all the rotor angles increase beyond w radians but all the angle differences are small, and the system will be stable if it eventually settles to a new angle, In case (b) itis evident that the machines are separated into two groups where the rotor angles continue to drift apart This system is unstable 1.4.2. Synchronous machine during a transient During a transient the system seen by a synchronous machine causes the machine terminal voltage, rotor angle, and frequency to change. The impedance seen “looking into” the network at the machine terminal also may change The field-winding voltage will be affected by: 1, Induced eutrents in the damper windings (or rotor iron) due to sudden changes in armature currents The time constants for these currents are usually on the order of less than 0.1 s and are often referred to as “subtransient” effects 2. Induced currents in the field winding due to sudden changes in armature currents The time constants for this transient are on the order of seconds and are referred to as “transient” effects 3. Change in rotor voltage due to change in exciter voltage if activated by changes at the machine terminal. Both subtransient and transient effects are observed Since the subtransient eflects decay very rapidly, they are usually neglected and only the transient effects are considered important Note also that the behavior discussed above depends upon the network well as the machine parameters The machine output power will be affected by the change in the rotor-winding EMF and the rotor position in addition to any changes in the impedance “seen” by the ma- chine terminals. However, until the speed changes to the point where it is sensed and corrected by the governor, the change in the output power will come from the stored energy in the rotating masses. The important parameters here are the kinetic energy in MW-s per unit MVA (usually called 7) or the machine mechanical time constant 7), which is twice the stored kinetic energy per MVA. pedance as 10 Chapter 1 When the impact is large, the speeds of all machines change so that they are sensed by their speed governors. Machines under load frequency control will correct for the power change. Until this correction is made, each machine’s share will depend on its regulation or droop characteristic Thus the controlled machines are the ones re~ sponsible for maintaining the system frequency The dynamics of the transition period, however, are important The key parameters are the governor dynamic characteristics, In addition, the flow of the tie lines may be altered slightly. Thus some machines are assigned the requirement of maintaining scheduled flow in the ties. Supplementary controls are provided to these machines, the basic functions of which are to permit each control area to supply a given load. The responses of these controls are relatively slow and their time constants are on the order of seconds This is appropriate since the scheduled economic loading of machines is secondary in importance to stability 1.8. Methods of Simulation If we look at a large power system with its numerous machines, lines, and loads and consider the complexity of the consequences of any impact, we may tend to think it is hopeless to attempt analysis. Fortunately, however, the time constants of the phenom- ena may be appreciably different, allowing concentration on the key elements affecting the transient and the area under study The first step in a stability study is to make a mathematical model of the system during the transient. The elements included in the model are those affecting the ac- celeration (or deceleration) of the machine rotors The complexity of the model de- pends upon the type of transient and system being investigated Generally, the com- ponents of the power system that influence the electrical and mechanical torques of the machines should be included in the model These components are: The network before, during, and after the transient The loads and their characteristics The parameters of the synchronous machines The excitation systems of the synchronous machines The mechanical turbine and speed governor Other important components of the power plant that influence the mechanical torque 7. Other supplementary controls, such as tie-line controls, deemed necessary in the mathematical description of the system Thus the basic ingredients for solution are the knowledge of the initial conditions of the power system prior to the start of the transient and the mathematical description of the main components of the system that affect the transient behavior of the synchronous machines The number of power system components included in the study and the com- plexity of their mathematical description will depend upon many factors. In general, however, differential equations are used to describe the various components Study of the dynamic behavior of the system depends upon the nature of these differential equations 1.5.1 Linearized system equal ns. If the system equations are linear (or have been linearized), the techniques of linear system analysis are used to study dynamic behavior The most common method is to Power System Stability u simulate each component by its transfer function The various transfer function blocks are connected to represent the system under study. The system performance may then be analyzed by such methods as root-locus plots, frequency domain analysis (Nyquist criteria), and Routh’s criterion. The above methods have been frequently used in studies pertaining to small systems ora small number of machines Fat larger systems the state-space model has been used more frequently in connection with system studies described by linear differential equa- tions Stability characteristics may be determined by examining the eigenvalues of the A matrix, where A is defined by the equation k= Ax +Bu ap where x is an n vector denoting the states of the system and A is a coefficient matrix The system inputs are represented by ther vector u, and these inputs are related mathe- matically to differential equations by ann x 7 matrix B_ This description has the ad- vantage that A may be time varying and u may be used to represent several inputs if necessary 1.5.2 Large system with nenlinear equations The system equations for 4 transient stability study are usually nonlinear Here the system is described by a large set of coupled nonlinear differential equations of the form % = fxn (2) where fis an nm vector of nonlinear functions Determining the dynamic behavior of the system described by (1 2) is a more diffi- cult task than that of the linearized system of (I 1), Usually cime solutions of the non- linear differential equations are obtained by numerical methods with the aid of digital computers, and this is the method usually used in power system stability studies Stability of synchronous machines is usually decided by behavior of their rotor angles, as discussed in Section 141 More recently, modern theories of stability of nonlinear systems have been applied to the study of power system transients to determine the stability of synchronous machines without obtaining time solutions Such efforts, while they seem to offer considerable promise, are still in the research stage and not in common use Both linear and nonlinear equations will be developed in following chapters Problems 1.1 Suggest definitions for the following terms: a Power system reliability b_ Power system security © Power system stability 12 Distinguish between steady-state (dynamic) and transient stability according to a The type of disturbance 2. The nature of the defining equations 13. What is tie line? Is every line a tie line? 14 What is an impact insofar as power system stability is concerned? 15. Consider the system shown in Figure PIS where a mass Mf is pulled by a driving force (1) and is restrained by a linear spring K and an ideal dashpot B 12 Chapter 1 Write the differential equation for the system in terms of the displacement variable x and determine the relative values of B and K to provide critical damping when f(z) is a unit step function ea mw Era Fig PLS 16 Repeat Problem 1 5 but convert the equations to the state-space form of (I 1) chapter 2 The Elementary Mathematical Model A stable power system is one in which the synchronous machines, when perturbed, will either return to their original state if there is no net change of power or will acquire a new state asymptotically without losing synchronism Usually the perturbation causes a transient that is oscillatory in nature; but if the system is stable, the oscillations will be damped The question then arises, What quantity or signal, preferably electrical, would enable us to test for stability? One convenient quantity is the machine rotor angle measured with respect to a synchronously rotating reference If the difference in angle between any two machines increases indefinitely or if the oscillatory transient is not sufficiently damped, the system is unstable The principal subject of this chapter is the study of stability based largely on machine-angle behavior 2.1. Swing Equation The swing equation governs the motion of the machine rotor relating the inertia torque to the resultant of the mechanical and electrical torques on the rotor; ie! Jé= 7, Nem QD where J is the moment of inertia in kg-m? of all rotating masses attached to the shaft, Gis the mechanical angle of the shaft in radians with respect to a fixed reference, and T, is the accelerating torque in newton meters (N-m) acting on the shaft (See Kim- bark [1] for an excellent discussion of units and a dimensional analysis of this equa- tion.) Since the machine is a generator, the driving torque Ij, is mechanical and the retarding or load torque 7, is electrical. Thus we write Ty= Te TE Nem (2.2) which establishes a useful sign convention, namely, that in which a positive Ty ac- celerates the shaft, whereas a positive 7, is a decelerating torque The angular refer- ence may be chosen relative to a synchronously rotating reference frame moving with The dot notation is used to signily derivatives with respect to time Thus 8 14 Chapter 2 constant angular velocity «,,? 4 = (wat + @) + 6, Tad (2.3) where a is a constant, The angle a is needed if 5, is measured from an axis different from the angular reference frame; for example, in Chapter 4 a particular choice of the reference for the rotor angle 4,, gives a = r/2and 8 = wal + #/2 + bn. From (23) we see that # may be replaced by é,, in (2.1), with the result Bea = Jin = T, Nom 24) where J is the moment of inertia in kg-m?, 5, is the mechanical (subscript m) torque angle in rad with respect to a synchronously rotating reference frame, wy is the shaft angular velocity in rad/s, and [, is the accelerating torque in N-m Another form of (2.4) that is sometimes useful is obtained by multiplying both sides by wp, the shaft angular velocity in rad/s. Recalling that the product of torque T and angular velocity w is the shaft power P in watts, we have Tiindm = Pa — Ps W (25) The quantity Jaq is called the inertia constant and is denoted by M. (See Kimbark (1] pp. 22-27 and Stevenson [2], pp. 336-40 for excellent discussions of the inertia constant) It is related to the kinetic energy of the rotating masses W,, where W, = (1/2) Jo} joules Then M is computed as M = Sein = 2Wy om VS 26 It may seem rather strange to call M a constant since it depends upon w, which certainly varies during a transient On the other hand the angular frequency does not change by a large percentage before stability is lost To illustrate: for 60 Hz, w, = 377 rad/s, and a 1% change in w, is equal to 377 rad/s A constant slip of 1% of the value of w, for one second will change the angle of the rotor by 3.77 rad Certainly, this would lead to loss of synchronism The equation of motion of the rotor is called the swing equation. It is given in the literature in the form of (2 4) or in terms of power, Mi, = Midy = Py - Pe W (27) where M is in J-s, 5, is in rad, w,, is in tad/s, and P is in W. In relating the machine inertial performance to the network, it would be more useful to write (2 7) in terms of an electrical angle that can be conveniently related to the position of the rotor Such an angle is the rorque angle 6, which is the angle between the field MMF and the resultant MMF in the air gap, both rotating at syn- chronous speed It is also the electrical angle between the generated EMF and the resultant stator voltage phasors The torque angle 6, which is the same as the electrical angle 4,, is related to the rotor mechanical angle 6,, (measured from a synchronously rotating frame) by 6 = 8, = (p/2)bm (2.8) where p is the number of poles. (In Europe the practice is to write 3, = péy, where p is the number of pole pairs ) 2 “The subscript R is used to mean rated” for ali quantities including speed. which is designated as 61 in ANSI standards ANSI Y 105. 1968 Hence wR =) in every case The Elementary Mathematical Model 15 For simplicity we drop the subscript ¢ and write simply 4, which is always under- stood to be the electrical angle defined by (2 8) From (2 7) and (2 8) we write QM/p)é = QM/p)o = P, W (2.9) which relates the accelerating power to the electrical angle @ and to the angular velocity of the revolving magnetic field In most problems of interest there will be a large number of equations like (2.9), one for cach generator shaft (and motor shaft too if the motor is large enough to warrant detailed representation) In such large systems problems we find it convenient to normalize the power equations by dividing all equations by a common three-phase voltampere base quantity Sy; _ Then (2 9) becomes a per unit (pu) equation (2M /pSx)8 = 2M /pSes)o = Pa[Ses = Poy a 2.10) where M, p, 6, and w are in the same units as before; but P is now in pu (noted by the subscript u) 2.2. Units Ithas been the practice in the United States to provide inertial data for rotating machines in English units The machine nameplate usually gives the rated shaft speed in revolutions per minute (r/min). The form of the swing equation we use must be in MKS units (or pu) but the coefficients, particularly the moments of inertia, will usually be derived from a mixture of MKS and English quantities We begin with the swing equation in N-m QU /p)b = QI/p)e = T, Nem QM) Now normalize this equation by dividing by @ base quantity equal to the rated torque at rated speed Ty 3 far = 60S ar /20my (2.12) where Spy is the three-phase VA tating and ng is the tated shaft speed in r/min Dividing (2 11) by (212) and substituting 120 fx /ng for p, we compute (J ?n} /900., 55) = T/Ty = Tay PU (2.13) where we have substituted the base system radian frequency we = 2x fe for the base frequency Note that o in (2 13) is in rad/s and Ty, is in pu The U.S. practice has been to supply /, the moment of inertia, as a quantity usually called WR?, given in units of lbm-ft?, “The consistent English unit for J is slug-ft? or WR?/g where g is the acceleration of gravity (32,17398 ft/s?) We compute the cor- responding MKS quantity as 7 WR? slug-ft? | 1 felbfis? | 746 Ws § 1 slug-ft? | 550 ft Ibe Substituting into (2 13), we write TA6(WR*) 0? 550 gO00)un Ses The coefficient of & can be clarified if we recall the definition of the kinetic energy of a (2.14) 16 Chapter 2 rotating body W/,, which we can write as 2 Ly, SUR?) Gam? 330 g 3600 2311525 x 10*(WR7)nk FT Then (2 14) may be written as (QW. /Spswp)e = Tor pu (2.15) ‘We now define the important quantity HE Wy/Ses § (2.16) where S33 = rated three-phase MVA of the system We = (2 311525 x 10°")PR?)a MI Then we write the swing equation in the form most useful in practice: QH/op)o = T,, pu Q17) where # is in s, a is in rad/s, and Z,is in pu. Note that w is the angular velocity of the revolving magnetic field and is thus related directly to the network voltages and currents For this reason it is common to give the units of w as electrical rad/s Note also that the final form of the swing equation has been adapted for machines with any number of poles, since all machines on the same system synchronize to the same wp ‘Another form of the swing equation, sometimes quoted in the literature, involves some approximation It is particularly used with the classical modet of the synchronous machine Recognizing that the angular speed w is nearly constant, the pu accelerating power P, is numerically nearly equal to the accelerating torque T, A modified (and approximate) form of the swing equation becomes (2H/wp)e & P, pu 218) The quantity H is often given for a particular machine normalized to the base VA rating for that machine. This is convenient since these machine-normalized H quantities are usually predictable in size and can be estimated for machines that do not physically exist Curves for estimating H are given in Figures 21 and 22 The quantities taken from these curves must be modified for use in system studies by converting from the machine base VA to the system base VA. Thus we compute Hse = Hesci(Snsmach / S035) $ (2.19) The value Of Hy, is usually in the range of 1-5 Values for Hy vary over a much wider range With Sys, = 100 MVA values of Hy, from a few tenths of a second (for small generators) to 25-30 s (for large generators) will often be used in the same study. Typical values of 7 (in MJ) are given in Appendix D. 2.3. Mechanical Torque The mechanical torques of the prime movers for large generators, both steam and waterwheel turbines, are functions of speed. (See Venikov [6], Sec 1 3, and Crary [7], Vol II, Sec 27) However we should carefully distinguish between the case of the un- regulated machine (not under active governor control) and the regulated (governed) case The Elementary Mathematical Model 7 MWe AVA reer o 100 20 30, “a0 co Generator Rating, VA ‘) 45) 4.) < 23 > 1800 r/min eucleor 339 —_— 2 1505 8001005 1200 T46OV800. 1809" 7200 200 Generator Rating, VA 6 Fig 2.1 Inertia constants for large steam turbogenerators: (a) turbogenerators rated 500 MVA and below [B, p. 120}, (b) expected future large turbogenerators (@ IEEE Reprinted from JEEE Trans val PAS-90, Nov /Dec 1971) 2.3.1 Unregulated machines For a fixed gate or valve position (ie, when the machine is not under active gov- ernor control) the torque speed characteristic is nearly linear over a limited range at rated speed, as shown in Figure 2 3(a) No distinction seems to be made in the literature between steady-state and transient characteristics in this respect Figure 2.3(a) shows that the prime-mover speed of a machine operating at a fixed gate or valve position will drop in response to an increase in load. The value of the turbine torque coefficient suggested by Crary [7] is equal to the loading of the machine in pu This can be veri- fied as follows, From the fundamental relationship between the mechanical torque A= 450 514 ¢/min B= 200 400 ¢/min C= 198 100 r/min D= 8 120e/min ‘Generator Rating, MVA, Fig 22 Inertia constants of large vertical-type waterwheel generators, including allowance of 15% for waterwheels (IEEE Reprinted from Elecir Eng vol. 56. Feb. 1937) 18 Chapter 2 ° Re Speed mas e Fig 23. Turbine torque speed characteristic: (a) unregulated machine, (b) regulated machine Tq and power Pa, Ty = Pao Nem (220) ‘we compute, using the definition of the differential, ay = FF a, + do Nem (2.21) Near rated load (2.21) becomes Ty = (1/oR)dPq — (Pup (ok )dio Nom (2.22) If'we assume constant mechanical power input, dP, = 0 and Tq = — (Pax fark }deo Nom (2.23) This equation is normalized by dividing through by Tx = Pax /wx with the result Iq = ~deo pu (224) where all values are in pu, This relationship is shown in Figure 2 3(a) 2.3.2 Regulated machines In regulated machines the speed control mechanism is responsible for controlling the throttle valves to the steam turbine or the gate position in hydroturbines, and the The Elementary Mathematical Mode! 9 mechanical torque is adjusted accordingly This occurs under normal operating condi- tions and during disturbances To bestable under normal conditions, the torque speed characteristic of the turbine speed control system should have a “droop characteristic”; ie, a drop in turbine speed should accompany an increase in load. Such a characteristic is shown in Fig- ure 2 3(b)_ A typical “droop” or “speed regulation” characteristic is 5% in the United States (4% in Europe). This means that a load pickup from no load (power) to full load (power) would correspond to a speed drop of 5% if the speed load characteristic is assumed to be linear The droop (regulation) equation is derived as follows: from Figure 2.3(b), Im = Tyo + Ta, and Tma = ~W,/R, where R is the regulation in rad/ Noms Thus Tm = Tro — (0 ~ @x)/R Nom (2.25) Multiplying (2 25) by og, we can write Pu % Tix = Pao — (ax /R)@s (2.26) Let Py = pu mechanical power on machine VA Pru £ Py /Sq = Pro /Se ~ (@r/SeR)Os or Pg = Prox ~ hes /SeR pu 227) Since P,y = Pa ~ Pros Praw = ~ RO /SpR = —0y,/R, pu (2.28) where the pu regulation R, is derived from (2 28) or Ry 2 SpR/uk pu 2.29) As previously mentioned, R, is usually set at 005 in the United States We also note that the “effective” regulation in a power system could be appreciably different from the value 0.05 if some of the machines are not under active governor control. If Sp is the sum of the ratings of the machines under governor control, and DSip is the sum of the ratings of all machines, then the effective pu regulation is given by Rue = RLS /ESs0) (2.30) Similarly, if @ system base other than that of the machine is used in a stability study, the change in mechanical power in pu on the system base Pmas is given by Paaw = ~(Ss@av/ Sey) pu (2.31) A block diagram representing (2.28) and (2.31) is shown in Figure 2.4 where K = Ss/Sin The droop characteristic shown in Figure 2.3(b) is obtained in the speed control system with the help of feedback. It will be shown in Volume 2 that without feed- back the speed control mechanism is unstable, Finally we should point out that the steady-state regulation characteristic determines the ultimate contribution of each machine to a change in load in the power system and fixes the resulting system fre- quency error 20 Chapter 2 1 “oy ale he SS Fig 24 Block diagram representation of the droop equation During transients the discrepancy between the mechanical and electrical torques for the various machines results in speed changes The speed control mechanism for each machine under active governor controt will attempt to adjust its output accord- ing to its regulation characteristic Two points can be made here: 1 For a particular machine the regulation characteristic for a small (and sudden) change in speed may be considerably different in magnitude from its overall average regulation 2. In attempting to adjust the mechanical torque to correspond to the speed change, time lags are introduced by the various delays in the feedback elements of the speed control system and in the steam paths; therefore, the dynamic response of the turbine could be appreciably different from that indicated by the steady-state regulation characteristic This subject will be dealt with in greater detail in Volume 2 2.4 Electrical Torque In general, the electrica] torque is produced by the interaction between the three stator cireuits, the field circuit, and other cireuits such as the damper windings. Since the three stator circuits are connected to the rest of the system, the terminal voltage is determined in part by the external network, the other machines, and the loads. The flux linking each circuit in the machine depends upon the exciter output voltage, the loading of the magnetic circuit (saturation), and the current in the different windings Whether the machine is operating at synchronous speed or asynchronously affects all the above factors. Thus a comprehensive discussion of the electrical torque depends upon the synchronous machine representation If all the circuits of the machine are taken into account, discussion of the electrical torque can become rather involved Such a detailed discussion will be deferred to Chapter 4. For the present we simply note that the electrical torque depends upon the flux linking the stator windings and the cuzrents in these windings. If the instantaneous values of these flux linkages and currents are known, the correct instantaneous value of the electrical torque may be determined. As the rotor moves, the flux linking each stator winding changes since the inductances between that winding and the rotor circuits are functions of the rotor position These flux linkage relations are often simplified by using Park’s transforma- tion A modified form of Park’s transformation will be used here (see Chapter 4) Under this transformation both currents and flux linkages (and hence voltages) are transformed into two fictitious windings located on axes that are 90° apart and fixed with respect to the rotor One axis coincides with the center of the magnetic poles of the rotor and is called the direct axis The other axis lies along the magnetic neutral axis and is called the quadrature axis Expressions for the electrical quantities such as power and torque are developed in terms of the direct and quadrature axis voltages (or flux linkages) and currents The Elementary Mathematical Model 21 A simpler mathematical model, which may be used for stability studies, divides the electrical torque into two main components, the synchronous torque and a second com- ponent that includes all other electrical torques We explore this concept briefly as an aid to understanding the generator behavior during transients 2.4.1 Synchronous torque The synchronous torque is the most important component of the electrical torque. It is produced by the interaction of the stator windings with the fundamental com- ponent of the air gap flux. It is dependent upon the machine terminal voltage, the rotor angle, the machine reactances, and the so-called quadrature axis EMF, which may be thought of as an effective rotor EMF that is dependent on the armature and rotor eur rents and is a function of the exciter response Also, the network configuration affects the value of the terminal voltage 242 Other electrical torques During a twansient, other extraneous electrical torques are developed in a syn- chronous machine The most important component is associated with the damper windings. While these asynchronous torques are usually small in magnitude, their effect on stability may not be negligible The most important effects are the following 1 Positive-sequence damping results from the interaction between the positive-sequence air gap flux and the rotor windings, particularly the damper windings. In general, this effect is beneficial since it tends to reduce the magnitude of the machine oscilla- tions, especially after the first swing It is usually assumed to be proportional to the slip frequency, which is nearly the case for small slips Negattve-sequence braking results from the interaction between the negative-sequence air gap flux during asymmetrical faults and the damper windings Since the nega- tive-sequence slip is 2 — s, the torque is always retarding to the rotor Its magnitude is significant only when the rotor damper winding resistance is high 3. The de braking is produced by the de component of the armature current during faults, which induces currents in the rotor winding of fundamental frequency. Their interaction produces 4 torque that is always retarding to the rotor It should be emphasized that if the correct expression for the instantaneous elec- trical torque is used, all the above-mentioned components of the electrical torque will be included In some studies approximate expressions for the torque are used, eg , when considering quasi-steady-state conditions Here we usually make an estimate of the components of the torque other than the synchronous torque 2.5 Power-Angle Curve of a Synchronous Machine Before we leave the subject of electrical torque (or power), we return momentarily to synchronous power to discuss a simplified but very useful expression for the relation between the power output of the machine and the angle of its rotor Consider two sources V = V/Q and E = £/5 connected through a reactance x as shown in Figure 2 S{a)> Note that the source V'is chosen as the reference. A current 3A phasor is indicated with a bur above the symbol for the ems quantity. For example if I is the ris value ofthe current, Tis the current phasor By definition the phasor 7 is given by the transformation @ where? 4 rel ~ I(cos 4 + jsin 8) = PLV7 I cos (wt + 6)],_ A phasor is 7 complex number felated to the ‘corresponding time quamity 12) by i) = Ge (V2 Ie!) = VIIcos(or + 6) = P-'(lel) 22 Chapter 2 1a @ ° we © ” Fig, 25. A simple two-machine system: (a) schematic representation, (b) power-angle curve T = 1/0 flows between the two sources We can show that the power P is given by P = (EV/x) sind (232) Since E, V, and x are constant, the relation between P and 6 is a sine curve, as shown in Figure 2 5(b). We note that the same power is delivered by the source F and received by the source V since the network is purely reactive Consider a round rotor machine connected to an infinite bus. At steady state the machine can be represented approximately by the above circuit if V is the terminal voltage of the machine, which is the infinite bus voltage; x is the direct axis synchronous reactance; and £ is the machine excitation voltage, which is the EMF along the quadra- ture axis We say approximately because such factors as magnetic circuit saturation and the difference between direct and quadrature axis reluctances are overlooked in this simple representation But (2.32) is essentially correct for a round rotor machine at steady state Equation (2.32) indicates that if £, V, and x are constant, EV/x is a constant that we may designate as Py to write P = Py sin 6; and the power output of the machine is a function only of the angle 5 associated with E Note that E can be chosen to be any convenient EMF, not necessarily the excitation voltage; but then the appropriate x and 8 must be defined accordingly studies 2.5.1 Classical representation of a synchronous machine in stabi The EMF of the machine (i, the voltage corresponding to the current in the main field winding) can be considered as having two components: a component E’ that cor- responds to the flux linking the main field winding and a component that counteracts the armature reaction, The latter can change instantaneously because it corresponds to currents, but the former (which corresponds to flux linkage) cannot change instantly, The Elementary Mathematical Model 23 ‘When a change in the network occurs suddenly, the flux linkage (and hence E") will not change, but currents will be ptoduced in the armature; hence other currents will be induced in the various rotor circuits to keep this flux linkage constant. Both the arma- ture and rotor currents will usually have ac and de components as required to match the ampere-turns of various coupled coils The flux will decay according to the effective time constant of the field circuit At no load this time constant is on the order of sev- eral seconds, while under load it is reduced considerably but still on the order of one second or higher From the above we can see that for a period of less than a second the natural char- acteristic of the field winding of the synchronous machine tends to maintain constant fiux linkage and hence constant £’. Exciters of the conventional type do not usually respond fast enough and their ceilings are not high enough to appreciably alter this picture. Furthermore, it has been observed that during a disturbance the combined effect of the armature reaction and the excitation system is to help maintain constant flux linkage for a period of a second or two. This period is often considered adequate for determining the stability of the machine. Thus in some stability studies the assump- tion is commonly made that the main field flux linkage of a machine is constant The main field-winding flux is almost the same as a fictitious flux that would create an EMF behind the machine direct axis transient reactance. The model used for the synchronous machine is shown in Figure 2.6, where xj is the direct axis transient reactance Fig 26 Representation of a synchronous machine by # constant voltage behind transient reactance The constant voltage source £/3 is determined from the initial conditions, ic, pretransient conditions, During the transient the magnitude £ is held constant, while the angle 6 is considered as the angle between the rotor position and the terminal voltage V Example 21 For the circuit of Figure 26 let V = 0 pu, xj operating at P = 08 pu at 08PF 02 pu, and the machine initially Solution Using Vas reference, 7 = 1.0/0 fy = 10/-36.9" = 08 ~ j0.6 E = E[s = 1.0 + j0.2008 — jo6) = 112 + j016 = 11314 /8.06° The magnitude of E is 1.1314 This will be held constant during the transient, although émay vary The initial value of 6, called dp, is 8.06" 24 Chapter 2 During the transient period, assuming that is held constant, the machine power as a function of the angle @ is also given by a power-angle curve Thus P = (EV/xj)sing = Pysind (233) For the example given above Py = 1.1314/02 = 5 657 25.2 Synchro Consider a synchronous machine the terminal voltage of which is constant. This is the case when the machine is connected to a very large power system (infinite bus). Let us assume that the machine can be represented by a constant voltage magnitude be- hind a constant reactance, as shown in Figure 26 The power is given by (2 32) Let the initial power delivered by the machine be P,, which corresponds to a rotor angle 5) (which is the same as the angle of the EMF £). Let us assume that 6 changes from its initial value 6 by a small amount 8,;i¢, 8 = 4, + 5, From (2 32) P also changes to P= P, + Py Then we may write Py + Py = Pysin(& + 4) = Pu(sin a c0s8, + coséy sin és) (2.34) If 4g is small then, approximately, cos 6, & 1 and sind % 44, or Py + Py % Py sin dy + (Py C08 55)6, ing power coefficients and since Py = Py sin by, Py = (Pig COS bo) by, (235) The quantity in parentheses in (2.35) is defined to be the synchronizing power co- efficient and is sometimes designated P, From (2.35}we also observe that oF P.& Py cosy = on (2:36) Equation (2 35) is sometimes written in one of the forms. Py oP Pibs = Fs (237) (Compare this result with dP, the differential of P.) In the above analysis the appropriate values of x and £ should be used to obtain Py. In dynamic studies xj and the voltage £’ are used, while in steady-state stability analysis a saturated steady-state reactance x, is used. If the control equipment of the machine is slow or inoperative, it is important that the machine be operating such that 0 S 6 S /2 for the operating point to be stable in the static or steady-state sense This is the same as having a positive synchronizing power coefficient This criterion was used in the past to indicate the so-called “steady-state stability limit.” 2.6 Natural Frequencies of Oscillation of a Synchronous Machine A synchronous machine, when perturbed, has several modes of oscillation with re- spect to the rest of the system. There are also cases where coherent groups of machines oscillate with respect to other coherent groups of machines. These oscillations cause fluctuations in bus voltages, system frequencies, and tie-line power flows. It is im- portant that these oscillations should be small in magnitude and should be damped if the system is to be stable in the sense of the definition of stability given in Section 1.2.1 The Elementary Mathematical Model 25 In this section we will illustrate the inherent oscillatory nature of @ synchronous machine by the following example Example 22 A bvo-pole synchronous machine is connected to an infinite bus with voltage 7 through a reactance x as in Figure 2 S(a)_ The voltage F remains constant, and a small change in speed is given to the machine {the rotor is given a small twist); ie, © = wy + eu(¢), where u(t) is a unit siep function, Let the resulting angle change be 5, Let the damping be negligible, Compute the change in angle as a function of time and determine its frequency of oscillation. Solution From (2 10) we write Mi/Sg, + P, = P, ButweletS = dy + 6ysuch that é = 3, and P, = Po + Pea; Pm is constant Then Més/Sp + Pig = Pu - Pap = 0 since 5, = 0. From (2 37) for small 6, we write P,, = P,é,, where from (2 36) P, is the synchronizing power coefficient Then the swing equation may be written as Mig /Sp + P.é, = 0 which has the solution of the form 8) = MIP, sin VP,Sq/Mt elect rad (238) Equation (2.38) indicates that the angular frequency of oscillation of the synchronous machine with respect to the rest of the power system is given by VP.3m/M_ This fre- quency is usually referred to as the natural frequency of the synchronous machine it should be noted that P, is a function of the operating point on the power-angle characteristic Different machines, especially different machine types, have different inertia constants Therefore, the different machines in a power system may have some- what different natural frequencies We now estimate the order of magnitude of this frequency From (2.6) and (2 16) we write M/Spy = 2H/oey OF P,Sqs/M = P,o_/2H where P, is in pu, &y is in 1ad/s, and Hisins, Now P, is the synchronizing power coefficient in pu (on a base of the machine three-phase rating). If the initial operating angle 8 is small, P, is approximately equal to the amplitude of the power-angle curve We must also be careful with the units For example, a system having P,/Sp; = 2 pu, H = 8, Woe = VEX STA x 8) = 685 rad/s fox = 685/28 = 109 Hz If MKS units are used, we write fous = (1/28) Vaf(PFSa 1) (239) where f= system frequency in Hz Sgs = three-phase machine rating in MVA H = inertia constant in s P, = synchronizing power coefficient in MW/rad Next, we should point out that a system of two finite machines can be reduced to a single equivalent finite machine against an infinite bus The equivalent inertia is Ih fd, + 42) and the angle is da — 624 26 Chapter 2 Thus we conclude that each machine oscillates with respect to other machines, each coherent group of machines oscillates with respect to other groups of machines, and soon. The frequencies of oscillations depend on the synchronizing power coefficients and on the inertia constants. 2.7 System of One Machine against an Infinite Bus—The Classical Model An infinite bus is a source of invariable frequency and voltage (both in magnitude and angle). A major bus of a power system of very large capacity compared to the rating of the machine under consideration is approximately an infinite bus, The inertia of the machines in a large system will make the bus voltage of many high-voltage buses essentially constant for transients occurring outside that system Consider a power system consisting of one machine connected to an infinite bus through a transmission line A schematic representation of this system is shown in Figure 2 7(a) @ * Fig 27 One machine connected to an infinite bus through a transmission line: (a) one-line diagram (b) equivalent circuit The equation of motion of the rotor of the finite machine is given by the swing equation (2.7) ot 2.10) To obtain a time solution for the rotor angle, we need to develop expressions for the mechanical and the electrical powers. In this section the simplest mathematical model is used This model, which will be referred to as the classical model, requires the following assumptions: 1 The mechanical power input remains constant during the period of the transient 2 Damping or asynchronous power is negligible 3. The synchronous machine can be represented (electrically) by a constant voltage source behind a transient reactance (see Section 2.5 1) 4, The mechanical angle of the synchronous machine rotor coincides with the electrical phase angie of the voltage behind transient reactance 5. Ifa local load is fed at the terminal voltage of the machine, it ean be represented by a constant impedance (or admittance) to neutral The period of interest is the first swing of the rotor angle 8 and is usually on the order of one second or less. At the start of the transient, and assuming that the impact initiating the transient creates a positive accelerating power on the machine rotor, the rotor angle increases. If the rotor angle increases indefinitely, the machine loses synchronism and stability is lost If it reaches a maximum and then starts to decrease, the resulting motion will be oscillatory and with constant amplitude Thus according to this model and the assumptions used, stability is decided in the first swing. (If damping is present the amplitude will decrease with time, but in the classical model there is very little damping ) The Elementary Mathem a7 Fig 28 Equivalent circuit for a system of one machine against an infinite bus The equivalent electrical circuit for the system is given in Figure 2.7(b) In Fig- ure 2.7 we define 7, = terminal voltage of the synchronous machine V = V/Q = voltage of the infinite bus, which is used as reference xj= direct axis transient reactance of the machine Zy = series impedance of the transmission network (including transformers) Z, = equivalent shunt impedance at the machine terminal, including local loads if any By using a ¥-A transformation, the node representing the terminal voltage V, in Figure 2.7 can be eliminated The nodes to be retained (in addition to the reference node) are the internal voltage behind the transient reactance node and the infinite bus. These are shown in Figure 2.8 as nodes 1 and 2 respectively Also shown in Figure 28 are the admittances obtained by the network reduction. Note that while three admit- tance elements are obtained (viz, yiz, vio, ANd V20), ¥20 is omitted since it is not needed in the analysis The two-port network of Figure 28 is conveniently described by the equation Al Yu Yay |B sltle v (2.40) hl [fa Fal 7 The driving point admittance at node 1 is given by Yi, = Fy (Ou + Pro where we use lower case ys to indicate actual admittances and capital ¥"s for matrix elements, The negative of the transfer admittance j,, between nodes 1 and 2 defines the admittance matrix element (1,2) 0t Fiz = Yo (8 = “Fr From elementary network theory we can show that the power at node | is given by P, = GeElf ot Pi = EY, cos0y + EV¥ 3008 (G2 — 8) Now define G, 2 Yu.cos6,, and y = 6): — 7/2, then Py = E?Gy, + EVYysin(5 ~ y) = Pe + Pysin(6 ~ 7) at) The relation between P, and é in (2.41) is shown in Figure 2.9 Examining Figure 29, we note that the power-angle curve of @ synchronous machine connected to an infinite bus is a sine curve displaced from the origin vertically by an amount P, which represents the power dissipation in the equivalent network, and horizontally by the angle y, which is determined by the real component of the transfer admittance Viz, In the special case where the shunt load at the machine terminal V, is open and where the transmission network is reactive, we can easily prove that Pc = and y =.0. In this case the power-angle curve becomes identical to that given in@33) 28 Chapter 2 Fig 29° Power output of a synchronous machine connected to an infinite bus, Example 2.3 A synchronous machine is connected to an infinite bus through a transformer and a double circuit transmission line, as shown in Figure 210. The infinite bus voltage V = 10pu The direct axis transient reactance of the machine is 020 pu, the trans former reactance is 010 pu, and the reactance of each of the transmission lines is 0.40 pu, all to a base of the rating of the synchronous machine, Initially, the machine is delivering 08 pu power with a terminal voltage of 105 pu The inertia constant H = 5MJ/MVA All resistances are neglected The equation of motion of the ma- chine rotor is to be determined Oe veo Fig 210. System of Example 2.3 Solution The equivalent circuit of the system is shown in Figure 2.11. For this system: Fae 1/i0S= -j20 Fy = -j20 Fo = 0 Oy = ~ 7/2 Fy = j20 Oi = w/2 therefore, Pc = Oand y = 0 The electrical power is given by Pow P= Pet EVYusin(@ - ¥) = Since the initial power is Py = 0.8 pu, then sing, = 04 EVY,sin6 = 2E sind D jom® jo WP io” @ Crret Fig 2.11 initial equivalent circuit of the system of Example 23. The Elementary Mathematical Medal 29 To find the initial conditions, we solve the network of Figure 211. We have the terminal condition Vai0/0 pu %=105/% pu P= 08 pu To find the angle of ¥,, we write, since resistance is zero, = 08 4 (VV,/x)sin 6» = (1.05/0.30) sin sin Bq = 08/35 = 02286 Oo = 132° The current is found from V; T= (V- Vy/Z = (105 /13.21" ~ 10 /0y/j03 = (1.022 + j0 240 — 1000)/j0.3 = 0.800 ~ j0.074 = 0,803 /—5.29° Then the internal machine voltage is E [8 = 105 /13.21" + (0.803 /~-$.29°)(0,2 /90") = 1022 + {0 240 + 0.0148 + jo 160 = 1.037 + j0400 = 1.111 /21,09" pu ZI+ Vor is E = 1111 is a constant that will be unchanged during the transient, and the angel is 5) = 2109° = 0 367 rad. We also may write P= (L111 x 10)/0.50|siné = 22228in6 Then the swing equation is given by 2H do 2H ds op _ p. we dt or a6 377 Ge ig O8 — 22228ind) rad/s* From this simple example we observe that the resulting swing equation is non- linear and will be difficult to solve except by numerical methods. We now extend the example to consider a fault on the system Example 24 Develop the equation of motion of the system of Figure 2 11 where a fault is applied at the sending end (node 4) of the transmission line For simplicity we will consider a three-phase fault that presents a balanced impedance of j0 1 to neutral, The network now is as shown in Figure 2 12, where admittances are used for convenience Solution By ¥-A transformation we compute Vir = —j[.333 x $)/18 333] = -j0.909 and since Yip = —Pu2, then Fig = j0 909. The electrical power output of the machine is now (0.909 x L1L1)sind ~ 1010sind 30 Chapter 2 saa @ 430 ® “hoo Fig 212. Faulted network for Example 2 4 in terms of admittances From Example 2 3 the equation of motion of the rotor is 2 “ = 37.708 — 1010sin5) rad/s* At the start of the transient sin, = 0 36, and the initia/ rotor acceleration is given by ae 37 7[0 8 ~ (1.010 x 0 368)] = 1645 rad/s? Now let us assume that after some time the circuit breaker at the sending end of the faulted line clears the fault by opening that line, The network now will have a series reactance of j0.70 pu, and the new network (with fault cleared) will have @ new value of transfer admittance, Fy, = j1 429 pu. The new swing equation will be 4°3 © 477(08 — 1.587sin5) rad/s? a Example 25 Calculate the angle 6 as a function of time for the system of Examples 2.3 and 24 Assume that the fault is cleared in nine cycles (0.15 s) Solution The equations for 6 were obtained in Example 24 for the faulted network and for the system with the fault cleared These equations are nonlinear; therefore, time solu- tions will be obtained by numerical methods A partial survey of these methods is given in Appendix B To illustrate the procedure used in numerical integration, the modified Euler method is used in this example This method is outlined in Appendix B First, the swing equation is replaced by the two first-order differential equations: be ot) op, = ea /2H)P, ~ P.O) (2.42) The time domain is divided into increments called Ar_ With the values of 5 and w and their derivatives known at some time f, an estimate is made of the values of these vari- ables at the end of an interval of time Ar, ie, at time 1+ At These are called the predicted values of the variables and are based only on the values of 5(t), #(/), and their derivatives. From the calculated values of 5(f + A‘) and w(s + 4), values of the derivatives at ¢+ Af are calculated A corsected value of 5(¢ + Az) and w(t + Ad) is obtained using the mean derivative over the interval. The process can be repeated until a desited precision is achieved At the end of this repeated prediction and correction a final value of 6(¢ + At) and w(¢ + Ar) is obtained. The process is then repeated for the next interval The procedure is outlined in detail in Chapter 10 of [8]. From Example 24 the initial value of 4 is sin~'0 368, and the equation The Elementary Mathematical Model 31 oO oa 08 68 1O 121% Time, Fig 213 Anglestime curve for Example? $ for w is given by © = 37.7(0800 - 1.010sind) 0 51< 015 = 37.7(0.800 — 1587sind) 6 2 015 The results of the numerical integration of the system equations, performed with the aid of a digital computer, are shown in Figure 213. The time solution is carried out for two successive peaks of the angle @ The first peak of 48 2° is reached at ¢ = 0.38 s, after which 6 is decreased until it reaches a minimum value of about 13 2° at ¢ = 082 s, and the oscillation of the rotor angle 6 continues For the system under study and for the given impact, synchronism is not lost (since the angle 5 does not increase indefinitely) and the synchronous machine is stable 2.8 Equal Area Criterion Consider the swing equation for a machine connected to an infinite bus derived previously in the form, 2H dé 2H d’s pps P, 2.43 on di? pe C8) where P, is the accelerating power From (2.43) P oy oo on (2.44) de” 2H" 2 Chapter 2 Multiplying each side by 2(d8/dt), (2.45) (2.46) (247) Integrating both sides, (2.48) or s = (3 [ ra)" (2.49) Equation (2 49) gives the relative speed of the machine with respect to a reference frame moving at constant speed (by the definition of the angle 6). For stability this speed must be zero when the acceleration is either zero or is opposing the rotor motion Thus for a rotor that is accelerating, the condition of stability is that a value Bom exists such that P,(Smux) © 0, and San f Pdi = 0 (2.50) If the accelerating power is plotted as a function of 8, equation (2.50) can be inter preted as the area under that curve between 6p and das. This is shown in Fig- Poh Pro") ree o Fig 214 Equal area criteria: (a) for stability for a stable system, (b) lor an unstable system The Elementary Mathematical Model 33 ure 2.14(a) where the net area under the P, versus 6 curve adds to zero at the angle Snax Since the two areas A, and A, are equal and opposite Also at daa the accelerating power, and hence the rotor acceleration, is negative, Therefore, the system is stable and Syay is the maximum rotor angle reached during the swing If the accelerating power reverses sign before the two areas 4, and 4, are equal, synchronism is lost This situation is shown in Figure 2.14(b), The area A, is smaller than 4), and as 6 increases beyond the value where P, reverses sign again, the arca Ay is added to A, The limit of stability occurs when the angle 4,,, is such that PByyx) = 0 and the areas Ay and A, are equal. For this case dyn, coincides with the angle 6, on the power-angle curve with the fault cleared such that P = P, and o> x/2 ‘Note that the accelerating power need not be plotted as a function of 6. We can ob- tain the same information if the electrical and mechanical powers are plotted as a func- tion of 6. The former is the power-angle curve discussed in Section 27, and in many studies P, isaconstant The accelerating power curve could have discontinuities due to switching of the network, initiation of faults, and the like 2.81 Critical cleoring ongle Fora system of one machine connected to an infinite bus and for a given fault and switching arrangement, the critical clearing angle is that switching angle {os which the system is at the edge of instability (we will also show that this applies to any two- machine system). The maximum angle 6,,., corresponds to the angle 4,, on the fault- cleared power-angle curve. Conditions for critical clearing are now obtained (see {1] and [2)) Let 7 © peak of the prefault power-angle curve ratio of the peak of the power-angle curve of the faulted network to Py ratio of the peak of the power-angle curve of the network with the fault cleared to Py sin” P,/Py < «/2 iy, = Sit! Bh fry Pye > #/2 Then for A, = A, and for critical clearing, = cos LCs — FIC Pa f Pa Gm = 89) + 726088q — 11 008 By] (2.51) Note that the corresponding clearing time must be obtained from a time solution of the swing equation cd if 282 Application toa one-machine system The equal area criterion is applied to the power network of Examples 2 4-25, and the results are shown in Figure 2.15. The stable system of Examples 2 4-25 is illus- trated in Figure 2.15. The angle at ¢ = 0 is 21.09° and is indicated by the intersection of P, with the prefault curve. The clearing angle 6, is obtained from the time solu- tion (see Figure 2 13) and is about 316° The conditions for 4, = 4, correspond to Bax & 48° This corresponds to the maximum angle obtained in the time solution shown in Figure 2 13 To illustrate the critical clearing angle, a more severe fault is used with the same system and switching arrangement A three-phase fault is applied to the same bus with zero impedance, The faulted power-angle curve has zero amplitude The prefault and 34 Chapter 2 Fig. 2 1S Application of the equal area criterion to a stable system postiault networks arc the same as before. For this system n=0 by = 21.09" ry = 1587/2222 = 0714 ob, = 149.73" ‘Calculation of the critical clearing angle, using (2 51), gives a, = cos'0.26848 = 74.43" This situation is illustrated in Figure 2.16 %, oO 8 120 130 190 Fig 216 Application of the equal area criterion to a critically cleared system The Elementary Mathematical Model 35 2.83 Equal area terion for a two-machine system It can be shown that the equal area criterion applies to any two-machine system since a two-machine system can be reduced to an equivalent system of one machine connected to an infinite bus (see Problem 2.10) We can show that the expression for the equal area criterion in this case is given by [° . Hy t20 (2.52) whered, = 5, — 6, In the special case where the resistance is neglected, (2 52) becomes. ba Padi, = 0 0 820 whete Hy = HyH,/(H, + Hy) 2.9 Classical Model of a Multimachine System The same assumptions used for a system of one machine connected to an infinite bus are often assumed valid for a multimachine system 1 Mechanical power input is constant 2. Damping or asynchronous power is negligible 3. Constant-voltage-behind-transient-reactance model for the synchronous machines is valid 4 The mechanical rotor angle of a machine coincides with the angle of the voltage behind the transient reactance 5. Loads are represented by passive impedances This model is useful for stability analysis but is limited to the study of transients for only the “first swing” or for periods on the order of one second Assumption 2 is improved upon somewhat by assuming a linear damping character- istic. A damping torque (or power) Dw is frequently added to the inertial torque (or power) in the swing equation The damping coefficient D includes the various damping, torque components, both mechanical and electrical. Values of the damping coefficient usually used in stability studies are in the range of 1-3 pu [9, 10, 11, 12) This repre- sents turbine damping, generator electrical damping, and the damping effect of electrical loads. However, much larger damping coefficients, up to 25 pu, are reported in the literature due to generator damping alone [7, 13] Assumption 5, suggesting foad representation by a constant impedance, is made for convenience in many classical studies Loads have their own dynamic behavior, which is usually not precisely known and varies from constant impedance to constant MVA. This is a subject of considerable speculation, the major point of agreement being that constant impedance is an inadequate representation Load representation can have a marked effect on stability results The electrical network obtained for an n-machine system is as shown in Figure 2.17. Node 0 is the reference node (neutral). Nodes 1, 2,...., are the internal machine buses, or the buses to which the voltages behind transient reactances are applied Passive impedances connect the various nodes and connect the nodes to the reference at load buses. As in the one-machine system, the initial values of Ej, E,.....E, are de- termined from the pretransient conditions. Thus a load-flow study for pretransient 36 Chapter 2 in -mochine sytem generators Fig 217 Representation of a multimachine system (classical model) conditions is needed The magnitudes £,, i = 1 ing the transient in classical stability studies The passive electrical network described above has m nodes with active sources. The admittance matrix of the n-port network, looking into the network from the terminals of the generators, is defined by . nate held constant dur- T-YE (253) where ¥ has the diagonal elements ¥;, and the off-diagonal elements ¥, By definition, ¥, ~ ¥,/% = driving point admittance for node i = G+ By ¥,, (4) = negative of the transfer admittance between nodes i and j = Gy + jBe 54) The power into the network at node i, which is the electrical power output of machine i, is given by B = Se EJF P, = ERG, + DEE, ¥,c0s (0, — 848) i= 1,2, yn ray = HG, + 2) F6IB,sin(8, — §) + Gjcost, ~ §)) t= 1,2, nm (2.55) The Elementary Mathematical Model 37 The equations of motion are then given by 2H des Da, = P| EG, + D> E,E,Y, cos(8, ~ 8, + 5) Oe di a 4, a (2.56) It should be noted that prior to the disturbance (tf = O°) Pan = Pao Paio = EP Gyo + Dy EE; Yin 008 (B50 ~ 89 + 8p) 37) The subscript 0 is used to indicate the pretransient conditions This applies to all machine rotor angles and also to the network parameters, since the network changes due to switching during the fault The set of equations (2.56) is a set of n-coupled nonlinear second-order differential equations. These can be written in the form X = (OX, Xo.2) (2.58) where x is a vector of dimension (2n x 1), 1 = [04,502,625 We Be] 259) and fis a set of nonlinear functions of the elements of the state vector x 2.10 Classical Stability Study of a Nine-bus System The classical model of a synchronous machine may be used to study the stability of a power system for a period of time during which the system dynamic response is de- pendent largely on the stored kinetic energy in the rotating masses For many power systems this time is on the ordes of one second or less The classical model is the simplest model used in studies of power system dynamics and requires a minimum. amount of data; hence, such studies can be conducted in a relatively short time and at minimum cost. Furthermore, these studies can provide useful information For ex- ample, they may be used as preliminary studies to identify problem ateas that require further study with more detailed modeling Thus a large number of cases for which the system exhibits a definitely stable dynamic response to the disturbances under study are eliminated from further consideration A classical study will be presented here on a small nine-bus power system that has three generators and three loads. A one-line impedance diagram for the system is given in Figure 2.18 The prefault normal load-flow solution is given in Figure 219 Gen- erator data for the three machines are given in Table 2.1. This system, while small, is large enough to be nontrivial and thus permits the illustration of a number of stability concepts and results. 2.10.1 Data preparation In the performance of a transient stability study, the following data are needed 1 A load-fiow study of the pretransient network to determine the mechanical power P, Of the generators and to calculate the values of £,/0y for all the generators The equivalent impedances of the loads are obtained from the load bus data 38 Chopter 2 why 230 lead € 00625 0.0119 +j0.1008 ‘0.010 + 0.065] 0.092 40.161 Fig 218 Nine-bus system impedance diagram: all impedances are in py om a 100-MVA base rokv 13.8kv Fig 219. Nine-bus system load-flow diagram showing prefault conditions; all Rows are in MW and MVAR, The Elementary Mathematical Model 39 Table2.1. Generator Data Generator 1 : 2 3 Rated MVA 2478 192.0 128.0 kv 165 i80 138 Power factor 10 085 0.85 Type hydro steam steam Speed 180 e/enin 3600 fin 3600 r/min Xy 0.1460 0.8958 13125 xy 0.0608 0.1198, 0.1813 yo 0.0969 0.8645 12578 4 0.0969 0.1969 0.25 x; (leakage) 0.0336 0.0521 0.0742 Tio 896 6.00 539 reo 0 0535 0.600 Slored energy atrated speed 2364 MWes, 640 MW: s 301 MW-s. fin. pu on a 100-MVA base, All time constants are ins (Several quantities fined in this Book ‘quantities are derived and justified in Chapter 4 bbut are given here to provide complete data for the sample system } 2. System data as follows: a. The inertia constant H and direct axis transient reactance xj for all generators b_ Transmission network impedances for the initial network conditions and the sub- sequent switchings such as fault clearing and breaker reclosings, 3. The type and location of disturbance, time of switchings, and the maximum time for which a solution is to be obtained 2.10.2 Preliminary calculations To prepare the system data for a stability study, the following preliminary calcula- tions are made: 1. All system data are converted to a common base; a system base of 100 MVA is frequently used 2. The loads are converted to equivalent impedances or admittances. The needed data for this step are obtained from the load-flow study. Thus if a certain load bus has a voltage V,, power P,, reactive power Q,, and current 7, flowing into a Joad ad- mittance ¥, = G, + jB,, then Pr + iQ = Wilf = VUPNG, ~ iB) = ViCG, - iB.) The equivalent shunt admittance at that bus is given by Ye = P/VE ~ (Qi /¥E) (2.60) 3. The internal voltages of the generators F, /djy are calculated from the load-flow data These internal angles may be computed from the pretransient terminal voltages V/a. as follows Let the terminal voltage be used temporarily as a reference, as shown in Figure 220. If we define J = 1, + jl, then from the relation P + jQ = VT* we have 1, + jh = (P ~ jQ)/V. But since £/3 = V+ jx}T, we compute Eb! = (V + Oxi/V) + i(Pxi/V) @61) The initial generator angle 4) is then obtained by adding the pretsansient voltage 40 Chopter 2 Fig 220 Generator representation for computing 8y angle @ to 8’, or a= 0 be (2.62) The ¥ matrix for each network condition is calculated. The following steps are usually needed: a. The equivalent load impedances (or admittances) are connected between the load ‘buses and the reference node; additional nodes are provided for the internal gen- erator voltages (nodes 1,2, in Figure 2.17) and the appropriate values of xjare connected between these nodes and the generator terminal nodes. Also, simulation of the fault impedance is added as required, and the admittance matrix is determined for each switching condition b All impedance elements are converted to admittance: c. Elements of the ¥ matrix are identified as follows: Y,, is the sum of all the ad- mittances connected to node i, and Y,, is the negative of the admittance between node i and node j Finally, we eliminate all the nodes except for the internal generator nodes and ob- tain the ¥ matrix for the reduced network The reduction can be achieved by matrix operation if we recall that all the nodes have zero injection currents except for the in- ternal generator nodes This property is used to obtain the network reduction as shown below - Let rs yv (2.63) where I= 0 Now the matrices Y and V ate partitioned accordingly to get Y,,| |V, Yee} [Ve (2.64) where the subscript is used to denote generator nodes and the subscript r is used for the remaining nodes. Thus for the network in Figure 2.17, V,, has the dimension (n x 1) and Y, has the dimension (r x 1) Expanding (2 64), = Y,V,+¥nV, 0=Y,V, + Y,V, The Elementary Mathematical Model 4 from which we eliminate V, to find DA = Wn = Ye¥5!¥ Vy . (2.65) The matrix (¥,, — ¥,,¥;'Yj,) is the desired reduced matrix ¥. It has the dimensions (nw x_n) Where n is the number of the generators The network reduction illustrated by (2 63)-(2 65) is a convenient analytical tech- nique that can be used only when the loads are treated as constant impedances. If the loads are not considered to be constant impedances, the identity of the load buses must, be retained Network reduction can be applied only to those nodes that have zero in- jection current Example 26 The technique of solving a classical transient stability problem is illustrated by con- ducting a study of the nine-bus system, the data for which is given in Figures 2.18 and 219 and Table 2.1 The disturbance initiating the transient is a three-phase fault ‘occurring neat bus 7 at the end of line 5-7. The fault is cleared in five cycles (0 083 s) by opening line 5-7 For the purpose of this study the generators are to be represented by the classical model and the loads by constant impedances. The damping torques are neglected The system base is 100 MVA Make all the preliminary calculations needed for a transient stability study so that all coefficients in (2.56) are known Solution The objective of the study is to obtain time solutions for the rotor angles of the gen- erators after the transient is introduced. These time solutions are called “swing curves * In the classical model the angles of the generator internal voltages behind transient reactances are assumed to correspond to the rotor angles. Therefore, mathematically, ‘we are to obtain a solution for the set of equations (2.56) The initial conditions, de- noted by adding the subscript 0, are given by Gj. = 0 and 59 obtained from (2.57) Preliminary calculations (following the steps outlined in Section 2 10 2) are: 1 The system base is chosen to be 100 MVA. All impedance data are given to this base 2. The equivalent shunt admittances for the loads are given in pu as Toad A: rs = 1.2610 ~ j0 5044 load B: Fz4 = 08777 — j0 2926 load C: Fre = 0.9690 — j0 3391 3. The generator internal voltages and their initial angles are given in pu by Ey/by = 1.0566/2.2717" Ey [by = 10502 /19.7315" Es {839 = 1.0170 /13.1752° 4, The ¥ matrix is obtained as outlined in Section 2,102, step 4. For convenience bus numbers I, 2, and 3 are used to denote the generator internal buses rather than the generator low-voltage terminal buses Values for the generator xj are added to the reactance of the generator transformers For example, for generator 2 bus 2 will be the internal bus for the voltage behind transient reactance; the reactance between Az Chapter 2 Table 2.2, Prefauit Network Impedance Admittance Bus 20 R : 3 Generators* No.1 1a 0 o1ss =o -8 4459 No.2 27 0 0.18230 =3 4855, No 3 39 0 023990 ~4 1684 Transmission lines 45 09100 © 0.0850 1.3652 11 604t 4-6 09170 =—-0.0920 1.9422 =10.5107 57 0.0320 0.1610 11876 ~5.9751 69 00390 ©0170 1.2820 5 5882 78 oooss = 0.0720 Lel7i ~13 6980 89 ooits 0. L008 Vissi —9 7843 Shunt admittancest Load 4 50 12610 0.2634 Load B 6-0 08777 0.0346 Load ¢ 80 0.9690 -0 1601 4-0 0.1670 70 0.2275 9.0 0.2835 ‘For each generator the transformer reactance js added to the generator x3 ‘{The line shunt susceptances are added to the loads. bus 2 and bus 7 is the sum of the generator and transformer reactances (0.1198 + 0.0625), The prefault network admittances including the load equivalents are given in Table 2.2, and the corresponding ¥ matrix is given in Table 23 The ¥ matrix for the faulted network and for the network with the fault cleared are similarly obtained The results are shown in Tables 2 4 and 2.5 respectively. 5 Elimination of the network nodes other than the generator internal nodes by net- work reduction as outlined in step 5 is done by digital computer. The resulting re- duced ¥ matrices are shown in Table 2.6 for the prefault network, the faulted net- work, and the network with the fault cleared respectively We now have the values of the constant voltages behind transient reactances for all three generators and the reduced Y matrix for each network. Thus all coefficients of (2.56) are available Example 27 For the system and the transient of Example 2.6 calculate the rotor angles versus time, The fault is cleared in five cycles by opening line 5-7 of Figure 2.18. Plot the angles 5, 8;, and 6, and their difference versus time—- = has Solution The problem is to solve the set of equations (2 56) form = 3 and D = 0. All the coefficients for the faulted network and the network with the fault cleared have been determined in Example 2.6 Since the set (2.56) is nonlinear, the desired time solutions for 6,, 6;, and 8, are obtained by numerical integration, A brief survey of numerical integration of differential equations is given in Appendix B (For hand calculations see [I] for an excellent discussion of a numerical integration method of the swing equa- PLSTIT— leer Z ——CvaL'6! + ISSTT- ceas's! + O78cI- y89r et 6 pa T= pceect ~ ongo'ent + 12191— 8 OB6p'EIE + LIST ~ © ssarst Z zees'st+ ozget~ - 610rP LOLs ol! + zene I 9 © suse it z9ea'e + e99e'1- s OLSON + Zep T— THOTTIT + es9e1— LeBE"Oe! = vLOE"E ostrsl b veorel w891 pt £ ssapst ssapst~ z ostrst oserat- 6 a t 9 5 Y € z PON y POrEaID UNA YUM HOMO JO MUTE A “SZAIORL, evav'el + 1st zaas'st + o7se I= 991 6 woro'ect — cipe't 8 L cwas'st + oesz'1- seersif- 610r'y + cere 9 oces'cit— geus'e zoe t— s LOISOIT + zepGI-~ HO TIF + Z69EI- LEGE OE - PLOT E osrrst pa9r'et sort € soar'st= z osrirat osrysf- 6 8 t 9 5 + € z 1 PON OMPN PoNEA JOXUIEW A “PT IMEL, evav'el + 1sst— ‘caas'st + occ 1 a9 ef 6 + iscri= vees'ert— Repent + 1L19"1- 8 OBEN L191 — 11¢6 Rel LHOR'Z Isto st + 908 ssarst L caas'sf + zee 1- Seergn= story + rene 9 1seo'st + 9080-1 s cous oral op 99r'ef peorrt— € ssapst ssavet— z osreat osrrat— 6 8 L 9 s ’ € t 1 SON MOMON TNEIG FO MEW A “ELE 44 Chapter 2 Table2.6, Reduced ¥ Matrices Type ot ode network Nod ' > 5 Prefault 0846 — [2988 0287 + j1S13. 0210 + j1.226 0138 + j0726 0.389 — jt 9530199 4 j1. 229 0.191 + 51.079 0.199 + 1.229 0.273 ~ [2.342 i 2 0.287 4 j1513 0.420 — j2724 0.213 + j1 088 3 0.210 + j1226 ©0213 + 1088 0.277 ~ 52.368 Faulted L 0657 - j3.816 0.000 + j0.000 0.070 + 0.631 2 0.000 + j0.000 0.000 ~ j5.486 9.000 + j0.000 3 0.070 + j0631 0.000 + jo000 0 174 — j2 796 Fault cleared 1 1181 ~ j2229 0.138 4 j0726 0191 + j1.079 2 3 tion, Also see Chapter 10 of [8] for a more detailed discussion of several numerical schemes for solving the swing equation) The so-called transient stability digital com- puter programs available at many computer centers include subroutines for solving non- linear differential equations Discussion of these programs is beyond the scope of this book ‘Numerical integration of the swing equations for the three-generator, nine-bus sys- tem is made by digital computer for 2.0 s of simulated real time, Figure 2.21 shows the rotor angles of the three machines A plot of 8 = 8; — 6, and by, = 5; — 00) STITT Thesis cleared in Seyclen o-oo 100m evel Ld ° O8 10) vs a0 Tine, + Fig 221 Plot of 6,62, and 43 versus time The Elementory Mathematical Model 45 "Ee oO a0 em Oa Lo ° OF To Teo Tine, = Fig 222 Plot ofS differences versus time in Figure 2 22 where we can see that the system is stable The maximum angle difference is about 85° This is the value of 6, at ¢ = 0.43 s Note that the solution is carried out for two “swings” to show that the second swing is not greater than the first for cither 8, or 8, To determine whether the system is stable or unstable for the par- ticular transient under study, it is sufficient to carry out the time solution for one swing only If the rotor angles (or the angle differences) reach maximum values and then decrease, the system is stable, If any of the angle differences increase indefinitely, the system is unstable because at least one machine will lose synchronism 2.11 Shortcomings of the Classical Model System stability depends on the characteristics of all the components of the power system. This includes the response characteristics of the control equipment on the turbogenerators, on the dynamic characteristics of the loads, on the supplementary control equipment installed, and on the type and settings of protective equipment used The machine dynamic response to any impact in the system is oscillatory In the past the sizes of the power systems involved were such that the period of these oscilla- tions was not much greater than one second. Furthermore, the equipment used for excitation controls was relatively slow and simple. Thus the classical model was adequate Today large system interconnéctions with the greater system inertias and relatively weaker ties result in longer periods of oscillations during transients. Generator control systems, particularly modern excitation systems, are extremely fast It is therefore 46 Chapter 2 questionable whether the effect of the control equipment can be neglected during these longer periods. Indeed there have been recorded transients caused by large impacts, resulting in loss of synchronism after the system machines had undergone several oscil- lations, Another aspect is the dynamic instability problem, where growing oscillations have occurred on tie lines connecting different power pools or systems As this situation has developed, it has also become increasingly important to ensure the security of the bulk power supply. This has made many engineers realize it is time to reexamine the assumptions made in stability studies This view is well stated by Ray and Ship- ley (14): ‘We have reached « time when it is appropriate that we appraise the state of the Art of Dynamic Stability Analysis. In conjunction with this we must: 1. Expand our knowledge of the characteristic time response of our system loads to changes in voltage and frequency—develop new dynamic models of system loads 2. Re-examine old concepts and develop new ideas on changes in system networks to improve system stability 3. Update our knowledge of the response characteristics of the various components of energy systems and their controls (boilers, reactors, turbine governors, generator regulators, field excitation, ete.) 4, Reformulate our analytical techniques to adequately simulate the time variation of all of the foregoing factors in system response and accurately determine dynamic system response. Let us now make a critical appraisal of some of the assumptions made in the classi- cal model: L. Transient stability is deciced in the first swing A latge system having many machines will have numerous natural frequencies of oscillations The capacities of most of the tie lines are comparatively small, with the result that some of these frequencies are quite low (frequencies of periods in the order of 5-6 s are not uncommon) It is quite possible that the worst swing may occur at an instant in time when the peaks of some of these nodes coincide It is therefore necessary in many cases to study the transient for a period longer than one second 2. Constant generator main field-winding flux linkage This assumption is suspect on two. counts, the longer period that must now be considered and the speed of many modern voltage regulators The longer period, which may be comparable to the field-winding time constant, means that the change in the main field-winding flux may be appreciable and should be accounted for so that a correct representation of the system voltage is realized, Furthermore, the voltage regulator tesponse could have a significant effect on the field-winding flux. We conclude from this discussion that the constant voltage behind transient reactance could be very inaccurate 3. Neglecting the damping powers A large system will have relatively weak ties In the spring-mass analogy used above, this is a rather poorly damped system, It is important to account for the various components of the system damping to obtain a correct model that will accurately predict its dynamic performance, especially in loss of generation studies (8} 4 Constant mechanical power If periods on the order of a few seconds or greater are of interest, it is unrealistic to assume that the mechanical power will not change The turbine-governor characteristics, and perhaps boiler characteristics should be in- cluded in the analysis 5. Representing loads by constant passive impedance. Let us illustrate in a qualitative manner the effect of such representation, Consider a bus having a voltage V to which a load P; + jQ, is connected. Let the load be represented by the static ad- The Elementary Mathematical Model 47 v 6 & Fig. 223 A load represented by passive admittance mittances G, = P,/¥? and B, = Q,/V? as shown in Figure 2.23. During a tran- sient the voltage magnitude V and the frequency will change. In the model used in Figure 2.17 the change in voltage is reflected in the power and reactive power of the load, while the change in the bus frequency is not reflected at all in the load power In other words, this model assumes P, « V2, 0, V2, and that both are frequency independent. This assumption is often on the pessimistic side (There are situa- tions, however, where this assumption can lead to optimistic results. This discussion is intended to illustrate the errors implied.) To illustrate this, let us assume that the transient has been initiated by a fault in the transmission network. Initially, a fault causes a reduction of the output power of most of the synchronous generators, Some excess generation results, causing the machines to accelerate, and the area fre- quency tends to increase At the same time, a transmission network fault usually causes a reduction of the bus voltages near the fault location In the passive im- pedance model the load power decreases considerably (since P, « V2), and the in- crease in frequency does not cause an increase in load power. In real systems the decrease in power is not likely to be proportional to V? but rather less than this An increase in system frequency will result in an increase in the load power Thus the model used gives a load power lower than expected during the fault and higher than normal after fault removal From the foregoing discussion we conclude that the classical model is inadequate for system representation beyond the first swing Since the first swing is largely an inertial response to a given accelerating torque, the classical model does provide useful information as to system response during this brief period. 2.12 Block Diagram of One Machine Block diagrams are useful for helping the control engineer visualize a problem. We will be considering the control system for synchronous generators and will do so by analyzing each control function in turn It may be helpful to present 2 general block diagram of the entire system without worrying about mathematical details as to what makes up the various blocks. Then as we proceed to analyze each system, we can fill in the blocks with the appropriate equations or transfer functions. Such a block dia- gram is shown in Figure 2.24 [15] The basic equation of the dynamic system of Figure 2.24 is (2.18); i¢, 1 = P,- P.= P, pu (2.66) where § has been replaced by o and J has been replaced by a time constant 1,, the numerical value of which depends on the rotating inertia and the system of units Three separate control systems are associated with the generator of Figure 2.24 The first is the excitation system that controls the terminal voltage Note that the excitation system also plays an important role in the machine’s mechanical oscillations, since it affects the electrical power, P,. The second control system is the speed control or governor that monitors the shaft speed and controls the mechanical power P,, 48 Chopter 2 Peciiation 5 ve ly, oa En Tagine -— Se Sate flows | Boiler ca [ET [bq Reece see, Fig 224 Block diagram of a synchronous generator control system Finally, in an interconnected system there is a master controller for each system, This sends a unit dispatch signal (UDS) to each generator and adjusts this signal to meet the load demand or the scheduled tie-line power It is designed to be quite slow so that it is usually not involved in a consideration of mechanical dynamics of the shaft. Thus in most of our work we can consider the speed reference or governor speed changer (GSC) position to be @ constant. In an isolated system the speed reference is the desited system speed and is set mechanically in the governor mechanism, as will be shown later In addition to the three control systems, three transfer functions are of vital im- portance The first of these is the generator transfer function, The generator equations are nonlinear and the transfer function is a linearized approximation of the behavior of the generator terminal voltage V; near a quiescent operating point or equilibrium state The load equations are also nonlinear and refiect changes in the electrical output quanti- ties due to changes in terminal voltage V, Finally, the energy source equations are a desctiption of the boiler and steam turbine or of the penstock and hydraulic turbine behavior as the governor output calls for changes in the energy input These equations are very nonlinear and have several long time constants To visualize the stability problem in terms of Figure 2.24, we recognize immediately that the shaft speed « must be accurately controlled since this machine must operate at precisely the same frequency as all others in the system Ii a sudden change in w occurs, we have two ways of providing controlled responses to this change. One is through the governor that controls the mechanical power P,, but does so through some rather long time constants. A second controlled response acts through the excitation system to con- trol the electrical power P, Time delays are involved here too, but they are smaller than those in the governor loop. Hence much effort has been- devoted to refinements in excitation control Problems 21 Analyze (2 1) dimensionally using a mass, length, time system and specify the units of each quantity (see Kimbark (1) 22. A rotating shaft has zero retarding torque T, = 0 and is supplied a constant full load accelerating torque; ie, Tm = Tr, Let 7, be the accelerating time constant, i.c., the time required to accelerate the machine from rest to rated speed we Solve the swing equation to find +, in terms of the moment of inettia J, we, and Tp, ‘Then show that 7, can also be related to Hf, the pu inertia constant. 23 24 25 26 27 28 29 210 2 212 213 214 The Elementary Mathematical Model 49 Solve the swing equation to find the time to reach full load speed ws starting from any initial speed wp with constant gocelerating torque as in Problem 22 Relate this time to rand the slip at speed wp Write the equation of motion of the shaft for the following systems: (2) An electric generator driven by a de motor, where in the region of interest the generator torque is proportional to the shaft angle and the motor torque decreases linearly with increased speed (b) Anclectric motor driving a fan, where in the region of interest the torques are given by Troer = 4-06 Thy = cB? where a, b, and ¢ are constants State any necessary assumptions Will this system have a steady-state operating point? Is the system linear? In (24) assume that 7 is in Nom, 6 is in elec deg, and./is in bm f@ What factor must bbe used to make the units consistent? In (2 T)assume that Pisin W and M in J-s/rad_ What are the units of 6? A.500-MVA two-pole machine is to operate in parallel with othet U.S machines. Compute the regulation R of this machine. What are the units of R? A 60-MVA two-pole generator and a 600-MVA four-pole generator are to operate in paral- lel with other U S. systems and are to share in system governing Compute the pu constant K that must be used with these machines in their governor simulations if the system base is 100 MVA. Repeat problem 2 8 if the constant K is to be computed in MKS units rather than pu In computer simulations it is common to see regulation expressed in two different ways as described below: (8) Py = Peo = SPR where Py, = mechanical power in pu on Sp Pao = initial mechanical power in pti on S.y if = system base frequency in Hz R,, ~ steady-state speed regulation in pu on a system base = R,S,n/Sp $= generator slip = (an — w)/2eHz (b) Pp ~ Pao = Kyo pu where P,, = turbine power in pu on S,p Pap = initial turbine power in pu'on S,y Ai = Sof RyoeSow Aw ~ speed deviation, rad/s Verify the expressions in (a) and (b) ‘A synchronous machine having inertia constant H = 40 MJ/MVA is initially operated in steady state against an infinite bus with angular displacement of 30 elec deg and delivering LO pu power. Find the natural frequency of oscillation for this machine, assuming small perturbations from the operating point A solid-rotor synchronous generator is driven by an unregulated turbine with a torque speed characteristic similar to that of Figure 2.3(a). The machine has the same characteris- tics and operating conditions as given in Problem 2 14 and is connected to an infinite bus. Find the natural frequency of oscillation and the damping coefficient, assuming small perturbations from the operating point. ‘Suppose that (2.33) is written for a salient pole machine to include reluctance torque term; ic., let P= Pysind + ksin23. For this condition find the expression for P, and for the synchronizing power coefficient Derive an expression similat to that of (2 7) for an interconnection of two finite machines that have inertia constants M, and M, and angles 4, and ,. Show that the equations for such a ease are exactly equivalent to that of a single finite machine of inertia M = M,M,/(M, + M,) and angle 6,, = 3, — dy connected to an infinite bus 50 Chapter 2 25 Derive linearized expressions (similar to Example 22) that describe an interconnection ofthree finite machines with inertia constants My, M;, and My and angles , 63. and 5) Is there a simple expression for the natural frequency of oscillation in this case? Desig- nate synchronizing power between machines 1 and 2.a8 Py», el¢ 2.16 The system shown in Figure P2 16 has two finite synchronous machines, each represented by a constant voltage behind reactance and connected by a pure reactance. The reactance x includes the transmission line and the machine reactances. Write the swing equation for each machine, and show that this system can be reduced to an equivalent one machine against an infinite bus. Give the inertia constant for the equivalent machine, the mechani cal input power, and the amplitude of its power-angle curve, The inertia constants of the two machines are 17, and Hy s Fig P216 217. Thesystem shown in Figure P2.17 comprises four synchronous machines. Machines 4 and B are 60 Hz, while machines C and D are $0 Hz; B and C are a motor-generator set (frequency changer) Write the equations of motion for thissystem Assume that the trans- mission networks are reactive Fig P217 218 The system shown in Figure P2.18 has two generators and three nodes Generator and transmission line data are given below. The result of a load-flow study is also given. A. three-phase fault occurs near node ? and is cleared in 0 1 s by removing line 5 Fig P218 (a) Perlorm all preliminary calculations for a stability study. Convert the system to a com- mon 100-MVA base, convert the loads to equivalent passive impedances, and calculate the generator internal voltages and initial angles. (b) Calculate the ¥ matrices for prefault, faulted, and postfault conditions (c) Obtain (numerically) time solutions for the internal general angles and determine if the system is stable. The Elementary Mathematical Model 51 Generator Data (in pu to generator MVA base) Generator y Xt 4 Rating ‘number (pu (ow (MW -s/MVA) (MYA) 1 028 008 5 50 3 0.25 0.07 4 120 “FX, = generator transformer reactance Transmission Line Data (resistance neglected) Line number: 3 4 5 6 X puto 008 = 006008013 100-MVA base Load-Flow Data bs Volaze Load Generator no. Magnitude pu Angle Mw MVAR Mw MVAR 1 1030 00 00 00 300 231 2 1.018 10 500 200 00 00 3 1.020 05 80.0 40.0 100.0 378 2.19 Reduce the system in Problem 2 18 to an equivalent one machine connected to an infinite 220 221 222 bus, Write the swing equation for the faulted network and for the network after the fault iscleared Apply the equal arca criterion to the fault discussed in Problem 2.18 What is the critical clearing angle? Repeat the calculations of Example 2 4, but with the following changes in the system of Figure 2 11 (a) Use a fault impedance of Z, = 0.01 + j0 pu. This is more typical of the arcing re- sistance commonly found in'a fault (b) Study the damping effect of adding a resistance to the transmission lines of R, in each fine where R, = 01 and 04 pu. To measure the damping, prepare an analog computer simulation for the system Implementation will require computation of ¥ 1, Yias the initial conditions, and the potentiometer settings. (©) Devise a method of introducing additional damping on the analog computer by adding a term K4é in the swing equation, Estimate the value of Ky by assuming that a slip of 2.5% gives a damping torque of 50% of full load torque (@) Make a parametric study of changes in the analog simulation for various values of FL For example, let H = 25,50,755. Repeat Problem 220 but with transmission line impedance for each line of R, + j0 8, where R, = 02, 05,08 pu Repeat the analog simulation and determine the critical clearing time to the nearest cycle This will sequire a means of systematically changing from the fault condition to the postfault (one ine open) condition after a measured time lapse. This can be accomplished by logical control on some analog computers or by care~ ful hand switching where logical control is not available Let V, = 095 Repeat Problem 2 21 using a line impedance of 0.2 + j08 Consider the effect of adding a “tocal” unity power factor load R,» at bus 3 fos the following conditions: Casel: Pip = 0.4 pu Po + jn = 0.4 + j020 pu Case 2: Pry = value to give the same generated power as Case 1 P. +i. = 0+ 10 pu Cased: Pro = 1.2 pu Po+ iQ, = -04% j02 pu (2) Compute the values of Ry and £ and find the initial condition for 5 for each case 52 Chapter 2 (b) Compute the values of F,, and Pip for the prefault, faulted, and postiault condition if the fault impedance is'Z, = 001 + j0. Usethe computer for this, writing the ad- imittance matrices by inspection and reducing to find the two-port admittances (¢) Compute the analog computer settings for the simulation (@) Perform the analog computer simulation and plot the following variables: Ty T.. Tyy 66a) 5,02 ~ 8 Also, make a phase-plane plot of w, versus 6 Compare these re- sillts with similar plots with no local load present (©) Use the computer simulation to determine the critical clearing angle References 1 Kimbark, EW. Power System Stabiliy, Vol 1 Wiley, New York, 1948 2. Stevenson, WD Elentonts of Power System Analysis. nd e6. McGraw-Hill, New York, 1962 3 Federal Power Commission. National Power Survey PC 2 USGPO Washington, DC , 1964 4 Lokay HE. and Thoits. P.O. Effects of future turbine-generator characteristics on transient sta- bility. LEEE Frans PAS-90:2427-31, 1971 5 ATE Subcommittee on Interconnection and Stability Factors First report of power system stabil ity Blecir Eng 56:261-82 1937 6 Venikov, VA. Transient Phewonena in Electrical Power Systems Pergamon Press. Macmillan Now York, 1964 1 Crary $B. Power Sy stem Stability Vol 2 Wiley. New York, 1947 & Suge GW. and El-abiad A H Computer Methods in Power System Analysis McGraw-Hill New York, 1968 % Concordia C. Effect of steam turbine reheat on speed-governor performance ASME } Eng Power 81-2016, 1959 10. Kirchmayer, LK. Economic Control of Interconnected Systems Wiley, New York. 1959 11 Young.C C. and Webler, RM. new stability program for predicting the dynamic performance ofeiectric power systems Proc Am Power Conf. 29:1126-38, 1967 12. Byerly RT” Sherman, DE. and Shortley, PB Stability. program data preparation manual Westinghouse Plectrie Corp Rept. 70-736. 1970 (Rev Dee 1971) 13 Concordia, C Synchronous machine damping and synchronizing torques AVE Trans 10:731-31 19st 14 Ray, ) J and Shipley, RB Dynamic system performance Paper 66 CP 709-PWR, presented at the IEEE Winter Power Meeting. New York. 1968 15 Anderson, P_M- and Nanakorn $ An analysis and comparison of certain low-order boiler models 184 Trans 14:17-23.1975 chapter 3 System Response to Small Disturbances 3.1. Introduction is chapter reviews the behavior of an electric power system when subjected to small disturbances. It is assumed the system under study has been perturbed from a steady-state condition that prevailed prior to the application of the disturbance. This small disturbance may be temporary or permanent. If the system is stable, we would expect that for a temporary disturbance the system would return to its initial state, while a permanent disturbance would cause the system to acquire a new operating slate after a transient period. In either case synchronism should not be lost. Under normal operating conditions a power system is subjected to small disturbances at ran- dom. Itis important that synchronism not be lost under these conditions Thus system. behavior is a measure of dynamic stability as the system adjusts to small perturbations We now define what is meant by a small disturbance. The criterion is simply that the perturbed system can be linearized about a quiescent operating state An example of this linearization procedure was given in Section 2.5, While the power-angle rela- tionship for a synchronous machine connected to an infinite bus obeys a sine law (2.33), it was shown that for small perturbations the change in power is approximately propor tional to the change in angle (2 35). Typical examples of small disturbances are a small change in the scheduled generation of one machine, which results in a small change in its rotor angle 6, or a small load added to the network (say 1/100 of system capacity cor less) In general, the response of a power system to impacts is oscillatory. If the oscil- lations are damped, so that after sufficient time has elapsed the deviation or the change in the state of the system due to the small impact is small (or less than some prescribed finite amount), the system is stable. If on the other hand the oscillations grow in magni- tude or are sustained indefinitely, the system is unstable For a linear system, modern linear systems theory provides a means of evaluation of its dynamic response once a good mathematical model is developed. The mathe- matical models for the various components of a power network will be developed in greater detail in later chapters Here a brief account is given of the various phenomena experienced in a power system subjected to small impacts, with emphasis on the qualita- tive description of the system behavior 53 54 Chapter 3 3.2. Types of Problems Studied The method of small changes, sometimes called the perturbation method [1,2, 3], is very useful in studying two types of problems: system response to small impacts and the distribution of impacts 3.2.1 System response to small impacts If the power system is perturbed, it will acquire a new operating state. If the perturbation is small, the new operating state will not be appreciably different from the initial one. In other words, the state variables or the system parameters will usually not change appreciably Thus the operation is in the neighborhood of a certain quiescent state xo. In this limited range of operation 2 nonlinear system can be de- scribed mathematically by linearized equations This is advantageous, since linear sys- tems are more convenient to work with. This procedure is particularly useful if the system contains control elements The method of analysis used to linearize the differential equations describing the system behavior is to assume small changes in system quantities such as 64, Us. P, (change in angle, voitage, and power respectively). Equations for these variables are found by making a Taylor series expansion about xo and neglecting higher order terms [4,56] The behavior or the motion of these changes is then examined. In ex- amining the dynamie performance of the system, it is important to ascertain not only that growing oscillations do not result during normal operations but also that the oscil- latory response to small impacts is well damped. If the stability of the system is being investigated, it is often convenient to assume that the disturbances causing the changes disappear The motion of the system is then free Stability is then assured if the system returns to its original state Such behavior can be determined in a linear system by examining the characteristic equation of the system. If the mathematical description of the system is in state-space form, ic., if the system is described by a set of first-order differential equations, ke Ax + Bu Gh the free response of the system can be determined {rom the eigenvalues of the A matrix 3.2.2. Distribution of power impacts When a power impact occurs at some bus in the network, an unbalance between the power input to the system and the power output takes place, resulting in a transient. When this transient subsides and a steady-state condition is reached, the power impact, is “shared” by the various synchronous machines according to their steady-state char- acteristics, which are determined by the steady-state droop characteristics of the various governors [5,7] During the transient period, however, the power impact is shared by the machines according to different criteria, If these criteria differ appreciably among groups of machines, each impact is followed by oscillatory power swings among groups of machines to reflect the transition from the initial sharing of the impact to the final adjustment reached at steady state Under normal operating conditions « power system is subjected to numerous ran- dom power impacts from sudden application or removal of loads As explained above, each impact will be followed by power swings among groups of machines that respond to the impact differently at different times. These power swings appear as power oscil- System Response to Small Disturbances 55 lations on the tie lines connecting these groups of machines This gives rise to the term “tie-tine osciltations.” In large interconnected power systems tie-line oscillations can become objectionable if their magnitude reaches a significant fraction of the tie-line loading, since they are superimposed upon the normal flow of power in the line. Furthermore, conditions may exist in which these oscillations grow in amplitude, causing instability, This problem is similar to that discussed in Section 3.21. It can be analyzed if an adequate math- ematical model of the various components of the system is developed and the’ dynamic response of this model is examined If we are interested in seeking an approximate answer for the magnitude of the tie-line oscillations, however, such an answer can be reached by a qualitative discussion of the distribution of power impacts. Such a discus- sion is offered here 3.3. The Unregulated Synchronous Machine. We start with the simplest model possible, ic. the constant-voltage-behind-tran- sient-reactance model. The equation of motion of a synchronous machine connected to an infinite bus and the electrical power output are given by (2 18) and (2.41) re- spectively or 2H #6 Py - Pe on a P= Pe + Pysin( ~ ¥) G2) Letting 6 = &) + 55, P, = Pay + Pa, Py = Paoand using the relationship sin(S ~ 7} = sin (by ~ ¥ + 84) © sin - ¥) + cos(o~ 5, G3) the linearized version of (3.2) becomes 2H Pos, ps. 20 G4) oe dP where ap, =f) Lp, - 3 . =, Pygc08 (65 ~ 1) 65) The system described by (3.4) is marginally stable (ie, oscillatory) for P, > 0. Its response is oscillatory with the frequency of oscillation obtained from the roots of the characteristic equation (2H /wg)s? + P, = 0, which has the roots 3 = ajV Pwo. /2H G.6) If the electrical torque is assumed to have a component proportional to the speed change, a damping term is added to (3 4) and the new characteristic equation becomes (2H/wn)? + (D/wn)s + P, = 0 By where D is the damping power coefficient in pu The roots of (3 7) are given by D . wp [/D\V 8HP)'? 1-7 Bl2)- al G8) 56 Chopter 3 Usually (Dwr)? < 8HP,/on, and the roots are complex; ie, the response is oscilla- tory with an angular frequency of oscillation essentially the same as that given by (3.6) The system described by (3.7) is stable for P, > 0 and for D > 0 Lf either one of these quantities is negative, the system is unstable Venikov [4] reports that a situation may occur where the machine described by (3.4) can be unstable under light load conditions if the network is such that # < 7. This would be the case where there is appreciable series resistance (see [4], Sec. 3.2) From Chapter 2 we know that the synchronizing power coefficient P, is negative if the spontancous change in the angle 6 is negative A negative value of P, leads to unstable operation 3.3.1 Demagneti The model of constant main field-winding flux linkage neglects some important effects, among them the demagnetizing influence of a change in the rotor angle 6. To account for this effect, another model of the synchronous machine is used It is not our concern in this introductory discussion to develop the model or even discuss it in detail, as this will be accomplished in Chapter 6. Rather, we will state the assump- tions made in such a model and give some of the pertinent results applicable to this discussion These results ate found in de Mello and Concordia [8] and are based on a model previously used by Heffron and Phillips [9]. To account for the field con- ditions, equations for the direct and quadrature axis quantities are derived (see Chap- ter 4) Major simplifications are then made by neglecting saturation, stator resistance, and the damper windings The transformer voltage terms in the stator voltage equa- tions are considered negligible compared to the speed voltage terms. Linearized rela- tions are then obtained between small changes in the electrical power P.., the rotor angle 4, the field-winding voltage v,,, and the voltage proportional to the main field-winding flux £5 For a machine connected to an infinite bus through a transmission network, the following s domain relations are obtained, Pry = Kiba + ES B9 Ks KsKy So 14 Kyrhs ing effect of armature reaction Ea T + Kyrgs * . G19 where K; is the change in electrical power for a change in rotor angle with constant flux linkage in the direct axis, Kz is the change in electrical power for a change in the direct axis flux linkages with constant rotor angle, re is the direct axis open cir- cuit time constant of the machine, K, is an impedance factor, and K, is the demag- netizing effect of a change in the rotor angle (at steady state). Mathematically, we write Ky = Pra /Baley a0 = Pes/Eblag-0 K, = final value of unit step v, response = lim £,()h,-0 ' - Kew ~ ¢ lim Eis) 311 Kes orane ew en The constants K,, K,, and K, depend on the parameters of the machine, the exter- nal network, and the initial conditions Note that X; is similar to the synchronizing power coefficient P, used in the simpler machine model of constant voltage behind ‘System Response to Small Disturbances 37 Fig. 31 Primitive linearized block diagram representation of generator model transient reactance Equations (3.9) and (3.10), with the initial equation (3 2), may be represented by the incremental block diagram of Figure 3 1 KKK K, Pa = (x, Kok: +o, 3.12 ( 1 ee 8 7 TF Bros ee) For the case where ¥,, = 0, KKK Pax = (k, - 22KoKs_) 5, («- ee) 1s) where we can clearly identify both the synchronizing and the demagnetizing compo- nents, Substituting in the linearized swing equation (3.4), we obtain the new characteristic equation, (with D = 0) [at ef — pM, «0 Wp T + Kyti05, or we have the third-order system a4! 2, Kin | eet tee + Ses +e K;KsK,) = 0 G14) Note that all the constants (3 11) are usually positive Thus from Routh's criterion (10] this system is stable if K, - K,K,K, > Oand K,KjK, > 0. The first of the-above criteria states that the synchronizing power coefficient K, must be greater than the demagnetizing component of clectsical power. The second criterion is satisfied if the constamts K;, Ky, and K, ate positive Venikov [4] points out that if the transmission network has an appreciable series capacitive reactance, it is possible that instability may occur This would happen because the impedance factor producing the constant K, would become negative. 3.3.2. Effect of small changes of speed In the linearized version of (3 2) we are interested in terms involving changes of power due to changes of the angle and its derivative. The change in power due to 58 Chapter 3 44 was discussed above and was found to include synchronizing power component and a demagnetizing component due the change in £4 with 6, The change in speed, ©, = dé,/dt, causes a change in both electrical and mechanical power. In this case the new differential equation becomes wth. aed) on -(2] a+ Z 4) (3.15) a0 oo ho op a? Go de 36 As in (3.7) the change in electrical power due to small changes in speed is in the form of Pes = (Dioq)os 3.16) From Section 2.3 the change in mechanical power due to small changes in speed is also Jinear Pas = 9Pq/Bed]ogs Gin where dP, /dal,, can be obtained (rom a relation such as the one given in Figure 23 If a transient droop or regulation & is assumed, we may write in pu to the machine base Pax = —(1/Ryes/eq) pu (3.18) which is the equation of an ideal speed droop governor The system block diagram with speed regulation added is shown in Figure 3 2 Fig 32. Block diagram representation of the linearized model with speed regulation added istic equation of the system now becomes 2H ify, 1 Bak Ke a aba +(s TH Res The charact (3.19) (3.20) System Response to Small Disturbances 59 Again Routh's criterion may be applied to determine the conditions for stability This is left as an exercise (see Problem 3 2) 3.4. Modes of Oscillation of an Unregulated Multimachine System The electrical power output of machine / in an n-machine system is, Pay = ERG, +, E/E, Y, cos (4, ~ 8) . tn = B2G, + D> E(B, sin i, + Gy cos 8,) G21 im E, = constant voltage behind transient reactance for machine j ¥, = G, + {By is a diagonal clement of the network short circuit admittance matrix ¥ G,, + jB, is an off-diagonal element of the network short circuit admit- tance matrix Y Using the incremental model so that 4, = dj + djs, We compute sin 6, = sin jp C0 jy + COS dp Sin dyy & SiN Bip + ys COS Djo 608 8 & 608 do ~ Sys SI Oyo Finally. for P.j., Pas = dy E:E,(By 008 by) ~ Gy Sin bg) By (322) m For a given initial condition sin 4,9 and cos 4,9 are known, and the term in parentheses in (3.22) isaconstant. Thus we write Pas = Dy Pa Sia G23) where oP, Py A Ta] £,6,(B, 008 0 ~ Gy sin dp) G24) ‘bao is the change in the electrical power of machine i due to a change in the angle between machines ¢ and j, with all other angles held constant. Its units are W/rad or pu power/rad, Itis a synchronizing power coefficient between nodes i and j and is identical to the coefficient discussed in Section 2.5.2 for one machine connected to an infinite bus We also note that since (3.21) applies to any number of nodes where the voltages are known, the linearized equations (3.22) and (3.23) can be derived for a given machine in terms of the voltages at those nodes and their angles Thus the concept of the syn- chronizing power coefficients can be extended to mean “the change in the electrical power of a given machine due to the change in the angle between its internal EMF and 60 Chapter 3 any bus, with all other bus angles held constant.” (An implied assumption is that the voltage at the remote bus is also held constant) This expanded definition of the syn- chronizing power coefficient will be used in Section 3.6. Using the inertial model of the synchronous machines, we get the set of linearized differential equations, . ® 2H dbs YY £.6(B, cos by — GySinBye)iy, =O = 1,2, 2 G25) a dP I fe or 2H, 2, : ede te Patan 0 F=1,2,. 0 (3.26) i fer The set (3.26) is not a set of n-independent second-order equations, since 28, = 0 Thus (3.26) comprises a set of (n — 1)-independent equations. From (3.26) for machine i, Po. ar Bde dys 0 = 1,2. sm B27 Subtracting the nth equation from the ith equation, we compute Pip Poy, Ge Get ay Padia ~ BE ep, Pri dus = 0 G28) ia Hi jet Equation (3.28) can be put in the form, - £ bast EY Py ays ~ BE rates <0 P2122. 2-1 G29 fon Since 85a = Sina ~ Sina (3.30) (3.29) can be further modified as Cas YE ovin O f= 12 wot 3H where the coefficients a, depend on the machine inertias and synchronizing power co- efficients Equation (3 $1) represents a set of n — 1 linear second-order differential equations or a set of 2(n ~ 1) first-order differential equations We will use the latter formula- tion to examine the free response of this system Let xy, x2) Xp be the angles dpa binas--- 14 1yua Tespectively, and let Xp Nanna be the time derivatives of these angles, The system equations are of the form System Response to Small Disturbances 61 (333) where U = the identity matrix X, = then — 1 vector of the angle changes 5,,, X, = them — | vector of the speed changes dé, /dt To obtain the free response of the system, we examine the eigenvalues of the charac- teristic matrix [11,12]. This is obtained from the characteristic equation derived from equating the determinant of the matrix to zero, as follows: -M | U det} ----4 = detM A [-.U where ) is the eigenvalue. Since the matrix —AU is nonsingular, we compute the de- terminant of M as iM] = {-AU| |(-AU) - A(-av)-'U | = (GRIT AU = Gayl = [RU - Al (335) (See Lefschetz [12], p 133) The system described by |MJ = 0, or |MU - A/ = 0, has 2(n — 1) imaginary roots, which occur in n — 1 complex conjugate pairs Thus the system has — 1 frequencies of oscillations (3.34) Example 31 Find the modes of oscillation of a three-machine system The machines are unregu- lated and classical model representation is used Solution For an unregulated three-machine system, the system equations are given by 2H, Poa, de + Prdaa + Prrdiss = 0 2h; dbs EGE + Paibna + Pandas = 0 2H d Pb, 228 + Pada + Pndng = 0 oe dt 2183 a baa, 62 Chopter 3 Multiplying the above three equations by «/2H, and subtracting the third equation from the first two, we get (noting that 6, = —4,) Pbisy , On cn On Oy Phas 4 am Pasbing + ca a Po ‘se + if Finda = 0 Pose Oe ps, a ina = 0 7 oh, “21d2a + an ; Prvdny + ih Pas ne 70} 8na If we eliminate 4124 by noting that 5;,, + 3:3, + dyia = 0, the following two equations are obtained: p « fon 5 Gir Pay + SE Pay + SR ‘em +( ae 2H; on on Pay — SE Pya\ina = 0 2H, 2H, bn Pony. [ow Wp ) a Set bsg Pet ~ app Patt Ouse Pai +E Pay + SE Pen)Oa = 0 ae * (2H, 2H, ene * oh auton 2H, or 2 2 Fis s aydng + dng = 0 SB + casting + arabe = 0 The state-space representation of the above system is boa 0 O | EF Of bn bag 0 OO 1] i bas Sus) | ay —an | 0 0 \Jwne fra] [-an an | 0 0} [wry To obtain the cigenvalues of this system, the characteristic equation is given by Now by using (3.35), tay ay det -0 ay Mba, OF + a)? + ap) ~ aren = 0 M+ (Qu + An) + (Aye = O24) = 0 = (/2M—Cn + am) # Mey + @P ~ Aen — a2)!" System Response to Small Disturbances 63 Examining the coefficients a,, we can see that both values of X* are negative real quantities. Let these given values beA = + j8, A= jy The free response will be in the form 6, = C, cos (Bt + $1) + C2 008 (yt + #2), where C,, Co, d), and @ ate constants Example 32 Consider the three-machine, nine-bus system of Example 2.6, operating initially in the steady state with system conditions given by Figure 2.18 (load flow) and the com- puted initial values given in Example 26 for E,/%g,i = 1,2, 3. A small 10-MW load (about 3% of the total system load of 315 MW) is suddenly added at bus 8 by adding a three-phase fault to the bus through 10.0 pu impedance ‘The system base is 100 MV. Assume that the system load after 1 = 0 is constant and consists of the original load plus the 10 pu shunt resistance at bus 8 Compute the frequencies of oscillation that will result from this small disturbance Then compare these computed frequencies against those actually observed in a digital computer solution Assume there are no governors active on any of the three turbines ‘Observe the system response for about two seconds. Solution First we compute the frequencies of oscillation From (3 24) Puy = ViVi(By 008 Bip — Gy Sit Bq) SV, VB, COS Byo From Example 2 6 we find the data needed to compute P,, with the results shown in Table3 1 Table 3.1. Synchronizing Power Coefficients of the Network of Example 2.6 ¥ “i % By yo Py 12 10566 1.0502, 1513 174598 16015 23 1.0502 10170 1 O88 6 5563 11544 31 4.0170 1.0566 1.226 10.9035, 1.2936 Note that the 4/9 are the values of the relative rotor angles at ¢ = 0. Since these are rotor angles, they will not change at the time of impact, so these are also the correct, values for r = 0*, Thisis also true of angles at load buses to which appreciable inertia is connected For loads that are essentially constant impedance, however, the voltage angle will exhibit a step change. Also from Example 26 we know H, = 23 64, 6.40, and 301 for = 1, 2, 3 respec- tively, Thus we can compute the values of a, from Example 31 as follows: ay = (wR /2)(Par/Hy + Pys/Hy + Posi /Hs) = 104.096 ez = (@p/2)(Paya/Hy — Pryn/Hy) = 59.524 @, = (we /2)(Po)/Hy — Py /H)) = 68.241 = (we /2)(Pari/Ha + Pars/Ha + Pen/ Hi) 119.065 Then (1/2). (ea: + a) & Wei Fan? = Maxi — 24) —(1/2)[-223.61 & 49860 83 — 33328 85] - —47.409 or — 175.752 64 Chapter 3 Now we can compute the frequencies and periods shown in Table 3.2 Table32. Frequencies of Oscillation of a Nine-Bus System Quantity Eigenvalue 1 —_Bigenvalue2 AN +)6 885, #113257 orad/s 6.885 13.257 fiz 1096 2.410 Ts 092 474 Thus two frequencies, about 1.1 Hz and 2.1 Hz, should be observed in the inter- machine oscillations of the system. This can be approximately verified by an actual so- lution of the system by digital computer. The results of such a solution are shown in Figure 3 3, where absolute angles are given in Figure 3 3(a) and angle differences rela- tive to 8, are given in Figure 3.3(b). As might be expected, neither of the computed frequencies is clearly observed since the response is a combination of the two frequen- cies A rough measurement of the peak-to-peak periods in Figure 3.3(b) gives periods in the neighborhood of 0.7 s. Methods have been devised (3, 11] by which a system such as the one in Example 3.2 can be transformed to a new frame of reference called the Jordan canonical form In Jordan form the different frequencies of oscillation are clearly separated. In the form of equations normally used, the variables 6, and 8,, (or other angle differences) contain oo 0m 1000 150 2000 2.500 Fig. 3 3 Unregulated response of the nine-bus system to a sudden load application at bus 8: (a) absolute angles, (b) angles relative to by System Response to Small Disturbances 65 “harmonic” terms generally involving all fundamental frequencies of oscillation, Hence we have difficulty observing these frequencies in measured physical variables. Example 3.3 Transform the system of Example 32 into the Jordan canonical form and show that in this form the system frequencies of oscillation are clearly distinguishable Solution The system equations for the three-machine problem are given by % : x & t Xa & t xy : ! . BY] [ean an! or & = Ax, where x is defined by %] Pons - x ay | | ons af Lens and the a coeflicients are computed in Example 3 2. ‘We now compute the eigenvectors of A, using any method [1, 3, 11] and call these vectors Ey, E;, Es, and E,. We then use these eigenvectors to define a matrix E. 0 06266 | —j0 06266 j 0.14523 | ~0.14523 j0.07543 | —j007543 | 0.13831 | 0.13831 083069 | 0.83069) 1.00000 | 1.00000 1.00000 | 1.00000 | -0.95234 | 0.95234 E = (EF, F, E, Bi) = where the numerical values are found by 2 suitable computer library routine We now define the transformation x = Ey tocomputeX = E¥ = Ax = AEy or ¥ = EX ABy = DywhereD = diag(ds.d:,As, Md Performing the indicated numerical work, we compute ~{3.9245 —j3.7008 02659 0.2792 3.5245 j3,7008 0.2659 0.2792 Ets —j1.9221 j1.5967 0.2792 0.2319 1.9221 —j1.5967 0.2792 -0.2319 -j13 2571 0.0 00 0.0 00 = j13.2571 (00 00 D ~ EAE = 00 00 = -j6. 8854 0.0 On) 00 0.0 56 8854 66 Chapter 3 Substituting into = Dy, we can compute the uncoupled solution y= Ge 7=1,234 where C; depends on the initial conditions. This method of computing the distinct frequencies of oscillation is quite general and may be applied to systems of any size. For very large systems this may not be practical, however, since the cigenvector computation may be too costly Finally, we note that the simple model used here assumes that no damping exists, In physical systems damping is usually present; therefore, the oscillatory response given above is usually damped, The magnitude of the damping, however, is such that the fre~ quencies of oscillation given by the above equations are not appreciably affected 3.5 Regulated Synchronous Machine In this section we examine the effect of voltage and speed control equipment on the dynamic performance of the synchtonous machine. Again we are interested in the free response of the system, We will consider two simple cases of regulation: a simple voltage regulator with one time lag and a simple governor with one time lag 3.5.1 Voltage regulator with one time lag Referring to Figure 2 24, we note that a change in the field voltage Us, is pro- duced by changes in either Vase or ¥;, If we assume that Vareg = Oand the transducer has no time lags, v-, depends only upon V,, modified by the transfer function of the excitation system Analysis of such a system is discussed in Chapter 7. To simplify the analysis, a rather simple model of the voltage regulator and excitation system is as- sumed This gives the following s domain relation between the change in the exciter voltage Ug, and the change in the synchronous machine terminal voltage Vj: Ure = — EK + 29 Ma 3.36) where K, = regulator gain 7, = segulator time constant To examine the effect of the voltage regulator on the system response, we return to the model discussed in Section 3.3 for a machine connected to an infinite bus through a transmission network. These relations are given in (3.9) and (3 10) To use (3 36), a relation between V, 6,,and Ej is needed Such a relation is de- veloped in reference (8) and is in the form Vig = Ks5y + Keka (337) where Ky = Via/Sale, — change in terminal voltage with change in rotor angle for constant E’ = change in terminal voltage with change in £’ for constant 6 Ke = Val Es The system block diagram with voltage regulation added is shown in Figure 3 4 From (3 36) and (3 37) Urs = —1K/L + 7 5)(Ks8s + KeFa) (3.38) Substituting in (3 10), we compute (Ksbg + Kes] - bers System Response to Smal Disturbances 67 Fig 3.4 System block diagram with voltage regulation or, rearranging, Ey =|- (3.39) From (3.39) and (3.9) s+ P= [K, - BE th o,/L, 1 ) 1+ KKK, eds(te ct) 4 tee ¢ Kyra) * Kitt. UN (3.40) Substituting in the s domain swing equation and rearranging, we obtain the follow- ing characteristic equation: tee(he a 5 u ( + he Sik) 2H \t.” Kyo tho + ey =0 G41) 4 oe [eit 2H | Ryton, Ker, Equation (3 41) is of the form Sb as + as’ tas +a = 0 (3.42) Analysis of this fourth-order system for stability is left as an exercise (see Problem 3.3) 68 ‘Chapter 3 3.5.2 Governor with one time lag Referring to Figure 2 24, we note that a change in the speed w ot in the load or speed reference (governor speed changer (GSC)] produces a change in the mechanical torque J; The amount of change in T,, depends upon the speed droop and upon the transfer functions of the governor and the energy source For the model under consideration it is assumed that GSC, = 0 and that the com- bined effect of the turbine and speed governor systems are such that the change in the mechanical power is in the form Pra, 1K (C+ 7,5) 43) where K, Ts gain constant = 1/R governor time constant ‘The system block diagram with governor regulation is shown in Figure 3.5 Then the linearized swing equation in the s domain is in the form (2H /og)s?5a(s) = =[Ky/( + 745)]85a(5) — Peals) G44) The order of this equation will depend upon the expression used for P..(s)._ If we as- sume the simplest mode! possible, Pz4(s) = P,6a(3), the elraracteristic equation of the system is given by (QHfon)s? + (Ke f(t + Ty)Is + Pe = 0 3.45) or SiQHr,/on) + 9°(2H/or) + (Ky + Pity)s + P= 0 (3.46) The system is now of third order. Applying Routh’s criterion, the system is stable if K, > Oand P, > 0 Ifanother model is used for P,.(3), such 28 the model given by (39) and (3 10), the system becomes of fourth order, as shown in Figure 3.5 Its dynamic response will change Information on stability can be obtained front the roots of the charactesistic ‘equation or from examining the eigenvalues of its characteristic matrix Ke he T Fig 35 Block diagram of a system with governor speed regulation. System Response to Small Disturbances oo osc, -0 37 Fig 36 Block diagram of a system with a governor and voltage regulator If both speed governor and voltage regulation are added simultaneously, as is usually the case, the system becomes fifth order, as shown in Figure 3.6 3.6. Distribution of Power Impacts In this section we consider the effect of the sudden application of a small load P; at some point in the network. (See also [7,5]) To simplify the analysis, we also as- sume that the load has a negligible reactive component, Since the sudden change in load P,, creates an unbalance between generation and load, an oscillatory transient results before the system settles to a new steady-state condition This kind of impact is continuously occurting during normal operation of power systems The oscillatory transient is in fact a “spectrum” of oscillations resulting from the random change in loads. These oscillations are reflected in power flow in the tie lines. Thus the scheduled tic-line flows will have “random” power oscillations superimposed upon them Our concern here is to make an estimate of the magnitude of these power oscillations Note that the estimates made by the methods outlined below are only approximate, yet they are quite instructive We formulate the problem mathematically using the network configuration of Fig- ure 3.7 and the equations of Sections 2.9 and 34 Referring to the (w+ 1)-port net- work in Figure 37, the power into node i is obtained from (3.21) by adding node k E.G, +L E,E(B, sind, + G,,c086,,) + E,¥e(Bu sind, + Gy C08 54) jak For the case of nearly zero conductance P,& >) EE,B,sind, + E,ViBy sind, (3.47) a 70 Chapter 3 Ty (ot Deport netwerk Fy ho \ \ } ye Fig. 37 Network with power impact at node k and the power into node & (the load bus) is, Py y VE, Byjsin 34, (348) jk Here we assume that the power network has a very high X/R ratio such that the conductances are negligible The machines are represented. by the classical model of constant voltage behind transient reactance. We also assume that the network has been reduced to the internal machine nodes (nodes 1,2,.. ., of Figure 2 17) and the node k, where the impact Pr . is applied The immediate effect (assuming the network response to be fast) of the application ‘of Pzq is that the angle of bus k is changed while the magnitude of its voltage %; is unchanged, os V; /dyq becomes V; /dg0 + dgg- Note also that the internal angles of the machine nodes 6), 5;,...,d, do not change instantly because of the rotor inertia, 3.6.1 Linearization ‘The equations for injected power (347) and (3.48) are nonlinear because of the transcendental functions. Since we are concerned only with a small impact P;4, we linearize these equations to find Poe Pot Piso Pr = Pro + Pea and determine only the change variables P,, and Pes The transcendental functions are linearized by the relations sin 5,, = sin (8x0 + dys) © Sin dy0 + (COS4430) 5455 60884; = COS (Bajo + diya) & COS S:;0 ~ (Sindis0)Oaja 3.49) for any k,j Note that the order kf must be carefully observed since 64, = — Bj. Sub- stituting (3 49) into (3.47) and (3 48) and eliminating the initial values, we compute the linear equations Pix = D> (EsE;By C085, bya + (VE: Bu 008840)5x2 = DL. Paydys + PeaBna i at irik Pas © 2 VaE Bry 008 5y0)8usa = Dy Payday (3.50) These equations are valid for any time ¢ following the application of the impact System Response to Small Disturbances 7 3.6.2 Aspecial case: # = 0* The instant immediately following the impact is of interest. In particular, we would like to determine exactly how much of the impact P,. is supplied by each generator Pim 12, At the instant ¢ = * we know that 4, = 0 for all generators because of rotor inertias Thus we can compute (with both j and j indicating generator subscripts) Sig = 0 Ba = bia — Bea = — 400°) Syn = Sea — Bya = Sea (O*) Thus (3.50) becomes Pig(0*) = —Pradsa(O*) — Pra") = z Paes (0") G5) £ Comparing the above two equations at 1 = 0*, we note that at node k Pus(0") = ~ D> Pia(0*) (352) oi This is to be expected since we are assuming a nearly reactive network, We also note that at node i Pi, depends upon By cosdy9. In other words, the higher the transfer susceptance By and the lower the initial angle duo, the greater the share of the im- pact “picked up” by machine i, Note also that P,y = —Prs, 0 the foregoing equa- tions can be written in terms of the load impact as. Pra) = => Pusdts(0") = D> Pal") (3.53) From (3.52) and (3 53) we conclude that 445(0*) = —Pra(0") / X Pa (354) PaO") = (ra/ xy a) Pis(*) te 12 355) fH Itis interesting that at the instant of the load impact (ie, at 1 = 0%, the source of energy supplied by the generators is the energy stored in their magnetic fields and is distributed according to the synchronizing power coefficients between i and k. Note that the generator rotor angles cannot move instantly; hence the energy supplied by the generators cannot come instantly from the energy stored in the rotating masses. This is also evident from the first equation of (3.51); Pi depends upon P,x or By, which depends upon the reactance between generator i and node k Later on when the rotor angles change, the stored energy in the rotating masses becomes important, as shown below Equations (3 52) and (3.55) indicate that the load impact P,4 at a network bus k is immediately shared by the synchronous generators according to their synchronizing power coefficients with respect to the bus k Thus the machines electrically close to the point of impact will pick up the greater share of the load regardless of their size Let us consider next the deceleration of machine i due to the sudden increase in its output power P,, The incremental differential equation governing the motion of machine Zs given by 72 Chapter 3 2H, de Bede t Pa =O Fa 1,2 ym G56) and using (3.55) 2H; doa : . - Gee (-./ x Pu) y=0 f= 12, yn Then if P;4 is constant for all ¢, we compute the acceleration in pu to be 1 dow _ _ Put fF age (Pio fz ra) Obviously, the shaft decelerates for a positive load Pr,. The pu deceleration of ma- chine i, given by (357), is dependent on the synchronizing power coefficient P,. and inertia H,. This deceleration will be constant until the governor action begins Note that after the initial impact the various synchronous machines will be retarded at differ- ent rates, each according to its size H, and its “electrical location” given by Pa 1,2, 0 (3.57) 3.6.3 Average behavior prior to governor action (t = f:) We now estimate the system behavior during the period 0 < 1 < t,, where f is the time at which governor action begins To designate this period simply, we refer to time as f, although there is no specific instant under consideration but a brief time period of no more than a few seconds Looking at the system as a whole, there will be an overall deceleration of the machines during this period To obtain the mean deceleration, let _us define an “inertial center” that has angle § and angular velocity @, where by definition, SL /LA)EGH, FS (U/LH) Lal, (3.58) ‘Summing the set (3 57) for all values of i, we compute 22 g (Hieia) = Pra = —Pral0*) (3.39) da, + /> Hane Pes |e 2H, G60) Equation (3.60) gives the mean acceleration of all the machines in the system, which is defined here as the acceleration of a fictitious inertial center. ‘We now investigate the way in which the impact Pr, will be shared by the various machines Note that while the system as a whole is retarding at the rate given by (3.60), the individual machines are retarding at different rates. Each machine follows an oscillatory motion governed by its swing equation. Synchronizing forces tend to pull them toward the mean system retardation, and after the initial transient decays they will acquire the same retardation as given by (3.60), In other words, when the transient decays, do,,/dé will be the same as di, /de as given by (3.60). Substituting this value of day,/d¢ in (3.56), at f = , > toy Piatti) = (u/% 1) Pra(0*) G61) Thus at the end of a brief transient the various machines will share the increase in load as a function only of their inertia constants. The time 1, is chosen large enough System Response to Small Disturbances 73 so that all the machines will have acquited the mean system retardation, At the same time ¢, is not so large as to allow other effects such as governor action to take place Equation (3.61) implies that the H constants for all the machines are given to a common base. If they are given for each machine on its own base, the correct powers are ob- tained if H is replaced by HS,;/S,», where Sy; is the machine rating and S,y is the chosen system base. Examining (3.56) and (3.61), we note that immediately after the impact Pra(ie., at 1 = 0*) the machines share the impact according to their electrical proximity to the point of the impact as expressed by the synchronizing power coefficients After a brief transient period the same machines share the same impact according to entirely differ- ent criteria, namely, according to their inertias. Example 3.4 Consider the nine-bus, three-machine system of Example 2.6 with a small 10-MW resistive load added to bus 8 as in Example 3.2. Solve the system differential equations and plot P,, and w,, as functions of time. Compare computed results against the- oretical values of Section 3.6 Paelts) = 1.94 Time, + Fig 38 Pig versus ¢ following application of a 10 pu resistive load at bus 8 Solution ‘A nominal 10-MW (0.1 pu) load is added to bus 8 by applying a three-phase fault through a 10 pu resistance, using a library transient stability program. The resulting power oscillations Ps, i = 1, 2, 3, are shown in Figure 3.8 for the system operating without governor action The prefault conditions at the generators are given in Table 3.1 and in Example 2.6 From the prefault load flow of Figure 2 19 we determine that Vio = 1016 and dj = 0.7°. A matrix reduction of the nine-bus system, retaining only nodes 1, 2, 3, and 8, azives the system data shown on Table 3.3 14 Chapter 3 Table3.3. Transfer Admittances and Initial Angles of a Nine-Bus System. a Gy ay Bu 1-8 0.01826 2.51242 15717 2-8 003530 3.55697 19 0315 3-8 — 0.00965 2.61601 12.4752 From (3.24) we compute the synchronizing power coefficients Pa = ViVi(By 00S 5uq — Gusin duo) These values are tabulated in Table 34, Note that the error in neglecting the Gx term is small Synchronizing Power Coefficients P, Pri ewoglecting Ga) (ith Gig term) 18 2.6961 26955 28 3 5878 3.6001 38 2.6392 2.6414. DP sn 8.9231 8.9370 The values of P;.(0*) are computed from (3 55) as Pig(0*) = (ea / x ra) Prs(0") i where P,,(0*) = 10.0 MW nominally, The results of these calculations and the actual values determined from the stability study are shown in Table 3.5. Table 3.5. Initial Power Change at Generators Due to 10-MW Load Added to Bus 8 w @ @ 6) © Pia Pig Pia Pie i (withGa) (Computer study) [NOL] —_—191% oF 3] 1 3.016 28 2749 2.745 2 4.028 36 3.659 3.665 3 2.956 27 2.692 2.690 EPs 10.000 on 9.100 9.100 Note that the actual load pickup is only 9.1 MW instead of the desired 10 MW, This is due in part to the assumption of constant voltage Yj at bus 8 (actually, the voltage drops slightly) and to the assumed linearity of the system. If the computed Pia are scaled down by 0.91, the results agree quite well with values measured from the computer study. These values are also shown on the plot of Figure 3.8 at time ¢ = 0* and are due only to the synchronizing power coefficients of the generators with respect to bus 8 The plots of P,, versus time in Figure 3 8 show the oscillatory nature of the power exchange between generators following the impact ‘These oscillations have frequencies that are combinations of the eigenvalues computed in Example 32 The total, labeled Pin, averages about 9 5 MW System Response to Small Disturbance 75 Tine, 0.1 0,2 0.3 04 05 0.6 07 0.8 0,9 1,0 1,1 12 1,9 14 1S 16 17 U8 19 20 Net Uy Ne Bocoms Fig 39 Speed deviation fotlowing application of a 10 pu resistive load at bus 8 Another point of interest in Figure 3.8 is the computed values of Pis(t,) that depend entirely on the machine inertia, These calculations are made from Pitts) = (Hi/E Hi) Pea = 10H,/23.64 + 6.40 + 301) = 10H,/33.05 71SMW ied 194MW i= 2 = 091MW pad and the results are plotted in Figure 3.8 as dashed lines [1 is fairly obvious that the Pile) oscillate about these values of Py.(%,) It is also apparent that the system has little damping and the oscillations are likely to persist for some time This is partly due to the inherent nature of this particular system, but the same phenomenon would be present to some extent on any system The second plot of interest is the speed deviation or slip as a function of time, shown in Figure 39. The computer program provides speed deviation data in Hz and these units are used in Figure 39. Note the steady deceleration with all units oscillating about the mean or inertial center This is computed as de Pes ON a ISH, 223.64 + 640 + 301) ~1513 x 107 pu/s = —0 570 rad/s? = —0.0908 H2/s The individual machine speed deviations w,, are plotted in Figure 3.9 and show graphi- cally the intermachine oscillations that occur as the system slowly retards in frequency The mean deceleration of about 0.09 Hz/s is plotted in Figure 3 9 as a straight line If the governors were active, the speed deviation would level off after a few seconds to a constant value and the oscillations would eventually decay Since the governors have a drooping characteristic, the speed would then continue at the reduced value as 7 Chopter 3 long as the additional load was present. If the. speed deviation is great, signifying a substantial load increase on the generators, the governors would need to be readjusted to the new load level so that additional prime-mover torque could be provided Example 3.5 Let us examine the effect of the above on the power flow in tie lines Consider a power network composed of two areas connected with @ tic line, as shown in Fig- ure 3.10 The wo areas are of comparable size, say 1000 MW each. They ate con- nected with a tie line having capacity of 100 MW. The tie line is carrying a steady power flow of 80 MW from area I to area 2 as shown in Figure 3.10. Now let a load impact P,, = 10 MW (1% of the capacity of one area) take place at some point in area I, and determine the distribution of this added load immediately after its applica- tion (1 = 0*) and a short time later (1 = 1) after the initial transients have subsided. Because of the proximity of the groups of machines in area I to the point of impact, their synchronizing power coefficients are larger than those of the groups of machines in area 2 If we define 3) Paclaess = Pry EPaelarcaz = Pay then let us assume that P,, = 2Pa Pn 1oMw. Fig 310 Two areas connected with a tie line Solution Since P,, = 2P;2, at the instant of the impact 2/3 of the 10-MW load will be sup- plied by the groups of machines in area 1, while 1/3 or 33 MW will be supplied by the groups of machines in area 2, Thus 33 MW will appear as a reduction in te-line flow In other words, at that instant the tie-line flow becomes 76.7 MW toward area 2 At the end of the initial transient the load power impact P:. will be shared by the machines according to their inertias, Let us assume that the machines of area | are Tie Line Poor Bae MW Tne, ¢ Fig 4.11 Tie-line power osciltations due ta the load impact in area 1 System Response to Small Disturbances 7 predominantly hydro units (with relatively small H), while the units of area 2 are of larger inertia constants such that Milena: = 2£Hileras Where all H's are on a com- mon base. Ihe sharing of the load among the groups of machines will now become 6.7 MW contributed from area 2 and 33 MW from area 1. The tie-line flow will now become 73 3 MW (toward area 2) From the above we can see that in the situation discussed in this example a sudden application of 2 10-MW load caused the tie-line flow to drop almost instantly by 3.3 MW, and after a brief transient by 67 MW The transition from 76.7-MW flow to 73.3-MW flow is oscillatory, and power swings of as much as twice the diflerence between these two values may be encountered. This situation is illustrated in Fig- ure 3.11 The time 1, mentioned above is smaller than the time needed by the various con- trollers to adjust the system generation to match the load and the tie-line flow to meet the scheduled flow Example 36 We now consider a slightly more complex and mote realistic case wherein the area equivalents in Figure 3.10 arc represented by their Thevenin equivatents and the tic- line impedance is given. The system data are given in Figure 3.12 in pu on a 1000-MVA base. The capacity of area 1 is 20,000 MW and that of area 2 is 14,000 MW. The inertia constants of the machines in the two areas are about equal (a) Find the equations of power for P, and P, (b) Find the operating condition when P; = 100 MW. This would correspond ap- proximately to a 100-MW tie-line flow from area I to area 2, (©) Find the synchronizing power coefficients (@) Consider a sudden load addition to area 2, represented by the resistive load Pry at bus 4 If this load is 200 MW (1.43% of the capacity of area 2), find the distri- bution of this load at ¢ = 0 andi = 4 ae ‘Area 1 equivalent Te Fine ‘Aveo 2 equivalent Fig 312 Two arcas connected by a tie line Solution Consider the system as a two-port network between nodes 1 and 2 Then we com- pute Zi. = 0.450 + j1 820 = 1.875 /76.112° pu Far = 1/Zq = 0533 /- 76.112" = 0128 ~ j0.518 pu 0 533 /103.888" Bo = g0 = 0 78 Chapter 3 ~0.128 bn = — 0518 (a) P. = Vigo + Vi ¥a(Gincosd + Brsin dn) — ViGa 0 + 10(-0 128cosd, + 0.518sind,) + 0.128 0.128 + 0.533sin(, ~ 13.796°) = Vigm + WV(Gnc0sby + Bysin’s,) - V3Gn = 0 + 10(-0 128 cos4, - 0 518sind,) + 0.128 = 0.128 — 0.533sin(d, + 13 796°) (b) Given that P; = 01 pu 0.100 = 0.128 + 0.533sin(, — 13796") 6, = 10.784" V,Va(Bi2 608 5329 — Giz sin d,20) = 1,0(0.518 cos 10 784° + 0 128sin 10.784") = 0.533 V, V(Bxs 6088399 ~ Ga, Sin By) . = 1.0[0 518 cos(—~10.784°) + 0.128 sin (—10.784")] = 0.509 (d) Now add the 200-MW load at bus 4; P,, = 200/1000 = 02 pu To complete the problem, we must know the voltage 7, ats = 0° Thus we com- pute (o) 100°) = (Ke - F)/2n = (1 0 £10.784" — 1 0 /0)/1 875 /76.112" = 0.100 /19.280" V0") = EB, + (0 100 + j0.012)f,, = 1.009 + j0.004 = 1.009 /0.252° ba = 0.252 yay = Bi — Su = 10532" yy = B29 — dig = ~ 0.252" From the admittance matrix elements Yn = Die = 1/2 = -O451 + j1 042 Pag = —Jre = 1/2 = -9 858 + j1 183 we compute the synchronizing power coefficients Pau = ViValBysc08 510 — Gu Sin dro) (1. 009)(1,042 cos 10 532° + 0.451 sin 10.532") = L117 Prag = VeVi(Bu C08 5249 — Gre SiN O20) = 1:009[1 183cos(—0.252") + 9.858sin(—0.252")] = 1 150 Then the initial distribution of Py is Pig(0*) = Pre(O.2)/(Pare + Pra) = (0.493)(0.2) = 0.0986 pu Prg(O*) = Pras(02)/(Prin + Pra) = (0.507)(0.2) = 0.1014 pu The power distribution according to inertias is computed as Prat.) = 0.2(20,000H /(20,000H + 14,000H)| = 0.11765 pu Pag(th) = 0.2[14,0004/(20,000H + 14,000/7)} = 0.08235 pu In this example the synchronizing power coefficients P,., and P, are nearly equal, while the inertias of the two areas are not. Thus while the fitial distributions of the System Response to Small Disturbances 79 load P,, are about the same, the distributions at a later time ¢ = f, are such that area | picks up about 59% of the load and area 2 picks up the remaining 41%, In general, the initial distribution of a load impact depends on the point of impact. Problem 3.10 gives another example where the point of impact is in area | (bus 3) In the above discussion many factors have been neglected, eg, the effect of the network transfer conductances, the effect of the reactive component of the load impact, the fast primary controllers such as some of the modern exciters, the load frequency and voltage characteristics, and others Thus the conclusions reached above should be considered qualitative and as rough approximations Yet these conclusions are basically sound and give a good “feel” for what happens to the machines atid to the tie-line fiows under the influence of small routine load changes, If the system is made up of groups of machines separated by tie lines, they share the impacts differently under different conditions. Hence they will oscillate with respect to each other during the transient period following the impact The power flow in the connecting ties will reflect these oscillations The analysis given above could be extended to include governor actions Following an impact the synchronous machines will share the change first according to their synchronizing power coefficients, then after @ brief period according to their inertias The speed change will be sensed by the prime-mover governors, which will act to make the load sharing according to an entirely different criterion, namely, the speed governor droop characteristic The transition from the second to the finat stage is oscillatory (see Rudenberg (7|, Ch 23) The angular frequency of these oscillations can be esti- mated as follows From Section 35.2, neglecting P,,, the change in the mechanical power P,,4 is of the form =U/R ws Ps = Ty oa ae (3.62) where R is the regulation and 7, is the servomotor time constant. The swing equation for machine / becomes, in the s domain, Wises, WR wa _ 9 we T+ ras oe The characteristic equation of the system is given by SU /rads + 1/2HRita = 0 (3.63) from which the natural frequeney of oscillation can be estimated Itis interesting to note the order of magnitude of the frequentcy of oscillation in the two different transients discussed in this section, For a given machine (or a group of machines) the frequency of oscillation in the first transient is the natural frequency with respect to the point of impact These frequencies are determined by finding the eigen- values of the A matrix by solving det (A - XU) = 0, where U is the unit matrix and A is defined by (3.1) For the second transient, which occurs during the transition from sharing according to inertia to sharing according to governor characteristic, the {requeney of oscillation is given by vi = [/2H,Rir.. Usually these two frequencies are appreciably different. 80 31 32 33 a4 35 36 37 38 39 Chapter 3 Problems A synchronous machine is connected to a large system (an infinite bus) through a long. transmission line The direct axis transient reactance xy = 020 pu The infinite bus voltage is 1.0 pu The transmission line impedance is Zige ~ 0.20 + j060 pu The synchronous machine is to be represented by constant voltage behind transient reactance with £' = 110 pu. Calculate the minimum and maximum steady-state load delivered at the infinite bus (for stability). Repeat when there is a local load of unity power factor having Rigg = 8 0 pu Use Routh’s criterion to determine the conditions of stability for the system where the characteristic equation is given by (3 14) Compute the characteristic equation for the system of Figure 31, including the damping term, and determine the conditions for stability using Routh’s criterion. Compare the results with those of Section 33.1 Using 34 as the output variable in Figure 32, use block diagram algebra to reduce the system block diagram to forward and feedback transfer functions Then determine the system stability and possible system behavior patterns by sketching an approximate root- locus diagram Use block diagram algebra to reduce the system described by (345) Then determine the system behavior by sketching the root loci for variations in Ky Give the conditions for stability of the system described by (3 20) ‘A system described by (3 41) has the following data: H = 4, rip = 50,1, = 010, Ki = 48, Ky = 26K; = 026, Ky = 330, Ks = Ol,and Ke = 05° Find the maximum and minimum values of K, for stability. Repeat for Ks = -020 ‘Write the system described by (3 46) in state-space form Apply Rovth’s criterion to (3 46) The equivalent prefault network is given in Table 2.6 for the three-machine system dis cussed in Section 2 10 and for the given operating conditions. The internal voltages and angles of the generators are given in Example 2 6. (a) Obtain the synchronizing power coefficients Pras Puss Pas. and the corresponding Coefficients «, [see (3 31)] for small perturbations about the given operating point (b) Obtain the natural frequencies of oscillation for the angles 6:35 and 5154 Compare with the periods of the nonlinear oscillations of Example 2 7 310 Repeat Example 3 6 with the impact point shifted to area I and let Pz3 = 100 MW as before. 311 Repeat Problem 3.10 for an initial condition of Pra = 300 MW. References 3 10 u 12. Korn,G A, and Korn, 1 M Mathematical Handbook for Sciemists and Engineers McGraw-Hill, New York, 1968 Hayashi, C Nonlinear Oscillations in Physical Systems McGraw-Hill, New York, 1964 Takahashi, Y , Rabins, M. J, and Auslander, D M_ Conzrol and Dynamic Systems Addison-Wesley, Reading, Mass, 1970 Venikov, VA. Transient Phenomena in Electric Power Systems Trans by B Adkins and D Ruten- berg Pergamon Press, New York, 1964 Hote, RA Advanced Studies wy Electrical Power System Design Chapman and Hall, London, 1966. Crary,8 B_ Power System Stabifiry Vols. 1, 2 Wiley, New Yosk, 1945, 1947 Rudenberg, R Transient Performance of Electric Power Systems. Phenomena in Lumped Networks McGraw-Hill, New York, 1950 (MIT Press, Cambridge, Mass 1967) de Mello, F B., and Concordia, C_ Concepts of synchronous machine stability as affected by excita- tion control JEEE Trans. PAS-88;316-29, 1969 Heffron, W.G ,and Phillips, RA” Effect of a modern amplidyne voltage regulator on underercited operation of large turbine generators AEE Trans 71 (Pt 3):692-97. 1952 Routh EJ. Dynamics of a System of Rigid Bodies Macmillan, London, 1877 (Adams Prize Essay ) Ogata. K State-Space Analysis of Control Systems Prentice-Hall, Englewood Clifis, NJ, 1967 Lefschetz, § Stability af Nontinear Controf Systems Academic Press New York, London, 1965 PART Il The Electromagnetic Torque chops The Synchronous Machine 4.1 Introduction In this chapter we develop a mathematical model for a synchronous machine for use in stability computations. State-space formulation of the machine equations is used. Two models are developed, one using the currents as state variables and another using the flux linkages. Simplified models, which are often used for stability studies, are dis- cussed This chapter is not intended to provide an exhaustive treatment of synchronous machine theory, The interested readet should consult one of the many excellent refer- ences on this subject (sce (1}-19)) The synchronous machine under consideration is assumed to have three stator windings, one field winding, and two amortisseur or damper windings These six wind- ings are magnetically coupled The magnetic coupling between the windings is a function of the rotor position. Thus the flux linking each winding is also a function of the rotor position The instantaneous terminal voltage v of any winding is in the form, v=4bnedk (4.1) where is the flux linkage, ris the winding resistance, and / is the current, with posi- tive directions of stator currents flowing out of the generator terminals The notation 42 indicates the stimmation of all appropriate terms with due regard to signs The expressions for the winding voltages are complicated because of the variation of \ with the rotor position 4.2 Park's Transformation A great simplification in the mathematical description of the synchronous machine is obtained if a certain transformation of variables is performed The transformation used is usually called Park's transformation [10,11]. It defines a new set of stator variables such as currents, voltages, or flux linkages in terms of the actual winding vari- ables. The new quantities are obtained from the projection of the actual variables on three axes: one along the direct axis of the rotor field winding, called the direct axis; a second along the neutral axis of the field winding, called the quadrature axis; and the third on a stationary axis. Park's transformation is developed mathematically as fol- jows ' 1. The transformation developed and used in this book is not exactly that used by Pack [10,11] but is sore nearly that suggested by Lewis [12] with certain other features suggested by Concordia (discussion 0 [12}) and Krause and Thomas(13} 83 84 Chopter 4 aonie aos Fig. 4 1 Pictorial representation of a synchronous machine, We define the d axis of the rotor at some instant of time to be at angle @ rad with respect to a fixed reference position, as shown in Figure 4.1. Let the stator phase cur- rents /,, ig, and /, be the currents leaving the generator terminals. If we “project” these currents along the d and g axes of the rotor, we get the relations ia = (2/3){f.8in@ + i,sin(6 — 2/3) + i,sin(@ + 2"/3)) (2/3){é, cos8 + i,cos(@ — 2x/3) + icos(@ + 2x/3)) 42) We note that for convenience the axis of phase a was chosen to be the reference Position, otherwise some angle of displacement between phase a and the arbitrary reference will appear in all the above terms The effect of Park’s transformation is simply to tansform all stator quantities from phases a, b, and ¢ into new variables the frame of reference of which moves with the rotor We should remember, however, that if we have three variables i,, /,, and i, we need three new variables Park's transformation uses two of the new variables as the d and q axis components The third variable is a stationary current, which is proportional to the zero-sequence current A multiplier is used to simplify the numeri- cal calculations Thus by definition tan (43) where we define the current vectors to lo fag =| fe is (44) iy a and where the Park's transformation P is defined as, . ve vr Vv P = V273) cos# cos(6 — 2/3) cos(# + 2/3) (45) sing sin(@ — 2x/3) sin(@ + 2/3) The main field-winding fiux is along the direction of the d axis of the rotor _It produces an EMF that lags this flux by 90°. Therefore the machine EMF £ is primarily along the rotor q axis. Consider a machine having a constant terminal voltage V For generator The Synchronous Machine 85 action the phasor £ should be leading the phasor 7 The angle between £ and V is the machine torque angle é if the phasor V is in the direction of the reference phase (phase a) Att = 0 the phasor V is located at the axis of phase a, ie, at the reference axis in Figure 41 The q axis is located at an angle 4, and the d axis is located at 6 = S64 2/2 At ¢> 0, the reference axis is located at an angle waf with respect to the axis of phase a Thed axis of the rotor is therefore located at Baap thea rad shes 4% (46) where we is the rated (synchronous) angular frequency in rad/s and 6 is the synchronous torque angle in electrical radians Expressions similar to (4,3) may also be written for voltages or flux linkages; ¢ g., Year =P Yate ody = PA (47) If the transformation (4 5) is unique, an inverse transformation also exists wherein we may write ave = Po "oag (48) The inverse of (45) may be computed to be Ww cos 6 sind Po! = VIP3II/V2_cos( — 2x/3) sin (@ - 24/3) 49) I/V2_ cos(@ + 2/3) sin(# + 2x/3) and we note that P-! = P’, which means that the transformation P is orthogonal Having P orthogonal also means that the transformation P is power invariant, and we should expect to use the same power expression in either the a-b-c or the O-d-g frame ofreference Thus P= Vala + Usly +, Uke = Vanedane = (Po '¥oay)' (BP inag) = YbaePo')'P agg = bap PP ay ~ atop = Lalo + Vag + Uyiy (410) 43° Flux Linkage Equations The situation depicted in Figure 4 1 is that of a network consisting of six mutually coupled coils These are the three phase windings sa-fa, sb-fo, and sc-fe; the field winding F-F'; and the two damper windings D-D' and Q-Q' (The damper windings arc often designated by the symbols kd and kg. We prefer the shortet notation used here. Phase-winding designations s and / refer to “start” and “finish” of these coils } We write the flux linkage equation for these six circuits as Ae] [Lea Far Loe Ler Leo Lag] [te stator : is ' ie | -| Wbums 411) : ie rotor 1 ip i 86 Chapter 4 where Lx = elf-inductance when j = k = mutual inductance when j = k and where L;, = L,;in all cases Note the subscript convention in (4.11) where lower- case subscripts are used for stator quantities and uppercase subscripts are used for rotor quantities Prentice [14] shows that most of the inductances in (4.11) are functions of the rotor position angle @. These inductances may be written as follows 4.3.1 Stator self-inductances The phase-winding self-inductances are given by Log = Ly + £,.00820 H Ly = Ly + L,c0s2(8 - 27/3) H La = Ly + Lp cos2(0 + 2/3) H (412) where L, > L,, and both L, and L,, are constants, (All inductance quantities such as 1, or M, with single subscripts are constants in our notation.) 43.2 Rotor self-inductances Since saturation and slot effect are neglected, all rotor self-inductances are constants and, according to our subscript convention, we may use a single subscript notation; i¢., Lie Lp Ho Lop = Lp H Log = Le H (413) 4.3.3. Stator mutual inductances The phase-to-phase mutual inductances are functions of @ but are symmetric, Lay = Lig = ~M, - L,cos2(0 + 2/6) H Lie = La = -M, — L,cos 20 - 2/2) H = La = ~M, ~ L,,cos2(0 + Sx/6) H (414) where | M,| > Lq. Note that signs of mutual inductance terms depend upon assumed current directions and coil orientations 4.3.4 Rotor mutual inductances The mutual inductance between windings F and D is constant and does not vary with & The coefficient of coupling between the d and g axes is zero, and all pairs of windings with 90° displacement have zero mutual inductance. Thus Lip = Lop = Mp Ho Lig= Lop = 0H Log = Loo = 0H (415) 43.5 Stator-to-rotor mutual inductances Finally, we consider the mutual inductances between stator and rotor windings, all ‘of which are functions of the rotor angle 8, From the phase windings to the field wind- ing we write Lat = Lee = Mycosd H Lu = La = M,cos(@ 2/3) H Lip = Lp = Mycos(6 + 2x/3) H (4.16) Similarly, from phase windings to damper winding D we have The Synchronous Machine 87 Lap = Lng = Mycosé H Lip = Lop = Mpcos(# — 2x/3) H Lip = Lp. = Mpcos(@ + 2x/3) H (417) and finally, from phase windings to damper winding Q we have Lig = Lon = Mosind H Leo = Mgsin(@ - 2x/3) H Lo = Mosin(@ + 2x/3) H (4.18) The signs on mutual terms depend upon assumed current directions and coil orienta- tion 4.3.6 Transformation of inductances: Knowing all inductances in the inductance matrix (4.11), we observe that nearly all terms in the matrix are time varying, since @ is a function of time Only four of the off-diagonal terms vanish, as noted in equation (4.15) Thus in voltage equations such as (4.1) the \ term is nota simple Li but must be computed as \ = Li + Li, We now observe that (4.11) with its time-varying inductances can be simplified by referring all quantities to a rotor frame of reference through a Park’s transformation (45) applied to the a-b-c partition. We compute ede)-F aie ay SEES] oe where Lye = stator-stator inductances Lats Lxe = stator-rotor inductances Lex = rotor-rotor inductances Equation (4 19) is obtained by premuhtiplying (4.11) by et where P is Park’s transformation and U, is thé 3 x 3 unit matrix Performing the operation indicated in (4.19), we compute Ae] flo 0 0 | Al [0 Le 0 1 kMy kM 0 | Jie ' ' I rg} [0 0 Ly | =|---------2--b-------- 2 * “| Wotrns (4.20) dr] [0 kM, 0 ' ' Yo} 0 kMy 0 | Me Lo 0 | fin eo) ' ° re & where we have defined the following new constants, Lg = Ly + M, + B/2Lm Ho Ly = L, + M,~ B3/2)La H Ly = L,- 2M, H k= Vif (4.21) 88 Chapter 4 In (4.20) dy is the flux linkage in a cireuit moving with the rotor and centered on the daxis Similarly, ), is centered on the g axis, Flux linkage Ap is completely uncoupled from the other circuits, as the first row and column have only a diagonal term tis important also to observe that the inductance matrix of (4 20) is a matrix of constants This is apparent since all quantities have only one subscript, thus conforming with our notation for constant inductances. The power of Park's transformation is that it removes the time-varying coefficients from this equation. This is very important We also note that the transformed matrix (4.20) is symmetric and therefore is physically realizable by an equivalent circuit This was not true of the transformation used by Park [10, 11], where he let Voie = Qvate With Q defined as 1/2 12 1/2 Q = 2/3 | cos@ —cos(# — 2x/3) cos (0 + 2x/3) (4.22) =sin® ~sin(@ — 2x/3) ~sin(@ + 2/3), Other transformations are found in the literature. The transformation (4.22) is not a power-invariant transformation and does not result in a reciprocal (symmetric) in- ductance matrix. This leads to unnecessary complication when the equations are nor malized , 4.4 Voltage Equations The generator voltage equations are in the form of (41) Schematically, the ci cuits are shown in Figure 4.2, where coils are identified exactly the same as in Fig- ure 4.1 and with coil terminations shown as well Mutual inductances are omitted from the schematic for clarity but are assumed present with the values given in Sec- tion 4 3 Note that the stator currents are assumed to have a positive direction flowing out of the machine terminals, since the machine is a generator For the conditions in- dicated we may write the matrix equation ve -ri-hty, ¥ ar Fig 42. Schematic diagram of a synchronous machine The Synchronous Machine 89 or v, 0 04 ie] [de us no | 0 0 Olle) [A b on! o 0 offal Jie ---J- -] -------- [onan “|| == v (423) =U, 0 0 0 |r O Offic] | de 0 0 0 0 | 0 tr» 9} fio] | ao 0 0 0.0 '' 0 0 rel Ligh {do 11 fe 1. 11 tfjie}- £a)1 1 Pot ule tu = —Raias — Lyle V (424) Itr, = 15 = 1, = r.asis usually the case, we may also define Ry. =U; @ (4.25) where U; is the 3 x 3 unit matrix, and we may rewrite (4.23) in partitioned form as follows: ee a Dow ally (426) Yeoe 8 Revo} firoo} | Moo] [0 br ip Ae vwoo= | 0 | ino =|iv} Arve =| Xo (427) 0 lie do. Thus (4.26) is complicated by the presence of time-varying coefficients in the \ term, but these terms can be eliminated by applying a Park's transformation to the stator partition. This requires that both sides of (4.26) be premultiplied by (ol EP alte Ee] » an for the left side of (4.26) For the resistance voltage drop term we compute where By definition 90 Chapter 4 Bally Jia] b odEy elt cle ee] Pe wlll TS ellis] ye The second term on the right side of (4.26) is transformed as POW ew Phew (4:30) 0 Usl broe|” [Soe We evaluate Pai. by. recalling the definition (47), Aow = PAu, ftom which we com- PULE Row = PRs, + Phe Then Phasc = Boy ~ Paaic = Rou ~ PP“'Mouy V (431) We may show that 0 0 Of ro 0 . PP, = of 0 0 -i]] Vf =]—wr} V (4.32) a) oh which is the speed voltage term Finally, the third term on the right side of (4 26) transforms as follows: 07 fr. 3] __ [oa = © v (433) o uso] Lo 0 where by definition mo, is the voltage drop from neutral to ground in the O-d-g co- ordinate system Using (4.24), we compute Nog = Py, = —PR,P*'Pigs ~ PL,P™'Pius, = —PRqP'iouy ~ PL, P> Bris] [BL aio -=-|oj}-|ojV¥ (434) 0 0 and observe that this voitage drop occurs only in the zero sequence, as it should ‘Summarizing, we substitute (4,28)-(4.31) and (4 33) into (4.26) to write . 2] JHEP BR] Note that all terms in this equation are known. The resistance matrix is diagonal For balanced conditions the zero-sequence voltage is zero. To simplify the nota- tion, let The Synchronous Machine 2 7% 0 0 a) fers R Re=[0 rm 0] S= Os, Lords. 0 0 49, Then for balanced conditions (4.35) may be written without the zero-sequence equa- tion as R 0 s| |) Mey ia 4 P}-|*] v (4.36) Ye0o 0 Re}l isco} [0] | Arvo] 4,5. Formulation of State-Space Equations Recall that our objective is to derive a set of equations describing the synchronous machine in the form & = f(x,u,) (437) where x = a vector of the state variables u = the system driving functions a set of nonlinear functions If the equations describing the synchronous machine are linear, the set (4.37) is of the well-known form X= Ax + Bu (438) Examining (4 35), we can see that it represents a set of first-order differential equa- tions. We may now put this set in the form of (4.37) or (4 38), ie, in state-space form. Note, however, that (4 35) contains flux linkages and currents as variables Since these two sets of variables are mutually dependent, we can eliminate one set to express (4.35) in terms of one set of variables only Actually, numerous possibilities for the choice of the state variables are available, We will mention only two that are common: (1) a set based on the currents as state variables; ie, x' = [igig/rin ig), which has the advantage of offering simple relations between the voltages v, and v, and the state variables (through the power network connected to the machine terminals) and (2) a set based on flux linkages as the state variables, where the particular set to be chosen depends upon how conveniently they can be expressed in terms of the machine currents and stator voltages. Here we will use the formulation x! = [As\gAr Ao Aol 4.6 Current Formulation Starting with (4.35), we can replace the terms in A and } by terms in é and jas fol- lows The d term has been simplified so that we can compute its value from (4 20), which we rearrange in partitioned form. Let Nowe) Lou Drool | be where Li, is the transpose of L,, But the inductance matrix here is a constant ma- trix, so we may write A = Li V, and the \ term behaves exactly like that of a passive inductance, Substituting this result into (4.35), éxpanding to full 6 x 6 notation, and rearranging, 2 Chapter 4 ° L, kMg kMy Lo (439) where k = V/37Zas before A great deal of information is contained in (4.39) First, we note that the zero-sequence voltage is dependent only upon ig and iy. This equation can be solved separately from the others once the initial conditions on ip are given. The remaining five equations are all coupled in a most interesting way. They are similar to those of a passive network except for the presence of the speed voltage terms These terms, consisting of wd of wLi products, appear unsymmetrically and distinguish this equation from that of a passive network Note that the speed voltage terms in the d axis equation are due only to g axis currents, viz ,i, and ig Simi- larly, the g axis speed voltages are due to d axis currents, ig, i, and ip Also observe that all the terms in the coefficient matrices are constants except w, the angular velocity This is a considerable improvement over the description given in (4.23) in the a-b-c frame of reference since nearly all inductances in that equation were time varying The price we have paid to get rid of the time-varying coefficients is the introduction of speed voltage terms in the resistance matrix. Since w is a variable, this causes (4 39) to be nonlinear If the speed is assumed constant, which is usually a good approximation, then (4.39) is linear In any event, the nonlinearity is never great, as w is usually nearly constant 4.7 Per Unit Conversion The voltage equations of the preceding section are not in a convenient form for en- gineering use. One difficulty is the numerically awkward values with stator voltages in the kilovolt range and field voltage at a much lower level. This problem can be solved by normalizing the equations to a convenient base value and expressing all voltages in pu (or percent) of base (See Appendix C ) An examination of the voltage equations reveals the dimensional character shown in Table 4 1, where all dimensions are expressed in terms of a v-i-t (voltage, current, time) system, {These dimensions are convenient here. Other possible systems are The Synchronous Machine 93 FL1Q (force, length, time, charge) and MZry (mass, length, time, permeability) | Ob- serve that all quantities appearing in (4 39) involve only three dimensions. Thus if we choose three base quantities that involve all three dimensions, all bases are fixed for all quantities For example, if we choose the base voltage, base current, and base time, by combining these quantities according to column 4 of Table 4 1, we may compute base quantities for all other entries Note that exactly three base quantities must be chosen and that these three must involve all three dimensions, v, i, and 1 Table 4.1, Electrical Quantities, Units, and Dimensions Quantity Symbot Units Dinvitions Relationship Voltage bv volts(V) fe) Current i amperes (A) li) Powerot voltamperes por S$ watts (W) [vil pau voltamperes (VA) . Flux linkage A weber turns (Wb turns) (on vai Resistance r ohm (2) (oft) vari, Inductance LorM — henry (H) ery) vedi Time 1 second(s) (") Angular velocity @ radians per second tf (rad/s) Angle Gors radian (rad) dimensionless 4.7.1 Choosing a base for stator quant The variables v2, Up, izy igs Avs ANG Ay are stator quantities because they relate di- rectly to the a-b-c phase quantities through Park’s transformation (Also see Rankin [15], Lewis [12] and Harris et al [9] for a discussion of this topic.) Using the subscript B to indicate “base” and R to indicate “rated,” we choose the following stator base quantities Let Sp = Sq = stator rated VA/phase, VA rms Vy = Va = stator rated ine-to-neutral voltage, V rms sp © we = generator rated speed, elec rad/s (440) Before proceeding further, let us examine the effect of this choice on the d and q axis quantities Fitst note that the three-phase power in pu is three times the pu power per phase (for balanced conditions) To prove this, let the rms phase quantities be V/a V and If. The three-phase power is 3 V/cos(a — y)W The pu power Pj, is given by Pig = BVI Vala)cos (a ~ y) = 3¥L.cos(w — ¥) (440) where the subscript w is used to indicate pu quantities, To obtain the d and q axis quantities, we first write the instantaneous phase voltage and currents. To simplify the expression without any loss of generality, we will assume that v,(1) is in the form, v, = Vy sin(@ +a) = V2Vsin(@ +a) V vy = V2Vsin(6 + a ~ 2x/3) V v, = ViVsin (6 + @ + 22/3) V (4.42) Then from (4.5), ¥oay = P¥y5e OF 94 Chapter 4 Ve 0 v, |= |-v3Vsina] Vv (4.43) vo] LVv3V cosa In pu Vg = Ua/Me = V3I(V/Vq)sina = V3V, sine (4.44) Similarly, Up = VIN cosa (445) Obviously, then Uh + Un = VE (4.46) The above results are significant, They indicate that with this particular choice of the base voltage, the pu d and g axis voltages are numerically equal to V3 times the pu phase voltages. Similarly, we can show that if the rms phase current is I/-y A, the corresponding d and q axis currents are given by, ty 0 i |= |v3rsiny| A (4.47) ud) [wa reosy and the pu currents are given by in = V3KsiNY — ig = V3I,co8y (448) To check the validity of the above, the power in the d and q circuits must be the same as the power in the three stator phases, since P is a power-invariant transfor- mation Pry = lade, + lye = 34, V.(Sinasiny + cosecos y) = 31,V,cos(a — 7) pu (4.49) We now develop the relations for the various base quantities From (4.40) and Table 4 1 we compute the following: Ty = SofVq = Sg/Vq A xms fg = Veg = Vfog 5 As = Vola = Valog = Ly ly Whturn Ry = Vo/ly = Va/ly 0 Ly = Vate/ly = Va/lnsoy HL (450) Thus by choosing the three base quantities Sp, Vp, and ig, we can compute base" values for all quantities of interest. To normalize any quantity, it is divided by the base quantity of the same dimension For example, for currents we write i = i(A)/Ig(A) pu (451) where we use the subscript u to indicate pu. Later, when there is no danger of ambiguity in the notation, this subscript is omitted The Synchronous Machine 95 4.7.2 Choosing a base for rotor quantities Lewis [12) showed that in circuits coupled electromagnetically, which are to be nor- malized, it is essential to select the same voltampere and time base in cach part of the circuit. (See Appendix C for a more detailed treatment of this subject) The choice of equal time base throughout all parts of a circuit with mutual coupling is the impor- tant constraint It can be shown that the choice of a common time base fy forces the VA base to be equal in all circuit parts and also forces the base mutual inductance to be the geometric mean of the base self-inductances if equal pu mutuals are to result; ie., Mug = (Lislos)'”, (See Problem 4 18.) For the synchronous machine the choice of Sy is based on the rating of the stator, and the time base is fixed by the rated radian frequency These base quantities must be the same for the rotor circuits as well. It should be remembered, however, that the stator VA base is much larger than the VA rating of the rotor (field) circuits. Hence some rotor base quantities are bound to be very large, making the corresponding pu rotor quantities appear numerically small, Therefore, care should be exercised in the choice of the remaining free rotor base term, since all other rotor base quantities will then be automatically determined. There is a choice of quantities, but the question is, Which is more convenient? To illustrate the above, consider a machine having a stator rating of 100 x 10° VA/ phase. Assume that its exciter has a rating of 250 V and 1000 A If, for example, we choose Tgp = 1000 A, Vg will then be 100,000 V; and if we choose Vea = 250 V, then Jag will be 400,000 A. Is one choice more convenient than the other? Are there other more desirable choices? The answer lies in the nature of the coupling between the rotor and the stator circuits. It would seem desirable to choose some base quantity in the rotor to give the correct base quantity in the stator For example, we can choose the base rotor current to give, through the magnetic coupling, the correct base stator flux linkage or open circuit voltage. Even then there is some latitude in the choice of the base rotor current, depending on the condition of the magnetic circuit The choice made here for the free rotor base quantity is based on the concept of equal mutual flux linkages This means that base field current ot base d axis amortisscur current will produce the same space fundamental of air gap flux as produced by base stator current acting in the fictitious d winding Referring to the flux linkage equations (4.20) let iy = Ia, ig = Ire, and ip = Ipy be applied one by one with other currents set to zero If we denote the magnetizing inductances (= leakage inductances) as Ing 2Ly- 2a Ho bmg £ Ey ~ 2, H Imp & lp-2¢ Ho Lng #@ Lo -4to o-foH (4.52) Luo * and equate the mutual flux linkages in each winding, Yt = Lats = KM lip = KMplyp WO ng = Lng ly = KMgIgp WD Qne = KMely = Lne len = Mploy Wo np = kKMgly ~ Lyolon Wb Ano = KMply = Mylex = Lo lop WO (453) Then we can show that 76 Chapter 4 Lnals = Lnelig = Luolbe = kMrlolen = KMolalon = Mylralon Lngli = KMolelon = Lnolbs (454) and this is the fundamental constraint among base currents From (4 54) and the requirement for equal S,, we compute Vee/Va = Soflew = Lne{Lms)'” = KMe/Lng = LmefkMy = Ma/kMy > ky Von! Ve = mol Ema)? = KMpfLng = EmofKMp = Mg/kM, © kp You/Ms Col Lng? = KMfo/Lng = LnolkMg * ky (455) These basic constraints permit us to compute Ris = KFRy 2 Ron = kbRa QD — Ron = kRRy D Lig = KiLy Ho Lop = kbLg H Lop = kelp H (456) and since the base mutuals must be the geometric mean of the base self-inductances (see Problem 4.18), Mig =kelg H Mop =kolp H Mon=kolp H Mg = kekpLy H (457) 4.7.3 Comparison with other per unit systems . The subject of the pu system used with synchronous machines has been contro- versial over the years. While the use of pu quantities is common in the literature, it is not always clear which base quantities are used by the authors Furthermore, synchro- nous machine data is usually furnished by the manufacturer in pu, Therefore it is important to understand any major difference in the pu systems adopted Part of the problem lies in the nature of the original Park's transformation Q given in (4.22) This transformation is not power invariant; ie., the three-phase power in watts is given by pac = 15 (igby + i,04) Also, the mutual coupling between the field and the stator d axis is not reciprocal When the Q transformation is used, the pu system is chosen carefully to overcome this difficulty. Note that the modified Park’s transformation P defined by (4.5) was chosen specifically to overcome these problems. The system most commonly used in the literature is based on the following base quantities: = three-phase rated VA Ve = peak rated voltage to neutral Ip = peak rated current and with rotor base quantities chosen to give equal pu mutual inductances This leads to the relations Ten = V2 (Lal Melby Vew = 3/V2)(Me/Lma) Vo This choice of base quantities, which is commonly used, gives the same numerical values in pu for synchronous machine stator and rotor impedances and self-inductances as the system used in this book. The pu mutual inductances differ by a factor of 3/2. Therefore, the terms kM; used in this book are numerically equal to M, in pu as found in the literature. The major differences lie in the following: 1. Since the power in the d and g stator circuits is the three-phase power, one pu cur- tent and voltage gives three pu power in the system used here and gives one pu power in the other system The Synchronous Machine 7 2 Im the system used here v3, + v3, = 33, while in the other system v3, + uh, = V3, where V, is the pu terminal voltage. The system used here is more appealing to some engineers than that used by the manufacturers [9, 12]. However, since the manufacturers’ base system is so common, there is merit in studying both Example 41 Find the pu values of the parameters of the synchronous machine for which the fol- lowing data are given (values are for an actual machine with some quantities, denoted by an astetisk, being estimated for academic study): Rated MVA = 160 MVA Lo = 1423 x 107 H* Rated voltage = 15kV, Y connected £, = f ,(unsaturated) = 05595 x 10° H Excitation voltage = 375 V kMp = 5782 x 10 H* Stator current = 6158.40 A kMg = 2779 x 10? H* Field current = 926 A 1(125°C) = 1542 x 107 2 Power factor = 085 7,(125°C) = 0371 @ Ly = 6341 x 10° HE Tp = 18.421 x 107 OF Lp = 2.189 H ro = 18.969 x 10> O* Lp = 5989 x 10? H* Inertia constant = 1.765 kW: s/bp 6118 x 10° H From the no-load magnetization curve, the value of field current corresponding to the rated voltage on the air gap line is 365 A Solution Stator Base Quantities: Sy = 160/3 = 53.3333 MVA/phase 1300/73 = 866025 V Jy = 6158.40 A ty = 26526 x 107 s Ay = 8660 x 265 x 10°? = 22972 Wb turn/phase Rs ~ 8660 25/6158.40 = 1.406 2 Ly = 8660/(377 x 6158) = 3.730 x 107 H Lind ~ Log ~ 4g = (6.341 = 0.5595)10" = 5.79 x 10> H To obtain Mf;, we use (4.11), (4.16), and (423) At open circuit the mutual in- ductance 14, and the flux linkage in phase a are given by Typ = Mp Cos Xy = ip My cos 8 The instantaneous voltage of phase a is v, = {pep M; sind, where wp is the rated syn- chronous speed Thus the peak phase voltage corresponds to the product ipapM, From the air gap line of the no-load saturation curve, the value of the field current at rated voltage is 365 A. Therefore, M, ~ 8660-V2/(377 x 365) = 89.006 x 10°? H kM, = W372 x 89006 x 10° = 10901 x 10" H Then ky = KMy/Lugg = 18 854 Then we compute, from (4 55}-(4.57), Ing = 6158.4/18.854 = 326.64 A Mz, = 18.854 x 3.73 x 107? = 70329 x 107° H x n 8 Chapter 4 Ven, = (53.33 x 10°)/326 64 = 163280 68 V Res = 163280 68/326.64 = 49989 @ Lyy = (18 845) x 3.73 x 10? = 1326 H Amortisseur Base Quantities (estimated for this example): kMp/Lj = 5.781/5.781 = 1.00 Low = Ly H Moy = Ly H Ron = Re & kMo/Lng = 2779/5 782 = 05 Rop = Re/4 = 0352 2 Log = Ly/4 = 0933 x 10° H Inertia Constant; H = 1.763(1 0/0746) = 237 kW-s/kVA The pu parameters are thus given by: 6 34/373 = 1.70 2.189/1 326 = 1.651 5,989/3 730 = 1.605 Aq = 05595/3.73 = 0.15 - = 6118/3.73 = 164 = 1423/0933 = 1526 = KMp = kKMy = Mg = 1.70 - 0.15 = 1.55 = kMg = 164-015 = 1.49 0.001542/1 406 = 0.001096 0.371 /499.9 = 0.000742 0.018/1.406 = 00131 rq = 18.969 x 1077/0351 = 0.0540 The quantities Ly» and Ligo are defined in Section 4.11 1 4.7.4 The correspondence of per unit stator EMF to rotor quantities ‘We have seen that the particular choice of base quantities used here’gives pu values of dand q axis stator currents and voltages that are «/3 times the tms values. We also note that the coupling between the d axis rotor and stator involves the factor k = 3/2, and similarly for the q axis. For example, the contribution to the d axis stator flux link- age A, due to the field current i, is kMyi, and so on In synchronous machine equations it is often desirable to convert a rotor current, flux linkage, ot voltage to an equivalent stator EMF. These expressions are developed in this section . The basis for converting a field quantity to an equivalent stator EMF is that at open circuit a field cusrent i; A corresponds to an EMF of igo,M, V peak If the rms value of this EMF is £, then i,wgM, = W/2E and ipwgkM, = V/3E in MKS units? 2. The choice of symbol for the EMF due to ipis not clearly desided The American National Stan- dards Institute (ANSI) uses the symbol E, {16]. A new proposed standard uses Ey[17), The International Electrotechnical Commission (IEC) in 4 discussion of [17], favors E, for this voltage The authors leave this voltage unsubscripted until a new standard is adopted The Synchronous Machine 99 Since M, and wp are known constants for a given machine, the field current corresponds toa given EMF by a simple scaling factor Thus & is the stator air gap rms voltage in pu corresponding to the field current i, in pu We can also convert a field flux linkage A, to a corresponding stator EMF. At steady-state open circuit conditions Ay = Lyi;, and this value of field current i,, when multiplied by @_Mz, gives a peak stator voltage the rms value of which is denoted by Ei, Wecan show that the d axis stator EMF corresponding to the field flux linkage \, is given by Alok M;/[Le) = V3Es (4.58) By the same reasoning a field voltage v, corresponds (at steady state) to a field cur- rent U;/r, This in turn corresponds to a peak stator EMF (vp/r,) oe Mp. If the rms value of this EMF is denoted by Esp, the d axis stator EMF corresponds to a field volt- age U; or (s/7)onkMy = V3 Eso (4.59) 4.8 Normalizing the Voltage Equations Having chosen appropriate base values, we may normalize the voltage equations (4.39), Having done this, the stator equations should be numerically easier to deal with, as all values of voltage and current will normally be in the neighborhood of unity. For the following computations we add the subscript u to all pu quantities to emphasize their dimensionless character, Later this subscript will be omitted when all values have been normalized The normalization process is based on (4.51) and a similar relation for the rotor, which may be substituted into (4.39) to give voVe r+, 0 0 0 0 0 vaVe 0 rr) 0 kM vaVs 0 -wly 7 -wkM, -ekM, 0 -onVm| | 0 0 0 & 0 0 0 0 0 0 0 ty 0 0 0 a) 0 0 'o io+3L, 0 0 0 0 Toute 0 Ly 0 kM, kMy ile 0 OL, 0 0 kMol dete (4.60) 0 kM, 0 Ly Mp inden 0 kM, 0 Mg Ly inatow 0 0 kM, 0 0 jowtow where the first three equations are on a stator base and the last three are on a rotor base Examine the second equation more closely. Dividing through by Yq and setting © = wytog, We have 100 Chapter 4 Fein pu (461) inn PU (4.62) We now recognize the following pu quantities nea Ry by = Lally My = Mporlea/Yo Lee Lolly Mov = Mpwnlon/Vy May = Moor on/Y 463) Incorporating (4 63), the d axis equation (4 62) may be rewritten with all values except time in pusie, Pag © Falan ~ WyLelys ~ Key M tes ~ HH ay — k i, — kK MO in, (468) The third equation of (4.60) may be analyzed in a similar way to write Leu: M, Dye = Liason — Nelge + OKMeviy + OkKMosing — — iy — iow pu 465) &n K where all pu coefficients have been previously defined. The first equation is uncoupled from the others and may be written as Ve = = — (+ Baba dow — Bz (Lo + FL y)y fon PU (4 66) Uf the currents are balanced, it is easy to show that this equation vanishes. The fourth equation is normalized on a rotor basis and may be written from (4 60) as Le eaten; , Me onlon ; Oe Ving Voy OY ee We now incorporate the base rotor inductance to normalize the last two terms as Lig = Le/Lep May = Ma /Men (4.68) The normalized field circuit equation becomes . : kMy: Lee; | Maw: = raj 4 Eee jg Mw 4. Ore = Trvlte tan + tre + oy (4.69) The damper winding equations can be normalized by a similar procedure The following equations are then obtained, KMow j. 4 Mao i, 4 Lo j,, (4.70) OR @R OR be Ve = 0 = toute + The Synchronous Machine 101 kM, ; qe = 0 = Fouigu + oe fy + —& iy (471) These normalized equations are in a form suitable for solution in the time domain with time in seconds However, some engineers prefer to rid the equations of the awkward /ax that accompanies every term containing a time derivative This may be done by normalizing time, We do this by setting Ld_d oy dt” dr 7) where T= wl (473) is the normalized time in rad Incorporating all normalized equations in a matrix expression and dropping the subscript u since all values are in pu, we write u% r 0 0 el, okMel[ -Y 0 n o |} 0 0 hy 0 0 0 my | 0 0 uy 0 Ly kM; kM | 0 0 [li kM, Ly Mg 0 0 We =| kM, My Lp 0 0 |fi|pu 4.74) ‘ 1 kMg Le |lig where we have omitted the v, equation, since we are interested in balanced system con- ditions in stability studies, and have rearranged the equations to show the d and q cou- pling more clearly. It is important to notice that (4.74) is identical in notation to (4 39) This is always possible if base quantities are carefully chosen and is highly desirable, as the same equation symbolically serves both as a pu and a “system quantity” equation Using matrix notation, we write (4.74) as y= -(R + eN)i- Li po (475) where R is the resistance matrix and is a diagonal matrix of constants, N is the matrix of speed voltage inductance coefficients, and L is a symmetric matrix of constant ductances. If we assume that the inverse of the inductance matrix exists, we may write i= -L"(R + eN)i- Lv pu (476) This equation has the desired state-space form It does not express the entire system be- havior, however, so we have additional equations to write Equation (4.76) may be depicted schematically by the equivalent circuit shown in 102 Chapter 4 Fig 43 Synchronous generator d-g equivalent circuit Figure 43, Note that all self and mutual inductances in the equivalent circuit are con stants, and pu quantities are implied for all quantities, including time. Note also the presence of controlled sources in the equivalent These are due to speed voltage terms in the equations Equation (4.74) and the circuit in Figure 4.3 differ from similar equations found in the literature in two important ways. In this chapter we use the symbols L and M for self and mutual inductances respectively. Some authors and most manufacturers refer to these same quantities by the symbol x or XY This is sometimes confusing to one learning synchronous machine theory because a term XJ that appears to be a voltage may bea flux linkage The use of ¥ for L or M is based on the rationale that w is nearly constant at 1.0 pu so that, in pu, ¥ = o£ & Z However, as we shall indicate in the sections to follow, w is certainly not a constant; it is a state variable in our equations, and we must treat it as a variable Later, in a linearized model we will let w be ap- proximated as a constant and will simplify other terms in the equations as well For convenience of those acquainted with other references we list a comparison of these inductances in Table 42. Here the subscript notation kd and kq for D and Q re- spectively is seen, These symbols are quite common in the literature in reference to the damper windings Table 4. Comparison of Per Unit Inductance Symbols Chapter 4 Za Ly Le by Lg kMp Mg KMp Mg Kimbark(2] Ly Ly Leg Leg Me M, Concordia(t] x, Xe Nha Bigg Fer Xd Sak aig Example 4.2 Consider a 60-Hz synchronous machine with the following pu parameters: : L,= 170 kMy = 1.49 Ly = 164 7 = 0.001096 L, = 165 ry = 0.000742 Ly = 1.605 r= 0.0131 Lg = 1.526 rq = 00540 kM, = My = kMy = 155 H 237s fen ty = O15 The Synchronous Mochine 103 Solution From (4.75) we have numerically 0.0011 0 0 | 1640 1490 0 000% 0 | oO o R+one| 0 0 oo | 0 0 | pw =1 70% =155e -1.55« 100011 0 0 0 0 ! 0 00540, [ 170 «155455 | 0 0 155165155 | 0 0 Le | 155 155 1605 ‘ 0 0 pu 0 0 o | 1.64 1.49 0 0 0 | 149 1526 from which we compute by digital computer 5405-1869 -3414' 0 -1869 7.110 -5060! 0 L-t. |-3414 -s060 8804; 0 pu 5.406 5.280 0 0 ' ° 0 0 1 -5280 58I1 Then we may compute 5.9269 1.3878, 447198 | ~88649e 8504.10 2.0498 5.2785 66 2818 | 30659 2785 dw -L-'(R EWN) = 10?) 37433 3.7564 115.3200 55989e 5086.86 | pu 9190.90 8379.9 8379 9w 1 -59279 284.857 =8975.2w ~81833@ -B8183.3w | 5.7888 ~313 534 and the coeflicient matrix is seen to contain w in 12 of its 25 terms. This gives some idea of the complexity of the equations 4.9 Normalizing the Torque Equations In Chapter 2 the swing equation JO = 21 /p)a = T, Nm (4.77) is normalized by dividing both sides of the equation by a shaft torque that corresponds to the rated three-phase power at rated speed (base three-phase torque), The result of this normalization was found to be 104 Chapter 4 QH/eg)o = T, pui¢) (4.78) where = angular velocity of the revolving magnetic field in elec rad/s T, = accelerating torque in pu on a three-phase base Wr/ Sass A and the derivative is with respect to time in seconds This normalization takes into account the change in angular measurements from mechanical to electrical radians and divides the equations by the base three-phase torque. Equation (4 78) is the swing equa- tion used to determine the speed of the stator revolving MMF wave as a function of time. We need to couple the electromagnetic torque T,, determined by the generator equations, to the form of (4 78) Since (4.78) is normalized to a three-phase base torque and our chosen generator VA base is a per phase basis, we must use care in combining, the pu swing equation and the pu generator torque equation Rewriting (4 78) as (2H/w)@ = Ty ~ T, puQdo) (4.79) the expression used for 7, must be in pu on a three-phase VA base Suppose we define T,, = pu generator electromagnetic torque defined on a per phase VA base = TAN m)/(Sp/#5) pu (4.80) Then T, = T.,/3 puG¢) (481) (A similar definition could be used for the mechanical torque; viz, Tye = 37m. Usu- ally, 7, is normalized on a three-phase basis ) The procedure that must be used is clear. We compute the generator electromag- netic torque in Nom. This torque is normalized along with other generator quantities on a basis of Sp, %, Ip, and fy to give T., Thus for a fully loaded machine at rated speed, we would expect to compute 7,, = 30 Equation (481) transforms this pu torque to the new value 7,, which is the pu torque on a three-phase basis. 4.9.1. The normalized swing equation In (4 79), while the torque is normalized, the angular speed w and the time are given in MKS units. Thus the equation is not completely normalized The normalized swing equation is of the form given in (2,66) Te — Toy = Toy DU (4.82) 1a where all the terms in the swing equation, including time and angular speed, are in pu Beginning with (4 79) and substituting = eet 0, = Ofay (483) we have for the normalized swing equation de, 2H, = 1, 4 84) Hoy Ge = Taw (484) thus, when time is in pu, a - 2Hey (485) The Synchronous Machine 105 4.9.2. Forms of the swing equation There are many forms of the swing equation appearing in the literature of power system dynamics, While the torque is almost always given in pu, it is often not clear which units of w and r are being used, To avoid confusion, a summary of the different forms of the swing equation is given in this section. We begin with w in rad/s and fins, 2H/ws)o = T., Iftand 7, are in pu (and © in rad/s), by substituting 4, = «atin (479), 2H da dw a dF as, If wand 7, are in pu (and 1 in s), by substituting in (4.79), = Tey PB (486) (487) It, , and 7, are all in pu, (4.88) fq is given in elec deg/s, (4.79) and (4.86) are modified as follows: mae Toy pu (4.89) ae =T. pu (490) It would be tempting to normalize the swing equation on a per phase basis such that all terms in (4.79) are in pu based on Sy rather than Sy). This could indeed be done with the result that all values in the swing equation would be multiplied by three This is not done here because it is common to express both T,, and 7, in pu on a three- phase base Therefore, even though S, is a convenient base to use in normalizing the generator circuits. it is considered wise to convert the generator terminal power and torque to a three-phase base Sy, to match the basis normally used in computing the machine terminal conditions from the viewpoint of the network (eg , in load-flow stud- ies). Note there is not a similar problem with the voltage being based on V4, the phase-to-neutral voltage, since a phase voltage of k pu means that the line-to-line volt- age is also k pu on a line-to-line basis 4.10 Torque and Power The total three-phase power output of a synchronous machine is given by Pow = ala + Usiy + Velo = Yoschane PU 49l where the superscript 1 indicates the transpose of v4, But from (48) we may write jute = Ploy with a similar expression for the voltage vector Then (4.91) becomes Pou = Wag(P*Y Pig Performing the indicated operation and recalling that P is orthogonal, we find that 106 Chapter 4 the power output of a synchronous generator is invariant under the transformation P; ie, Pou = Vala + Ugly + rio (4.92) For simplicity we will assume balanced but not necessarily steady-state conditions. Thus vy = i = Oand Pou = Usis + U,i, (balanced condition) (493) Substituting for v, and v, trom (4.36), ida + phy) + (ipdg — Wyle — AZ + 2) (4.94) Concordia (1] observes that the three terms ate identifiable as the rate of change of stator magnetic field energy, the power transferred across the air gap, and the stator ‘ohmic losses respectively. The machine torque is obtained from the second term, Tog = 8Wag/00 = Ppa dio = 8/80 ((igAy — WA)JOl = iy — UA, PU (4.95) Poo = The same result can be obtained from a mote rigorous derivation, Starting with the three armature circuits and the three rotor circuits, the energy in the field is given by Moa D5 Gaited (496) m which is @ function of @. Then using 7 = 4444/90 and simplifying, we can obtain the above relation (see Appendix B of [1)). ‘Now, recalling that the flux linkages can be expressed in terms of the currents, we write from (4.20), expressed in pu, Ng = Lgig + KMpip + KMpip = Lyiy + KM gig (497) Then (4.95) can be written as iy is Tog © (Lely KMily KMoiy | —Lyig — kMgit] | '0 | pu (4.98) which we recognize to be a bilinear term Suppose we express the total accelerating torque in the swing equation as T.-T, (4.99) where Ty is the mechanical torque, T, is the electrical torque, and T, is the damping torque. It is often convenient to write the damping torque as T, = Ty — Tyo} — Te = Tm Ty= Dw pu (4.100) where D is a damping constant. Then by using (4 81) and (4,98), the swing equation may be written as The Synchronous Machine 107 101) where 1; is defined by (4.85) and depends on the units used for w and ¢ Finally, the following relation between 6 and w may be derived from (4.6) baw-1 (4 102) Incorporating (4.101) and (4, 102) into (4.76), we obtain ‘| : in| [obey -L-4R + oN) on) by i - : + ' fo ; ' . I, e 3 é 6 =I (4.103) This matrix equation is in the desired state-space form x = f(x,u,1) as given by (4.37). Itis clear from (4.101) that the system is nonlinear. Note that the “inputs” are vand Ty, 4.11 Equivalent Circuit of a Synchronous Machine For balanced conditions the normalized flux linkage equations are obtained from (4.20) with the row for \y omitted & hy I, 0 kM, kMp 0 Vly yy Oo L 0 0 kM li, Ml=|kKMp 0 Ly My 0 | lie (4.104) Xp kMp 0 Ma Ly 0 |lip do. 0 kM 0 0 Lolli We may rewrite the d axis flux linkages as 108 Chapter 4 he = (Le = £4) + tabi + KMeie + KMoly Ap = KMpig + (Le — 25) + Celie + Main Xp = KMpig + Mai + (Lp — 20) + Polip (4.108) where £4, Zp, and Lp are the leakage inductances of the d, F, and D circuits respec- tively. Let ip = i, = 0, and the flux linkage that will be mutually coupled to the other circuits is My ~ Lyig or (Ly — €u)ip As stated in Section 47.2, Ly — £4 is the magnetizing inductance Lg. The flux linkage mutually coupled to the other d axis circuits is then Lygig, The flux linkages in the F and D circuits, 4; and Xp, are given in this particular case by Ay = kMi,, and Ay = KMpiy From the choice of the base rotor current, to give equal mutual flux, we can see that the pu values of Zyig, hy, and Ap must be equal. Therefore, the pu values of Lng, kM, and kM) are equal This can be verified by using (4 57) and (4 55), kM, kM, Lng yn (Myla) Ty” G09) kMn = In pu, we usually call this quantity L495 i€., Lap * Ly - by = kM; = kKMp pu (4.107) We can also prove that, in pu, Lap = Ly ~ Xn = Ly - bp = bg — dy = KMp = KMy = Mg (4-108) Similarly, for the g axis we define Lag * Ly — ty = Lg - 29 = kKMo pu (4 109) If in cach circuit the pu leakage flux linkage is subtracted, the remaining flux linkage is the same as for all other circuits coupled to it_ Thus Ns ~ tele = Ae Cele = dy — Lolo ® May pu (4110) where Raw = tally ~ La) + KMyie + KMoig = Leolis + ie + ty) pu @an Similarly, the pu g axis mutual flux linkage is given by dao = (La — dally + KMoto = Laoli, + to) (4112) Following the procedure used in developing the equivalent circuit of transformers, we can represent the above relations by the circuits shown in Figure 44, where we note that the currents add in the mutual branch. To complete the equivalent circuit, we Fig 4.4 Flux linkage inductances of a synchronous machine The Synchronous Machine 109 Fig 4$ Direct axis equivalent circuit consider the voltage equations vy = —rig - Ay = wd, a rig — bag ~ La ~ Lads + KMplp + KMpin] — wh, or vg = rig — Lig — Lavlls + ip + ip) — wry (4.113) Similarly, we can show that mop = tye = bale = Laplis + ip + in) (4.114) vp = 0 = =ryip — Foip - Lanlly + ip + ty) (4.115) The above voltage equations are satisfied by the equivalent circuit shown in Fig ure4.5 The three d axis circuits (d, F, and D) ate coupled through the common mag- netizing inductance £4», which carries the sum of the currents iy. ip, and ip Thed axis circuit contains a controlled voltage source wA, with the polarity as shown. Similarly, for the q axis circuits vy = rig — tee — Laglly + fg) + ory (4.116) 0 = ~rei9 — Lalo - Laaliy + fo) (4.117) These two equations are satisfied by the equivalent circuit shown in Figure 46, Note the presence of the controlled source wAyin the stator g circuit & 4.12 The Flux Linkage State-Space Model We now develop an alternate state-space model where the state variables chosen are Yas Aes Aas Ay aN Ag. From (4.110) ig = (/4a)s - Avo) te = U/EDOAr- Mav) tp = (UE d)Ap ~ Mav) (4 118) but from (4.111) Ag = (iy + ip + fy) Lap, which we can incorporate into (4 118) to get Fig 4.6 Quadrature axis equivalent circuit 10 Chapter 4 Maoll/Lav + Vda + W/be + 1/f0) = Malta + Mf te + do/ to Now define YL & VLav + Vda + Abe + M20 then Yao = Euv/ta)rv + Luv/ beds + Lavi to) do Similarly, we can show that Mo = (Laoltadd + Luolo)ro where we define H/Luo * Whig + Wey + Veto and the @ axis currents are given by i, = (1/Eq) A, — Mao) ig = (/29)(Ao — ug) Writing (4.118) and (4 123) in matrix form, uj [ve 0 0 -uA | x ip 0 Wee 0 =1/t, | 0 XL bb 0 0 Wey =1/y i ” - ‘ Mao | Ne 4 0 Me Oo -1/t, he io 19 Wee -Wto)| Mo 4.121 The voltage equations The voltage equations are derived as follows from (4.36) For the d equation bg = ~riy— Ay - od, Using (4 124) and rearranging, By = —1Qe/2a— Mola) ~ od = 04 on Ky = -C/£a) da + P/E a) Aan — hy ~ By Also from (4 36) 0p = “tle - Sy Substituting for iy Ae = —reOs/Er ~ Mol£e) + Ue or Ay = (s/f eds + Ur/€e)dav — (UF) (4119) (4120) (12 (4122) (4123), 104) (4.128) (4129 (4.127) (4.128) The Synchronous Machine m Repeating the procedure for the D circuit, ‘0 = —Va/20)do + (ro/ Lo) av (4.129) The procedure is repeated for the q axis circuits. For the v, equations we compute Ry = —U/E DN + (1/2) dag + OAs — U4 (4.130) and from the g axis damper-winding equation, Ro = —(roldeddo + (ral to) do (CoEDD) Note that Xyp OF Aso appears in the above equations This form is convenient if satura tion is to be included in the model since the mutual inductances Lyp and L4g are the only inductances that saturate. If saturation can be neglected the Ay and Xyg terms can be eliminated (see Section 4.12 3) 4.12.2. The torque equation From (495) Z,, = ighy — A, Using (4 124), we substitute for the currents to com- pute T= ag hag + t Aad (x - ze (4.132) We may also take advantage of the relation 2, = 2, (called £, in many references) The new electromechanical equation is given by B= —Cue/4adry + Oso/Le3—)de ~ D/ ne + Tals 133) Finally the equation for 6 1s given by (4.102). Equations (4.126)-(4.131), (4.133), and (4.102) are in state-space form. The auxiliary equations (4.120) and (4.121) are needed to relate Ayp and Ayo to the state variables The state variables are Ay, Xey Apy Ags Age w, and 6 The forcing functions, ase Ug, U,V, and T,, This form of the equations is particularly convenient for solution where saturation is required, since saturation affects only Ayp and Aug Dry at) ty 4.12.3 Machine equations with saturation neglected Hi saturation is neglected, Lyp and yg ate constant. Therefore, Lyp and Lug are also constant. The magnetizing flux linkages A,» and A,g will have constant re~ lationships to the state variables as given by (4 120) and (4121). We can therefore climinate Azo and Ayo from the machine equations Substituting for A,p, as given in (4.120), in (4.118) and rearranging, ig (1 — hue) de — Lao Me _ Leo Yo. . Tafa te te Fa bo inn Hada (1 Bas) he — bye tp ts te Ai] ts Ee To iy = — Leo Xa. Euro de (-4e)e on Ete bn de® ene M2 Chopter 4 These currents are substituted in the d axis voltage equations of (4 36) to get, Ye gp Lue Me, Luo Xo Lae = (i) ~ Se! — wry - ¥, ( a te te te vot s Lup Ly Ay Lyp ¥, Sohal Be wo Aw. y OEE Ty ( Ey, a, + 5 Lup Luo L, Xx Ng = ry SMO ME a ry SMO AE, (- igo) 2 (4.135 oO tT hy ey to » Similarly, the q axis equations are a) da, , Luo de — Et Et wy — te to . L, X rel - EN (4.136) and the equation for the electrical torque is given by Lu Lu Le = AsA St dp Me + AAA, (4.137) Rte Ra Ta » The state-space model now becomes » he h he wo + Lys thay 1 a | x) |-eb-4#) Le ° ool) bw ve be ois (4138) The system described by (4.138) is in the form x = f(x,u,4) Again the description of the system is not complete since vy and v, are functions of the currents and will de- pend on the external load connections. The 7 x 7 matrix on the right side of (4.138) contains state variables in several terms, and this mat form of the equation is nor an appropriate form for solution. It does, however, serve to illustrate the nonlinear nature of the system Example 43 Repeat Example 4.2 for the flux linkage model Solution From the data of Example 4 1: The Synchronous Machir v3 fe= 4,= 0150 pu z(- 2p = 1651 = 1550 = 0.101 pu 4, 2p = 1.605 — 1.550 = 0.055 pu te Luo. 9.991387 2g = 1526 - 1490 = 0036 pu te ta toi ,t a cc ( - = 0.005278 Lo” 135 * 015 * O10 tr oa Zr Luo. 9 003756 + ggg 7 382381 pu qt Lyo = 0.028378 pu he ~ 0.008789 Poa yt yt a Fo Tyo 149 * 015 * 036 fo Luo 42 SMe _. 0 286058 = 352381 pu to 2, Lyo = 0.028378 pu Jo ( _ ta) 0 308485 , L te to + (1 - 54) — 0005927 ta ( #2) Luo _ 9.000706 Eun ~ 9.002049 344 tet Luo - 9.001046 5 Lue ~ 9.003743 Sata te ub Lup. 0.001910 to Lu _ 9.944720 31,4445 fot hue = 0.0029 wo 34 Sp Se 0 0s0ns2 352, lo pte Luo e( ~ 2) - onssss0 35,45” (9.000705 to to and we get for the state-space equation NV -$927 2.050 3.743 -« 0 de -¥4 Ne 1388 3.278 «3.756 0 0 Me Up No 10” 44720 66.282 -115 330 0 0 Ao 0 «| = 10" + 5 © 0 0 5928 5.789 |], -Y he 0 0 6 284.854 313.530} | Ao| 0 e -07062, -1046, -1910A, 0.7052, 2954r,}|e} | 0.000567, 4.12.4 Treatment of saturation The flux linkage state-space model is convenient for considering the effect of satura- tion because all the terms in the state equations (4 126)-(4.133) are linear except for the magnetizing flux linkages Ap and dag ‘These are affected by saturation of the mutual inductances Lp and L4g, and only these terms need to be corrected for saturation. In the simulation of the machine, either by digital or analog computer, this can be accom- 114 Chapter 4 jar fo ins i Fig 47 Saturation curve for Map plished by computing a saturation function to adjust (4 120) and (4.121) at all times to reflect the state of the mutual inductances As a practical matter, the q axis inductance Lg seldom saturates, so it is usually necessary to adjust only X4p for saturation. The procedure for including the magnetic circuit saturation is given below [18]. Let the unsaturated values of the magnetizing inductances be Lane and L4gq. The compu- tations for saturated values of these inductances follow For salient pole machines, Lin = KLav0 Lag = Lago Ky = f(a) @ 139) where K, isa saturation factor determined {rom the magnetization curve of the machine For a round-rotor machine, we compute, according to [16] Lan = Khao Lag = KL ago K, =f) d= io + Nig)? (4 140) To determine K, for the d axis in (4 139), the following procedure is suggested. Let the magnetizing current, which is the sum of ig + ig + ip, be iy. The relation be- tween \qo and iy is given by the saturation curve shown in Figure 47 For a given value of Ap the unsaturated magnetizing cuttent is iyo, corresponding to Lavo while the saturated value is iys The saturation function K, is a function of this mag- netizing current, which in turn isa function of \4p To calculate the saturated magnetizing current iys, the current increment needed to satisfy saturation, iva = ius ~ lwo, is first calculated Note that saturation be- gins at the threshold value Aypr corresponding to a magnetizing current yr. For flux linkages greater than Xypr the current iya increases monotonically in an almost expo- nential way. Thus we may write approximately ius = 4,€xp[B.(Xao — Mor) Aan > avr 4141) where A, and B, are constants to be determined from the actual saturation curve. Knowing iva for a given value of Azo, the value of iys is calculated, and hence K, is determined The solution is obtained by an iterative process so that the relation DapK.Oran) = Lavoixs is satisfied 4.13 toad Equations From (4 103) and (4.138) we have a set of equations for each machine in the form % = 0,1 7p) 142) The Synchronous Machine ns where x is a vector of order seven (five currents, w and 6 for the current model, or five flux linkages, w and 6 for the flux linkage model), and v is a vector of voltages that includes vj, U4, and v, Assuming that uy and 7, are known, the set (4.142) does not completely describe the synchronous machine since there are two additional variables vy and v, appeat- ing in the equations. Therefore two additional equations are needed to relate v, and v, 0 the state variables, These are auxiliary equations, which may or may not increase the order of the system depending upon whether the relations obtained are algebraic equations or differential equations and whether new variables are introduced To ob- tain equations for v, and v, in terms of the state variables, the terminal conditions of the machine must be known In other words, equations describing the load are required There are a number of ways of representing the electrical load on a synchronous generator For example, we could consider the load to be constant impedance, con- stant power, constant current, or some composite of all three For the present we re quire a load representation that will illustrate the constraints between the generator voltages, currents, and angula: velocity These constraints are found by solving the net- work, including loads, given the machine terminal voltages For illustrative purposes here, the load constraint is satisfied by the simple one machine-infinite bus problem illustrated below. 4.13.1 Synchronous machine connected to an infinite bus Consider the system of Figure 4.8 where a synchronous machine is connected to an infinite bus through a transmission line having resistance R, and inductance L,. The voltages and current for phase a only are shown, assuming no mutual coupling between phases, By inspection of Figure 4.8 we can write vy = Ves + Reig + Lyi, or . iy ofa] v..| + RUA] + LU (4143) ve] | Bae ie i In matrix notation (4 143) becomes Vato > Veate + ReUinte + LeUhane (4.144) which we transform to the O-d-q frame of reference by Park’s transformation: You = Pratc = Precte + Reloay + LePigs. Vor pu (4.145) The first term on the right side we may call v.94 and may determine its value by as- suming that ¥...5. i8 a set of balanced three-phase voltages, or Fig 48 Synchronous generator loaded by an infinite bus 116 Chapter 4 cos(ugt + a) Vout: = V2V. | cos (wet + a ~ 120°) (4 146) cos(wR! + a + 120°) where V. is the magnitude of the rms phase voltage. Using the identities in Appendix A. and using @ = wat + 6 + /2, we can show that 0 V3 |-sin(6 - ) (4.147) cos(é — a) Yaad = PYecte The last term on the right side of (4 145) may be computed as follows. From the definition of Park’s transformation jn = Pi, We compute the derivative ing, = Piste + Plage Thus Prine = iggy ~ Phase = dogg — PPioug (4.148) where the quantity PP-! is known from (4 32), Thus (4.145) may be written as 0 0 Voag = Va V3 |—sin(6 ~ a)) + Reiog + Lelong — ol. |-i,; Vorpu (4.149) cos(5 — a) ig which gives the constraint between the generator terminal voltage voy and the gen- erator current igy for a given torque angle § Note that (4.149) is exactly the same whether in MKS units or pu due to our choice of P and base quantities Note also that there are two nonlinearities in (4 149) The first is due to the speed voltage term, the wL,i product. There is also a nonlinearity in the trigonometric functions of the first term . The angle é is related to the speed by 5 = © ~ I pu or, in radians, 5+ fw onda (4.150) Thus even this simple load representation introduces new nonlinearities, but the order of the system remains at seven 4.13.2 Current model Incorporating (4.149) into system (4.75), we may write —Ksiny + Reig + Leis + wl, -o (R + wN)i + 0 (4.151) Keosy + Rig + Lely — wheig 0 The Synchronous Machine Ww where K = VV. aud y = 6 ~ a. Nowlet ReartR Lew Qth, be ltk (4.152) Using (4.152), we may replace the », Le, and L, terms in L,R, and N by &, 2,, and £, to obtain the new matrices Land (R + oN). Thus [se -o -fi- (+ oNi+) 0 (4.153) Koosy 0 Premultiplying by —L~' and adding the equations for & and é, -L(R + oN) a a) 4 fle i fo o ' Ty 3) -3 NE e tof & of ssl oo 6 ho ap - (4.154) The system described by (4 154) is now in the form of (4.37), namely, X = f(x, wD, where x! = [iziripi, in wd] The function f is @ nonlinear function of the state variables and 1, and u contains the system driving functions, which are, Ue and Ty. The loading effect of the transmis- sion line is incorporated in the matrices R, L,and N- The infinite bus voltage V_ appears in the terms K sin y and K cos y Note also that these latter terms are not driving functions, but rather nonlinear functions of the state variable 6 Because the system (4 154) is nonlinear, determination of its stability depends upon finding a suitable Liapunov function or some equivalent method This is explored in greater depth in Volume 2 4.13.3 The flux linkage model From (4 149) and substituting for i, and i, in terms of flux linkages (see Sec- tion 4.12.3), _ ReLwo v= -V3Ve snG a) + FEI Ea), — Relate ny - 1 pte ho _ obeLuo be - be) — belo y _ Leluo Jn Spat no $e (1 Girlie - Seine a, tei (4.155) 118 Chapter 4 = V3V.cos(é - a) + z(-4 ee) 7 ~ FG de - oe ~ tn the 4, 4 Ley — Lue) 5, — Lebo 3, 4.156) Tt * ths wer Be) he FE be ano Combining (4.155) with (4.135), Sef — Lae), — le, Lelie 5, Bf baw RLuo y p+ #6 tal Take bat © Bat edt Tate Ay + Seta ng + VV. sin(@ — @) vee (4.157) Lu), , Blame La) s+ FEN ~ Ghetu y, — VIV, 005 (6 - a) «to (4.158) Equations (4 157) and (4 158) replace the first and fourth rows in (4 138) to give the complete state-space model. The resulting equation is of the form Tx=Cx+D (4.159) where x’ = [Ay Me Ap Ay Ag we 6), als ( _ a) _Lebun _Lebup ta\ a) "Fade ~ Beko ° ° e e The Synchronous Machine ug and the matrix C is given by ° and VIV. sin(6 — a) uo 0 D = |~V3V, cos(s — a) (4 162) 9 Taft) -1 If I! exists, premultiply (4.159) by I~! to get ke Tx + I'D (4163) Equation (4 163) is in the desired form, ie. in the form of x = f(x,u,1) and completely describes the system It contains two types of nonlinearities, product nonlinearities and trigonometric functions Example 4.4 Extend Examples 42 and 43 to include the effect of the transmission line and torque equations The line constants are R, = 0, L, = 0.4 pu, tj = 2Hwg = 1786.94 tad. The infinite bus voltage constant K and the damping torque coefficient D are left unspecified Solution R=r4R,= 000106 i, = 1.4L, = 210 Ly +L, = 204 120 Chopter 4 Then ooo. 0 0 | 2040 1.490 0 000072 0} 8 ° R+oN=| 0 6 oot | 0 0 2100 -1 550 0 0 0 | 0 cos 2.400 1.550 1550 | 0 0 1550 1651 1550} 0 0 L=|iss0 1550 1605 | 0 0 0 0 0} 2.040 1490 0 0 0 | 1490 1,526 By digital computer we find 1709-0591 —1.080 |} —0.591 6.668 —5 867 1 0 i-=|-1080 -5867 —7.330 1) 11 = 1669 ° 1 1669 2.286 Then 000187 -0.00044 -00141 | 34870 2.5470 ~0.00065 0.00495 —0.0769 LR + oN) =|—0.00118 0.00436 0.0960 ~1206e 088le 359 = -2 6500 = 2.650 3-506. 2588e 2.588 | -0.00183 0.12332 and we compute HKsiny] [=171Ksiny + 0589v, ~u 0 589K sin y ~ 6 69uy L') 0 |=} 108Ksiny + 5.89uy Koosy 171K cosy 0 = 167K cosy Therefore the state-space current model is given by The Synchronous Machine 121 -0000559D 0] & 1 olla @ ~0.00095i, -0.00087i, —0.00087i, 0.00092i, 0.000833, é 0 0 0 0 0 fs 000193 -000037 00143 -349%% = 255e@ | 0 0] | te ip 0.00067 0.00496 0.0778. 1.20w = 0878 0 04) ar ip 000122 © 000837 = 0071 22w ele | 0} | te |=] 3590 2650 2656 00019 0091 ° oll a te -3:500 =2590 -259@ 00018-01234 | 0 olf ig 171K sin y — 0.58905] 0 589K sin-y — 6 69, = 108K siny — 5.980, + LT Keosy 167K cosy 0.000559 7, “1 The flux linkage model is of the form 1 = CA + D, where T, C, and D are given by (4 159)-(4 162). Substituting, 31622 0.7478 -13656 1 0 10 0 0 10 nr) 109 0 0 0 i ) | 0 0 T Oo oO oO 0 0 0 0 0 0 10 03162 0.2365 04319 0 10 o ° 03162 0.6678 0 1.0 1 0 ' The matrix C is mostly the same as that given in Example 4.3 except that the w terms are modified 122 Chapter 4 3927 2050 3.743 3162e 21120 i : 1388 5.278 3.756 FO o + 0 44720 66282 115330) 0 o 4 an = a fo 3620-747 Tw 13660 | 5928 $789 | - ! io 10 0 0 0 — | 284.854 313.530} t 4 -07058X, 1.046A, -1.910A,/ 0705, 2954A, '-05596D 0 I ' 0 0 o 1 0 ort ® hw), 17766 28024 47.733 | 10000 667 Bu | | he 1388-5278 3.756 | 0 o 1 0 »' ho 44.720 66 282, 115.330; 0 o | dy 3, | = 10° | 1000e — -236 dw = 431 Bw + 188.337 -207.529 ' ° Ny he 0 0 0 |284.854 ~313 530) de - 3 - o 0.706, -10464, -1910,0705A, 2954A, | -0.5596D 0] | w 0 0 an) oi o4 0} Ls 0316 Ksin-y + 02290, + 0 0.000559 T,, -l 4.14 Subtransient and Transient Inductances and Time Constants If all the rotor ciscuits are short circuited and balanced three-phase voltages are suddenly impressed upon the stator terminals, the ux linking the d axis circuit will de- pend initially on the subtransient inductances, and after a few cycles on the transient inductances. Let the phase voltages suddenly applied to the stator be given by Ug cos 6 vy | = WEP cos(# ~ 120) | ule) (4 164) % 0s (0 + 120) where u(t) is a unit step function and V is the rms phase voltage Then from (47) we The Synchronous Machine 123 can show that by 0 v, b= | V5 Vu) 4.165) v, oO d Immediately after the voltage is applied, the flux linkages \, and Ap are still zero, since they cannot change instantly. Thus aty = 0* Ae 0 kMpiy + Lele + Maino kMpig + Mais + Lyin (4 166) Therefore KMpLy — kMoM, kM, kMpM, ~ - KM foMs = — EM Me (4.167) * os Lilp= My 416) Substituting in (4 20) for d,, we get (att = 0°) Ne (: _ RM iLo + LekeMi - 2kMekMoM, « LyLp — Mi The subtransient inductance is defined as the initial stator flux linkage per unit of stator current, with all the rotor circuits shorted (and previously unenergized) Thus by definition ) ia (4.168) Ns Lia (4 169) subtransient reactance. From (4.168) and (4.169) KM) Lo + (kKMp) Ly — 2KMeKMoMx Thy — MR _ Ly + bp ~ 2Lay (LeLo/Lin) -3 where L 4p is defined in (4.108) If the balanced voltages described by (4.164) are suddenly applied to a machine with no damper winding, the same procedure will yield (att = 0°) ip kM y/E dia (in) Dem Ua — Mp? /Lelig = Lota 173) where £3 is the d a Ly a (4.170) =, (4171) where L}is the d axis transient inductance; ic, Ly = Ly = KM P/Ly = by Lolly (aim) Ina machine with damper windings, after a few cycles {rom the start of the transient described in this section, the damper winding current decays rapidly to zero and the effective stator inductance is the transient inductance If the phase of the impressed voltages in (4 164) is changed by 90° (v, = V2 ¥ sin @), vs becomes zero and v, will have a magnitude of V3 V. Before we examine the q axis inductances, some clarification of the circuits that may exist in the axis is needed. For a salient pole machine with amortisseur windings a q axis damper circuit exists, but there is no other q axis rotor winding. For such a ma- chine the stator flux linkage after the initial subtransient dies out is determined by ¢s- 124 Chopter 4 sentially the same circuit as that of the steady-state q axis flux linkage, Thus for a salient pole machine jt is customary to consider the q axis transient inductance to be the same as the q axis synchronous inductance. The situation for a round rotor machine is different “Here the solid iron rotor pro- vides multiple paths for circulating eddy currents, which act as equivalent windings during both transient and subtransient periods. Such a machine will have effective q axis rotor citcuits that will determine the q axis transient and subtransient inductances, Thus for such a machine it is important to recognize that a g axis transient inductance (much smaller in magnitude than L,) exists. Repeating the previous procedure for the q axis citcuits of a salient pole machine, dg = 0 = kMoi, + Lolg (4175) or ig = KMo/Lo)ig (4.176) Substituting in the equation for \y, Lyi + kKMoig (4.177) or Ay = [Le - KMo/Loli, 4 L where £7 is the g axis subtransient inductance Ly = Ly ~ (kMo}/Lg = Ly ~ Bolko (4.179) We can also see that when ig decays to zero alter a few cycles, the q axis effective in- ductance in the “transient period” is the same as L,, Thus for this type of machine (4.178) Loa Ly 4180) Since the reactance is the product of the rated angular speed and the inductance and since in pu a = 1, the subtransient and transient reactances ate numerically equal to the corresponding values of inductances in pu We should again point out that for a round rotor machine Lf < Lj > re, while Ly and L, are of similar magnitude Therefore we can write, approximately, . Mais ip + 4 —Malts 4.186 OT Milly? Lp — MiJ Ly @ 189) Equation (4 186) shows that ip decays with a time constant 2 hy = SOS Mahe (4.187) This is the d axis open circuit subrransient time constant It is denoted open circuit because by definition the stator circuits are open When the damper winding is not available or after the decay of the subtransient current, we can show that the field current is affected only by the parameters of the field circuit; ie, rely + Lois = Veult) (4 188) The time constant of this transient is the d axis transient open circuit time constant ion where tle = Le/re (4.189) Kimbark [2] and Anderson [8] show that when the stator is short circuited, the cor- responding d axis time constants are given by ri = rhLt/Ls (4 190) Ta = TaolalLe (4.191) A similar analysis of the transient in the g axis circuits of a salient pole machine shows that the time constants are given by a = Loto 4192) af = ribLi'/Ly (4.193) For a round rotor machine both transient and subtransient time constants are present Another time constant is associated with the rate of change of direct current in the stator or with the envelope of alternating currents in the field winding, when the ma- chine is subjected to a three-phase short circuit This time constant is 7, and is given by (see {8],Ch 6) te = Lalr (4.194) where L, is the negative-sequence inductance, which is given by Ly = (Lh + £,)/2 (4.195) Typical values for the synchronous machine constants are shown in Tables 43, 4.4, and 45 126 Chapter 4 Table 4.3. Iypical Synchronous Machine Time Constants in Seconds Time ‘Turbogenerators Waterwheel generators Synchronous condensers constant Low Avg. ‘High Low Avg. ‘High Low Avg. High rio 28 56 92 15 (56 95 60 90 us th o4 1h 18 05 18 3300 12020 28 rg = ry 0.02 0035 00S 001 0035 005 002 0035 00s te 0.06 0.16 0.35 0.03.5 0.25 DONT 0.30, Source: Reprinted by permission from Power System Stability vol. 3, by EW Kimbark @ Wiley, 1956 Table 4.4. Typical Turbogenerator and Synchronous Condenser Characteristics Generators Synchronous condensers Parameter Recom- Recom- Range mended == Range mended average average ‘Nominal rating 300-1000 MW 50-100 MVA Power factor 0.80-0.95 0.90 Direct axis synchronous reactance xg 140-180 160 170-270 220 Transient reactance x4 23.35 25 45-65 55 Subtransient reactance xf 15-23 20 35.45 40 ‘Quadrature axis synchronous reactance xy 150-160 155 100-130 us Negative-sequence reactance x2 18-20 19 35-45 40 Zero-sequence reactance x» i214 B 15-25, 20 Short circuit ratio 050-0 72 064 035-065 050 Inertia constant #2, (EW=8) {3600r/min 30-50 40 (kVA) [1800r/min _5.0-8.0 60 : . ‘Source: From the 1964 National Power Survey made by the U.S Federal Power Commission USGPO. Note: All reactances in percent on rated voltage and KVA base. kW losses for typical synchronous condensers in the range of sizes shown, excluding losses associated with step-up transformers, are in the order of I 2-1 5% on rated kVA base. No attempt has been made to show EW losses associated with gen- ferators, since generating plants are generally rated on a net power output basis and losses vary widely de- pendent on the generator plant design Table 4.5. Typical Hydrogenerator Characteristics Smal Lage Parameter tints ri Nominal rating (MVA) 0-40 40-200 Power factor 0 80-0.95 0180-095 Speed (1/min) 70-350 70-200 Inertia constant H, “W-8). 15-40 30-55 (kVA) Dircot axis synchronous reactance x4 90-110 80-100 Transient reactance x3 25-45 20-40 Subtransient reactance x 20-35 15-30 Quadrature axis synchronous reactance xy Negative-sequence reactance 20-45 20-35 Zero-sequence veactanice Yo 10-35 10-25 Short cieuit ratio 1.0-2.0 1.0-2.0 Source: From the 1964 National Power Survey made by the U $ Federal Power Commission. USGPO. ‘Note: All reactances in percent on rated voltage and kVA base. No attempt has been made to show KW losses associated with generators, since generating plants are generally rated on a net power output basis and losses vary widely dependent on the generator plant design '*These power factors cover conditions for generators installed either close to or remote from load cen- ters The Synchronous Machine 127 4.15 Simplified Models of the Synchronous Machine In previous sections we have dealt with a mathematical model of the synchtonous machine, taking into account the various effects introduced by different rotor citcuits, i.e., both field effects and damper-winding effects. The model includes seven nonlinear differential equations for each machine In addition to these, other equations describing the load (or network) constraints, the excitation system, and the mechanical torque must, be included in the mathematical model, Thus the complete mathematical description of a large power system is exceedingly complex, and simplifications are often used in modeling the system Ina stability study the response of a large number of synchronous machines to a given disturbance is investigated. The complete mathematical description of the system would therefore be very complicated unless some simplifications were used, Often only a few machines are modeled in detail, usually those nearest the disturbance, while others are described by simpler models The simplifications adopted depend upon the location of the machine with respect to the disturbance causing the transient and upon the type of disturbance being investigated Some of the more commonly used simplified models are given in this section The underlying assumptions as well as the justifica- tions for their use are briefly outlined In general, they are presented in the order of their complexity Some simplified models have already been presented. In Chapter 2 the classical representation was introduced In this chapter, when the saturation is neglected as tacitly assumed in the current model, the model is also somewhat simplified An ex- cellent reference on simplified models is Young [19] 4.15.1 Neglecting damper windings—the E, model The mathematical models given in Sections 4 10 and 4 12 assume the presence of three rotor circuits Situations arise in which some of these circuits or theit effects can be neglected Machine with solid round rotor {2} The solid round rotor acts as a q axis damper winding, even with the d axis damper winding omitted. The mathematical model for this type of machine will be the same as given in Sections 4 10 and 4.12 with ip or Ap omitted For example, in (4.103) and (4 138) the third row and column are omitted. Amonisseur effects neglected This assumption assumes that the effect of the damper windings on the transient under study is small enough to be negligible. This is particularly true in system studies where the damping between closely coupled machines is not of interest In this case the effect of the amortisseur windings may be included in the damping torque, i e , by increasing the damping coefficient D in the torque equa- tion Neglecting the amortisseur windings can be simulated by omitting ip and ig in (4 103) or Ap and Ag in (4.138). Another model using familiar machine param- eters is given below From (4.118), (4.123), (4.120), and (4.121) with the D and Q cir- cuits omitted, ja} | Cam Luo ti —Lwo/tabe i uoltate (he ~ Luo Lt i 0 (4.196) 128 Chapter 4 or = |=] ----+---|] -- (4.197) iy 0) uel, We can show that £5' is given by ° ep al te hand bite ss) -Leo [Lily LafLile Therefore, the currents are given by ie vin ~Lavf/Lil, 0 | Ay in|=|—Lao/Lale La/Eile 0 IL (4.199) i 0 0 T/L, || dy The above equations may be in pu or in MKS units This follows, since the choice of the rotor base quantities is based upon equal flux linkages for base rotor and stator currents. From the stator equation (4 36) and rearranging, Ay = -rig ~ why — 04 pu (4.200) or from (4.199) and (4 200) Ay = wp, and Lj! = 1,’ Note that while some simplifying as- sumptions are used in this model, the field effects and the effects of the damper cixcuits are included in the machine representation Stator subtransient flux linkages are defined by the equations Made Lig Mo =A, - Lil, (4.230) where Lj and LY’ are defined by (4170) and (4179) respectively Note that (4,230) represents the more general case of (4 169), which represents a special case of zero initial flux linkage. These flux linkages produce EMF’s that Jag 90° behind them. These EMF’ ate defined by er RON = ogdf ef & uM = — a dy (4.231) (See [8] for a complete derivation.) From (4.36) the stator voltage equations, under the assumptions stated above, are given by Ug = —rig — Rdg Uy = Ty + ORY (4.232) Combining (4 230) and (4.232), Uy = mtg ~ wei Le — onde Now from (4 231) and (4.233), vgs mrig ix" tel vy = mri, + ig” + ef (4.234) sri, + opighd + ogdf (4.233) The Synchronous Machine 133 Fig 412. Voltage behind subtransient reactance equivalent where, under the assumptions used in this model, x" 2 gl? = ol} (4235) The voltages ef and ef are the d and g axis components of the EMF e” produced by the subtransient flux linkage, the d and g axis components of which are given by (4230) This EMF is called the voltage behind the subtransient reactance Equations (4.234) when transformed to the a-b-c frame of reference may be repre- sented by the equivalent circuit of Figure 412. If quasi-steady-state conditions are assumed to apply at any instant, the relations expressed in (4.234) may be tepresented by the phasor diagram shown in Figure 4 13. In this diagram the q and d axes represent the real and imaginary axes respectively. “Projections” of the different phasors on these axes give the q and d components of these phasors. For example the voltage £" is repre- sented by the phasor F” shown Its components are £7 and Ej respectively From the above we can see that if at any instant the terminal voltage and current of the ma- chine are known, the voltage E” can be determined, Also if Ey and £/ are knowa, E” can be calculated; and if the current is also known, the terminal voltage can be deter- mined We now develop the dynamic model for the subtransient case. Substituting (4 230) into (4.134), we compute If) Law Li |b aye [ abe ( - wah + Bekt 4, » Bett y, (4236) We can show that _ fp Hines + to) sir ; - [- Tite t Flt +09) 7 4 Gan Fig 413 Phasor diagram for the quasi-static subtcansient case 134 Chapter 4 since by definition » Lio(tr + F0' Wo ba AT Therefore we may write (4.236) as Mo= (LE Luo/ 2a te)de + (LE Luo! tao) do Using (4 203), we can rewrite in terms of E; as Mo = (Li Lwole/ te PeLan) V3 Ey + (Li Luo/ ta fo)do Now we can compute the constants 2 Likwols, Li ~ 44 Xd = Xe aE Fy xy =x =1-M%=*% 1k xi x Substituting in (4.240) and using (4.231), we compute in pu ef = OF — 1 fg — XUV TE, ~ Ay) +o Similatly from (4 230) and (4.104), Mi = (Lyig + Lago) ~ Lyi = (Ly - Li Vig + Lagi which can be substituted into (4 231) to compute ef = 0, 2), = ee where we define the voltage €y = @RLagig We can also show that Mom A = Lyi = (Lag/Lo)e (4.238) (4.239) (4.240) (4241) (4.242) (4.243) (4.244) (4 245) (4 246) (4.247) Now from the field flux linkage equation (4.104) in pu, we incorporate (4.203) and (4.226) to compute Ew By (xp ~ xViy + io) V3 From the definition of Lj (4.174) we can show that Lg ~ ha = Lolly We can also show that (Ly ~ LOIS ~ 46¥ = bef php ~L30) Then from (4 104) in pu Xo = Lats + Lavin + Loin Xe = Lavig + Le in + Lavin Eliminating i, from (4 251), (4.248) (4.249) (4.250) (4251) (4.252) The Synchronous Machine 135 Now substituting (4.203), (4 249), and (4.250) into (4 252), VIE, = dy = (La ~ Ls = Landis ~ § by (4253) which can be put in the form ip = 2224 In, — VIE) — x) - uid) (4.254) Gs In addition to the above auxiliary equations, the following differential equations are obtained From (4.36) we write — ay roip + hp = 0 (4.255) Substituting (4.187) and (4 250) in (4.255), (xg = xe ho = - aoe ip (4256) Similarly, from (4.36) we have rolg + which may be written as lop rolLuo/Lollio + Wer Lagh{Lolho = 0 4257) Now from (4.246), (4.247), (4.231), (4.192), and (4.257) we get the differential equation ef = eafth (4258) The voltage equation for the field circuit comes from (4.36) Up = Trip +e (4.259) which can be put in the same form as (4 228) thE, = Ep — E (4.260) where E is given by (4.248) Equations (4 256), (4 258), and (4.260) give the time rate of change Ap, ef, and Ez in terms of ip, e,. and £ The auxiliary equations (4 245), (4.248), and (4 254) re- late these quantities to i, and i,, which in turn depend upon the load configuration The voltage ey’ is calculated from (4 243). To complete the model, the torque equation is needed From (4.95), Teg = tga — tahy By using (4 230) and recalling that in this model it is assumed that L} = 22, Tyg = ig - WN (4261) and if @ in pu is approximately equal to the synchronous speed, (4.261) becomes Teg = Cd iy + ef ig (4.262) If saturation is neglected, the system equations can be reduced to the following: ~te¢-4Le,- xy, (4.263) Ta 770 136 Chapter 4 ho- bv3e-4tay+to- iy (4.264) Ta Tao Ta0, + SERGE =), VE f) Ga = dG Tass — 5) Tie @- x) Dod =) Teo(Xg — Xe)? Now from (4.243) and using K, and K, as defined in (4.241) and (4 242) respectively, we may write + (4 265) ey = WOK, Ey + Kzo (4.266) To complete the description of the system, we add the inertial equations & = (fn) Ta ~ eblg/34 ~ fe Br, - Dory (4267) bew-1 (4.268) The currents i, and i, are determined from the load equations. The block diagrams for the system may be obtained by rearranging the above equa- tions In doing so, we eliminate the ¥/3 from all equations by using the rms equivalents, similar to (4 212), Ap = dof VIE" = ef VF = Ef + jEY 4.269) Then (4 263)-(4.266) become (+ THEE =~, XL, (4 rhs) Ay = Eh + Oh — xe (+ ro Ey = Exp — Kak + %ssly + Kohn EX = KE, + Kydp (4.270) r 7 b—fgny [ig 5 be Si tt oe — % ete Ty Fig. 414 Block diagram for the £" model The Synchronous Machine 137 Fig 415 Block diagram for computation of torque and speed in the £" model where we have defined Ke xf — x, K xy Moy @y — Kel RK, oxy = Get AGE = x4) (4271) x x The block diagram for (4.270) is shown in Figure 4 14 The remaining equations are given by (D + a)o = Ty — (Eth, + Ef) sb = w — 1 (4272) The block diagram for equation (4.272) is given in Figure 4 15. Also the block diagram of the complete system can be obtained by combining Figures 4.14 and 4.15 If saturation is to be included, a voltage increment £4, corresponding to the in- crease in the field current due io saturation, is to be added to (4 248), E = El + Ey ~ (e — xlig + ip)/V3_ (4273) Example 46 Use the machine data from Examples 4.1-4 $ to derive the time constants and gains for the £” model Solution The time constant rf = 003046 s = 72 149 rad is already known from Exam- ple 4.5. For the £” model we also need the following additional time constants From (4.192) the q axis subtransient open circuit time constant is rly = Lofro = 1.423 x 109/18 969 x 10°? = 0075s = 28.279 rad which is about twice the d axis subtransient open circuit time constant We also need the d axis transient open circuit time constant. It is computed from (4.189). Ty = Le[ts = 2189/0371 = 990s = 222425 rad Note that this time constant is about 30 times the subtransient time constant in the d 138 Chapter 4 axis This means that the integration associated with jp will be accomplished very fast. compared to that associated with Tp To compute the gains, the constant xj or Lj is needed It is computed from (4.174): Li = Ly — Lip/Ly = 1.10 ~ (155)/1.651 = 0.245 pu We can now compute from (4 271) = aXe, 0.185 ~ O15 _ 9 368 K xj~ x, 0245 — 015 K, = 1 ~ K, = 0.632 (ge = xy)(xi — xf) _ (1,70 — 0.245)(0.245 — 0.185) K, = Sea a — a) | O70 OOO) = 9.673 “ Ga ~ xy (0.245 — 0.150" , = (a= NGG = 24) _ (1.70 — 0.245)(0.185 = 0.150) _ 4 55, Xe yak 0.245 ~ 0150 0536 From (4.179) we compute Ly = L, ~ Lig/Le = 1.64 ~ (1499/1526 = 0.189 pu Then, from (4.270), we compute the gain, x, ~ x7 = 1.64 - 0185 = 1455 pu ing iy and X, for a cylindrical rotor machine—the two-axis model In the two-axis model the transient effects are accounted for, while the subtransient effects are neglected [18] The transient effects are dominated by the sotor circuits, which are the field circuit in the d axis and an equivalent circuit in the q axis formed by the solid rotor An additional assumption made in this model is that in the stator voltage equations the terms A, and }, are negligible compared to the speed voltage terms and that o> &, = 1 pu The machine will thus have two stator circuits and two rotor circuits. However, the number of differential equations describing these circuits is reduced by two since hy and }, ate neglected in the stator voltage equations (the stator voltage equations are now algebraic equations) The stator transient flux linkages are defined by MEAN bhi NE (4274) and the corresponding stator voltages are defined by ey 8 -wh, = —ogd; 1S why = wry (4.275) Following a procedure similar to that used in Section 4 15.2, Ug = rig — welt, + ef Ug = —ri, + whi + &} (4 276) or ep = Uy + rig + Hei, + (x, — XDA, (4277) f= Dy riy — by (4.278) Since the term (x; — 4})é, is usually small, we can write, approximately, eS Uy + re + xy (4279) The Synchronous Machine 139 + 5? Fig 416 Transient equivalent circuit of a generator The voltages ¢; and e are the g and d components of a voltage e' behind transient re- actance Equations (4.279) and (4 278) indicate that during the transient the machine can be represented by the circuit diagram shown in Figure 4.16 It is interesting to note that since e) and ef are d and q axis stator voltages, they represent times the equiva- lent stator rms voltages For example, we can verify that e; = V3Ej, as given by (4.203) Also, in this model the voltage e’, which corresponds to the transient flux link- ages in the machine, is not a constant Rather, it will change due to the changes in the flux linkage of thed and g axis rotor circuits We now develop the differential equations for the voltages ¢} and ef The d axis flux linkage equations for this model are Na Leia + Laois PY Me = Lavla + Lely pu (4.280) By eliminating i, and using (4.174) and (4 203), Ne — VEE, = Lily pu and by using (4275), = VE; pu (4.281) Similarly, for the g axis Ay = Lele + Lagig PU Ng = Lagi, + Loig pu (4.282) Eliminating ip, we compute Ay — Cao/ Lodo = (La - Hro/Lodiy pu (4.283) by defining Ly = Lg/ke pu (4284) and by using (4.284) and (4.275) we get ey 8 VIE, = —(Lig/La)dy pu (4.285) We also define VIE = Levis pu VI Eg = eg = Lagig pu (4.286) We can show that [8], 4 y+ tl, (4.287) From the Q circuit voltage equation rig + dAg/dt = 0, and by using (4282) with (4.286), Bt wily = Bet ile Eat xd tphs = Ey - (% - x) (4.288) where, for uniformity, we adopt the notation 140 Chanter 4 Fig 417 Block diagram representation of the two-axis model Ho = th = Lolto (4.289) ‘Similarly, from the field voltage equation we get a relation similar to (4.228) b= | Ey - BD (4.290) Too Equations (4 288), (4 290), and (4 287) can be represented by the block diagram shown in Figure 417. To complete the description of the system, the electrical torque is ob- tained from (4.95), Tig = Adg ~ Ayla» Which is combined with (4.274) and (4275) to compute 1. =. le + Bil, ~ (Ly — Lid lily (4291) Example 4.7 Determine the time constants and gains for the two-axis model of Figure 4.17, based on the machine data of Examples 4 1-46 In addition we obtain from the manu- facturer’s data the constant xj = 0380 pu Solution Both time constants are known from Example 4.7. The gains are simply the pu re- actances x, — x) = 164-0380 = 1260 pu x, ~ x = 170 = 0245 = 1455 pu The remaining system equations are given by Ty — Deo ~ [Bly + Ej, ~ (Lh - LOL) b-w-1 (4292) The block diagram for (4 292) is shown in Figure 4.18 By combining Figures 4.17 and 4 18, the block diagram for the complete model is, obtained Again saturation can be accounted for by modifying (4.287), Ew Ey a lat Es (4.293) where E, is a voltage increment that corresponds to the increase in the field current due to saturation (see Young [19]) The procedure for incorporating this modification in the block diagram lar to that discussed in Section 4 15.2 The Synchronous Machine 141 Fig 418 Block diagram reprasentation of (4 292) 4.15.4 Neglecting amortisseur effects and 1, and i, terms—the one-axis model This model is sometimes referred to in the literature as the one-axis model. It is similar to the model presented in the previous section except that the absence of the Q cirouit eliminates the differential equation for £5 or ¢) (which is a function of the current ig). The voltage behind transient reactance e” shown in Figure 4 16 has only the component e; changing by the field effects according to (4 290) and (4.293) The component ¢ is completely determined from the currents and vy. Thus, the system equations are tok = Ey -E pu E = Ej ~ (xe - xi)ly pu (4294) ‘The voltage E} is obtained from (4 36) with dy = 0, and using (4.274) and (4 275), Ey Vet yl try pu (4.295) The torque equation is derived from (4.95), T,, = Aig — Agi Substituting (4 274) and Fig 419 Block diagram representation of the one-axis model 142 Chapter 4 noting that, in the absence of the Q circuit, A, = Lyiy, Ty = Ful, Ug ~ Lilly Pu (4.296) Thus the remaining system equations are 7 = Ty ~ Deo ~ (Eig ~ (Ly ~ Lila} pu b= @-1 pu (4.297) The block diagram representation of the system is given in Figure 4.19. 4.15.5 Assuming constant flux linkage in the main field winding From (4.228) we note that the voltage £{, which corresponds to the d axis field flux linkage, changes at a rate that depends upon ry This time constant is on the order of several seconds The voltage E,, depends on the excitation system characteristics. If Ey does not change very fast and if the impact initiating the transient is short, in some cases the assumption that the voltage E; (or e?) remains constant during the transient can be justified Under this assumption the voltage behind transient reactance £' or e" has a q axis component £; or e/ that is always constant, The system equation to be solved is (4.296) with the network constraints (to determine the currents) and the condi- tion that £; is constant The next step in simplifying the mathematical model of the machine is to assume that £; and £' are approximately equal in magnitude and that their angles with respect to the reference voltage are approximately equal (or differ by a small angle that is con- stant). Under these assumptions E' is considered constant. This is the constant voltage behind transient reactance representation used in the classical model of the synchronous machine Example 4.8 The simplified model used in Section 4 15.2 (voltage behind subtransient reactance) is to be used in the system of one machine connected to an infinite bus through a trans- mission line discussed previously in Section 4.13. The system equations neglecting saturation are to be developed Solution For the case where saturation is neglected, the system equations are given by (4 263)-(4 268), This set of differential equations is a function of the state variables ef, Yo» Ey, w, and 6 and the currents i and 4, Equation (4.266) expresses ef as a linear combination of the variables Ey and Ap For the mathematical description of the system to be complete, equations for i, and i, in terms of the state variables are needed. These equations are obtained from the load constraints From the assumptions used in the model, ie, by neglecting the terms in A, and \, in the stator voltage equations (compared to the speed voltage terms) and also by as- Fig 4.20 Network representation of the system in Example 48 The Synchronous Machine 143 suming that © 2 ea, the system reduces to the equivalent network shown in Fig- ure 4.20, By following a procedure similar to that in Section 4 15 2, equations (4 234) are given by Vig = Ri, + kh, + EF (4.298) where RareR kh axe x, (4299) and Vig = ~V3V,sin(6 — a) v, 4 = V3Vcos(é ~ a) (4.300) From (4.298) I, and J, are determined —_—_!_ (RY + 'P 1 (RF + ry Equations (4 147) and (4.301) along with the set (4.263)-(4 268) complete the mathe- matical description of the system [-RW ag = BA) + P"aq = EY) ey — BD + RW, -— ED (4301) 4.16 Turbine Generator Dynamic Models The synchronous machine models used in this chapter, which are in common use by power system engineers, ate based on a classical machine with discrete physical windings on the stator and rotor, As mentioned in Section 4.14, the solid iron rotor used in large steam turbine generators provides multiple paths for circulating eddy cur rents that act as equivalent damper windings under dynamic conditions The represen- tation of these paths by one discrete circuit on each axis has been questioned for some time. Another source of concern to the power engineer is that the value of the machine constants (such as Lj, LJ, ete.) used in dynamic studies are derived from data ob- tained from ANSI Standard C42.10 [16] This implicitly assumes two rotor circuits in each axis—the field, one d axis amortisseur, and two g axis amortisseurs This in turn implies the existence of inductances Ly, Lj Lf, Ly Lf, and L,! and time constants 7%, Ths Ty and 1%, all of which are intended to define fault current magnitudes and decte- ments, In some stability studies, discrepancies between computer simulation and field data have been observed. It is now suspected that the reason for these discrepancies is the inadequate definition of machine inductances in the frequency ranges encountered in stability studies Studies have been made to ascertain the accuracy of available dynamic models and data for turbine generators [21-25]. These studies show that a detailed representation of the rotor circuits can be more accurately simulated by up to three discrete rotor circuits om thed axis and three on the g axis Data for these circuits can be obtained from fre- quency tests conducted with the machine at standstill To fit the “conventional” view of rotor circuits that influence the so-called subtransient and transient dynamic behavior of the machine, it is found that two rotor circuits (on each axis) are sometimes adequate but the inductances and time constants are not exactly the same as those defined in IEEE Standatd No. 115 ‘The procedure for determining the constants for these circuits is to assume equiva- 144 Chapter 4 lent circuits on each axis made up of a number of circuits in parallel, The wansfer func- tion for each is called an operational inductance of the form L(s) = ING)/DONL (4.302) where Z is the synchronous reactance, and N(s) and D(s) are polynomials ins Thus for the d axis we write C+ ays) + bys) + es) LEGO AS + €18) 4.303) C+ as) + bys) + es) i ) and the constants £4, a), @2, b,, 6, ¢, and ¢, are determined from the frequency do- main response If the operational inductance is to be approximated by quadratic polynomials, the constants can be identified approximately with the transient and subtransient param- eters, Thus, for the d axis, £,(s) becomes pt Calta rigs + (Li [Livros (4304 (1 + rYos)l + Tas) The time constants in (4.304) are different {rom those associated with the exponential decay of d or q axis open citcuit voltages, hence the discrepancy with [EEE Standard No. 115, An example of the data obtained by standstill frequency tests is given in [24] and is reproduced in Figure 4.21, Both third-order and second-order polynomial representa tions are given Machine data thus obtained differ {rom standard data previously ob- tained by the manufacturer from short circuit tests Reference (24) gives a comparison’ between the two sets of data for a 555-MVA turbogenerator, This comparison is given in Table 4.6 LAs) = L, 1,6) = Speed, pu ,90001___0.0001, 0.001 9.01 9.10 1.0 39 & Sg 3 Pa teyang 9008 So arahrsas od : 3 04 ! BO gy team +009 4 +o. ode eee sagas + O0TEY om Frequency response plots 555 5-MVA unit ee ore TE ied ots for sinlton of Avid ass a ‘S008 0 08 0.08 08 6 0 Frequency, He Fig 421 Frequency response plot for a 555-MVA turboalternator (@ IEEE Reprinted {rom IEEE Trans vol PAS-93, May/June 1974 ) The Synchronous Machine 145 Table 4.6. Comparison of Standard Data with Data Obtained from Frequency Tests for a 555-MVA turboalternator Constants Standard data Adjusted @ Za pu 197 Ly pu 027 Lj pu 0175 Ly pu 1.867 iy pu 0.473 Ly pu 0213 Ly pu 016 th 8 43 th oS 0031 oS 056 ro S 0.061 Source:o IEEE Reprinted from JEBE Trans vol PASS 1974 The inductance versus frequency plot given in Figure 4.21 is nothing more than the amplitude portion of the familiar Bode plot with the amplitude given in pu rather than in decibels. The transfer functions plotted in Figure 421 can be approximated by the superposition of multiple first-order asymptotic approximations, If this is done, the break frequencies should give the constants of (4 304). The machine constants thus ob- tained are given in the third column of Table 46 If, however, the machine constants obtained from the standard data are used to obtain the breakpoints for the straight-line approximation of the amplitude-frequency plots, the approximated curve does not pro- vide a good fit to the experimental data For example, the d axis time constant 7% of the machine, as obtained by standard methods, is 4.3 s If this is used to obtain the first break frequency for log |1/(1 + 7493)J, the computed break frequency is \fthy = 1/43 = 02326 rad/s = 0.00062 pu (4305) The break point that gives a better fit of the experimental data corresponds to a frequency of 0 1282 rad/s or 0.00034 pu. Since the amplitude at this frequency is the reciprocal of the d axis transient time constant, this corresponds to an adjusted value, denoted by zy, given by rif = 1/0.1282 = 78 s (4306) Reference {24] notes that the proper ajustment of ria, 7jo. and Ly are all particu- larly important in stability stu ‘A study conducted by the Northeast Power Coordinating Council (26] concludes that, in general, it is more important in stability studies to use accurate mackine data than to use more elaborate machine models Also, the accuracy of any dynamic ma- chine model is greatly improved when the so-called standard machine data are modified to match the results of a frequency analysis of the solid iron rotor equivalent circuit At the time of this writing no extensive studies have been reported in the literature to support or dispute these results Finally, a comparison of these results and the machine models presented in this chapter are in order. The full model presented here is one of the models investigated in the NPCC study [26] for solid rotor machines. It was found to be inferior to the more 146 Chapter 4 elaborate model based on two rotor windings in each axis This is not surprising since the added detail duc to the extra g axis amortisseur should result in an improved simula- tion, Perhaps more surprising is the fact that the model developed here with F, D, and Q windings provided practically no improvement over a simplet model with only F and Qwindings Furthermore, with the F-Q model based on time constants tj and r/o, larger digital integration time steps are possible than with models that use the much shorter time constants 7% and 7/5, as done in this chapter Asa general conclusion itis apparent that additional studies are needed to identify the best machine data for stability studies and the proper means for testing ot estimating these data This is not to imply that the work of the past is without merit. The tra~ ditional models, including those developed in this chapter, are often acceptable But, as in many technical areas, improvements can and are constantly being made to pro- vide mathematical formulations that better describe the physical apparatus Problems. 41 Park's transformation P as defined by (4.5) is an orthogonal transformation Why? But the transformation Q suggested originally by Park (10, 11] is that given by (4.22) and is not orthogonal. Use the transformation Q to find voltage equations similar to (4 39) 42. Verify (4 9) by finding the inverse of (4 5) 43 Verify (4.12) by sketching the stator coils as in Figure 4.1 and observing how th tance changes with rotor position 44 Verily the following equations: (a) Equation (4 13) Can you explain why these inductances are constant? (b) Equation (4 14) Why isthe sign of M, negative? Why is |_M,|_ > Z,? (©) Explain (4.15) in terms of the coefficient of coupling of these coils. 45 Verify (4.16)-(4 18) Explain the signs on these equations by referring to the currents given on Figures 411 and 4 2 46 Verify (4.20) 47 Explain the signs on all terms of (4 23). Why is the A term negative? 48 Consider a machine consisting only of the phase winding sa-fa shown in Figure 4.1 and the field winding F Sketch @ new physical arrangement where the field flux is stationary and coil sa-fa turns elockwise Are these two physical arrangements equivalent? Explain 49 For the new physical machine proposed in Problem 4 8 we wish to compute the induced EMP in coil sa-fa Do this by two methods and compare your results, including the polarity of the induced voltage {a) Use the rate of change of flux linkages A (b) Compute the Blv or speed voltage and the transformer-induced voltage. Do the results agree? They should! 410 Verify (4 24) for the neutral voltage drop 411 Check the computation of PP~! given in (4.32) 412 The quantities hy and 2, are given in (4 20) Substitute these quantities into (4.32) and compute the speed voltage terms Check your result against (4 39) 413. Verify (4.34) and explain its meaning 414 Extend Table 4.1 by including the actual dimensions of the voltage equations in an MLye system Repeat for an FL2Q system AIS Let a(t) = Vy COS (wont + &) ¥,,cos (wat + « ~2/3) pcos (ugt + a + 2x/3) (a) For the pu system used in this book find the pu voltages vj and v, as related to the sms voltage (©) Repeat part (a) using a pu system based on the following base quantities: Sy = three- phase voltampere and Vy = line-to-line voltage (©) For part (b) find the pu power in the d and q circuits and igand jin pu 416 47 418 419 420 421 422 423 424 426 427 4.28 4.29 430 The Synchronous Machine 147 Using the transformation Q of (4.22) (originally used by Patk) and the MKS system of units (volt, ampere, ete ), find: (a) Thed and g axis voltages and currents in relation to the rms quantities, (b) Thed and g axis circuit power in relation to the three-phase power ‘Normalize the voltage equations as in Section 4.8 but where the equations are those found from the Q transformation of Problem 4.1 Show that the choice of a common time base in any coupled circuit automatically forces the equality of VA base in all circuit parts and requires that the base mutual inductance be. the geometric mean of the self-inductance bases of the coupled windings; ie Si = Sp Mon = (LypLap)'? Show that the constraint among base currents (4 54) based upon equal mutual flux linkages is the same as equal MMF"s in cach winding, Show that the I/wg factors may be eliminated from (4.62) by choosing a pu timer ~ wpe rad Develop the voltage equations for a cylindrical rotor machine, ie, a machine in which the inductances are not a function of rotor angle except for totor-stator inductances that are as given in (4 16)-(4 18) Consider a synchronous generator for which the following data are given: 2 poles, 2 slots/pole/phase, 3 phases, 6 slots/pole, 12 slots, 5/6 pitch. Sketch the slots and show two coils of the phase a winding, coil | beginning in slot 1 (0°) and coil 2 beginning in slot 7 (180°), Label coil | saj-fa, (start a and finish a,) and coil 2 s@s-fay Show the position of Nand S salient poles and indicate the direction of pole motion, Now assume the machine is operating at | 0 PF (internal PF) and note by + and notation, looking in at the coil ends, the direction of currents at time fo, where at fo fy = tmx ty = —(/2Minox ie = = (1/2 nae Plot the MMF as positive when radially outward +4, enters sa, and +i, enters sb, but +4, enters fe, Assume the MME changes abruptly at the center line of the slot | The MMF wave should be a stepwise sine wave Is it radially outward along dor q? Verify (4 138) Derive formulas for computing the saturation function parameters A, and B, defined in (4.141), given two different values of the variables Xap faro, and fas Compute the saturation funetion parameters 4, and B, given that when Aap = V3, (ines — iao)/ino = 0.13 Map = 123, (tus — 1 2iwo)/} Zin = 0.40 where vs and Io correspond to Ayy = V3 and sis is the saturated current at Ayo = 12Vv8 Compute the saturation function K, at Xo = 1.8, using the data and results of the previous problem Let yor = 08 V3 The synchronous machine described in Examples 4 2 and 4 3 is connected to a resistive load Of R, = 10 pu Derive the equations for the state-space current model using v, and Tyas forcing functions. Use the current model Repeat Problem 4 27 using the flux linkage model Derive the state-space model for a synchronous machine connected to an infinite bus with a focal load at the machine terminal The load is to be simulated by a passive resistance Repeat Problem 4 29 for a focal toad simulated by a passive impedance The load has a reactive component Obtain the state-space model for a synchronous machine connected to an infinite bus through a series resistance, inductance, and capacitance. Hint: Add two state variables related to the voltage (or charge) across the capacitance, Incorporate the load equations for the system of one machine against an infinite bus (shown in Figure 4 8) in the simplified models giver in Section 4 15: (a) Neglecting damper effects. 148 Chapter 4 (b) Neglecting Ay and i, for « machine with solid round rotor (G) Neglecting damper effects and the terms Xyand ky. 433. Show that the voltage-behind-subtransient-reactance model of Figure 4 14 can be tear- ranged to give the model of Schulz [20] given in Figure P4 33 %,-% a ro [Fam 0% Tg) Fig P433 434 Using the third-order transfer functions for £4(s) and £,(3) given in Figure 4.21, sketch Bode diagrams by making straight-line asymptotic approximations and compare with the given test results, 435 Repeat Problem 4 34 using the second-order transfer functions for £4(s) and L,(s) 436 Repeat Problem 4 35 using the second-order transfer functions of (4.304) and substituting the standard data rather than the adjusted data References 1 Concordia, ©. Synchronous Machines. Wiley New York, 1951 2. Kimbark, E,W Power Spstem Stability, Vols 1,3 Wiley, New York. 1956 3 Adkins. B The General Theory of Blectrical Machines Chapman and Hall, London 1964 The Synchronous Machine 149 4 Craty $B, Power System Stability Vols 1,2. Wiley New York, 1945. 1947 5 Lynn, T Wand Walshay, MH Tensor Anaivsis of @ Synchronous Two-Machine System IEE (British) Monograph Cambridge Univ Press. London, 1961 6 Taylor,G D Analysis of Synchronous Machines Connected to Power Network EE (British) Mono- graph Cambridge Univ. Press London, 1962 7 Westinghouse Electric Corp Electrical Transmission and Distribution Keference Book Pittsburgh Pa, 1980. 8 Anderson PM. Anabsis of Faulted Power Systems. ow State Univ. Press Ames, 1973 9 Harris M_R. Lawrenson. PJ and Stephenson, 1M. Per Unit Systems With Special Reference to Electrical Machines VEE (British) Monograph Cambridge Univ. Press, London, 1970. 10 Park. RH Two reaction theory of synchronous machines PUI. AVE Trans. 48:716-30 1929 11 Park, RH, Two teaction theory of synchronous machines, Pt 2 AZEE Trans. 52:382-58. 1933 12 Lewis, W.A, A basic analysis of synchronous machines, Pt 1. ATER Trans PAS-77:436-55, 1958 13, Krause P.C. and Thomas CH” Simblation of symmetrical induction machinery IEEE Trans PAS- 84:1036-52, 1965 14 Prentice, B. R- Fundamental concepts of synchronous machine reactances. AEE Trans $6(Suppl 1) 76-20 1939 Is Rankin, A.W. Per unit impedances of synchronous machines AVEE Trans, 64:569-72, 839-41, 1945, 16 IEEE. Test procedures for synchronous machines. Standard No 115, March, 1965 17 IEEE Committee Report Recommended phasor diagram lor sychronous machines EEE Trans PAS-86:1593-1610 1969 18 Prubuaskankat K.. and Janischewskyj W. Digital simulation of multimachine power systems for stability studies. ZEEE Tram PAS7.73-80, 1968 19 Young, CC. Equipment and system modeling for large-scale stability studies IEEE Trans PAS~ 91:99-109 1972 20 Schule RP Synchronous machine modeling Symposium on Adequacy and Philosophy of Modeling System Dysamic Pesformance IEEE Publ 78 CH O970- PWR. 1975, 21 Jackson, W. Band Winchester. RL. Direct and quadrature axis equivalent circuits for solid-roter tur- bine generators TEBE Trans PAS-88:1121. 36, 1969 22. Schulz R P., Jones. WD. and Ewart DN. Dynamic models of turbine generators derived from solid rotor equivslent cicuits, ZEEE Trans PAS-92926-35. 1973 28 Wetson, Wand Manchu G_ Synchronous ouachine operational impedunces from low voltage sca urements atthe stator terminals. [EEE Trans PAS-93:777-84, 1974 24 Kundur Pend Dandeno. P-L Stability peviormance of 353 MVA turboalternators-Digital com parisons with system operating tests IEEE Trans. PAS-93767-16. 1974 25 Dandeno, PL, Hauth. RL. and Schulz, RP Effects of synchronous machine modeling in large- scale system studies JELE Trans PAS-92:374-82. 1973 26 Northeast Power Coordinating Council. Elects of synchronous machine modeling in large-scale sys- {em studies Final Report, NPCC-10 Task Force on System Studies System Dynamic. Simulation Techniques Working Group 1971 chopter The Simulation of Synchronous Machines 5.1, Introduction This chapter covers some practical considerations in the use of the mathematical models of synchronous machines in stability studies Among these considerations are the determination of initial conditions, determination of the parameters of the machine from available data, and construction of simulation models for the machine In all dynamic studies the initial conditions of the system are required This in- cludes all the currents, flux linkages, and EMF’s for the different machine circuits. The number of these circuits depends upon the model of the machine adopted for the study The initial position of the rotor with respect to the system reference axis must also be known. These quantities will be determined from the data available at the terminals of the machine The machine models used in Chapter 4 require some data not usually supplied by the manufacturer. Here we show how to obtain the required machine parameters from typical manufacturer's data The remainder of the chapter is devoted to the construc tion of simulation models for the synchronous machine, Both analog and digital simulations are discussed 5.2. Steady-State Equations and Phasor Diagrams The equations of the synchronous machine derived in Chapter 4 are differential equations that describe machine behavior as a function of time When the machine operates in a steady-state condition, differential equations are not necessary since all variables are either constants or sinusoidal variations with time. For this situation phasor equations are appropriate, and these will be derived. It is common to tacitly assume all machines to be in a steady-state condition prior to a disturbance, The so- galled “stability study” examines the system behavior following the disturbance The phasor equations derived here permit the solution of the initial conditions that exist prior to the application of the disturbance, This is a necessary part of any stability investigation From (4.74) at steady state all currents are constant or, mathematically, isis 61) Then from (4 74) 0 = ior 0 igre 62 Simulation of Synchronous Machines 151 or at steady state ip = ig = 0 (53) Using (5.1) we may write the stator voltage equation from (4 74) as Oy = —Fig — Wl gig Vy = rig + OL gig + KM wiz G4) From (4.5) with balanced conditions, ») = 0 Therefore, from (4 9) we may compute v4 = VIP(vicosO + v, sind) (55) where by definition @ = wa! + 6 + +/2, Then from (5.4) and (5 5) VIP (ria + @Ly1,) COS (wnt + 6 + 4/2) +(-riy + WL yig + kMpwig)sin (wat + 6 + #/2)) = V273[-(is + wLgi,)cos (wat + 6 + x/2) + (rig + OL gig + kMpwiz) 008 (wR! + 8) (6.6) At steady state the angular speed is constant, w = wg, and wl. products may be de- noted as reactances, or ol, els =X (5.7) From (4.226) we also identify won Myig = VBE (58) where £ is the stator equivalent EMF corresponding to is Using phasor notation,! the ¥/2 multiplier of (5.6) is conveniently used to define the rms voltage phasor “Jb + EB (5.9) where the superior bar indicates a total phasor quantity in magnitude and angle (a com- plex number) By using the relation j = 1 /x/2in (59), ve (best Hi) Gy bean xe Rea -(pbtisg ling Bt wig Bt Eb (5.10) Note that in this equation ¥, and £ are stator rms phase voltages in pu, while i and i, are de currents obtained trom the modified Park transformation. The choice of this particular transformation introduced the factor 1/3 in the equation To simplify the notation we define the rms equivalent d and g axis currents as let ifV3 yb tiVF 61D The stator current i, expressed as a phasor will have the two rectangular components J, and 1, Thus if the phasor reference is the q axis, T= 1, + ila (5.12) 1 We define the phasor 4 = Ae!" as 3. complex umber dg Felted fo the socesponsing time do- main quantity a(1) by the relation a(t) = Re (VFde") = VTA cos (w! +a) 152 Chapter 5 gon Fig 51 Phasor diagram representing (5 14) Substituting (5 12) and (5.11) in (5 10) and rearranging, El = Va + rhe + ix ly (B ~ xabe lB (513) and by using E = £/8,1, = 1,/8, and Ty = jal, BaV, +00, + ix, + jxle (5.14) The phasor diagram representing (5 14) is shown in Figure 51 [1] Note that the phasor jx,J, leads the g axis by 90° The phasor jx,I, makes a 90° angle with the nega- tive d axis since [, is numerically negative for the case illustrated in Figure 51 To ob- tain v, and v, from (5.4), we compute the rms stator equivalent voltages Vz % vy/V3 = ~rly — thy VY, & vJV9 =~, + Xale +E (S18) Note that V, and V, are the projection of ¥, along the q and d axes respectively ‘Also note that in the phasor diagram in Figure 5.1 both Vj and J, are illustrated as negative quantities. Thus the magnitude of ri, is subtracted from x,J, to obtain the magnitude of Vy This situation is shown in Figure 5 1 since lagging current (nega- tive J,) is commonly encountered in practice Examining Figure 5.1 and (5.15), we note that if the angle 6 is known the phasor diagram can be constructed quite readily If the position of the g axis is not known but the terminal conditions of the machine Fig 52. Location of the g axis from a known terminal cutrent and voltage ‘Simulation of Synchronous Machines 153 are given (ic., if ¥, %, and the angle between them are known), construction of the phasor diagram requires some manipulation of (5 15) However, an alternate pro- cedure for locating the position of the g axis is illustrated in Figure 5.2, where it is assumed that ¥,, J,, and the power factor angle are known, Starting with V, (used here as reference) the voltage drop 7/, is drawn parallel to T,. Then the voltage drop jx,J, is added (this is a phasor perpendicular to 7,). The end of that phasor (Ey, in Figure 5 2) is located on the g axis, This can be verified by noting that the d axis component of the phasor jx,T, is x,J,, which is similar to that shown in Figure $1 Its q axis component however is x,[,, Which is different from that shown in Figure 5.1 Thus to locate the phasor F in Figure 5 2, we add the phasor (xy — x,)Iy to the phasor E, 5.3 Machine Connected to an Infinite Bus through a Transmission Line To illustrate more fully the procedure for finding the machine steady-state condi- tions, we solve the simple problem of one machine connected to an infinite bus through a transmission line Although this one-machine problem is far simpler than actual systems, it serves well to illustrate the procedure of finding initial conditions for any machine As we shall see later, this simple problem helps us concentrate on concepts without becoming engulfed in details The differential equations for one machine connected to an infinite bus through a transmission line with impedance Z, = R, + joel, is given by (4 149) Under bal- anced steady-state conditions with zero derivatives, (4 149) becomes vg = — V3Vasin(8 — a) + Reig + oheiy by = V3V, 008 (6 = a) + Reig — wLeie (5.16) Substituting for v, and v, from (5 4) into (5.16), mrig — wlgig = — V3, sin(5 — a) + Reig + Lei, mri, + wlsig + KMywis = V3V. cos — a) + Reiy — wh cig By using (5 7) and (5 11) and rearranging the above equations, we compute Ea Vicos(5 - a) +0 + Rly ~ (es + XM 0 = —V.sin(S — @) + + Ry + Oy + XO 617) where X, = wl, Equations (5 17) represent the components of the voltages along the g and d axes respectively The phasor diagram described by these equations is shown in Figure $3, where the phasot representing the infinite bus voltage V., with the g axis as reference, is given by Vi, = Vag + Vou = Vo cos (5 = a) — jV2sin(5 — a) (5.18) Note that Figures 5 1 and 5.2 can be combined since the same q and d axes, the same EMF £, and the same current J, are applicable to both Thus in Figure 5.3 the machine terminal voltage components V, and V, can be obtained using (5.15) An alternate procedure would be to start with the phasor V, in Figure 5.3, then add the voltage drop RI, - X,lq in the g axis direction and the voltage drop Rely + Xely in the d axis direction to obtain the phasor F, ‘Again remember that in Figure 5.3 both I, and Vj are shown as negative quanti- ties. The remarks concerning the focation of the q axis starting from V. and [, are also applicable here 154 Chapter § Fig $3. Phasor diagram of (5 17) 5.4 Machine Connected to an Infinite Bus with Local Load at Machine Terminal The equations that relate the infinite bus voltage V. to the stator equivalent EMF E are given by (5.17). Note that this form of the equations does not give the machine terminal voltage explicitly. Since the terminal voltage is a quantity of considerable interest, we seek a solution in which ¥, and ¥, are given explicitly One convenient method is to add a local load at the machine terminals, as shown in Figure 5.4. For the system shown in Figure 54, the steady-state equations for the machine voltages, EMF's, and currents are the same as given by (5.14), (5 15), and (5 12) re- spectively, Equations (4.149), which at steady-state conditions are the same as (5.16), are still applicable except that the currents jz and i, should be replaced by the currents ig andi These are the d and q axis components of the transmission line current é In other words, with the q axis as a reference, Lig + ja (5.19) where we define Ig = igiV3 hag = bal V3 (5.20) The transmission line equations are then given by Fig $4 One machine with a local load connected to an infinite bus through a transmission line ‘Simulation of Synchronous Machines 155 vg = — VV. sin(6 — @) + Reig + OL cing vy = V3V, cos(5 — a) + Rely — wleiy ($21) which can be stated in the form Vy = —Vesin(d — a) + Rely + Xelg V, = V.cos(d — a) + Rely = Xela (5.22) To obtain a relation between 7, and [,, we refer to Figure $4 By inspection we can write the phasor relations Le=te-h Ge - TR + 1X) K+ Ve (5 23) where we define 2; = Ry + jX,. Separating the real and imaginary components, Uy be — a = Nad Xe = % (a ~ fa) Re + (lq ~ Ia) Xt = Va (524) From (5.24) we can solve for J, and Lig, fa = Ty + Xe Rp, wp, — Ve + ake (5.25) RE+ Xt Ri+ Xt The equations for the g and d axis voltage drops can then be obtained from (5.25), (5.15), and (5 22) 5.4.1 Special cass For this case X, the resistive load, Z, = R, + jO From (5.25) Ta = la ValRe dy = Substituting (5 26) into (5.22), V, = —Vsin( = 0) + Rs — Ve/Ri) + XU, — Vy/Ri) V, = Vo cos(5 ~ a) + Relig ~ Ve/Rr) - Xela ~ Fa/Ri) — Y/R (5 26) or Vil + Re/Rr) + Vy(Xe/ Rr) = ~Vo sin — a) + Rela + Xely VAX Re) + Vill + Re[Rr) = Va 00s (6 - a) + Relp—Xely (S27) ‘Substituting (5. 15) into (5.27) and rearranging, pe- —Pesin — @) + (Re 1 Re R dese), (+S)e- roost = a) = (Reo rf ony EA) 1 Now define 156 Chapter 5 f, ext Bay Fig 55 Phasor diagram of a ynchronous machine connected to an infinite bus with local resistive oad 6 Ret Ri X.xg Ryo Re + EE Aes +R Rr Xl + /Ry) + HY + RIK) Re Then (5 28) can be written as (X./Ri)E = —V.sin(6 ~ a) + Rela + Rely Ret Re Kame Ry Re X14 1/Ri) + x4(l + R/R1) Ro= R.+r (1+ R/R DE = Ve cos - a) ~ Rule + Rely (5.29) Let us define a phasot E; Evm (1+ RR jE + (X/RIDE (5.30) where the phasor Ey makes an angle y with the q axis y = arctan[¥,/(R, + Re)] (531) The phasor diagram for (5.29)-(5 31) is shown in Figure 5.5, 5.4.2 The general case: Z, arbitrary For Z, arbitrary the equations are more complicated Substituting (5 25) into (5 22) and rearranging, u(t + Beka) y, (Be ~ *) = Va sin(6 ~ a) + Rela + Xely Zi Zi BMS BR (hy BELEN) «yon 0) = Ry = Ke (532) or Vil +A) + edz = —Vesin (6 ~ a) + Ride + Xl = Fads + Vl + di) = Va cos(6 ~ a) - Xela + Rely (533) where Ar = (RAR, + XX DZ a = (RX, — HP R)/ZE (5.34) ‘Combining (5.33) and (5 15), ‘Simulation of Synchronous Machines 187 MiB = —Vesin(d ~ a) + (Re + (1 + da) ~ wedally + 1% + (1 + As) + ally (1+ ME = Va cos — a) + [-Xe ~ ha — wal + Asda + [R, — Xph2 + (1 + AD 6.35) Again, by defining E, & (1 + X,)E + jdoF, RAR +L +) — ads RAR + (+) — xe REX A+ Dt Pe, KP Xt aye di) tre 636) we may write (5 35) in the form AE = —Vasin(6 — a) + Rely + Kyl (1 + ADE = Va cos - a) — Kale + Rel, (537) Since (5 37) is of the same form as (5.29), it can be represented by the same phasor diagram in Figure 5 5 5.5 Determining Steady-State Conditions The most common boundary conditions are the terminal voltage V, and either the current [, and the power factor F, or the generated power P and the reactive power Q (per phase) In either case V,, f,, and (the power factor angle) are assumed to be known Resolving [, into components with V, as a reference, we write Tod. + ile G38) where f, is the component of [, in phase with V,, and J, is the quadrature component (which carries its own sign) We also define the power factor F, as F, = cos 639) where o is the angle by which /, lags V,. Then I= 1,cosé 1, = -/,sing 40) The phasor E,, in Figure 5 2 is given by Eg * Va (+ ixy Vly = Vo + (he + ir + IX) = Ve = gle trl) + 10h + rhe) (641) The angle between the g axis and the terminal voltage V, (ie, the angle 6 ~ 8 in Figures 5 | and 5.2) is given by 15 — 6 = tan"[lxgl, + dM + th = Xgl) (5.42) The above relations are illustrated in Figure 5 6. Then we compute V,= —V,sin(s - 8) % = Vec0s(6 ~ 8) 643) and vg and v, can then be determined from their relationship to Vz and ¥, given by (5 15). The currents are obtained from Ig= -hsin(@- 8+) I, = 1,c05(6 ~ 8 + 4) (S44) and the rotor quantities ig and i, can be determined from (511), The remaining 158 Chapter 5 Fig $6 Phasor diagram illusteating (5 41) and (5 42) currents and flux linkages can readily be determined once these basic quantities are known. In the case of a synchronous machine connected to an infinite bus the same pto- cedure is followed if the conditions at the machine terminals are given. The voltage of the infinite bus is then determined by subtracting the appropriate voltage drops to the machine terminal voltage 7, If the terminal conditions at the infinite bus are given as the boundary conditions, the position of the q axis is determined by a procedure similar to the above The machine d and q axis currents and voltages and the machine terminal voltage can then be determined. This is illustrated in Examples 5.| and 5.2 5.5.1 Machine connected to an infinite bus with local load Case I: V.,£, and the machine load angle 6 — a are known. In this case Jy and J, can be determined ditectly {rom (5.37) Then from (5 15) we can determine Vy and ¥, The three-phase power of the machine can be determined from the relation Py, = 3(Vals + V,1,). The terminal current I, is determined from (5.25), and knowing V., we can also determine the power and power factor at the in- finite bus Case 2: Machine terminal conditions ¥,, [,, and power factor are known From /,, ¥,, and the power factor the position of the quadrature axis is deter- mined (see Figure 5 2), From this information J,, Vz, J,,and V, can be found, Also E can be calculated {rom (5.13). From (5 36) and (5.37) the phasor E, can be constructed The infinite bus voltage can then be determined by drawing Rul, + Xq/, parallel to the daxis and R,I, ~ Red parallel to the g axis, as shown in Figure $7 Thus V. and the angle 5 — @ are found, from which we can determine Vg and Veg. The current J, is determined from (5.25), and the power at the infinite bus is given by 3(Vealu + Vogl) ‘Case 3: Conditions at infinite bus are known From V,, Z,, and Z, the machine terminal voltage V, is calculated. Then from V and Z, we can determine T,. From J, and J, T, is found. Now the conditions at the terminals of the machine are known and the complete phasor diagram can be con- structed Simulation of Synchronous Machines 159 Fig 57 Construction of the phesor diggram for Case 2 5.6 Examples The procedures described are illustrated by several examples where different initial conditions are given Example 51 The machine described in Examples 4.1, 4.2, and 43 is to be examined at rated power and 0.85 PF lagging conditions (nameplate loading) The terminal voltage is 10 pu. Calculate the steady-state operating conditions. If this machine is connected by a transmission line of 0 02 + j0.40 pu impedance to a large system, find the infinite bus voltage Solution From previous examples and the prescribed boundary conditions the following data are available: 1700 pu ¥, = 1000 pu 1640 pu R, = 002 pu r= 0001096 pu Z, = 0.4 pu F, = cos = 0850 Z, = 0.4005 /87.138" From the given power, power factor, and voltage we compute J, = 10/085 = 1176 pu The angle @ is computed from F, as ¢ = cos“!0.85 = 31 788° Then from (5 40) 1, = [,cosg = 1.000 I, = Ising = -0620 From (5 42) and Figure 57 _ wa 1.00 x_ 1.64 — 0.001096 x 0.620 (5 B) = srolan TG 4 0.630 x 164 + 100 x 0 001055 = arctan 0.8126 = 39,096" and 6 — 8 + ¢ = 31788 + 39096 = 70.884" = angle by which J, lags the g axis, Then from (5 44) 100 Chapter 5 Icos(é — 8 + 6) = 0385 pu, = 0667 pu and I, = ~Lsin(@d- B+ 6) = -1112 pu y= -1925 pu From (5.42) ¥, = V,cos39 09 = 0.776 pu uy = 1.344 pu ¥, = —V,sin39 09" = -0631 pu vy = ~1092 pu From Figure 5 | by inspection B= Vi 4 ty ~ vals = 0.776 + 0.001096 x 0.385 +170 x 1112 = 2,666 = Ezy at steady state [from (4 209) and (5.8)] Now using (58) in pu, ip = V3E/L,p where, from Example 41, Ly) = 155 pu Then ig = (V3 x 2 666)/1 55 = 2.979 pu The currents ip and ig ate both zeto. The ftux linkages are given in pu by Ny = Lely + KMpip = 1. 70(- 1,925) + (1 55)(2.979) = 1.345 Nap = (is + iedKM, = (2.979 ~ 1925)(1.55) = 1634 Ay = Lely = 164 x 0667 = 1,094 Dag = kKMgi, = 149 x 0667 = 0.994 De = KMyig + Unig = 1.55(-1.925) + (1 651)(2.979) = 1.935 No = kKMoig + Mpip = 155(2979 — 1.925) = 1634 = Ayo Ag = kMoi, = 0994 = Aug ‘Asa check we calculate the electrical torque 7,, which should be numerically equal to the three-phase power in pu, Teg = ga — tady = 0.667 x 1345 + 1,925 x 1.094 = 3.004 Then 7, = 1001 pu If we subtract the three-phase [°r losses, we confirm the generated power to be exactly P= 7, 1000. We also calculate the infinite bus voltage for this operating condition, We can write V, = ¥, - ZT Let ¥, = ¥,/8 = 10/68 Then T, = 1/8 = = 1176 (6 — 31.788" V. fa = 1.08 ~ (0.4005 /87.138°)(1.176 /8 = 31.788") or V, [a = 8 = 10 ~ 0.4712 /55.349° = 0828 /~27.899" pu Thus we have V, = 0.828 pu, and @ — a = 27.899" = the angle by which V7, leads ¥,,. The angle between the infinite bus and the g axis is computed as 6 ~ a = (6 — 8) + (B — a) = 39.096 + 27 899 = 66,995" ‘Simulation of Synchronous Machines ier Example 52 Let the same synchronous machine as in Example 5.1 be connected to an infinite bus through a transmission line having R, = 0.02pu, and L, = X, = 04pu. The in- finite bus voltage is 10 pu. The machine loading remains the same as before (P = 10 puat0 85 PF) The boundary conditions given in this example are “mixed”; ie, the voltage is known at one point (the infinite bus), while the power and reactive power arc known at a different point (the machine terminal). A slight modification of the procedure of Example 5 1 is needed Solution A good approximation is to assume that the power at the infinite bus is the same as at the machine terminals by neglecting the ohmic power loss in the transmission line (since R, is small), A better approximation ‘is to assume a power loss in the transmis- sion line based on some estimate of current (say 1 0 pu current) Let 2R, = (1 00){0,020) = 0.02pu Then the power at the infinite bus is 0.980 pu and the component of the current in phase with V., is [, = 0.980pu The angle @ between 7, and P is given by tané = L/f, = 10201, The angle between V, and V7, is given by an equation similar to (5.32), viz, Xi, + Robe _ 0.392 + 0.021, V.— XI, + RI,” 1020-041, The power factor angle at the machine terminal ¢ is given by = B+ 8 = costO.8S = 31788" tanB = These angles are shown in Figure 58, with V_ used as reference; i¢, a = 0 Then tang = tan (cos'0 85) = 0620 Using the identity tan = (tan + tan6)/(1 — tan ftané) we compute =1.020/, + (0.392 + 0.02/,)/(1.020 - 0.44.) T+ [1 020(0 392 + 0.027,)4,1/(1 020 — 041.) from which we get/, = 0.217 pu 0.620 = Big $8 Phasor diagram of V, and Vic 162 Chapter 5 From the known value of /, we can now determine 8. (0,392 ~ 0,004) ] _ . [oapzeee 19310 8 = tan Also, = tan-¥(0 213/0.980) = 12.483" = 19.310 + 12.483 = 31.793" which is a good check (see above) The terminal voltage V, is given by Fo = (Vo — Kelp + Rel) + j(Xd, + Rel) = 1106 + j0 388 = 1172/19.31° pu The generator phasot current is T, = 0.980 — {0217 = 1,003 /~12.48" pu and P = ¥,1,cos@ = 10001 pu (on a three-phase basis) The position of the g axis can be determined from an equation similar to (5 41), With a = 0, + XL + + Rds Vi- G+ XO + + RO, The currents, voltages, and flux linkages can then be calculated as in Example 5.1 The results are given below in pu: b= tant = 53.736" ij = -1 591 Ay = 1676 0.701 Ap = Aan = 1914 ip = 2.826 dy = 1150 E = 2.529 Ao = Aug = 1.045 by = 1148 T,, = 3.004 v, = 1.675 T, = 1001 In steady-state system studies (often called Joad-flow studies) it is common to spec- ify the generator boundary conditions in terms of generated power and terminal voltage magnitude, ie, P and ¥, (Both ¥, and ¥; are commonly used for the terminal volt- age and both are used in this book ) In studies of large systems these boundary condi- tions are satisfied by iterative techniques, using a digital computer For the one machine-infinite bus problem the system may be solved explicitly. We now consider the ‘one machine-infinite bus problem with a local load connected to the F, bus consisting of a shunt resistance R, and @ shunt capacitance C,, representing the transmission line susceptance The system of generator, local load, and line may be conveniently described as a two-port network (Figure 5 9) for which we write, with V,, as reference (« = 0), q Ys Yn] | % (545) TL |¥u Yai [h The apparent power injected at node | may be computed as 5, = P+ iQ) ~ Wilk = VAR + KOE G46) Simulation of Synchronous Machines 163 Fig 59° One-machine system as a two-port network Then we may compute P, = GV? + UV.(G200s8 + B,ysinB) (547) where we define Fey = Gig + {Bin for allkandm In (547) P,, ¥, and V,, are speci- fied, while G,,, G,,, and By; are known or computed system parameters. Thus we may solve ($ 47) for the angle 8. In doing so, it is convenient to define a constant angle y related to the admittance element ¥}) = Y,,/7. Then from (5 47) we define F = cos(y ~ 8) = (Py - GVA Fah Vo (5.48) from which 8 can be found. Obviously, there are limits on the magnitude of P, that can be specified in any physical situation, as the cosine function is bounded in (5 48), Example 53 Compute the steady-state conditions for the system of Examples 5 I and 5.2, where the given boundary conditions in pu are P= 10 (oma three-phase basis) y= Lit Ve = 100 and where the local load is given in pu as R= 100 B, =C, = 001 Solution For the numerical data and boundary conditions given, we compute Z, = R. + jX, = 0.02 + j04 = 0.4005 /87.138" pu Y2= Tn = “WZ = Yoda = —0 1247 + j2.4938 = 2.4969 /92.862" pu ory = 92.862° We are also given that R, = 100 pu and B, = 0.01 pv. Thus the admittance from node | to reference is Fp = 001 + j0.01 pu. We then compute Fu = io + Fe = Gy + iBy = 0.1347 — j2 4838 pu We now compute the quantity F defined in (5.48) as, Fa (Pi - GuiV/YahY, = 02792 Then y -— B = cos! F = 73788" B = 92862 — 73788 = 19.074" or Ve = 1.17 {19.074 164 Chapter 5 To find the currents, we note from Figure 59 that 7, = T+ T, Now T= Ty + Te = (U/RD LB + Y/X0) {8 + 90° = 0.0072 + j0.0149' pu We also write L-G~ Paz. _ [RA ¥,c0s8 ~ Ve) + XV sin 8) + j[RMsinB — X,(i,cos8 — Vie)} Zz = 09667 ~ j0.2161 pu Then, noting that J, lies at an angle from V,, (Figure $ 8), T= 17 +T, = 1, {8 = 09739 - jo 2012 = 0.9945 (= 11.672" pu We may now compute, as a check, P+ jQ = FiIt = 1.000 + j0.595 1.164 (30.7467 pu The power factor is E, = tan“'(Q/P) = 0.859 The quantity Ey, of Figure 5.2 may be computed as a means of finding 6 Thus with @ = Owe compute, as in Figure 5.6, Fug = El V(B + ral + ine (8 = 2.446 /54.024° pu and 6 = 54024° Then we compute 6 8 = 34950" b= 048 = 30746" 6-8 + 6 = 65696" With all the above quantities known, we compute d-q currents, voltages, and flux linkages in pu as in Example 5.1, with the result iy = -1570 dy = 1.662 i, = 0709 Mao = Xp = 1.897 vg = -L 161 Ay = 1.163 v, = 1.661 Dag = dg = 1.056 E = 2500 dy = 2180 i = 2794 Tog = 3003 = " 1.000 Example 54 The same machine at the same loading as in Example 5 1 has a focal load of 04 pu power at 08 PF It is connected to an infinite bus through a transmission line having R, = 0.1 puand X, = 0.4 pu, Find the conditions at the infinite bus Solution The internal machine currents, flux linkages, and voltages are the same as in Example 51. Thus, in pu, ‘Simulation of Synchronous Machines 165 =-112 ¥,=0776 I, = 0.385 V, = -0.631 5-8 = 30096 — E = 2666 From the local load information |i] = 04/110 x 08) Therefore J, = 04 ~ j03 pu We can also determine that, in pu, R=16 X=12 Z%=20 Thus we compute from (5.34) A= (16 K OL 4 12 x 04/207 = 0.16 A= (16x 04-12 x O1/Q0% = 013 05 pu Then Ry = 01 + 0001096 x 116 — 013 x 17 = -01197 R, = 0.1 + 0.001096 x 1.16 - 013 x 1.64 = -0119 ¥,= 04417 x 116 + 0.001096 x 0.13 = 2372 X, = 04 + 164 x 116 + 0.001096 x 013 = 2.303 From (5.37) Vig = Va sin(S ~ a) = ~(=1.112)(-0 1197) = (0.385)(2.303) + (0 13)(2.666) = -0673 Vig = Va. cos(6 = a) = (1 1122372) — (0 385)(-0.119) + (1.16)(2.666) 0501 V, = 0.673" + (5011? = 0.839 From (5.25) Iya “1112 + SIIB X12 40631 16 | 9 6268 = x 12 L = 0385 - 9776 x 16 — 0.631 12 9.9639 The power delivered to the infinite bus is P.. = (~0.673)(—0 6268) + 0.2639 x 0501 = 0.554 pu The power delivered to the local load is P, » 04 pu. Then the transmission losses are 0 14 pu, which is verified by computing Ryli 57 iol Conditions for a Multimachine System To initialize the system for a dynamic performance study, the conditions prior to the start of the transient must be known These are the steady-state conditions that exist before the impact. From the knowledge of these conditions we can assume that the power output, power factor, terminal voltage, and current are known for each machine If they are not specifically known, a load-flow study is run to determine them ‘Assume that a reference frame is adopted for the power system This reference can 166 Chapter 5 be chosen quite arbitrarily. Once it is chosen, however, it should not be changed during the course of the study, In addition, during the study it will be assumed that this refer ence frame is maintained at synchronous speed Consider the ith machine. Let its terminal voltage phasor 7,, be at an angle 8; with respect to the arbitrary reference frame, and Jet the g axis be at an angle 8, with respect to the same reference Note that 6; is determined from the load-flow study data, while 4, is the desired initial angle of the machine q axis, which indicates the rotor position. The difference between these two angles (3, — 8,) is the load angle or the angle between the q axis and the terminal voltage From the load-flow data we can determine for each machine the component J, of the terminal current in phase with the terminal voltage and the quadrature component, 1, By using an equation similar to (5.42), we can determine the angle 4, — 8, for this machine. Then by adding the angle 8, we get the angle 6,, which is the initial rotor angle of machine / From V,, and 8, we can determine I, Ig, Vas and V,,, which can be used in (5.14) or (5 15) to determine &, Then from (5.7) is, can be determined. The flux linkages can also be calculated once the d and g components of /, are known 5.8 Determination of Machine Parameters from Manufacturers’ Data The machine models given in Chapter 4 are based upon some parameters that are very seldom supplied by the manufacturer. Furthermore, the pu system used here is somewhat different from the manufacturer's pu system It was noted in Section 47.3 that the pu self-inductances of the stator and rotor circuits are numerically equal to the values based on a manufacturer's system, but the mutual inductances between rotor and Stator circuits differ by a factor of W372 We shail attempt to clarify these matters in this section For a more detailed discussion see Appendix C Typical generator data supplied by the manufacturer would include the following Ratings Three-phase MVA Stator line current Frequency and speed Power factor Stator line voltage Parameters. Of the several reactances supplied, the values of primary interest here are the so-called unsaturated reactances They are usually given in pu to the base of the machine three-phase rating, peak-rated stator voltage to neutral, peak-rated stator cur- rent, and with the base rotor quantities chosen to force reciprocity in the nonreciprocal Park's transformed equations. This is necessary because of the choice of Park trans- formation Q (422) traditionally used by the manufacturers. ‘The following data are commonly supplied Reactances (in pu) Synchronous d axis = x, Subtransient g axis = xj Synchronous axis = x, Negative-sequence = x; Transient d axis = x} Zero-sequence = x5 Transient g axis = xj Armature-leakage = x Subtransient d axis Simulation of Synchronous Machines 167 Time constants (ins) Field open circuit = jy Subtransient of amortisseur (d axis) = 7’ Subtransient of amortisseur (g axis) = 1," Resistances (in Q) Stator resistance at 25°C Field circuit resistance at 25°C Other data Moment of inertia in Ibm ft? or WR? (sometimes separate data for generator and turbine are given) No-load saturation curve (at rated speed) Rated load saturation curve (at rated speed) Calculations. The base quantities for the stator are readily calculated from the rat- ing data: Sy = VAtating/phase VA ¥, = stator-rated line-to-neutral voltage V J, = stator-rated current A 2n x rated frequency rad/s The remaining stator quantities follow: Voy s Ry = Yh @ Vly Woturn Ly = Voty/lp H fe de Also the stator pu inductances are known from the corresponding reactance values Thus Ly, Lj, Li, Ly, Lj, Lf Ly, Lo, and ty are known Rotor base quantities: If 2, in pu is known, then Ly» in pu is determined from Lap = Ly — 44 the corresponding value of L4y in H is then calculated. The mutual field-to-statot inductance M, in H is determined from the air gap line on the no-load saturation curve as V/2V, = weMyi,, where i, is the field current that gives the rated voltage in the air gap line The base rotor quantities are then determined from (4.55) and (456); the base matual inductance Myy is calculated from (4.57) Rotor per unit quantities: Calculation of the rotor circuit leakage inductances is made with the aid of the equivalent circuits in Figure 5 10 The field-winding leakage inductance £, is calculated from Figure 5 10(a) by inspection: Ly = Fat Lavde!ELan + 44) pa (349) which can be put in the form de = Lalli — 2a)i(ba - LI) (5.50) 168 Chapter 5 4 % m bo =—" ) ’o ‘ap ® Fig 510 Equivalent circuit for d axis inductances: (a) transient inductance. (b) subtransient inductance Similarly, by inspection of Figure 5.10(b), Len hat (8.51) 1 Ikan + 1/2n + 1/ee from which we can obtain ty = Lavta(Ld - 2ad/Lants — LL ~ £0) 52) The self-inductances of the field winding L; and of the amortisseut Ly are then calcu lated from Lae totba Le= tet Lan (53) The same procedure is repeated for the g axis circuits oth - fy (5.54) where 1, = £,and £4 is determined from Figure 5, 11 by inspection: Li = Ay + tolsol(Le + Leo) (555) from which we can obtain bg = Lalli ~ LOML, ~ £39) (5.50) and the self-inductance of the g axis amortisseur is given by Le = Lig + Lo (557) Resistances. The value used for the stator winding resistance should be that which corresponds to the generator operating temperature at the rated load. If this data is not available, a temperature rise of 80-100°C is usually assumed, and the winding resistance 4 he Fig 5.11 Equivalent circuit of theg axis subtransient inductance Simulation of Synchronous Machines 169 is calculated accordingly Thus for copper winding the stator resistance for 100°C tem- perature rise is given by rag = hs{(234.5 + 125)/(2345 + 25)] 2 (5.58) The same procedure can be used to estimate the field resistance at an assumed operating temperature. However, other information is available to estimate the field resistance From (4 189) we compute n= Lefrig po (5.59) where rip is given in pu time The damper winding resistances may be estimated from the subtransient time constants Frm (4 187) and (4 190) the d axis subtransient time constant is given by td = (Lobe ~ Lio)froLela/ba) pu (5.60) Since all the inductances in (5,60) ate known, rp can be determined Similarly, from (4.192) and (4 193) rg can be found, 1 = LiL Lehto) pu 66 Again note that rj and rf are given in pu Finally, data supplied by the manufacturer may not be available in the complete form given in this section We should also differentiate between data obtained from verified tests and those obtained from manufacturers’ quotations. The latter are usually estimated for a machine of given size and type, often long before the machine is fabri- cated. This may also explain apparent inconsistencies that may be found in a given set of data. This section illustrates the procedure that can be used to determine the parameters of the machine When some of the data is not available, the engineer may find it con- venient to assign values for this data from typical data available in the literature for machines of the same size and type, We should always ascertain that the parameters thus calculated ate self-consistent Actual values for several existing machines are given in Appendix D. Example 55 The data given by the manufacturer for the machine of Example 4 | are given be- low. The machine parameters are to be calculated and compared to those obtained in Example 4.1 xg = by=170 po x, =hy a4, = 015 pu 1, = 164 pu thy = 59 8 Lj = 0245 pu rf = 0.023 s xy = Lj = 0380 pu th = 0.075 s ay = Lf = 0185-2" pu 7, = 024 5 Solution We begin by calculating the pu d axis mutual inductance Lap = 1 70~ 015 = 155 This is also the same askM,,kM,, and Mg Similarly, 170 Chopter 5 Lag = kMg = 164-015 = 149 pu Now, from (5 50) Lp = 1$5{(0 245 ~ 0.15)/(1.70 — 0.245)] = 0101 pu Ly = 0.101 + 155 = 1.651 pu From (5.52) - —__(.55)(0.101)(0.185 ~ 0.15) (155) 101) — (1.651) (0 185 — 0.15) Lp = 1.550 + 0.055 = 1.605 pu Also, from (5 56) Lo = 1.49{(0.185 — 0.150)/(1 640 — 0.185) = 0.036 pu Lg = 1490 + 0.036 = 1526 pu = 0.055 pu From the open circuit time constant Ty = 59 s = 222425 rad We compute from (5 59) ty = 1651/2224 25 = 7423 x 10 pu and from (5.60) _ (1.605 x 1.651 — 1.55 x 1.55)(0.185) (1 651)(0 023 x 377)(0 245) = 00131 pu From 1% = 0.075 we compute Tf = (L2/L)rh = 846 ms = 3.19 rad Then from (5 61) ro = (1.526/3.19)(0 185/1 64) = 0.054 pu These values are the same as those calculated in Example 4 1 5.9 Analog Comput ‘Simulation of the Synchronous Machine ‘The mathematical models describing the dynamic behavior of the synchronous ma- chine were developed in Chapter 4. The remainder of this chapter will be devoted to the simulation of these models by both analog and digital computers. We begin with the analog simulation Note that the equations describing the machine are nonlinear. For example (4 154) and (4 163) have two types of nonlinearities, a product nonlinearity of the form x,x, (where x, and x; are state variables) and the trigonometric nonlinearities cos y and siny These types of nonlinearities can be conveniently represented by special analog computer components, Also, the analog computer can be very useful in representing other nonlinearities such as limiters (in excitation systems) and saturation (in the mag netic circuit). Thus in many ways the analog computer is very well suited for studying synchronous machine problems. A brief description of analog computers is given in Appendix B, Simulation of Synchronous Machines i” To place the matter in the proper perspective, recall that the state-space model of a synchronous machine connected to an infinite bus is a set of seven first-order, non- linear differential equations When the equations for the excitation system (for ,) and the mechanical torque (for T,,) are also added, the system is typically described by 14 differential equations Complete representation of only one synchronous machine with its controls would occupy the major part of a large-size analog computer Thus while the analog computer is well adapted for the study of synchronous machine dynamics, s usually limited to problems involving one or two machines with full representation or to a small number of machines represented by simplified models (2, 3,4, 5} The model most suited for analog computer representation is the flux linkage model. Thus the equations developed in Section 4 12 are used for the analog simula- tion. The differential equations will be modified, however, to avoid differentiation For example the state-space equation of the variable x, is A = f(xuye) (5 62) where x, 7 = 1,2, ,m, are the state variables, and mj, = 1,2, ,7, are the driv- ing functions, Fot analog computer simulation (5 62) is written as = af (xu, dt + x (0) (563) where a is the computer time scale factor and wy is requited if time is to be in seconds (see Appendix B) 5.9.1 Direct axis equations From (4 126) Ne a f lé Quo — N) — @, = via +A) (5.64) From (4.128) d SP Oe — dod + ofa + 2400 5.65) aS, [ie ) (0) ¢ and from (4.129) Ayo a A z Quo — Ao)at + dp(0) (5.66) The mutual fiux linkage \,» is computed from (4.120) Rap = Luoa/ta + Ae/ Le + Av/ Zo) (567) Then from (4.118) the d axis and field currents are given by dy = (hada — dav) (5.68) dp = (U/L Ar = Aan) (5.69) The analog representation of the d axis equations is shown in Figure 5.12 Note that all integrand terms ate multiplied by wy to compute time in seconds and divided by the time scaling factor a 72 Chapter 5 Ph te A a = “ae Soto oF 4 44 Or ho ate /tee * rot 1 oO | ay rou » oO o_O | PO “sof te Fig 512. Analog representation of thed axis equations 5.9.2 Quadrature axis equations From (4 130) « = f [ee ~ Ay) + why ~vja + 2,(0) and from (4.131) EO — Ap)dt + Ay(O) The mutual flux linkage is computed from Yag = LucOs/te + Ro/Le) Then the g axis current is given by, from (4.123), ig = (1/4gAy ~ ao) The analog simulation of the q axis equations is shown in Figure 5 13 Fig. 513 Analog simulation of theg axis equations. (5.70) 67) (8.72) 673) Simylation of Synchronous Machines 173 5 5 \ =~ % * \ Fig. $14 One machine-infinite bus system with local resistive load 5.9.3 Load equations In (4 149) a = 0 will be used for convenience Therefore, ue of [VV sind + oy ~ Ryly — oL,i,]dt + 140) (574) a [-V3V. cosd + 0, = Reig + WLeigldt + 1,(0) (S75) oS Equations (5.74) and (5.75) are useful in generating the voltages v, and v, How- ever, if they are used directly, differentiation of i, and i, will be required, which should be avoided in analog computer simulation. To generate v, and v,, the following scheme, suggested by Krause [2}, is used The machine is assumed to have a very smal] resistive load located at its terminal, as shown in Figure 5.14. This load is represented by a large resistance R From Figure 5 14 the machine terminal voltage and current for phase a are given by v= (is ~ ia)R (5.76) where i,, is the phase a current to the infinite bus Sa/ip Ven 8 OY az . +g —- OF San l te = “aN ww 4 =Gs/ih, " Hf Rg 4 Fig $15 Analog simulation of the load equations 174 Chapter 5 Fig. 516 Simulation of the electrical torque Tes Following a procedure similar to that used in Section 54, the current i, can be re- solved into d and g axis components i,, and i,, given by (5.74) and (575). The cur- rents ig and i, ate given by (5.68) and (573) The v, and , signals are obtained from re 5.14 by inspection, Og = ig = igR By = Cig = bg) R (877) where iy and i, are obtained from (5.74) and (575) respectively, with subscript ¢ added as required by Figure 5.14. The analog computer simulation of the load equa- tions is shown in Figure 5.15 5.9.4 Equations for wand 5 From (4.90) and (4.99), with &, = din pu and 7, = 2Hap, we can write dtgy dt, where T, = (iA — igAy)/3 Equation (5 78) is integrated with time in seconds to compute, with zero initial conditions and with a time scale factor of a, dese 2H, 2 = 2H = Tq —T,- Dox pu (5.78) taf. (Ty = Te — Dooy.)dt pu (5.79) mu = Sha Note that the load damping signal used is proportional to w, (pu slip), requiring appro- priate values of D Most analog computers require that 5 be expressed in degrees to find sin 5 and 0s 6 [6]. Therefore, since § = wp(w, — 1) = epi, pu, we compute 180 a, 7a b= f “oy at + 180 ‘so 5(0) elec deg (5 80) ° The analog computer simulation of (5.78)-(5 80) is shown in Figures 5 16 and $17 The generation of the signals —w and —6 is shown in Figure 5.17. The analog repre~ 180 | ~ Fig $17 Simulation of wg, 1, and 6 ‘Simulation of Synchronous Machines 175 sentations shown in Figures 5.12, 5.13, and 5 15-5 17 generate the basic signals needed to simulate a synchronous machine connected to an infinite bus through a transmission line, However, other auxiliary signals are needed. For example to produce the signals ed, and oh, shown in Figures 512 and 5.13, additional multipliers are needed To produce the signals V_ sin 6 and V. cos 4, an electronic resolver is needed. The complete analog representation of the system is shown in Figure 5.18. It is important to 10 eee | taps Fig 518 Analog computer patching for a synchronous machine connected to an infinite bus through a transmission line 176 Chapter 5 note that signals are added by using the appropriate setting for the potentiometers associated with the various amplifiers and integrators scaled to operate within the analog computer rating, This scaling is best illustrated by an example, and in Example 5.6 the scaling is given in detail for the simulation of the synchronous machine. The initial conditions may be calculated from the steady-state equations (as in Examples 5.1~5 3), and these values may be used to initialize the integrators. However, the analog computer may be used to compute these initial conditions To initialize the system for analog computation, the following procedure is used. The integrator for the speed is kept at hold position, maintaining the speed constant. The integrators for the flux linkages are allowed to operate with the torque 7,, at zero. This builds the flux linkages to values corresponding to the no-load conditions, The load 7, is then applied with the speed integrator in operation The steady-state conditions thus reached cor- respond to initialization of the system for transient studies Example 56 The synchronous machine discussed in Examples 4 1-4 3, 5.1, and 5 2 is to be simu- lated on an analog computer, The operating conditions as stated in Example 51 represent the steady-state conditions. The system sesponse to changes in v, and T,, is to be examined Solution The data for the synchronous machine and transmission line in pu is given by: Ly = 1.700 Luo = 0.02838 ZL, = 1.640 Ly = 0.02836 Ly = 1.605 + = 0.001096 Lg = 1526 re = 0.00074 Lap = 1850 ty = 00131 Lag = 1.490 Jo = 0.0540 Ly = 1651 R = 1000 dg = 24 = 0.150 R, = 0.02 4, = 0101 Ly = 0.085 H=237 8 ho = 0.036 Tie = 590 8 L, = 0.400 V,, = 0.828 The additional data needed is Ty = 1.00 pu and Exp = 2.666. Note that Eyp = & in the steady state This value of Ep with the proper scaling is introduced into the in- tegrator for Ay As explained in Section 5.9 5, the analog computer is made to initialize itself by allowing the integrators to reach the steady-state conditions in two steps In the first step Ep is applied with 7, = 0 and @ = wp = constant, Then T,, is switched on with all integrators, including the w integrator, in operation The basic connection diagrams for the analog simulation are given in Figures 5 12- 517. The overall connection diagram is shown in Figure 5.18 In that figure the analog unit numbers and the scaling factors for the various signals are given; eg , the scaling factor for Ay is 10, which is given in parentheses The time scaling used is 20. The settings of the various potentiometers and the scaling are listed in Table 5.1 7 Simulation of Synchronous Machines verro | of | — | bere 199 + % | 990] os | oz | coo | cor . sro _"% on . vero | ot | — | wee 1999 ae | e999 | oc | oz | £00 | ¢00 sro y _ - 2 _ 1 esto | 4 26810, Tea O aeeieazo * 7 NM 0 ot | 0€ 200 | 20% egagirsso'e _ a rpm z Bee Te - 1 uw rio 1is0 1Is0 queaueacoo "7 x ot} oc | oe | coo | cor . srizozior'0 _-"Y ye zipso} 1 | — | etpso 08z'0 Sleaniazoo ~ Ty x ot} or | oe | 200 | z00 7 oy y- 7 esero| 1 | oor] escro eser0 fay x or) or | or | toz | toe 4 , Coates) vary a. wero To | vor} H95100 | H9SI00 | Glee oteHTOEA 7 REA ‘7 or} or | or | ioz | zoe . ' , (oz)(coz101'0) my an . o9v'0 | ro | oor | 609700 eser'0 TaaG-or ¥ 9¢cr00) 7 Fos v | eeeeo} of | or | roe | 107 , . . . ovIyss00 _ > ~ osrro | or | oF ory grr Uotcor00) * 7 ™ OF) 0€ | o€ | 100 | Jor . . fozcipssoo) _ ey an, ssero | or | ot | osry osr'e Gapeeaci00) 7 v ot} oc | o¢ | 100 | t00 ee sero | or | ro | sero sLer'0 = x ot | oc | 000 | oot 0 ro st seer wae o- | sev oz | oc | 000 | 02 gsez'o | oot 9sez it ¥ ILO } oor} V0 pred sBsl eee fa~ |) seo | ov | OF | 000 | oor y “ue & , , , Corso) OY a seioo | or | ro | scero sLer0 Tew) 7 i v ot | o¢ | oe | 00 | 000 a [aay | Su foeen | OS) yumsuea, way | 77°79 | 7 | 7 | gy | 0a (07 = »)sainawoD Fore Ka uONwINUHS aUIYDEN sTOUOANOUAS 40; SBUMAg UID puE oTWONUAIOd “T'S 148, Chopter 5 178 ssec'o | oot | v0 ses -} sev | 91 | oc | 10% | cor wero] or | = 1997 ist | 9 | or | coz | cou iro | or | 1991 ivr | 9 | or | coz | om vrevo | or | — | pee i999 ‘= | i990 | o€ | oz | clo | ety sto av perv | ar | — | vere 9 eo % | c999'0 | o€ | oc | £10 | 10 ooaro | — | ossro 06810 cana N=! or) o€ | 0 | cio | ar cepeseszoo | OM o sosseoo Oy b i eo - 1 e160 616L'0 616L70 Zinistacoe "OR x or} oe | of | cio | cto vosco {oor | ro ov'ee or'se Loortengseoo) _ Py Xx} ot] os ho} ann " Ciaylss6eso0) ~ Hs f | oF (oey(6osseo0) _ vy o . . {oenisosseoo) ” 1 oso ro or sz or'sz tapeteon ~ ws v or} oe | o¢ | 1 0 ry 4% seioo | oF | ro | siero suer'0 me v-]} ot! of ) of | of | uz ssezo | oot | 10 ogee See = mo | cer| ve | oc | oto | or . we » 4 vivo | oor) v0 rier se'sr lie ’a-) seo} or | of | oto } on ouMsr) Oy an ; seloo | or | ro | sucro suero Taco v or] oc | oe | or | oro we Lty [Se |aearen | OS? swsis409 vor fiver [or [oo | get, | ee (panuiuo>) 1'S 1981, 179 Simulation of Synchronous Machines 40 Py soyye 40) posn st a1QP) Sp Uy ->10N, coro] + | —~ | cosro o or se Foro] oos] o8 | cic | fiz ooovo | © | = | coovo o or or | oso} oo} og | cz osoro | or | oF 80" 080 = "so | 1000 | os} so | uz | o1F oso't Tiotoai) ~ ost oo WOUEOS | eH9 . ; sseroj © | or | aero |e-o1 x 91S"¢ see "z-| os | or | 00s} o1c | 0¢ OH (op)ooT sooo} — | — ” sooo a MIT O7 | oc | or | etm | ett OH en , (0¥)001 gy hom . s00'0 ~ 000 a TT a | h oz | oc | o | zr > gsezo | oot | V0 osrez ses te mim | sev} or | oz | oor | 10s wT ze9r0 | oo ze91 wis Tex #%7A-| TO] oor! ox | oor | o0s vr , gsezo | oor ose ever Te ‘a so} or | oc | oor | tor rT treo | ot | ro | szveo swv60 iy oz | oz | oor | oo» evev0 | ot | 10 seveo wee. ST ™ oz | tor | os sev6'0 Goeoo) ye “ . Cow) __ vt . , zearo | oo | Fo eo ws Taper " HER eM zo} oo1| oz | tor | 20s -) vy gsez'o | 001 over erly wows) 24 fa so} or | oz | tor | cor Lue wo 180 Chapter 5 Fig 5.19 Response of a machine initially at 90% load and 90% excitation to a 20% step change in excita tion The steady-state conditions reached by the analog computer are listed in Table 5.2 They are compared with the values computed in Example 5.1 Figures 5.19-5.21 show the following analog computes outputs: the change in the exciter voltage Ep, the mechanical torque Tye, the electromagnetic torque Tyas the field flux linkage Ay, the stator d axis current ig, the terminal voltage error Via, the angular velocity error w,, and the rotor angle 6. The results of the simulation are shown in Figures 5.19-5.23, where all plotted quantities are given in pu, Eaample 5 1 is used as a base for the computer runs. Thus a 10% change in Eyp is 0.2666, which is 10% of the nominal value computed in Example 5.1 Similarly, 10% Ty, is 03 pu, and zero V,, corresponds to a terminal voltage ¥, of 3 pu (or F, = 10) Simulation of Synchronous Machines 181 Fig 520 Response of a machine initially at 100% load (Example 5 | conditions) to a 10%, increase in Typ followed by a 10% increase in Ey, to assure stable operation Figure 5.19 shows the response of the loaded machine to a 20% change in E;) The generator is initially loaded at 90% of rated load (T,, = 27). Note that the response to this change in Ey) does not excite an oscillatory response except for a small, well- damped oscillation in w, The terminal voltage responds nearly as a first-order system with a time constant of about 4s(rjy = 5.95) Figure 5.20 shows the system response to 10% step changes in both 7, and Ey», The system is initially in exactly the condition calculated in Example 5.1 with computer voltages given in Table 5 2. A 10% increase in T, is the first disturbance This excites a well-damped oscillatory response, particularly in T,, iy, Vj, «, and 6 (as well as other variables that are not plotted). A good degree of damping is evident. However, this 182 Chapter 5 ch Fig 521 Response of a machine initialy at 907 load to a 20% increase in Ty, followed by a 20%, in Ego to restore stability overload on the system results in a gradual increase in 6 with time, which if not arrested will cause the machine to fall out of step Repeated runs of the system have indicated that corrective action is required before 6 reaches about 95° The corrective action chosen was a 10% inctease in Es», This quickly restores the system to a stable operating state at about the same angle 5 as the initial angle, but at a higher A, than the initial value Figure 5.21 is similar to 5 20 except that the increments of Tj, and Egy are each 20%, She system is initially at 90% load and 90% £,p(0.9 x 2666 = 2.399) Then a 20% step increase in J,, is applied The result is a fast movement toward instability, as evidenced by the rapid increase in 6 and the drop in terminal voltage A 20%, increase in Epp is Simulation of Synchronous Machines 183 Table5.2. Comparison of Digital and Analog Computed Variables Analog computed values arial ruted value pu Percent Variable Computed value p y me falas v, 1732 68.66 77 -090 % - 1092 44.12 = 1.103 =101 % 1344 5263 1316 210 iy —1925 ~ 38.39 -1920 029 i 0.667 13.42 0671 0.60 tb 2979 Xap 1.634 4812 1604 =184 de 0.994 3010 1.003 094 ae 1345 39.49 1316 -213 My 1.094 33.10 1.103 085 Ms 1935 19.04 1904 160 Tx 3.004 2997 2997 010 & 66.995 33.89 61.78 Lit *Angle between q axis and inf applied at about the time 6 reaches 100", and the system is quickly restored to a stable operating state Finally, the excess load and excitation are removed Figure 5.22 shows a plot in the phase plane, or ws versus 5, for exactly the same dis- turbances as shown in Figure 5.20. The system “spirals” to the right, first very fast and later very slowly, following the 10% increase in J, Just prior to loss of synchronism a Fig $22 Phase-plane plot wg versus 5 for a 10% step increase in T followed by a 10% step increase in Exp (see Figure 5.20) Initial conditions of Example 5 1 184 Chapter 5 Fig 523 Phase-plane plot wg versus & for a 10% step increase in Tzy with initial conditions Ty = 09, Exp = 2666 10% increase in Ezy causes the system to return to about the original 6, following along, the lower trajectory Figure 5.23 shows an example of a stable phase-plane trajectory. The system is initially at 90% load but with 100% of the Example 5.1 computed value of Esp, or 2 666 A 10% increase in Z,, causes the system to oscillate and to seck a new stable value of 6 A comparison of Figures 5.22 and 5 23 shows the mote rapid convergence to the target, value of 6 in the stable case. 5.10. Digital Early efforts in solving synchronous machine behavior by digital computer were simply digital applications of the constant-voltage-behind-teansient-reactance model, using a step-by-step solution method similar to that of Kimbark (7] As larger and faster computers became available, engineers quickly realized that the digital computer was a powerful tool for handling very large systeins of differential equations. This caused an expansion in power plant modeling to include exciters, governors, and tur- ines, It also introduced more detailed synchronous machine models into many com- puter programs, usually in the form of one of the simplified models of Section 4.15 More recent research {8, 9] has been aimed at finding the best machine model for system dynamic studies All digital computer simulations must solve the differential equations in a discrete manner; ie, the time domain is broken up into discrete segments of length t4 and the equations solved for each segment A simple flow chart of the process is shown ulation of Synchronous Machines Simulation of Synchronous Machines 185 iy eet cne reel Fig $24 Flow chart of digital integration in Figure $24 There are several proven methods for performing the actual numerical integration, some of which are presented in Appendix E Our coneetn in this book is not with numerical methods, although this is important Our principal concern is the mathematical model used in the simulation A number of models are given in Chap- ter 4 We shalll use the flux linkage model of Section 4 12 to illustrate a digital pro- gtam for calculating synchronous machine behavior in a numerical exercise 5.10.1 Digital computation of saturation One of the problems in digital calculation of synchronous machine behavior is the determination of saturation. This is difficult because saturation is an implicit func- tion: 1¢., Mao = f(Aan). Actually, Ngo is a function of iyo = ty + ix + in, which flows in the magnetizing inductance Ly». But the currents i, is, and ip depend upon Aso, as shown clearly in the analog computer representation of Figure 5.12. Each integra- n step gives us new 2's by integration From these \’s we compute yo From Jn we estimate saturation, which gives a new Ao, and this gives new currents, and soon The first requirement in computing saturation is to devise some means of deter- mining the amount of saturation corresponding to any given operating point on the saturation curve For this procedure the saturation curve is represented by a table of data of stator EMF corresponding to given field current, by @ polynomial approxi- mation, or by an exponential estimate The exponential estimate is often used since exponentials are easy to compute It is based upon computing the offset from the ait gap line in pu based on the field current required to produce rated open circuit voltage, shown in Figure 525 as iro Usually it is assumed these is no saturation at 98 pu 186 Chapter 5 Teminal Voltoge, Vj, PU Foie bs Fig $25 Estimating saturation as an exponential function. voltage. We then compute the normalized quantities =12 Sa = BE Sen “a tee a in esp Then any saturation may be estimated as an exponential function of the form Sq = Age’e”s (5.82) where Vy = V, — 08. Since at open circuit Ayo = W/3V,, we can also compute satura- tion in terms of Kao, Sq = Acexp{(so/V3) — 08) (5.83) This is appealing since Xap = (ig + é + ip)Lan and Lap is the only inductance that saturates appreciably If Sq and Sg2 are given, these values can be substituted into (5.82) to solve for the saturation parameters dg and Bg. From (5 81) and (5 82) we write Sor = Age? 1.2Sq, = Age ?* (5.84) Rearranging, we compute In(Se1/4e) = 02Bg —In(1.2S¢2/4e) = 0.4Be (5.85) Then 0 4B, = In(1.28¢;/Ag) = In(Sei /Ae? or Ag = Sbi/1 Sex (5.86) This result may be substituted into (5 85) to compute Bo = SIn(1 28¢2/Se1) (5.87) Appendix D shows a plot of Sq as a function of ¥, The function S¢ is always positive and satisfies the defined values Sg, and Sgz at V; = 1.0 and 1.2 respectively. Although we define saturation to be zero for ¥, < 0.8 pu, actually S, assumes a very small posi- Sinivlation of Synchronous Machines 187 tive value in this voltage range The exponential function thus gives a reasonably accurate estimate of saturation for any voltage From (5.81) we can write for any voltage level, So = (ir ~ Kiro)/kizo (388) where ip is the field current required to produce an open circuit voltage V,, including the effect of saturation. If the air gap line has a slope (resistance) R we have V, = Rkiso. Then, from (5.81) So(V;) = (Riz ~ Rkigo)/Rkizg = (Rip ~ V;)/¥, from which we may write the nonlinear equation Vi = Rix — ViSo(¥.) (5.89) where Ri, is the voltage on the air gap line corresponding to field current ip Be- cause of saturation, the actual terminal voltage is not Ri; but is reduced by an amount V,Sg where Sq is a function of ¥, Equation (5 89) describes only the no-load condi- tion. However, we usually assume that saturation has a similar effect under load; ie, it reduces the terminal voltage by an amount V, S« from the unsaturated value, Example $7 Determine the constants Ag and Bg needed to compute saturation by means of the exponential definition, given the following data from the saturation curve W210 pu Sq = 30 A V=12 pu Sq. = 120 A The field current corresponding to V, = 10 on the air gap line is iz) = 365 A. Solution From (5.81) we compute in pu So: = 30/365 = 0.08219 Sea = 120/1 2(365) = 0.27397 Then from (5 86) Ag = (0.08219)?/1,2(0 27397) = 0.0205 and from (5.87) Be = Sin[1.2(0.27397/0,08219] = 6.9315 5.10.2 Updating the integrands ‘After computing the new value of saturation for each new time step, we are ready to update the integrands in preparation for numerical integration. This process is illustrated by an example Example 5.8 Prepare a FORTRAN computer program to compute the integrands of the flux linkage model for one machine against an infinite bus using the machine data of the Chapter 4 examples Include in the program a treatment of saturation that can be 188 Chapter 5 SeRACONTINUGUS SYETEM MODELING PROGRAMREES 348 VERSION 1.3 44% INITIAL MACRO SG2GENSAT(WADS) KEXPERGSAT ( (WANS /UT3) 0.0) SGEAGSAT#ERE (KEXP) FRDMAC INITTAL CONSTANT —PT=3,141592H5+8L.02100409CL0=0.07 ©) MUST SPFCTFY GEVEWATO® POWFHEPGEN AND GENEKATOK TEBMINAL vO, TAGE ® VT AND INFINITE BUS VOLTAGE* VINE PGFNE1, 00 vistel? VINFS1,000 TITLE SATURATED SYACHKONOUS GENFRATOR WITHOUT EXCITER CONST NP=2,09HC=2,57 eRMVAS 160, 04KK E15, 06RPF=0,HDeX0=1, 704 TIC CONST XO=1,64 4 KPE0.265 4 KOPPHU 1859 XUPP=O, 1654XL CONST RFOHMEN, 7578715544 FLUH365, 00 10PP= Paka xF 20.4 Pakam WEEN 02 CONST SATCHO=0,0822«SATGI2=0.3288 PARAM EXCON=0401 PaRaw TSTART S062 Panam A09KEFELQU FIXED © SATUMATION FUNCTION FOR GEWERATOR PROCENUMAL AGSATHHGSAT=SATUR (SATGIU+SATGI2) IF (SAT610.E9,0,0) 40 TO 20 AGSAT=(SATG102#7)/(1.28SATG12) HGST=#5.M@AL06 ((1420SAT612) /SATRIOD Go To 30 20 assaT=o.0 RGSAT=0.0 30 CONTINUE, ENDPRO ® Compute INITIAL CONDITIONS 29 A 1S;RAGHMER.OOLITS. B34 TOUPHRU 008K RY IESE Lawexu: AN (XOP LAD (LADSADPSL AD ADOLE, LCowexpee ca FOL CONAL,AD/ (LF LAD=LFE A CON) SL AUS (AUPP MLA) /(LAG@XOPP EL AD Thastel 607 (24819604) VHASESSHKVO1000,0/2T3 SHASESRMVAP10RO2I0—0/3 60 THASESSSHASE /vetanh S RHASESEVBASE S/ [4 ASES NAASESeVRASESOTHASE CHASE S=HR ASE S8 THOSE LOMExDSLHASES. LASLHASES Dest an RT?S50RT (260) MP HeRT2OVHASESSTHASE/IFLD RAPHRRTSOMPH/RT2 TEL DRS THASES®L ADH/KMFH Fig 5.26 CSMP program for computing initial conditions ‘Simulation of Synchronous Machines 189 VFI DRESHASE/TELOR RFU DBEVFLDA/TFLON LALDR=RF LOB THASE MF RASE=SQRT (LRASES*LFLOAD wELOMEVELDROTRASE RAS (HAQHM® 359451 7 (RBASES #25965) Umbed O70 (Le ZLAD) + (LQ UZAY 01, 97LE) +0 eOZLEID CMGELeO7 CCL SO/LAQ) + (1¢0/L.8) + (1607EK0) Le sxe © PROVIDE LOGIC FOX CASE WHERE TDPP AND TAPP AWE MISSING PROCEDURAL KKDARKOSTFIN(LKD LRGs OMHy TOPPOTOPP SL AD sLAGHL ACLFD IF(TOPP 4EQ. 5.0) GO TO 196 RKDELKHZ (O4HOTNUP) ¢ (LADSL ASL FD /(OMROTNDBE (| ADSL ASL ADSL SL ARLEN) Go To 105 106 RKM=1. OER 105 IF (TOP Ea. 040) GOTO 19e RRUELKOZ (OM ACTORP) 6 (LAGSLAD /(OMROTOPHA (LAD+L AD) 60 To 110 106 eKn21.0b +8 lio Owurl 60 ENDERA * PRAVINE LOGIC FOR CALCULATING BEST POSSTHLE RF PROCEDURAL RF=THFID(TUGP +LEF ¢LFLDS +HFLUK SPE OHM) IF (TDOP.E4040) GO TO 120 R= (LEFOLELIMD / CTHOPERFLUMD Go TO 125 120 BF = (REOHM#359,5) /(RELDROZS9,S) 125 CONTINUE FRDPRO BT32S0KT 340) PRR THO .O/ET p21 20.081 ZESERES#Z + XE SOR G12=-RE77ES B12sxe/7ES: GaMMaspLeaTan (HLe/612) G1G21.07KL0 ie=G12 YIPESUKT (G1 7HRCSH12902) Nunepatnso) Leyree2 Deneylzev levine FACEHUM/ EN ONT Lenk aceeey TAN (UM/FAC AMMAR ZETA cosal scons tera STWALSSTN (HE TAD TUES VT® ((COSALZHLD) #(SINAL SCL) ) ILI mavT8 (CS TNALZHLD) # (COSALSCLDT TIRES (RES (VISCOSAL “VINE ) exXPAVT®STNALI/ZES IT Te (HES VTOSINAL @XE® (VT ECOSELAVINE}/ZES. TARESIL@E+[1RE TatMe re Twel fin THETASATAN (TAIN/TORE) TassarT (ave eezsTarmee2) EMRE HVTACHSAL +2A8TASCOS (THE TA) =¥ 99, AOSTN (THETA EQTMSVTOSINAL +A 1AOSIN( THETA) +#Q8TA*COS(THFTAD Eqnssnyut (016s 82eeoTue#2) TECHKEAPGENS3HWAN TARE? OL SATAN (EOIN/EQRE} PROCEDURE VD VOeTU +1040 +404 WANS em 00S 9SG0 «S604 TFUNe [MU VTEDFL eELOL 9 TReNNSFUNC (OIL +VT ¢ TA9RE Tae THETA SRT 342A sLADsLAGLFOLAQs TECHEY Neo Fig. 526 (continued) 190 Chapter 5 100 vos-RTRSVTESIN (DL =HETAD Vee T3@VTOCOS (1 -HETA) IDS=RT39 TAS IN (UL =THETAD Ty2QT381A9C0S (DL=THETAD * UNSATURATED FIELD CURRENT = TFUN TFUINS ((VQ+RA®T OY /L AD) = ((LB+LAD9 BI} AL AD. TMUsID6 FUN wanseL ane Tau © SATURATED UAXIS FLUX LINKAGES SGN=GENSAT (WANS) WOE AO THe WADS 4 SATURATED WeAXIS FLUX LINKAGES waozeLanero WAOSETMPL (WAZ ¥0,00001 +6898) SGO=GENSAT (WAQS) GANSEL AUS 127 (1604569) WORL Ae TUSWADS, TEEWD® TU-"Oe TD TENEL =TE=TECHK DELDL=O41® (TENEL /TECHR) @T AN (OLY IF(DELDL@T.UsnyoUL) GO TH 209 IF(DELDL«LT.-U40:0001) 89 TO 200 G0 To 400 200 NL=nl eDeLOL NNENNE 1 TF (YN.6T.50) GOTO 400 60 To 100 400 CUNTINUE ENuPRO © FINAL INITIAL CnmouTATIONS TEFSIFUN*S6D8 TMU WRSLEOIFF*KADS wknzwans: Wnozweas, VE RRESIFE ULUSDPREDL TTMaG2SONT (TIRE CMDs TTINERZ) PSLSATAN(ITIM/ITHE) WT IOLTHAGESIN(DL=PST) Toreat3éj Wagecis (0L=PS1) Thos t0=fbT TLno=te-tar TMETE Taed.0 VICHK= (1e/hT3) @SURT (VOS#Z4 4802) wOz=00 ME Z=0F DoMZ=00MU DLZ=0L, 1oTZ= 10 yorz=iaT THz=T™ vozevo, Vozsve KP ORRTSORE/LAD EF D2VF/KFD) » NosoRT Fig $26 (continved) Simulation of Synchronous Machines 91 executed prior to integration at each time step. Include a local load on the generator bus in the computation Use the Continuous System Modeling Program (CSMP) [10] for solving the equations and plotting the results Solution An essential part of the computer program is a routine to compute the initial condi- tions. As noted in Examples 5.1-5 3, this computation depends upon the boundary conditions that are specified The boundary conditions chosen for this example are those of Example 53, viz,, P and ¥; at the generator terminals, The FORTRAN coding for this section of the program is included in the portion of the program listing in Figure 5 26 called INITIAL Note that the statement of the problem does not give any explicit numerical boundary condition. This is one of the advantages of a com- puter program; once it is written and verified, problems with different boundary condi- tions but of the same type can be solved with case. The boundary conditions specified in Figure 5.26 give P = 1,00 (PGEN), ¥, = 1.17(VT), and V, = 1.00(VINF), 1 Make a preliminary estimate of Xp (Aan is named WADS in the program; W being used for ) and S meaning “saturated”). Dao = Molina = WADSIiscar (590) 2 Compute the new currents. From the equations ty = Os > Mo)/ ta te = Qu — Mao)/te io = Qo - Mote imo = ta + ip + io (5.91) we compute an estimate of the new currents, This estimate is not exact because the value of Ayo used in (591) is the value computed at the start of the fast Ar, whereas the flux linkages Ay, Az, and Ap are the integrated new values. Thus iavo computed by (5.91) does not correspond to point A of Figure 5 27, but to some new point B Since \gp is a function of the currents and of saturation, we must find the correct new Ayo iteratively. We do this by changing our estimated 4p slightly until iyp agrees with X4p on the saturation curve, or until points A and B of Figure $27 coincide. 3. To estimate the new Ago, we compute the saturation function Sg = f(Aso) in the Fig $27 Saturation curve for the magnetizing inductaace Lao 192 Chapter 5 ornseitc nosoT 3 entaarow cunnens, wansoeeans OL #A0S00 404.0001 ¥F KADY T= (xD=6a0S) Aa ipestaronabs) AP Troe (annenas) 7160 Tapelbetee ede GF usaT (wA0S) Ganset Abel <7 (2.08560) Fratimeatise (€a0S=WRNS) 8EXCON VehgSeTHht (42080404001 -FuAD? Tos tweens) 78 prrertvery ete SuneGensat (aus) BagseLAceteus (1 aneSsay Fanaesauge road52aacs XEON sont 2 rawour Teempernenueto mere 2 seers Janqwerar cma vexey Dowustntises (0424 15LOKD HtimODme Le 3 aN Tene somennen USTNTIseL (DL 24 T69U) blbeveneut © Loca coat TRVES Come 76bO9 # (T= tT c¥O/RL DN = ComUECLDBYED vierntawe (ube 20? Bove (Om C7 @ {Tem LT = (WOsRL + COM/eCL EVD Y VasTNF ML (vnZaT SVD © Teansmissrom Lise TGLOT# (Owi/ ED © (ven t3e9 IN COLI =PFSTOT-UMeLE*TOT) Totstareat (iptzeioi0T) TOPOrs (OmK/LEDe (vlnHeT SeCUS (OL =HEATOR sOMINLEETETY toretnraay cturee retary : HTS FLUE Linanses. rabsnnns {-vo~ (waeTO> ~neUeWa® wOsIN Tae exey Ean) veeerostry foFsoube uFant eTFE SEatMaol Cur re FOF Toxyeomie (acter Beebe ENT aa wRDZSTORDD BATS FLUE LINKAGES reustiulie (vas (RAS Te) eoKUeRD) NoeTTEAL (in zs 16) Foxa=onne (wwe TRO) wkue TNTSAL CwrinZe 1 6KU TERMINAL VOLTAGE WIEC(ubeszovaen2) #90,5) 7873 TAFLCIbesestoeae) ooo es) 7873 praoneles3uo © oRIvENG FoNctToys. TeeTHZ sxTHOTAL7 10, 00STERCTSTANE) EXDnER OU “KEP eFFOLZ2USUSSTED(TATzET) Roser Fig $28 CSMP program for updating integrands, Simulation of Synchronous Machines 193 remerea, fance Oe) aurpuT #n PAGE SenNPs (2.5541, 75) aureur WF PAGE GHOUP* (2142039 euTPur «#0 BAGE GuOIPE(L aMe?aut ureur ans PAGE. GROUPE (1.5920) pureut S60. PAGE OHNIPE(0.1240.24) outeur Te Pree GHnUPe(U9Re IH) nutvur IF PAGE GROUPE(34043.264 aerbur THD PAGE AROIER (=U 0240.96) purvur PAGE GHMIMECL 591.15) eurour ol Page. guarIb= (50442) ouTPoT unwy PAGE GunuiPa (=0,0018¢04902) euTeur TF Pace RAMID=(20R 1206) THaEe INTIS? 5 sUTDEL #0 .0597ELT=0,0001 FNO stor Fig. 5.28 (continued) usual way, using (5 83). Then we compute Ag and Ay, defined in Figure 5 27, Yo = Auo(l + Seo) Aw = Laotian Then the error measured on the air gap line is Ap = Ay — Ao, and the error mea- sured on the saturation curve is approximately Ag = Ae/(I + Seo) Now define a new Ayp t0 be Gap, defined as Gao Gan = Nav + (Aw — Ao)/(l + Seo) = Lavtuo/(l + Sen) 4 Now we test Gyp to see if itis significantly different from Ago; i¢., we compute Dao + Ax Then we compute J Gyo - dal ee where € is any convenient precision index, such as 10+ If the test fails, we esti- mate a new gp from NeW Neo © Fan = dav ~ Gan — dav) where A is chosen to be a number small enough to prevent overshoot: typically, h = 0.01. Now the entire procedure is repeated, returning to step | with the Av = Fp, finding new currents, ete As the process converges, we will know both the new current and the new saturated value of Xap The second part of the program computes the integrands of aif equations in prepa- ration for integration (integration is indicated in the program by the macro INTGTL) The computer program for updating the integrands is shown in Figure 5 28 The computed output of several variables to a step change in I, and Esp is shown in Figures 5 29-5.40 Computer mnemonics are given in Table 5.3. In both cases, the step input is applied at 1 = ISTART = 02s Response to a 10% step increase in Ty, 1.7023 — Time, + 5% step increase in Erp Response toa 569 — 1 Fig 5.29 daxis flux linkages Ag Response to a 10% step inereuse in Tp 2.2548 — 5% step increase in Exp Response to a Fig. 5.30. Field flux linkages he Response to a 10% step increase in Tr, %4 step increase in Exp Response to 2 5 Time, 5 axis amortisseur flux linkages Ap Fig $31 196 Response to a 10% step increase in Ty é g % Response to a 5 Fig 532. Saturated d axis mutual flux linkages Ans 197 Response to a 10% step increase in Ti, 0.21119 — Time, z : 3 i 2 . 3 3 i a i 0.17446 —: we Response to a 10% step increase in Tm, step increase in Erp Response 10 a 5% 1.1886 ~~ = = =e eo - 1 Fig 534 Line currentig 199 7, step increase in Ty, Response to a I 3.1063— step increase in Exp Response to a 5% Time, Fig 535. Field current step inerease in Ty Response to a 10% Time, s Response to 2 5% step increase in Epp 0.03075 — Fig 536 daxis amortisseur current ip 201 Response to a 10% step increase in J, step increase in Exp, Response to a 5% 1,.1708— Terminal voltage ¥; Fig 537 Response to a 10% step increase in Ty, 5 Time, 3% step increase in Exp Response (0 a 5 36— 6 Fig 538 Torque angle in degrees. 203 Response to a 10% step increase in Ty step increase in Exp Response to a 5% 0.00185 — Fig 539 Speed deviation wy in pu 204 3 5 g $ a 2 $ § be & g | s se g = a 2 & 3 a 2 2 g & & = 2 é 2 9.5638— 205 206 Chapter 5 Table$.3. Computer Mnenomics of Output Variables Figure Variable Computer mnemonic 529 de wD 5.30 ae Wwe 331 Xo WKD 532 avs WADS 533 Sop SGD 5.34 i, TA 535 ip IFF 5.36 ip IKD 5.37 % vr 538 8 (in degrees) DLD 539 wa (in pu) DOMU 5.40 Tes TE Problems 51 Thesynchronous machine discussed in Examples 5.1 and 5 2 is operating at rated terminal voltage, and its output power is 0.80 pu The angle between the q axis and the terminal voltage is 45°, Find the steady-state operating condition: the d and q axis voltages, currents, flux linkages, and the angle é 52 The same synchronous machine connected to the same transmission line, as in Examples 5.1 and 5.2, has a local load of unity power factor, which is represented by @ resistance R = 10 pu’ The infinite bus voltage is 1.0 pu. The power at the infinite bus is 09 pu at 09 PF lagging Find the operating condition of the machine. 53 Repeat Problem 5.2 with the machine output power being 0.9 pu at 0.9 PF lagging 54 In the system of one synchronous machine connected to an infinite bus through a trans- mission line (discussed in Examples 5 1, 5 2, and 5 6) the synchronous machine is to be represented by the simplified model known as the one-axis model given in Section 415 Prepare a complete analog computer simulation of this system Indicate the signal levels for the operating conditions of Example 5.1, the amplitude and time scaling, the po- tentiometer settings, and the amplifier gains Note: In the load equations, assume that Leia = Leig % 0. 55 Repeat Problem 5.4 using the two-anis model of Section 4 15 56 Repeat Problem 5 4 using the voltage-behind-subtransient-reactance model of Section 4.15 57 Inthe analog computer simulation shown in Figure 5 13 and Table 5.1, the time sealing is (20). If the time scaling is changed to (10), identify the amplifiers and potentiometers in Table 5.1 that will be affected 58 In Figure 513 the signal to the resolver represents the infinite bus voltage If the level of this signal is reduced by a fuctor of 2 while the level of all the other signals are maintained, identify the potentiometer and amplifier settings that need adjustment References 1 2 3 IEEE Committee Report Recommended phasor diagram for synchronous machines JEEE Trans PAS.88:1593-1610, 1969 Krause. PC. Simulation of a single machine-infinite bus system Mimeo notes, Electr Eng Dept Purdue Univ , West Lafayette, Ind , 1967 Buckley, D. F Analog computer representation of a synchronous machine Unpubl MS. thesis, lowa State Univ , Ames, 1968 Riaz, M. Analogue computer representations of synchronous generators in voltage regulator studies AIEE Trans, PAS-T5:1178-84, 1956 Schroder, DC, and Anderson, P.M Compensation of synchronous machines for stability Paper C73 313-4, presented at the IEEE Summer Power Meeting, Vancouver, B C., Canada, 1973 Electronic Associates Ine Handbook of Analog Computation 2nd ed Publ 00800 0001-3. Princeton, NJ. 1967 10 Simulation of Synchronous Machines 207 Kimbark, E W Power System Stability, Vol 1 Wiley, New York, 1948 Dandeno, PL. Hauth,R Land Schulz, RP. Effects of synchronous machine modeling in large-scale, system studies. IEEE Trans PAS.92:S74-82, 1973 Schulz, R-P Jones, W D., and Ewart DN Dynamic models of turbine generators derived trom solid rotor equivalent circuits. IEEE Trans PAS-92:926~33, 1973, International Business Machines System/360 Continuous System Modeling Program Users Manual, GH20.0367-4 IBM Corp., 1967 chapter © Linear Models of the Synchronous Machine 6.1. intreduction A brief review of the response of a power system to small impacts is given in Chap ter 3. It is shown that when the system is subjected to a small load change, it tends to acquire a new operating state. During the transition between the initial state and the new state the system behavior is oscillatory. If the two states are such that all the state variables change only slightly (ie, the variable x, changes from xo to Xe + Xq Where xq is a small change in x,), the system is operating near the initial state The initial state may be considered as a quiescent operating condition for the system To examine the behavior of the system when it is perturbed such that the new and old equilibrium states are neatly equal, the system equations are finearized about the guiescent operating condition By this we mean that first-order approximations are made for the system equations The new linear equations thus derived are assumed to be valid in a region near the quiescent condition The dynamic response of a linear system is determined by its characteristic equation (or equivalent information) Both the forced response and the free response are de cided by the roots of this equation. From a point of view of stability the free response gives the needed information If it is stable, any bounded input will give a bounded and therefore a stable output The synchronous machine models developed in Chapter 4 have two types of non- linearities: product nonlinearities and trigonometric functions The first-order approxi- mations for these have been illustrated in previous chapters and are outlined below. ‘As an example of product nonlinearities, consider the product x;x;. Let the state variables x; and x; have the initial values xq and xj» Let the changes in these variables be xis and xj. Initially their product is given by x;oxjo. The new value becomes Gin + Xiao + ja) = Xioxye + Mw%a + KYoXia + Xara The last term is a second-order term, which-is assumed to be negligibly small. Thus for a first-order approximation, the change in the product x,x, is given by Cio + Xia)hXo + Xa) — XoXo = AjoXg + Xo%a (6.1) We note that xj9 and vo are known quantities and are treated here as coefficients, while x, and x, are “incremental” variables 208 Linear Models of the Synchronous Machine 209 The trigonometric nonlinearities are treated in a similar manner as 08 (6g + 84) = C08 6y.c08dy ~ sindysin dg with cosy & J and sindy % 63. Therefore, £05 (65 + 34) ~ cos 8y & (sin do)og (62) The incremental change in cos 5 is then (—sin 6)4q; the incremental variable is 64 and its coefficient is -sin 4. Similarly, we can show that the incremental change in the term sin 6 is given by sin (6) + 54) — sin dy & (COS do) 5s (63) 62 Linearization of the Generator State-Space Current Model Let the state-space vector x have an initial state x5 at time ¢ = fo; eg., if the eur- rent model is used, Xb = Liao tro 40 Hg0 igo @o Bo} (64) At the occurrence of a small disturbance, ie. after ¢ = #%, the states will change slightly from their previous positions or values. Thus X= Xo + Xy (65) Note that xq need not be constant, but we do require that it be known The state-space model is in the form x = 60,1) 66) which, by using (6 5), reduces to Ke + Ky = HK + Xd) 67) In expanding (6.7) ail second-order terms are neglected; ie, terms of the form Xia%yq Ate assumed to be negligibly small The system (6 7) becomes Ko + Ka (Xo,t) + AlK)xs + Bixo)e (68) from which we obtain the linearized state-space equation iy = AGa)xa + BOo)e (69) The elements of the A matrix depend upon the initial values of the state vector Xo. For a specific dynamic study it is considered constant. The dynamic properties of the system described by (6.9) ate determined from the nature of the eigenvalues of the ‘A matrix The state space may be thought of as an n-dimensional space, and the operating conditions constrain the operation to a particular surface in this m space. Being non- linear, the surface is not flat, although we would expect it to be continuous and rela- tively smooth, The quiescent operating point x» and the functions A(xq) and B(Xo) are different for every new initial condition, We may also compute the A(xo) by finding the total differential dx at xp with re- spect to all variables; ie, with dx = xy 210 Chapter 6 where the quantity in brackets defines A (Xo) We begin by linearizing (4.74), proceeding one row at a time For the first equa- tion (of thed circuit) we write Pye + Vas = —Tliso + daa) — (Wo + ws )Lgliga + tgs) — (wo + wa)k Mo lige + igs) —Lalisa + jen) — kM lira + fra) — KM (ioe + tna) Expanding the product terms and dropping the second-order terms, Veo + Vag = (—rigg — @oLgign — wokKM eign — Lely — kKMrizo — kM ipo) wring = Walgiga — alga — VokMoiga ~ igokMgus —Lilaa - kMpizy — kKMoing exactly equal to vg, Rearany The quantity in parenthesis on the right side remaining quant ves = ~tlea — Wolgina ~ wokMoioa ~ (iol, + kMoign)ey ~Loias — KMeiny — KM ping (610) which is equal to oa — Lalag — KMrizy — KMolog (611) Vea = — Naa — Col gia — wokM eign — Similarly, for the axis voltage change we write vy = erlaisg + GOKMeicy + wokMins + (ile + ieakMr + inokMy)ws ~ tiga — Lgign ~ kKMoign (612) which is equal to gs = Wolgias + WokKMsirs + wokM pigs + AuoWs — Ties — Lolya — KMoloa (6.13) For the field winding we compute bra = Pring ~ kM risa ~ Lrirg — Matos (614) The linearized damper-winding equations are given by 0 = =rpina — kKMpiss ~ Miss - Loins (6.15) 0 = ~rpigs — kKMoiza — Liga (6 16) From (4 101) the linearized torque equation may be established as 18s = (1/3 —Leigoiea — Lalas ~ KMrigoira — KMriroiga — KMpigoina ~ KMoioviea + Lalavica + Leigoian + KMpiwign + kMoigaiza) - Dus + Tra (617) Linear Models of the Synchronous Machine 211 which can be put in the form Ty3 = Tma — (1/3)[Latga — Agodiea - Oso ~ Leisodion — KMrigotes ~ KMpigving + KMgizviga) — Dos (618) Finally, the torque angle equation given by (4.102) may be written as bs = 5 (619) Equations (6.11)-(6 19) are the linearized system equations for a synchronous machine (not including the load equation) If we drop the A subscript, since all variables are now small displacements, we may write these equations in the following matrix form: ve r 0 el, wok Mg ~ or in matrix form ve -Kx — Mk pu (621) Note that the matrix M is related to the matrix L of equation (4,74) by Oj -1 0 ho 1 Assuming that M~! exists, the state equation for the synchronous generator, not in- cluding the load equations, is M“Kx — M-'y pu (6.22) 212 Chapter 6 which is the same form as k= Ax + Bu (6.23) Example 6.1 ‘As a preparation for later examples involving a loaded machine, determine the matrices M and K for the generator described in Examples 41-43, Let 7, = 2Mon = 1786.94 rad Solution The matrix M is related to the matrix L of Example 4 2 as follows Lj; 00 M=lo}-7, 0 Oo; o1 Then we write 1.700 1550 1550 1550 1651 1.550 0 0 1550 1550 1506 M= 1 4640 1.490 | 0 ' { 0 | 1490 1526 | ! | = 1786.94 0 0 1 0 i ' 1 0 4 The matrix K is defined by (6 20) 0.0011 0 of 164 149 do 0 9 00007 =o 0 0 o 0 0 0 ool | 3 3 3 0 eo 4 When the machine is loaded, certain terms in these matrices change from the numeric values given to reflect the impedance of the connecting system For example, when loaded through a transmission line to a large system, r, Lz, and L, change Linear Models of the Synchronous Machine 213 to R, L,, and L, as noted in Section 413. Other terms are load dependent (such as the currents and flux linkages) and must be determined from the initial conditions 63. Linearization of the Load Equation for the One-Machine Problem Equation (4 149) is repeated here for convenience: = 1" ~Ksin(6 - a) + Reis + Leig + wLeiy vy = Keos(é - a) + Rei, + L. where K = V3V. and a is the angle of P. The same procedure followed previously is used to linearize this equation, with the result (624) Vea, = —K 008 (bq = 5g + Rela + Oobetga + tyol ews + Leigy Uys = —Ksin (So — aby + Reign + Lela — @oLetea — iaoleox (6.25) Substituting (6 25) into (6 11) and (6.12), ~Kcos(do — @)4 + Reis + Leigy + WoLsigy + ok ewa Naa ~ WoL gia — okMoigg — Ayows ~ Laisa — kKMeiza ~ kKMvins ~Ksin (6) — aby + Relea + Lela ~ wolelas — toLews = wolgigs + @okMsizg + @okMping + Neowa — Tiga ~ Lelja — KMgigg (6 26) Rearranging (6 26) and making the substitution MaMa bee Ly = by be Neat hig Rar tR bya bgt Le (627) we get, after dropping the subscript A, 0 = -Rig— wolzig — wokMoig — Ayow + Kc0s(bo - a)6 — Egig — KMeip — kKMyip pu 0 = —Rig + wolsig + wokMels + wok Moin + Xuow + Ksin (6) - a)5 - Lyi, — KMoig pu (6.28) Combining (6.28) with (6 14)-(6 16), (6 18), and (6.19), we get for the linearized sys- tem equations 24 Chapter 6 ‘, (629) Equation (6 29) is a linearized set of seven first-order differential equations with constant coefficients In matrix form (6.29) becomes v = —Kx ~ MX, and assuming that M~ exists, x =M*'Kx - M-4y = Ax + Bu (6.30) where A = -M-'K. Note that the new matrices M and K are now expanded to i clude the transmission line constants and the infinite bus voltage It is convenient to compute A as follows Let Then (6.31) M;'Ks; Note that the only driving functions in the system (6.29) are the field voltage U4 and the mechanical torque I, Initially, the machine is spinning at synchronous speed and is delivering some known power to the infinite bus A change in either Us or T» will cause the system to seck a new operating point, and this change is usually accompanied by damped oscillations of the variables Example 6.2 Complete Example 61 for the operating conditions described in Example 5 2, taking into account the load equation Find the new expanded A matrix. Assume D=0. Linear Models of the Synchronous Machine 215 Solution From Example 5 2 we compute R = 0.0011 + 0.020 = 00211 Es = 1.700 + 0.400 = 2.100 £, L, = 1.640 + 0.400 = 2040 a The matrix M is given by 2100 1550 1.550 1550 1651 1.550 1550 1.550 1.605 —17869 0 0 0 i 0 1 We also compute, in pu, Kuo = 1.676 + (—1991)(0.4) = 1.039 Ago = 1150 + (0.701)(0 4) = 1.430 K cos (39 — a) = V3(cos $3,735" 1.025 Ksin (jy — a) = Wisin 53.735") = 1397 4 oq = Laty) = ESD = 10x 0701. o14 4 (kblsi) = SESSA OTOL 9 369 4 ebtyi) = Oot LF 1590) 1 gg -0 790 LG Meln) _ 1.490(~ 1.591) The matrix K is given by 0021 0 0 1430-1025 0 0 2,040 1 1039-1397 0 216 Chapter 6 36062 0.439 14142 | ~348718 2547.01 | -244463 1751.33 12472-4950 76857 | 120601 $8ORE | BAS 4G 605.68 ' 290243 1608.63! 2776 © 4356 © -96017 1543.98 —1106 10 A=| 358995 264972 264972 ~ 3505.70 —258754 —258754 -36.064 90072 35218 — 123.320 177671 2387.40 [1079 =1735 01-2331 37 0.0078 02027-02027 | 07993-04822 | 90 00 00 00 oo | 00 oo ft 10 00 Example 6.3 Find the eigenvalues of the A matrix of the linearized system of Example 62 Examine the stability of the system. Generator loading is that of Example 5.2 Solution To perform the computation of the eigenvalues for the A matrix obtained in Ex- ample 6.2, a digital computer program is used, The results are given below Ar = 0.0359 + 0.9983 As = -0:0016 + j0.0289 Ar = ~0.0359 - j0.9983 Ag = -0 0016 — j0.0289 ds = 0.0991 dy = -0.0007 Ag = 0.1217 All the eigenvatues are given in rad/rad, Note that there are two pairs of complex eigenvalues The pair As and A, correspond to frequencies of approximately 1.73 Hz3 they are damped with a time constant of 1/(0.0016 x 377) or 1668 This complex pair and the real pole due to \; dominate the transient response of the system. The other complex pair corresponds to a very fast transient of about 60 Hz, which is damped at a much faster rate This is the 60-Hz component injected into the rotor circuits to balance the MMF caused by the stator de currents Note also that the real parts of all the eigenvalues are negative, which means that the system is stable under the conditions assumed in the development of this model, namely small perturba- tion about a quiescent operating condition Example 64 Repeat the above example for the system conditions stated in Example 5 | Solution A procedure similar to that followe: results: Examples 6.2 and 63 gives the following ~36.062 0439-14142 | 3487 18-2547 01! -232701 958.54 12472 ~4.950 76857 | 120601 $80.86 | 80478 ~331 50 22776 4.356 «= 96017 | 220243 160863} 1469.69 —605.39 90071 | 98266 2257.70 }0* = 1233 20 | -959.60 -2204 72 A=) 3589.95 264972 2649.72} 3505.10 258754 —2587 5: -00075 0.1929 -0 1929 00 00 00 00 00 Linear Models of the Synchronous Machine 217 and the eigenvalues are given by A, = 0.0359 + 0.9983 A, = -0.0009 + j0.0248 dy = -0.0359 ~ j0.9983 A, = 0.0009 ~ j0.0248, ds = 00991 >, = —0.0005 dy = 01230 Note that this new operating condition has a slightly reduced natural frequency (1 49 Hz) and a greatly increased time constant (2.955) compared to the previous example. Thus damping is substantially reduced by the change in operating point 6.4 Linearization of the Flux Linkage Model We now linearize the flux linkage model of a synchronous machine, following a pro- cedure similar to that used above for the current model From (4.135) we can compute the linear equations hye = — & (1 - Ee’ 7p Ewe Luo Ba & (FBP) Na PB a Ea = Wg — Ayowa — Pua (632) hg =n, Ein, te (y — Ea Lup ‘ Keg =r TEEN () 7 ) + FA dos #0 (639) _ >, bu. Lup ~ 22 (1-42) 6 Boa = to pe Nat te GMP Dea — (1 PB) Row (6.34) Similarly the g axis equation (4 136) can be linearized to give i at (he ug wy =v, 6 Kya = OMe t(i Fe) Na + IE Ne + Nets dys (635) Lng Kos = te BE Na The torque equation (4 137) becomes {6 36) a Lugo — L L L, L Tag = (28O GME Ne — PAE Ae) + FEN ME Dob Lu ~ Lug Lup Lup _ wo yy (637 + Tye Ne + EAP eo + ) ha FEE Made ) Similarly, the swing equation becomes Oy 1 [tue y baw — bug (EN aN) 1 {Luo Lf Luo ~ La L, Luo = [FHP Ayo | og — Lb ( MP me FH Npy + FBZ Apo) A, Lee +) . + Git hE hn = PD) D 1fL i +L (PE) te Po th tn ate zi 218 Chopter 6 and finally by = ws (6 38) Fora system of one machine connected to an infinite bus through a transmission line, the load equations are given by (4.157) and (4.158), These are then linearized to give [+i ( a] ie Bie - bl "| ns Leku Lekuo R ~ OT Ma = “Th, 2 hon # (1-8), +R oad Dos + Res + [K'sin (By — a)]55 (6 40) ato where 4] LeLavo ie) No - Eee 42) de — Esta y,, — Lekwo 9, te)| ale ~ alg O° and R =7 +R, and K = V3¥.._ The linearized equations of the system are (6 33), (6 34), (6 36), and (6.37)-(6.40) and é, = wy In matrix form we write =CA+D (641) where the matrices I, C, and D are similar to those defined in Section 4.133 for the nonlinear model If the state equations are written out in the form of (6 41) and compared with the nonlinear equations (4 159)-(4 162), several interesting observations can be made. First, we can show that the matrix T is exactly the same as (4.160) The matrix C is similar, but not exactly the same as (4.161). If we write C as dFD 40 wd (6.42) Linear Models of the Synchronous Machine 219 with partitioning as in (4 161), we can observe that C,, C,, and G ate exactly the same as in the nonlinear equation Submatrices C, and C, are exactly as in (4.161) if o is replaced by we Submatrices C,, C,, C;, and C, are considerably changed, however, and C, and C,, which were formerly zero matrices, now become Kyo V3V. cos(5 — a) G=] 0 ° o 0 ae hee ree o| 643) 0 0 where « is the angle of V., and 4, is the initial angle of the q axis, each measured from the arbitrary reference ‘We may write matrices C, and C, as 1 (y Lwode\ | Luod | Low , =| Bata te] 3 (6.44) where X4oo and Aygo ate the initial values of Asn and Aug fespectively Finally, we note the new D matrix to be D 0 v4 010 0) Tra/ry OF (645) Assuming that the inverse of T exists, we can premultiply both sides of (6.42) by T-! to obtain Ae TCA + TD (6.46) which is of the form k= Ax + Bu (647) The matrices A and B will have constant coefficients, which are dependent upon the quiescent operating conditions ‘Note that the matrices A and B will not be the same here as in the current model Since the choice of the state variables is arbitrary, there are many other equations that could be written The order of the system does not change, however, and there are still seven degrees of {reedom in the solution Example 65 Obtain the matrices T, C, and A of the fiux linkage model for the operating condi- tions discussed in the previous examples. Solution Machine and line data are taken from previous examples in pu as: 220 Chopter 6 Ly = 1.100 = 0036 1, = 1640 y= 00011 ty 4, = 0150 Tx = 0.00074 Lay = kM; = kM, ~ 1550 ty = 00131 Lyg = KMg = 1.490 rq = 0.0540 L, = 1651 Lup = 0.02838 , = 0.101 Lyg = 002836 Lp = 1605 0.020 ty = 0.055 L, = 0400 Lg = 1.526 7 = 178694 rad The matrix T is independent of load and is given by 31622-07478 -13656! 0 o 10 0 0 10 0 0 o to 0 0 and T-! is computed as 03162 02364 0.4318 | 0 10 0} 0 0.3162 0.6678 ; 0 10 0 ‘ 0 To caleulate the matrix C, the following data is obtained from the initial operating conditions as given in Example 5 2: Xo = 1150 WIV, cos(dg — a) = 1.025 Xoo = 1.045 = W3V, sin(éy — @) = 1.397 No = 1676 Ago = 2200 Yoo = 1914 The matrix C corresponding to Example 5.2 loading is then calculated to be Linear Models of the Synchronous Machine 221 114035 39.438 72022 | -316253 2111.78 1388-3278 «3.756 | 0 0 44.720 66.282 115330) 0 0 — 1430.11 1024.53 0 0 oO 0 C=) 316246 -747.76 —1365.58 | 114055 1113784 103932 139655 |10-* 0 0 O | 284854 313530, 0 0 =1,0285 -0.4009 07322 | -19867 16503 0 0 o 1 06 6 14 ‘Note that some of the elements of the matrices C, and C, in this example are somewhat different from those in Example 44 since the resistance & is not the same in both ex- amples The A matrix is given by 16.422 39848 —26 141 | — 1000 12 667 83 \ —45226 324.00 1388-5278 3756 «| 0 o ; 0 0 44720 66282 -115330} 0 0 i; 0 0 A 99988-23644 —431 80 0 0 0 -174.142) 32863 441.59 [10> -313530} 0 10285-04009 —0 7322 0 0 0 The eigenvalues of this matrix are the same as those obtained in Example 63 and cor- respond to the loading condition of Example 5 2 For the operating condition of Example 5.1 we obtain the same matrix I For this operating condition the initial conditions in pu are given by dy = 1.345, Ney = 1.935, Apo = 1.634, Xgo = 1094, Ngo = 0.994, Kcos(dy — a) = 0.5607, and Ksin (5) — «) 13207, The matrix C for the operating conditions of Example 5 1 is given by = 114.035, 39.437 72022 | 316253 QML 7B | = 1361.30 $60.75 1388-5278 3786 | a o a) o 44720 66282 = 115.330 | 0 0 10 0 ce 31216 = -74776 = 1365.58 =114055 111378 | $74.48 1320 68 |10°? 284854 -313530 | 0 0 ° 0 0 09790 03816-06969 0 0 0 <171SS 13246 ° 0 and the matrix A is given by 22 Chapter 6 16422 3984826141 | 10001266783 | 4305017733 1388-5278 3756 | 0 o | 0 0 44720 66282-11530 10 o 1 oo ° Aw | 99988-23648 asso} sis -7414 | 18176 41760)10° ° ° © 2485-3353 | oO 09790-03816 06999 | 1715513246, ° ° ° o | 0 ° 10 The eigenvalues obtained are the same as those given in Example 6.4 and correspond to the loading condition .{ Example 5.1 65 Simplified Linear Model A simplified linear model for a synchronous machine connected to an infinite bus through a transmission line having resistance R, and inductance L, (or a reactance X,) can be developed (see references (1) and (2}) Let the following assumptions be made: 1 Amortisseur effects are neglected 2. Stator winding resistance is neglected 3. The A, and A, terms in the stator and Joad voltage equations are neglected compared to the speed voltage terms wa, and why pevn4 The terms w) in the stator and load voltage equations are assumed to be approxi- mately equal to @,A 5 Balanced conditions are assumed and saturation effects are neglected Under the assumptions stated above the equations describing the system are given below in pu 6.5.1 The’ equation From (4.74) and (4 104) the field equations are given by Up = reip + Rg Mp = Leip + KMpiy (6.48) Eliminating i,, we get Up = e/ Lede + Xe = ef LeK Meg (649) Now let ¢; = V3; be the stator EMF proportional to the main winding flux link ing the stator; ie, V3Ej = wgkM,A,/L; Also let Ezo be the stator EMF that is produced by the field current and corresponds to the field voltage uy; ie . V3 Egy = wk MpUs fry Using the above definitions and zo defined by (4.189), we get from (6.49) in the 5 do- main Exp = (1 + tio S)By — Xa ~ Xda (6.50) where Ly = ig/V3 and s is the Laplace transform variable. Also using the above definition for £,, we can arrange the second equation in (6 48) to give Ey = Oe kMpip| V3 + (Xq = Xie = E+ Oty ~ xDIp (6.51) Linear Models of the Synchronous Machine 223 where & is as defined in Section 4.7.4. Note that (6.50) and (6 51) are linear From (4.149) and (4.74) and from the assumptions made in the simplified model, we compute uv, and ¥, for infinite bus loading to be Wy = = Gly = — VIV. sin 6 — a) + Rely + ely by = @plaly + OgkMpi, = V3V. cos (6 — a) + Ri — walaig (6.52) Linearizing (6.52), 0 = -Ruigg + (te + Xan + @aKMpipg + [Ksin (3, ~ 0)]85 0 = —Riiga — (% + XDigg + [Kc08 (8) — )]55 (6.53) where K = V3V,, and V, is the infinite bus voltage to neutral Rearranging (6 51) and (6 53), 04 + X)as + Relya = Eqn + [Va sin (dy — e155 Rilus + Oy + Xdlgs = (¥.008 (55 — ald, (6.54) Solving (6.54) for 14, and f,,, we compute fa] 4 [et X,) Rcos(dy ~ a) — (x, + X,)sin (i ~ a) | [Ets (655) Is R, (xh + X,)005(5) - a) + Resin (@ - a) | | ¥o5s where Ky = URE + Oy + XOXO (6.56) We now substitute J, into an incremental version of (6 50) to compute Exp = (/Ky + Tie S)Eja + Kiedy (6.57) where we define (in agreement with [2}) 1/Ky = 1 + KAxy — xa)(x, + X,) Ky = VeKiCtg ~ xD{C% + X_)sin (By — a) = Rec0s()- a) (6.58) Then from (6,58) and (6.57) we get the following s domain relation Ky KK Te ay Bios — oe ba 6.59) 14 Kyrigs 1 Ky rigs (659) Ely = [Note that (6.59) differs from (3 10) because of the introduction here of Eyp rather than v,] From (6 59) we can identify that K, is an impedance factor that takes into account the loading effect of the external impedance, and K, is related to the demagnetizing ef- fect of a change in the rotor angle; ic, Ken (6.60) a, | Eppsvantn 6.5.2 Electrical torque equation The pu electrical torque 7, is numerically equal to the three-phase power. There- fore, T, = (1/3) aig + Vi.) = Vila + VQ) pu (661) where under the assumptions used in this model, 224 Chapter 6 Vem —%le My = Aula + WRKM he /VS (662) Using (6 51) in the second equation of (6 62), Wo= xh Vy = wile + By (6.63) From (6 63) and (6 61) T, = [Ey — (%y — xablally (6 64) Linearizing (6.64), we compute Tag = lobia + (Bio ~ Cy — Dlaltea — Oe — xD yoles = Ieobia + Beales — Ce ~ goles (6.63) where we have used the q axis voltage E,, defined in Figure 5 2as Ey, = E + (xy — x)ly with E taken from (6.51) to write the initial condition E yay = Ey + a ~ %)lae = Ego — (8a — Daa + (a — Xen = Ey — Oy - Xb (6.66) Substituting (6.55) and (6.56) into (6 65). we compute the inctemental torque to be Tea = K,V. {EgaulRe Sin (By — a) + (4 + X_)008(8) — @)] + LyolXy — Oy + XYsin (Gy — a) — Re cos (dy — a))}5, + Kllyol RE + (%y + Xd") + BaoRel Eqn & Kids + KaBiy (6.67) Where K, is the change in electrical torque for a small change in rotor angle at constant d axis flux linkage; ie., the synchronizing torque coefficient = KV otEgaolR, Sin (bp — @) + (xy + X,) cos (dy — a] + Lol, — xX, + %_) sin (Bp — ae) ~ R, cos (by — a) ]h K, is the change in electrical torque for small change in the d axis flux linkage at con- stant rotor angle aa] sty 7 | = KR Egg + LolR? + (%y + XN We should point out the similarity between the constant K; in (6,67) and the synchroniz~ ing power coefficient discussed in Chapter 2 and given by (2.36). If the field flux linkage is constant, £; will also be constant and K, = 0 The model is reduced to the classi cal model of Chapter 2 6.5.3. Terminal voltage equation From (4 41) the synchronous machine terminal voltage V, is given by V3 = (1/3)@3 + Op ‘or in rms equivalent variables Vie Ve4 V2 (6.68) Linear Models of the Synchronous Machine 225 This equation is linearized to obtain Vig = (Yeo! Ya) Yes + Hoa! Yo) Ys 6 69) Substituting (6.63) in (6 69), Ya = —WVao/ Vad%lea + Vol Vo) Xilaa + Fjs) (6 70) Substituting for /,, and J,, from (6 55), Yay = (&iVaxiMo/ Yo) 608 (5) — a) — (ry + X,)sin (By ~ 0] — KV ¥aed Vo) Xd + X-) 008 (5g — 2) + R, sin (6, — @)]} 85 + A Meo/ Mad — Kyxs(x, + X.)) — Vaal Via) Kit gel Eon Ky5y + KE ly (6.71) where K; is the change in the terminal voltage V, for a small change in rotor angle at constant d axis flux linkage, or and K, is the change in the terminal voltage V; for a small change in the d axis flux linkage at constant rotor angle, or 6.5.4 Summary of equations Equations (6.59), (6.67), and (6.71) are the basic equations for the simplified linear model, ie., , kK Kk Te Rts Te Krys Tag = Kybq + Kobla Vig = Kyby + Kets (6.72) We note that the constants Kj, Ks, Ky, Ks, Ks, and Kg depend upon the network pa- rameters, the quiescent operating conditions, and the infinite bus voltage To complete the model, the linearized swing equation from (4 90) is used 1 = Ta — Ten (6.73) The angle 6, in radians is obtained by integrating on &, twice In the above equations the time is in pu to a base quantity of 1/3775, T is the total torque to a base quantity of the three-phase machine power, and 7, = 2a, Example 66 Find the constants K; through K, of the simplified model for the system and condi- tions stated in Example 5.1 Solution We can tabulate the data from Example 5.1 as follows. 226 Chapter 6 Transmission line data R= 002 X,= 040 pu Infinite bus voltage V, = 0.828 Synchronous machine data x= 1700 pu x, = 1640 pu x} = 1700 = [(1.95)°/1651] = 0.245 pu Also, from Example 5 1 ipo = 2979 Tyg = 0.385 Tq = 1112 Veg = 0.776 Vag = =0.631 ¥, = 1.000 We can calculate the angle between the infinite bus and the g axis to be 5 — @ = 66995° Then sin (4 — a) = 09205, cos(6) — ax) = 0 3908, From (6 66) we compute Eggo = 155 x 2.979/VF ~ 1112(1 70 — 164) = 2.5995 Also, Y/K, = RE + (xy + Xx + X,) = 13162 K, = 0.7598 Then we compute from (6 $8) Ky = [1 + (1/1.3162)(1 455)(2 04)" = 0.3072 K, = 0828 x 0.7598 x 1455(204 x 09205 — 002 x 03908) = 17124 We then calculate K, and K; from (6 67) Ky = Kyu tEqualResin (Sy ~ a) + (xj + X,)e08(6y ~ a)] + Mgh%q ~ Xi) ay + X,) sin (Oy — @) = R,c05 (45 — @)l} 0.7598 x 0 828[2 5995(0.02 x 0.9205 + 0.645 x 0.3908) + 0.3853 x 1 395(2.04 x 0.9205 — 0.02 x 0.3908)] 1.0755 Kill RE + Oey + XP + EgaoRet = 0-7598{0.385{(0 02)? + (2.04)"} + 2 5995 x 0.02} = 12578 Ky ¥ K, and K, are calculated from (6.71): Ks = (Ky¥axi¥oa/ Vo) Re 008 (by ~ a) — (%, + X,)sin (5 — @)] ~ (KV ox, Vio/ Vo (xa + X-)008 (bo — &) + Resin (Go — @)] = {(0 7598)(0.828)(0 245)(0.776/1 0)]{(0.02)(0 3908) ~ (2.04)(0.9205)) — (0.7598)(0.828)(1,64)(—0.631/1 0){(0.645)(0 3908) + (0.02)(0.9205)} = -0.0409 Linear Models of the Synchronous Machine 207 Ky = (Veo) Yo) — Kies, + X0) -— Yoo! Med KR, = 0.776{1 ~ (0 7598)(0 245)(2.04)] + (0.631)(0 7598)(1 64)(0.02) = 0.4971 Therefore at this operating condition the linearized model of the system is given by Eis = (03072/(1 + 1.8135)1Espq — (0.5261/(1 + 1813 9)dy T.y = 107555, + 1.2578 EB! Vig = -0.04095, + 0.4971 B/, Example 67 Repeat Example 6 6 for the operating conditions given in Example 5 2 Solution From Example 5 2 gg = 2.8259 Ig = 0.4047 pu Iq = —0.9185 Veo = 0.9670 pu Vig = —0.6628 ¥, = 1.000 pu Vo = 1172 by — a = 53.736" and sin (d, — @) = 0.8063, cos(d, — @) From this data we calculate Bi and Ey Ej = 1.55 x 2.826/V3 — 1.455 x 0.9185 = 1.1925 Eq = 1.1925 ~ 1.395(—0 9185) = 2.4738 W)Ky = RE + (5, + Kay +X) = 13162 K, = 0.7598 Then = 04 x 3.455\~' y= (15 RO Lass ) 0.3072 Ky = EOXEASS (204 x 0.8063 ~ 0.02 x 0.5915) = 1 805 tio = 5.90 8 ‘The effective field-winding time constant under this loading is given by Kytip = 0.3072 x $9 = 18125 5 K, = (0.7598)(1.0){(2 474)(@ 02) (0.8063) + (0.645)(0 5915)] + (0.4047)(1,395)[(2.04)(0 8063) — (0 02)(0.5915)]} = 1.4479 We note that for this example the constant X, is greater in magnitude than in Ex- ample 6.6, The constant K, corresponds to the synchronizing power coefficient dis- cussed in Chapter 2. The greater value in this example is indicative of a lower loading condition or a greater ability in this case to transmit synchronizing power K, = 0.7598{0.4047{(0.02) + (2.04)] + (2.474)(0.02)} = 1.3174 228 ‘Chapter 6 Ky = (0.7598)(1 0)(0 245) (2a) {(0.02)(0 5915) — (2.04(0.8063)] = (0.7598)(1 0)(1 wy (=2608) [{0.645)(0.5915) + (0.02)(0 8063)] = 0.0294 (0.9670) Ke= 29a — (0.7598)(0 245)(2.041)] - (2s) (0.7598)(1 64)(0 02) = 0.5257 The linearized model of the system at the given operating point is given in pu by Ely = (0.3072/(1 + 18135))Esoa — (0.5546/(1 + 1.813 9)]6s Tey = 144798, + 13174 Eta V,, = 0.02945, + 0.5257 Es 65.5 Effect of loading Examining the values of the constants K, through K, for the loading conditions of Examples 6 6 and 6.7, we note the following 1 The constant K; is the same in both cases From (6.57) and (6 58) we note that Ks is an impedance factor and henee is independent of the machine loading 2. The constants Ki, K;, Ky, and K, are comparable in magnitude in both cases, while , has reversed sign From (6.58), (6 67), and (6,71) we note that these con- stants depend on the initial machine loading The cases studied in the above examples represent heavy load conditions. Certain effects are clearly demonstrated. In the heavier loading condition of Example 6 6, Ks has @ value of —0.0409, and in the less severe loading condition of Example 67 its value is 0.0294 This is rather significant, and in Chapter 8 it will be pointed out that in machines with voltage regulators, the system damping is affected by the constant K, If this constant is negative, the voltage regulator decreases the natural damping of the system (at that operating condition). This is usually compensated for by the use of sup- plementary signals to produce artificial damping From Examples 6 6 and 6.7 we note that the demagnetizing effect of the armature reaction as manifested by the E;, dependence is quite significant This effect is more pronounced in relation to the change in the terminal voltage To illustrate the demagnetizing eflect of the armature reaction, let Epos = 0; then Ejay = [KsKa/(L + Ky rho sM8y (6.74) and substituting in the expression for J, we get, Teg = [Ky — KiKsKof(l + Ky aq N15 (675) The bracketed term is the synchronizing torque coefficient taking into account the effect of the armature reaction initially the coefficient K, is reduced by a factor KKa/ ta Similarly, substituting in the expression for Vj, Vig = [Ks - KiKaKof(l + Ky rin 8)]0q (6.76) The second term is usually much larger in magnitude than K,, and initially the change in the terminal voltage is given by Vial,g = 7 (KaKo/ 740), (677) % Linear Models of the Synchronous Machine 229 My val 1 19 oe ash o7 ° oF 1 oy fo=e 5 re 0.0 \ oog- sre 1 (0.1,0.0)} 1 00} og fy oii . 1 1 H . Sra 8 ect Powe, P o4 06 od - 01 oO? 04 08 0.8 TO) 0.5 2219) 0. esl Poet? od 3 a Voom She | on oy ° at a Sree ea ect Pover P| Fig 61 Variation of parameters Ki, Ag with loading: (2) A, versus P (real power) and Q (reactive power) as parameter, (b) Ky versus P and Q, (c) Ky versus P and Q, (d) Ks versus P and Q, (e) Ke versus PandQ (IEEE Reprinted from IEEE Trans vol PAS-92, Sept /Oct 1973) The effects of the machine loading on the constants Ky, Ky, Ky Ks, and K, are studied in reference [3] for a one machine-infinite bus system very similar to the system in the above examples except {or zero external resistance. The results are shown in Fig- ure 6 1 65.6 Comparison with classical model The machine model discussed in this section is almost as simple as the classical model discussed in Chapter 2, except for the variation in the main field-winding flux. tis interesting to compare the two models The classical model does not account for the demagnetizing effect of the armature reaction, manifested as a change in E{ Thus (6 67) in the classical model would have Ky = 0. Also in (6.59) the effective time constant is assumed to be very large so that £, % constant In (6 72) the classical model will have K, = 0. 230 Chapter 6 To illustrate the difference between the two models, the same system in Example 6.7 is solved by the classical model Example 6.8 Using the classical model discussed in Chapter 2, solve the system of Example 6 7. _ven attr [ Fig 62 Network of Example 6.7 Ly Solution The network used in the classical model is shown in Figure 6.2. The phasor E = E {ais the constant voltage behind transient reactance, Note that the angle 6 here isnot the same as the rotor angle 6 discussed previously; it is the angle of the fictitious voltage E The phasors V; and V, are the machine terminal voltage and the in- finite bus voltage respectively For convenience we will use the pu system used (or implied) in Chapter 2, ie, based on the three-phase power. Therefore, E = E/8 = 1 + j0.0 + (0.020 + j0.645)(0.980 ~ j0.217) = 1.3186 /28.43° The synchronizing power coefficient is given by P, # = EV(By, 0088) ~ Gy, Sind) = (EV, /Z°)V(xh + X,) e085) + Ry sin bo)) 1s | bay 13164 x LO LAT n” (0685 x 08794 — 0.02 x 0.4761) = 1826 To compare with the value of K, in Example 6 7 we note the difference in the pu sys- tem, K, = 1448. Thus the classical model gives a larger value of the synchronizing power coefficient than that obtained when the demagnetizing effect of the armature re action is taken into account, To obtain the linearized equation for ¥,, neglecting R, we get [, = [(1.3186 cos 6 — 1.00) + j1.3186 sin 4]/j0 645 V, = 1.000 + j00 + j040%, Substituting, we get for the magnitude of ¥, V3 = (0.3798 + 0.8177 cos 5}? + (0.8177) sin? S WeaVis = — (0 62sin 6) 55 or = ~ 01261, Linear Models of the Synchronous Machine 231 The corresponding initial value in Example 6 7 is given by Vas) ge * ~ (KeKeo/ tie) 8g = —0.1252 64 6.6 Block Diagrams The block diagram representation of (6.73) and the equation for 6, is shown in Figure 63, This block diagram “gencrates” the rotor angle 6. When combined with (6 59), (6.67), and (6.72) the resulting block diagram is shown in Figure 64. In both diagrams the subscript A is omitted for convenience Note that Figure 6.4 is similar to Figure 3.1 Figure 6 4 has two inputs or forcing functions, namely, E¢p and T,,. The output is the terminal voltage change V, Other significant quantities arc identified in the dia- gram, such as £;, 7, ©, and 6 The diagram and its equations show that the sim- plified model of the synchronous machine is a third-order system. 1- tLe Fig 63 Block diagram of (6 73) | selec oe 67 State-Space Representation of Simplified Model From Section 6,5 the system equations are given by KytnEgs + Eis = KiEsoa ~ Ks Kady Tey = K\8n + KBs Vis = Ksba + KoEja yy = Ty ~ Tes by = wy (6 78) Eliminating V,, and 7, from the above equations, Ela = — (1/Ksrio) Egs ~ (Kal té0)8a + (/ tio) Exva oy = — (Ki/) gs — (Ki) 5 + (1/4) Tr by = ws (679) By designating the state variables as £/,, wa, and 5, and the input signals as E,), and K i " Fig64 Block diagram of the simplified linear model of a synchronous machine connected to an infinite bus. Ke 232 Chapter 6 Tyas the above equation is in the desired state-space form & = Ax + Bu where, Irs 0 Eros i = UBs ey ba) we] B=} 0 I/y, (6.80) ” o 0 1/Kytie © —Ka/ tio Ae|-K/z 0 -Ki/t, (681) 0 1 0 In the above equations the driving functions Esp, and T,,. are determined from the detailed description of the voltage regulator-excitation systems and the mechanical turbine-speed governor systems respectively The former will be discussed in Chapter 7 while the latter is discussed in Volume 2 Problems 61 Thegenerator of Example 5 2 is loaded to 75% of nameplate rating at rated terminal volt- age and with constant turbine output. The excitation is then varied from 90% PF lagging to unity and finally to 90% leading Compute the current model A matrix for these three power factors How many elements of the A matrix vary as the power factor is changed? How sensitive are these elements to change in power factor? 62 Usea digital computer to compute the eigenvalues of the three A matrices determined in Problem 61. What conclusions, if any, can you draw from the results? Let D = 0 63 Using the data of Problem 6 | at 90% PF lagging, compute the eigenvalues of the A matrix with the damping D = 1,2,and3. Find the sensitivity of the cigenvalues to this parameter 64 — Repeat Problem 6 1 using the flux linkage model 65 Repeat Problem 6.2 using the flux linkage model 66 Repeat Problem 6 3 using the flux linkage model 67 Make an analog computer study using the lipearized model summarized in Section 6 5 4 Note in particular the system damping as compared to the analog computer results of Chapter 5 Determine a value of D that will make the linear model respond with damping similar to the nonlinear model 68% Examine the linear system (6 79) and write the equation for the eigenvalues of this system. Find the characteristic equation and see if you can identify any system constraints for stability using Routh’s eriterion 69 For the generator and loading conditions of Problem 61, calculate the constants K, through Ke for the simplified linear model 610 Repeat Example 6.8 for the system of Example 66 Find the synchronizing power co- efficient and ¥,, as @ function of 6, for the classical model and compare with the corresponding values obtained by the simplified linear model References 1 Hefson, W G,, and Phillips, R_A. Effect of a modern voltage regulator on underexcited operation of large turbine generators APEE Yrans 71-092-97, 1952, 2 de Mello, F. P., and Concordia, C. Concepts of synchronous machine stability as affected by excitation control. ZEEE Trans PAS-88:316-29 1969 3 El-Sherbiny. MK and Mehta. D.M_ Dynamic system stability Pt 1 /EBE Trans PAS.92:1538-46 1973 shooter 7 Excitation Systems Three principal control systems directly affect a synchronous generator: the boiler control, governor, and exciter. This simplified view is expressed diagramatically in Figure 7.1, which serves to orient our thinking from the problems of representation of the machine to the problems of control. In this chapter we shall deal exclusively with the excitation system, feaving the consideration of governors and boiler control for Vol- ume 2 7.1 Simplified View of Ex« ion Control Referring again to Figure 7 1, let us examine briefly the function of each control ele- ment. Assume that the generating unit is lossless This is not a bad assumption when total losses of turbine and generator are compared to total output Under this assump- tion all power received as steam must leave the generator terminals as electric power. Thus the unit pictured in Figure 7.1 is nothing more than an energy conversion device that changes heat energy of steam into electrical energy at the machine terminals. The amount of steam power admitted to the turbine is controlled by the governor The excitation system controls the generated EMF of the generator and therefore controls not only the output voltage but'the power factor and current magnitude as well An example will illustrate this point further, team at prose, Py Power oF valtepe Vv fact Tite Lfeholbys Poteet weae[ oo L_ cient I | Fring contol <1 Governor - Power set-point Rete HEY Fig 71 Principal controls of a generating unit Refer to the schematic representation of a synchronous machine shown in Figure 7 2 where, for convenience, the stator is represented in its simplest form, namely, by an EMF behind a synchronous reactance as for round rotor machines at steady state Here 239 234 Chapter 7 Fig 72. Equivalent circuit of « synchronous machine the governor controls the torque or the shaft power input and the excitation system con- trols E,, the internally generated EMF Example 71 Consider the generator of Figure 7 2 to be operating at a lagging power factor with a current /, internal voltage £,, and terminal voltage V. Assume that the input power is held constant by the governor. Having established this initial operating condition, as- sume that the excitation is incteased to a new value E? Assume that the bus voltage is held constant by other machines operating in parallel with this machine, and find the new value of current /’, the new power factor cos 0; and the new torque angle 3° Solution This problem without numbers may be solved by sketching a phasor diagram. In- deed, considerable insight into learniig how the control system functions is gained by this experience The initial operating condition is shown in the phasor diagram of Figure 73 Under the operating conditions specified, the output power per phase may be expressed in two ways: first in terms of the generator terminal condi P = Vicos@ (ay and second in terms of the power angle, with saliency effects and stator resistance neglected, P= (E,V/X)sind (72) Inout problem P and V are constants. Therefore, from (7 1) Toosd = k, (73) where k; isa constant. Also from (7.2) E, sind = ky a4 where &; is a constant t 7 ff a. SL) ae Fig 73. Phasor diagram of the initial condition Excitation Systems 235 Fig. 74 Phasor diagram showing control constraints. Figure 7 4 shows the phasor diagram of Figure 7 3, but with k, and k, shown graph- ically. Thus as the excitation is increased, the tip of &, is constrained to follow the dashed line of Figure 7 4, and the tip of Tis similarly constrained to follow the vertical dashed line. We also must observe the physical law that requires that phasor /¥ and phasor Tlie at right angles Thus we construct the phasor diagram of Figure 7.5, which shows the “before and after” situation. We observe that the new equilibrium condition requires that (1) the torque angle is decreased, (2) the current is increased, and (3) the power factor is more lagging; but the output power and voltage are the same. By similar reasoning we can evaluate the results of decreasing the excitation and of changing the governor setting These mental exercises are recommended to the student as both interesting and enlightening Fig 75. Solution for increasing £, at constant Pand ¥ Note that in Example 7.1 we have studied the effect of going from one stable op- erating condition to another. We have ignored the transient period necessary to accom- plish this change, with its associated probtems—the speed of response, the nature of the transient (overdamped, underdamped, or critically damped), and the possibility of saturation at the higher value of £, These will be topics of concern in this chapter 7.2 Control Configurations We now consider the physical configuration of components used for excitation sys- tems. Figure 7 6 shows in block form the arrangement of the physics! components in 236 Chapter 7 froot rae TG cor | Dutt voteve & cement on eee prime mover Deira Exciter powgs | Exciter sales Fig 76 Arrangement of excitation components, any system, In many present-day systems the exciter is a de generator driven by either the steam turbine (on the same shaft as the generator) or an induction motor, An in- creasing number are solid-state systems consisting of some form of rectifier or thyristor system supplied from the ac bus or from an alternator-exciter The voltage regulator is the intelligence of the system and contiols the output of the exciter so that the generated voltage and reactive power change in the desired way. In earlier systems the “voltage regulator” was entirely manual, Thus the opetator ob- served the terminal voltage and adjusted the field rheostat (the voltage regulator) until the desired output conditions were observed. In most modern systems the voltage regu- lator is a controller that senses the generator output voltage (and sometimes the current) then initiates corrective action by changing the exciter control in the desired direction The speed of this device is of great interest in studying stability. Because of the high inductance in the generator field winding, it is difficult to make rapid changes in field current. This introduces considerable “lag” in the control function and is one of the major obstacles to be overcome in designing a regulating system The auxiliary contro} illustrated in Figure 7.6 may include several added features For example, damping is sometimes introduced to prevent overshoot. A comparator may be used to set a lower limit on excitation, especially at leading power factor opera- tion, for prevention of instability due to very weak coupling across the air gap. Other auxiliary controls are sometimes desirable for feedback of speed, frequency, accelera- tion, or other data [1] 7.3. Typical Excitation Configurations To further clarify the arrangement of components in typical excitation systems, we consider here several possible designs without detailed discussion 7.3.1. Primitive systems First we consider systems that can be classified in a general way as “slow response” systems. Figure 7.7 shows one arrangement consisting of a main exciter with manual or automatic control of the field. The “regulator” in this case detects the voltage level and includes a mechanical device to change the control theostat resistance. One such direct- acting rheostatic device (the “Silverstat” regulator) is described in reference [2] and consists of a regulating coil that operates a plunger, which in turn acts on a row of spaced silver buttons to systematically short out sections of the rheostat In application, the device is installed as shown in Figure 78 In operation, an increase in generator out- put voltage will cause an increase in de voltage from the rectifier, This will cause an increase in current through the regulator coil that mechanically operates a solenoid to insert exciter field resistance elements. This reduces excitation field flux and voltage, thereby lowering the field current in the generator field, hence lowering the generator Excitation Systems 237 a a ©) He, Voltage regulator Morus! cena Fig 77 Main exciter with cheostat control voltage. Two additional features of the system in Figure 78 are the damping wans- former and current compensator The damping transformer is an electrical “dashpot” or antihunting device to damp out excessive action of the moving plunger The current compensator feature is used to control the division of reactive power among parallel generators operating under this type of control. The current transformer and compen- sator resistance introduce a voltage drop in the potential circuit proportional to the line current. The phase relationship is such that for lagging current (positive generated reactive power) the voltage drop across the compensating resistance adds to the voltage from the potential transformer This causes the regulator to lower the excitation voltage for an increase in lagging current (increase in reactive power output) and provides a drooping characteristic to assure that the load reactive power is equally divided among. the parallel machines The next level of complication in excitation systems is the main exciter and pilot i ste S Ker Ga iil a are emt | 4 serasree" 7 El Aagieeg ames ean Fig. 78 Selt-excited main exciter with Silverstat regulator (Used with permission from Elecirical Trans- ‘mission and Disteibution Reference Book, Westinghouse Electric Corp.. 1950) 238 Chapter 7 Mein Filet Compurotor excites Commutator exeltor stig LoL 2 let |< Manus! eee ‘control if ea Fig 79 Main exciter and pilot exciter system. exciter system shown in Figure 79 This system has a much faster response than the self-excited main exciter, since the exciter field control is independent of the exciter output voltage. Control is achieved in much the same way as for the self-excited case Because the rheostat positioner is electromechanieal, the response may be slow com- pated to more modem systems, although it is faster than the self-excited arrangement The two systems just described are examples of older systems and represent direct, straightforward means of effecting excitation control In terms of present technology in control systems they are primitive and offer little promise for really fast system re- sponse because of inherent friction, backlash, and lack of sensitivity The first step in sophistication of the primitive systems was to include in the feed- back path an amplifier that would be fast acting and could magnify the voltage error and induce faster excitation changes Gradually, as generators have become larger and interconnected system operation more common, the excitation control systems have be- come more and more complex The following sections group these modern systems ac- cording to the type of exciter [3] Commurator Ragustor Exciter tomer” eld Feld breoker | ‘Exciter Tiel | OF Fig 710 Excitation control system with de generator-commutator exciter (@ IEEE. Reprinted from IEEE Trans., vol PAS-88. Aug 1969) Example: General Electric type NA143 amplidyne sys- tem [a] Excitation Systems 239 ova _ # ie eu Dal pen “T yee iS | siete | tn | od { | bape 26 | — Stetion KM, Suxiliory: i Ae ya Fig 711 Excitation control system with de generator-commutator exciter (w IEEE Reprinted from JEEE ‘Trans. vol PAS-88, Aug 1969) Example: Westinghouse type WMA Mag-A-Stal system [6}. 7.3.2. Excitation control systems with de generator-commutator exciters Two systems of US. manufacture have de generator-commutator exeiters. Both have amplifiers in the feedback path; one a rotating amplifier, the other a magnetic amplifier Figure 7.10 [3] shows one such system that incorporates a rotating amplifier or amplidyne (51 in the exciter field circuit, This amplifier is used to force the exciter field in the desifed direction and results in much faster response than with a self-excited machine acting unassisted Another system with a similar exciter is that of Figure 7.11 where the amplifier is a static magnetic amplifier deriving its power supply ftom a permanent-magnet gen- erator-motor set Often the frequency of this supply is increased to 420 Hz to increase the amplifier response. Note that the exciter in this system has two control fields, one for boost and one for buck corrections. A third field provides for self-excited manual operation when the amplifier is out of service 7.3.3 Excitation control systems with alternator-rectifier exciters With the advent of solid-state technology and availability of reliable high-current rectifiers, another type of system became feasible In this system the exciter is an ac gen erator, the output of which is rectified to provide the de current required by the gen- erator field, The control circuitry for these units is also solid-state in most cases, and the overall response is quite fast (3] An example of alternator-rectifier systems is shown in Figure 7.12 In this system the alternator output is rectified and connected to the generator field by means of slip tings, The alternator-exciter itself is shunt excited and is controlled by electronically adjusting the firing angle of thyristors (SCR’s) This means of control can be very fast 240 Chapter 7 Fig 712 Excitation control system with alternator-rectifier exciter using stationary noncontrolled rec- lifiers (o IEEE Reprinted from JEEE Trans. vol PAS-88, Aug 1969) Example: General Blectrie Alterrex excitation system [7] since the firing angle can be adjusted very quickly compared to the other time constants involved. Another example of an alternator-rectifier system is shown in Figure 7.13. This sys- tem is unique in that it is brushless; i ¢, there is no need for slip rings since the alterna- tor-exciter and diode rectifiers are rotating with the shaft. The system incorporates a pilot permanent magnet generator (labeled PMG in Figure 7.13) with a permanent mag- net field to supply the (stationary) field for the (rotating) alternatot-exciter Thus all coupling between stationary and rotating components is electromagnetic Note, how- ever, that it is impossible to meter any of the generator field quantities directly since these components are all moving with the rotor and no slip rings are used PA" tig Excitation ___omplifier Shoat ' WTA regulate a Other ‘nts Fig 713 Excitation conttol system with allernator-rectifier exciter employing rotating rectifiers (© IEEE Reprinted [rom JEEE Trans, vol PAS-88 Aug 1969.) Example: Westinghouse type WTA Brushless excitation system (8.9) 241 Excitation Systems Exeter, ia a rs ‘ening eae Fig 714 Excitation control system with alternator-SCR exciter system (@ IEEE Reptinied from IEEE Trans. vol PAS-88, Aug 1969) Example: General Electric Althyrex excitation system am The response of systems with alternator-rectifier exciters is improved by designing the alternator for operation at frequencies higher than that of the main generator. Re- cent systems have used 420-Hz and 300-Hz alternators for this reason and report ex- cellent response characteristics (8, 10) 7.3.4 Excitation control systems with alternator-SCR exciter systems, Another important development in excitation systems has been the alternator-SCR design shown in Figure 7.14 (3] In this system the alternator excitation is supplied di- near roetor Baetoror Regulator Fatentot Sr power [yeieee Frartfoones [elemeot Auwiiery Power for ‘tore scr requlotor other inp Fig 71S Excitation control system with compound-rectifier exciter (0 IEEE Reprinted from /EEE Trans., vol PAS-88, Aug 1969.) Example: General Electric SCTP static excitation system 112.13) 242 Chopter 7 rectly from an SCR system with an alternator source Hence it is only necessary to adjust the SCR firing angle to change the excitation level, and this involves essentially no time delay. This requires a somewhat larger alternator-exciter than would otherwise be necessary since it must have a rating capable of continuous operation at ceiling voltage In slower systems, ceiling voltage is reached after a delay, and sustained opera- tion at that level is unlikely 7.3.5 Excitation control systems with compound-rectifier exciter systems The next classification of exciter systems is referred to as a “compound-rectifier” exciter, of which the system shown in Figure 7 15 is an example [3] This system can be viewed as a form of self-excitation of the main ac generator Note that the exciter input comes from the generator electrical output terminals, not {rom the shaft as in previous examples This electrical feedback is controlled by satur- able reactors, the control for which is arranged to use both ac output and exciter values as intelligence sources. The system is entirely static, and this feature is important, Al though originally designed for use on smaller units {12, 13], this same principle may be applied to large units as well Self-excited units have the inherent disadvantage that the ac output voltage is low at the same time the exciter is attempting to correct the low voltage This may be partially compensated for by using output current as well as voltage in the control scheme so that (during faults, for example) feedback is still sufficient to effect adequate control. Such is the case in the unit shown in Figure 7.15, 7.3.6 Excitation control system with compound-rectifier exciter plus potential- source-rectifier exciter A variation of the compound-rectifier scheme is one in which a second rectified out put is added to the self-excited feedback to achieve additional control of excitation Excitation Ausitiory [ Voliege tee, Te arte“ RE | aie, she T Taner Whee LI feces eguotor Litt, Fig 716 Excitation control system with compounc-rectifier eaciter plus potentiaksource-rectifier ex citer (© IEBE. Reprinted from /EEE Trans., vol PAS-88 Aug 1969) Example: Westing- house type WTA-PCY static excitation system [14] Excitation Systems 243 This scheme is depicted in Figure 7.16 [3], Again the basic self-excited main generator scheme is evident. Here, however, the voltage regulator controls a second rectifier sys- tem (called the “Trinistat power amplifier” in Figure 7.16) to achieve the desired ex- citation control Note that the system is entirely static and can be inherently very fast, the only time constants being those of the reactor and the regulator 7.3.7 Excitation control systems with potential-source-rectifier exciter The final category of excitation systems is the self-excited main generator where the rectification is done by means of SCR’s rather than diodes. Two such systems are shown in Figure 7.17 and Figure 7.18 [3] Both circuits have static voltage regulators that use potential, current, and excitation levels to generate a control signal by which the SCR gating may be controlled This type of control is very fast since there is no time delay in shifting the firing angle of the SCR's 7.4 €xcitation Control System Definitions Most of the foregoing excitation system configurations are described in reference {3}, which also gives definitions of the control system quantities of interest in this ap- plication. Only the most important of these are reviewed here. Other definitions, in- cluding those referred to by number here, are stated in Appendix E. Allexcitation control systems may be visualized as automatic control systems with feed forward and feedback elements as shown in Figure 7.19. Viewed in this way, the excitation control systems discussed in the preceding section may be arranged in a gen- eral way, as indicated in Figure 7.20 and further described in Table 7.1. Note that the synchronous machine is considered a part of the “excitation control system,” but the control elements themselves are referred to simply as the “excitation system.” The type of transfer function belonging in each block of Figure 7 20 is discussed in reference [15]« The reference to systems of Type 1, Type 3, etc., in the last column of Table 7 1 also refers to system types defined in that reference This will be discussed in greater detail in Section 79 Our present concern is to learn the general configuration = PTs uxifory Field breaker ower Sig Slip tinge vectfr awstis ie (She oe repsitoe Fig 717 Excitation control system with potential-source-rectfier exciter (® IEEE Reprinted from IEEE Trans. vol PAS-88. Aug 1969) Example: General Electric SCR static excitation sys tem [14] 244 Chapter 7 eS hae @—-t+1— Bom ‘ee son te ome SN esti sme tee [GSES | ree fh" fl | Taste at (Sel) tonto] | -—. [7 + Paferenes] —}-— | oso (oastee! (Ee ecoaly L a ee Let cegulotor mou cov Regulating sytem en - -_.J Fig 718 Excitation control system with potentialsource-tectifier exciter (© IEEE Reprinted from IEEE Trans. vol PAS-88 Aug 1969) Example: Westinghouse type WTA-Trinistat excitation system of modern excitation control systems and to become familiar with the Janguage used in describing them 7.4.1 Voltage response ratio An important definition used in describing excitation control systems is that of the response ratio defined in Appendix E, Def 3.15-319. This is a rough measure of how fast the exciter open circuit voltage will rise in 05s if the excitation control is ad- justed suddenly in the maximum increase direction In other words, the voltage refer- ence in Figure 7.20 is a step input of sufficient magnitude to drive the exciter voltage to its ceiling (Def. 3.03) value with the exciter operating under no-load conditions. Figure 7.21 shows a typical response of such a system where the voltage u, starts at the rated load field voltage (Def. 321) that js the value of v,, which will produce rated Mtr | Actuoting Dieectly contlles signal ta) “signal Fetword | woriblo (Bef 3.41) Gian aD hers BH |_sloments ethos) Fordbock onal (06F3 50) Ooi Z0H Fig 719 Essential elements of an automatic feedback control system (Def £02) (© IEEE. Reprinted from JEEE Trans vol PAS-88 Aug 1969) Note: In excitation controt system usage the ace ‘wating signal is commonly called the error signal (Def 329) (See Appendix E for definitions ) 245 Excitation Systems odes amasuae 9a ‘uatsts 241 Jo snvautp 10 nor’ so yeawys Ave punose ySeqpao1 e182 01 Bey-pea| SOUS WEL SOAK je 10 OS UED :stazIEGEIS WAIEAS JonIUED UONWIIR 24] q wntursew woKe}O WuNUHA YORE) WNL 15 weWAs Siovesuadulos 1ualing 34n9e PLE aaide=)—sindul AOUMY (¥) “HAIAPOW TRUBS "9 "alg aon Pos Annan 8099548 Jou uo nq 9K fe yo eTEUO “aes? inous a8pluq pu Souepadit Runsorsvus ude dae (euswey)p BUIpMIOU UEVAS AUE YO SIMA JO SOUR) AUBLE Ye YsHSUOD UES INduI svavID: pu UDLIE® duSeOp AseMAA “ ‘6961 “DOV "9R-SWA TOA "O4) 99:9 WOAg poatsdEy "FFA 5 0100S ‘3 “paods “Acuanbe) “seMod 03 sbuonadosd sUss Buz sante> win-.syudiye ' ‘apse ed heresy saya tov | qos ‘ore shay “owe | Seat Sn Me. souonoais| _stouorains rawornna | ssutual nos Lrovorwenerad sway | gnusio ‘sno Snr © pews | “Sura ovata oust | ora soning] enous persuading | sowstaa pede aaa | gum me Saxoou | "Bayoout sosuay sagen ‘ cre snowsnehs) swans powroyes | “tent enact) sevens toxyene ows ounoont “iron ‘oansat ' ween Seouasyuig veaveosnea | somaya] women es <5 ow ue yout Sod uegpoeysen | Sate sm : Toveenig| _souelareds poms Tair ieare 17 Taysagmoe 1 a wee | | rewonon 3 ‘eae | 2 | mmanen 1 | grt) rot aos Son] mom | aces | scenes | 'aumoypposagg| rots | _“amrmuag 2° eonesta| unis soe sow Candia | west 3 1 | aupos pos |jonuon envy | NE sovo¥0 2843 goad, — | wipisas “$201008 «2g ‘4 Sussarap ‘ied : t o [ie e fe | ' SWBRA [OHUOD We (Os UI SEAL ATUOUIUIOD s\uDUOGUOD “ELaMEL 246 Chapter 7 vt ot i Ot | OL | | ee] feel] | fee] | ! | Biel LSE | (A i | oO | ic ol le) be fern Preonplier} 2B amelie ower reckons Es P i ae fC ae wir | Seo cae C851), V'x.0ay « ’ -42.10.01.015)} ' 1 9 | 2) |e ee [lost 2.14 fide 2. Ff (DeF 2-09) (Dek 2 mI depo Manvel 1 |__ eet oe eo Excitation system (Def 1.03) mashing [tection cota owed Notes The mumerls on this dogrom refer to the columns in Table 7,14 Fig 720 Excitation control systems (© IEEE, Reprinted from JEEE Trans, vol PAS-88, Aug 1969) Note: The numerals on this diagram refer co the columns in Table 71 (See Appendix E for definitions ) generator voltage under nameplate loading Then, responding to a step change in the reference, the open-circuited field is forced at the maximum rate to ceiling along the curve ab Since the response is nonlineas, the response ratio is defined in terms of the area under the curve ab for exactly 05s We can easily approximate this area by 2 straight line ac and compute Response ri cdj(0a)(0.5) pu V/s 7) Kimbark [16] points out that since the exciter feeds a highly inductive load (the gen- erator field), the voltage across the load is approximately » = kd@/dt. Then in a short time Af the total flux change is Ae= Lf vdt = arca under buildup curve (76) Eppot Ye any wnt eld vahioge ° Tine, Fig 721 Definition of a voltage response ratio Excitotion Systems 247 Spite attining 9556 ng 100% ceiling in er Hime [otep ‘esporte) fing Voltage (Def 2.03) taining 95% Fraving en exponent Fespanse LL sreironos machin oo lod field voltoge (DeF 2 21) or? ss se 78s we Resporee Roto (Def 2 18) Fig 722 Exciter ceiling voltage as a function of response ratio for a high initial response excitation sys tem (IEEE Reprinted from /EEE Trans. vol PAS-88, Aug 1969 ) The time Ar = 0,5 was chosen because this is about the time interval of older “quick- response” regulators between the recognition of a step change in the output voltage and the shorting of field rheostat elements Buildup rather than build-down is used be- cause there is usually more interest in the response to a drop in terminal voltage, such asa fault condition In dynamic operation where the interest is in small, fast changes, build-down may be equally important Equation (7.5) isan adequate definition if the voltage response is rather slow, such as the one shown in Figure 721 It has been recognized for some time, however, that modern fast systems may reach ceiling in 0 Is or less, and extending the triangle acd out to 0.5 sis almost meaningless. This is discussed in reference (3], and a new defini- tion is introduced (Def. 1.05) that replaces the 0.5-s interval Oe in Figure 7.21 by an interval Oe = 0.1 s for “systems having an excitation voltage response time of 01 s or less” [the voltage response time (Def 3.16) is the time required to reach 95% of ceiling] A comparison of three systems, each attaining 95% ceiling voltage in 0 1s, is given in Figure 7 22 (3] and shows how close the 0.1-s response is to the ideal system, a step function 7.4.2. Exciter voltage ratings Some additional comments are in order concerning certain of the excitation voltage definitions First, it may be helpful to state certain numerical values of exciter ratings offered by the manufacturers (see [2] for a discussion of exciter ratings) Briefly, ex- citers are usually rated at 125 V for small generators, say 10MVA and below Larger units usually have 250-V exciters, say up to 100 MVA; with still larger machines being equipped with 350-V, 375-V, or 500-V exciters The voltage rating and the ceiling voltage are both important in considering the speed of response [1,17] Reference [1] tabulates the pattern of ceiling voltages for various response characteristics in Table 7.2, which shows the improved response fox higher ceiling voltage ratings (and the lower ceiling voltage for solid-state exciters) It is reasonable that an exciter with a high ceiling voltage will build up to a particular volt- 248 Chapter 7 age level faster than a similar exciter with a lower ceiling voltage simply because it saturates at a higher value This is an important consideration in comparing types and ratings of both conventional and solid state excites as shown in Table 7 2. Table7.2. Typical Ceiling Voltages for Various Exciter Response Ratios Response Per unit ceiling voltage rm conventional exciterst SCR exciters 05 125-135 1.20 10 140-150 120-1 25 Is 135-165 1 30-1.40 20 170-180 145-155 40 : 2.00-2.10 “Based on rated exciter voltage In adopting a pu system for the exciter, there is no obvious choice as to what base voltage to use, Some possibilities are (also see [2)): (A) exciter rated voltage, (B) rated load field voltage, (C) rated air-gap voltage (the voltage necessary to produce rated voltage on the air gap line of the main machine in the case of a de generator exciter), and (D) no-load field voltage. The IEEE [3] recommends the use of system B, the rated load field voltage. Consider, as an example, an exciter rated at 250V_ For this rating some typical values of other defined voltages are given in Figure 7.23. The pu system A IL typed 2.78 oted voltage 2.22 Reted load eld voltage foted oie 989 voltoge Fig. 723 Per unit voliages for a 250-V exciter - of Figure 7 23 has little merit and is seldom used System B is often used System C is often convenient since, with rated air gap voltage as a base, pu exciter voltage, pu field current, and pu synchronous internal voltage are all equal under steady-state conditions with no saturation System D is not illustrated in Figure 7.23 and is seldom used. 7.43 Other specifications Excitation control system response should be compared against a suitable criterion of performance if the system is to be judged or graded. System performance could be measured under any number of forcing conditions It is generally agreed that the quan- tity of primary interest is the exciter voltage-time characteristic in response to a step change in the generated voltage of from 10 to 20% [18,19]. The problem is how to state in words the various possible slopes, delays, overshoots, damping, and the like One useful description, often used in control system specification, is that based on the Excitation Systems 249 Sytem Responee, ps Tine, © Fig, 724 Time domain specifications [22] curve shown in Figure 724 Here the curve is the response to a step change in one of the system variables, such as the terminal voltage This response, based on that of a second-order system, is 2 reasonable one on which to base time domain specifications since many systems tend to exhibit two “least-damped” poles that give a response of this general shape at some value of gain {20,21} Three quantities describe this re- sponse: the overshoot, the rise time, and the settling time The overshoot is the amount that the response exceeds the steady-state response— in Figure 724, a, pu The rise time is the time for the response to rise from 10 to 90% of the steady-state response The settling time is the time required for the response to a step function to stay within a certain percentage of its final value Sometimes it is given as the time re- quited to artive at the final value after first overshooting this value. The first definition is preferred . The damping ratio is that value for a second-order system defined by ¢ in the ex- pression GQ) = KMS + 2Fa,s + 03) an and is related to the values a, and a, of two successive overshoots [23] The naru- ral resonant frequency 1, is also of interest and may be given as a specification. In the case of the second-order system (7 7), the response to a step change of a driv- ing variable is c(t) = 1 — e-#fooswt + [f/(L — Psinw,r} (7.8) where we, = @,(1 — 2)" 79) When ¢ = 0, the system is oscillatory; when { = 07, it has very little overshoot (about 5%). Critical damping is said to occur when ¢ = 10. In dealing with an exciter being forced to ceiling due to a step change in the voltage regulator control, the system is often “overdamped”; ie, ¢ > 1. In this case the voltage rise is more “sluggish,” as shown in Figure 7.25. Here the overshoot is zeto, the settling time is T, (ie,, the time for the response to settle within & of its final value), and the rise time is Ty Reference [19] suggests testing an excitation system to determine the response, such as in Figure 7.25. Then determine the area under this curve for 0.55 and use this as a specification of response in the time domain. For newer, fast systems reference [3] suggests simulation of the excitation as preferable to actual testing since on some systems certain parameters are unavailable for measurement (8, 9] 250 Chapter 7 001 02 03 04 05 Tine Fig 725 Response of an excitation system 7.5 Voltage Regulator In several respects the heart of the excitation system is the voltage regulator (Def. 2.12). This is the device that senses changes in the output voltage (and current) and causes corrective action to take place. No matter what the exciter speed of the response, it will not alter its response until instructed to do so by the voltage regulator, If the regulator is slow, has deadband or backlash, or is otherwise insensitive, the system will bea poor one. Thus we need to be very critical of this important system component In addition to high reliability and availability for maintenance, it is necessary that the voltage regulator be a continuously acting proportional system. This means that any corrective action should be proportional to the deviation in ac terminal voltage from the desired value, no matter how smail the deviation Thus no deadband is to be tolerated, and large errors are to receive stronger corrective measures than small errors. In the late 1930s and early 1940s several types of regulators, electronic and static, were developed and tested extensively [24, 25] These tests indicated that continuously acting proportional control “increased the generator steady-state stability limits well beyond the limits offered by the theostatic regulator” (24,26). This type of system was ther fore studied intensively and widely applied during the 1940s and 1950s, beginning with application to synchronous condensers; then to turbine generators; and finally, in the early 1950s, to hydrogenerators (Reference (24] gives an interesting tabulation of the progress of these developments.) 7.5.1 Electromechanical regulators The rather primitive direct-acting regulator shown in Figure 7.8 is an example of an electromechanical regulator. In such a system the voltage reference is the spring tension against which the solenoid must react. It is reliable and independent of auxiliaries of any kind, The response, however, is sluggish and includes’ deadband and backlash due to mechanical friction, stiction, and loosely fitting parts Two types of electromechanical regulators are often recognized; the direct-acting and the indirect-acting, Direct-acting regulators, such as the Silverstat (2) and the Tirtell (24], have been in use for many years, some dating back to about 1900 Such devices were widely used and steadily improved, while maintaining essentially the same form. As machines of larger size became more common in the 1930s the indirect- acting rheostatic regulators began to appear. These devices use a relay as the voltage- sensitive element [24]; thus the reference is essentially a spring, as in the direct-acting device. This relay operates to control a motor-operated theostat, usually connected between the pilot exciter and the main exciter, as in Figure 79. This regulator i ited in its speed of response by various mechanical delays Once the relay closes, to Excitation Systems 251 short out a rheostat section, the response is quite fast In some cases, high-speed relays are used to permit faster excitation changes. These devices were considered quite suc- cessful, and nearly all large units installed between about 1930 and 1945 had this type of control, Many arestill in service Another type of indirect-acting regulator that has seen considerable use employs a polyphase torque motor as a voltage-sensitive element [27]. In such a device the output, torque is proportional to the average three-phase voltage This torque is balanced against a spring in torsion so that each value of voltage corresponds to a different angular position of the rotor, A contact assembly attached to the rotor responds by closing contacts in the rheostat as the shaft position changes A special set of contacts closes very fast with rapid rotor accelerations that permit faster than normal response due to sudden system voltage changes. The response of this type of regulator is fairly fast, and much larger field currents can be controlled than with the direct-acting regu- lator. This is due to the additional current “gain” introduced by the pilot ex main exciter scheme. The contact type of control, however, has inherent deadband and this, coupled with mechanical backlash, constitutes a serious handicap a 7.5.2. Early electronic regulators About 1930 work was begun on electronic voltage regulators, electronic exciters, and electronic pilot exciters used in conjunction with a conventional main exciter (24, 25], In general, these carly electronic devices provided “better voltage regulation as well as smoother and faster generation excitation control” [24] than the competitive indirect- acting systems. They never gained wide acceptance because of anticipated high mainte- nance cost due to limited tube life and reliability, and this was at least partly justified in later analyses [25]. Generally speaking, electronic voltage regulators were of two types and used either to control electronic pilot exciters or electronic main exciters [25] The electronic exciters or pilot exciters were high-power de sources usually employing thyratron or ignitron tubes as rectifying elements 7.5.3. Rotating amplifier regulators In systems using a rotating amplifier to change the field of a main exciter, as in Figure 7 10, it is not altogether clear whether the rotating amplifier is a part of the “voltage regulator” or is a kind of pilot exciter. Here we take the view that the rotating amplifier is the final, high-gain stage in the voltage regulator The development of rotating amplifiers in the late 1930s and the application of these devices to generator excitation systems (28, 29] have been accompanied by the develop- ment of entirely “static” voltage sensing circuitry to replace the electromechanical de- vices used earlier Usually, such static circuits were designed to exclude any electronic active components so that the reliability of the device would be more independent of component aging. For example, devices employing saturable reactors and selenium rectifiers showed considerable promise Such circuits supplied the field windings of the rotating amplifiers, which were connected in series with the main exciter field, as in Figure 7.10. This scheme has the feature that the rotating amplifier can be bypassed for maintenance and the generator can continue to operate normally by manual regula~ tion through a field rheostat This connection is often called a “boost-buck” connec tion since, depending on polarity, the rotating amplifier is in a position to aid or oppose the exciter field The operation of a typical rotating amplifier regulating system can be analyzed by reference to Figure 710 The generator is excited by a self-excited shunt exeiter, The 252 Chapter 7 Fela Tine Seta Fras Sa shag Field Valioge, vp ‘Exciter Shunt Field Current Fig 726 V-I characteristie defining boost and buck regions field circuit can be controlled either manually by energizing a relay whose contacts bypass the rotating amplifier or automatically, with the amplifier providing a feedback of the etror voltage to increase or decrease the field current, The control characteristic may be better understood by examining Figure 726 The field theostat is set fo intersect the saturation curve at a point corresponding to rated terminal voltage, ic, the exciter voltage required to hold the generated voltage at rated value with full load Under this condition the rotating amplifier voltage is zero. ‘Now suppose the generator load is reduced and the generator terminal voltage be- gins to rise The voltage sensing circuit (described later) detects this rise and causes the rotating amplifier to reduce the field current in the exciter field. This reduces the exciter voltage, which in turn reduces j;, the generator field current Thus the shaded area above the set point in Figure 7 26 is called the buck voltage region A similar reasoning defines the area below the set point to be the boost voltage region Rotating amplifier systems have a moderate response ratio, often quoted as about 0.5 (c.g. sce Appendix D) The speed of response is due largely to the main exciter time constant, which is much greater than the amplidyne time constant The ceiling voltage is an important factor too, exciters with higher ceilings having much faster response than exciters of similar design but with lower ceiling voltage (see [17] for a discussion of this topic) The voltage rating of the rotating amplifier in systems of this type is often comparable to the main exciter voltage rating, and the voltage swings of the amplifier change rapidly in attempting to regulate the system (24) 7.5.4 Magnetic amplifier regulators Another regulator-amplifier scheme capable of zeto deadband proportional control is the magnetic amplifier system (6, 30, 31) (We use the generic term “magnetic ampli- fier” although those accustomed to equipment of a particular manufacturer use trade names, ¢ g, Magamp of the Westinghouse Electric Corporation and Amplistat of the General Electric Company.) In this system a magnetic amplifier, ie.,a static amplifying device (32, 33], replaces the rotating amplifier. Usually, the magnetic amplifier consists ofa saturable core reactor and a rectifier. It is essentially an amplifying device with the advantages of no rotating parts, zero warm-up time, long life, and sturdy construc tion It is restricted to low or moderate frequencies, but this is no drawback in power applications Excitation Systems 253 sono Lad Soturble ood T rol ing Fig 727. Magnetic amplifier Basically, the magnetic amplifier is similar to that shown in Figure 727 [33] The current owing through the load is basically limited by the very large inductance in the saturable core main windings. As the core becomes saturated, however, the current jumps to a large value limited only by the load resistance. By applying a small (low- power) signal to the control winding, we control the firing point on each voltage (or current) cycle, and hence the average load current. This feature, of controlling a large output current by means of a small control current, is the essence of any amplifier The fact that this amplifier is very nonlinear is of little concern One type of regulator that uses a magnetic amplifier is shown in block diagram form in Figure 7 10 {4} Here the magnetic amplifier is used to amplify a voltage error signal to a power level satisfactory for supplying the field of a rotating amplifier The rotating amplifier is located in series with the exciter field in the usual boost-buck con- nection. One important feature of this system is that the magnetic amplifier is relatively insensitive to variations in line voltage and frequency, making this type of regulator favorable to remote (especially hydro) locations. Another application of magnetic amplifiers in voltage regulating systems, shown in Figure 7.11 [6], has several features to distinguish it from the previous example. First, the magnetic amplifiers and reference are usually supplied from a 420-Hz system sup- plied by a permanent-magnet motor-generator set for maximum security and reliability. The power amplifier supplies the main exciter directly in this system Note, however, that the exciter must have two field windings for boost or buck corrections since mag- netic amplifiers are not reversible in polarity. The main exciter also has a self-excited, theostat-controlied field and can continue-to operate with the magnetic amplifiers out of service The magnetic amplifier in the system of Figure 7.11 consists of a two-stage push- pull input amplifier that, with 1-mW input signals, can respond to maximum output in three cycles of the 420-Hz supply. The second stage is driven to maximum output when the input stage is at half-maximum, and its transient response is also about three cycles. The figures of merit [34] are about 200/cycle for the input stage and 300/eycle for the output stage This compares with about 500/s for a conventional pilot exciter The power amplifier has a figure of merit of 1500/cycle with an overall delay of less than 0,01. (The figure of merit of an amplifier has been defined as the ratio of the power amplification to the time constant It is shown in [34] that for static magnetic amplifiers it is equal to one-half the ratio of power output to stored magnetic energy.) Reference 6] reviews the operating experience of a magnetic amplifier regulator in- stallation on one 50-MW machine in 2 plant consisting of seven units totaling over 300 MW, only two units of which are regulated, The experience indicates that, since 254 Chapter 7 the magnetic amplifier regulator is so much faster than the primitive rheostatic regula- tor, it causes the machine on which it is installed to absorb much of the swing in load, particularly reactive load In fact, close observation of operating oscillograms, when operating with an arc furnace load, reveals that both exciter voltage and line currents undergo rapid fluctuations when regulated but are nearly constant when unregulated. This is to be expected since the regulation of machine terminal voltage to a nearly constant level makes this machine appear to have a lower reactance, hence it absorbs changes faster than its neighbors In the case under study, the machine terminal volt- age was regulated to 40.25%, whereas a -t1% variation was observed with the regulator disconnected [6} 75.5 Solid-state regulators Some of the amplification and comparison functions in modern regulators consist of solid-state active circuits [3] Various configurations are used depending on the manufacturer, but all have generally fast operation with no appreciable time delay com- pated to other system time constants The future will undoubtedly bring more applica~ tions of solid-state technology in these systems because of the inherent reliability, ease of maintenance, and low initial cost of these devices 7.6 Exciter Buildup Exciter response has been defined as the rate of inctease or decrease of exciter volt- age when a change is demanded (see Appendix E, Def 3.15). Usually we interpret this demand to be the greatest possible control effort, such as the complete shorting of the field resistance. Since the exciter response ratio is defined in terms of an unloaded exciter (Def. 3.19), we compute the response under no-load conditions. This serves to satisfy the terms of the response ratio definition and also simplifies the computation or test procedure The best way to determine the exciter response is by actual test where this is pos- sible The exciter is operated at rated speed (assuming it is a rotating machine) and with no load. Then a step change in a reference variable is made, driving the exciter voltage to ceiling while the voltage is recorded as a function of time This is called a “buildup curve.” In a similar way, a “build-down” curve can also be recorded Curves thus recorded do not differ a great deal from those obtained under loaded conditions If itis impractical to stage a test on the exciter, the voltage buildup must be computed We now turn our attention to this problem 7.6.1 The de generator exciter In dealing with conventional de exciters three configurations (ie. separately ex- cited, seliexcited, and boost-buck) are of interest They must be analyzed indepen- dently, however, because the equations describing them are different. (Portions of this analysis parallel that of Kimbark [16], Rudenberg (20), and Dahl [35] to which the reader is referred for additional study ) Consider the separately excited exciter shown in Figure 728. Summing voltage drops around the pilot exciter terminal connection, we have Ae + Ri = vy (7.10) where Ag = flux linkages of the main exciter field, Wb turns R = main exciter field resistance, 2 i= current, A’ », = pilot exciter voltage, V Excitation Systems 255 Gonerctor Moin exciter Contactor Fig 728 Separately excited exciter It is helpful to think in terms of the field flux @, rather than the field flux linkages If we assume the field flux links turns, we have Noy + Ri =v, qn The voltage of the pilot exciter v, may be treated as a constant [16] Thus we have an equation in terms of i and ¢, with all other terms constant. The problem is that i de- pends on the exact location of the operating point on the saturation curve and is not linearly related to vp. Furthermore, the flux @, has two components, leakage flux and armature flux, with relative magnitudes also depending on saturation Therefore, (7.11) is nonlinear Since magnetization curves are plotted in terms of vy versus i, we replace 6, in (7.11) by a term invélving the voltage ordinate ¥, Assuming the main exciter to be run- ning at constant speed, its voltage v, is proportional to the air gap flux $,5ie., Ue = kb, 712) The problem is to determine how 4, compares with ¢, The field flux has two com- ponents, as shown in Figure 7.29 The leakage component, comprising 10-20%, of the total, traverses a high-reluctance path through the air space between poles It does not link all NV turns of the pole on the average and is usually treated either as proportional to @, of proportional toi Let us assume that 4, is proportional to @, (see [16] for a more detailed discussion), then p= C6, ay where C is a constant. Also, since (14) Fig 729 Armature of air gap flux gg, leakage flux #2, and field flux de = oe + $2 (Reprinted by per- mission from Power System Stability vol 3, by EW. Kimbark © Wiley. 1956 ) 256 Chapter 7 we have be = (1 + C)d. = ody (7.18) where a is called the coefficient of dispersion and takes on values of about 11 to 12 Substituting (7.15) into (7.11), tpg + RI= vy (7.16) where re = (No/k) s, and where we usually assume ¢ to be a constant, This equation is still nonlinear, however, as Uy is not a finear function of i We usually assume u, to be a constant Ina similar way we may develop the differential equation for the self-excited exciter shown in Figure 7.30, where we have Ag + Ri = Us or Nop + Ri = (7.17) Fig 730. Seli-excited exciter Following the same logic regarding the fluxes as before, we may write the nonlinear equation Tey + Ri = Up (7:18) for the self-excited case where rp is the same as in (7.16) Ina similat way we establish the equation for the self-excited exciter with boost- buck rotating amplification as shown in Figure 7.31, Writing the voltage equation with the usual assumptions, rep + Ri = Uy + Up (19) Kimbark [16] suggests four methods of solution for (7.16)-(7 19) These are (1) formal integration, (2) graphical integration (area summation), (3) step-by-step inte- gration (manual), and (4) analog or digital computer solution. Formal integration re- quites that the relationship between v, and i, usually expressed graphically by means of ‘the magnetization curve, be known explicitly An empirical relation, the Frohlich equa- tion [35] Fig 731 Self-excited exciter with a rotating amplifier (boost-buck) Excitation Systems 257 vy = aif(b +1) (720) may be used, or the so-called modified Frohlich equation vp = aif(b + i) +i (721) can be tried. In either case the constants a, b, and ¢ must be found by cut-and-try techniques. If this is reasonably successful, the equations can be integrated by separa- tion of variables, ‘Method 2, graphical integration, makes use of the saturation curve to integrate the equations, This method, although somewhat cumbersome, is quite instructive. It is unlikely, however, that anyone except the most intensely interested engineer would choose to work many of these problems because of the labor involved. (See Kim- bark [16], Rudenberg [20}, and Dahl {35} for a discussion of this method ) Method 3, the step-by-step method (called the point-by-point method by some authors [16, 35]), is 2 manual method similar to the familiar solution of the swing equa- tion by a stepwise procedure [36] In this method, the time derivatives are assumed constant over a small interval of time, with the value during the interval being de- pendent on the value at the middle of the interval Method 4 is probably the method of greatest interest because digital and analog computers are readily available, easy to-use, and accurate The actual methods of com- putation are many but, in general, nonlinear functions can be handled with relative case and with considerable speed compared to methods 2 and 3 In this chapter the buildup of a de generator will be computed by the formal inte- gration method only "However, an analog computer solution and a digital computer technique are outlined in Appendix B To use formal integration, a nonlinear equation is necessary to represent the satura tion curve For convenience we shall use the Frohlich equation {7 20), which may be solved for /to write i= bu-/(a — 04) (722) We illustrate the application of (7 22) by an example Example 72 A typical saturation curve for a separately excited generator is given in Figure 7.32 Approximate this curve by the Frohlich equation (7.22) Solution By examination of Figure 7 32 we make the several voltage and current observa- tions given in Table 7.3 Table7.3. Exciter Generated Voltages and Field Currents i AO 4 23 4 5 6 7 8 9 W ve V 0 30 60 90 116 134 147 156 164 172. 179 Since there are two unknowns in the Frohlich equation, we select two known points on the saturation curve, substitute into (7.20) or (7.22), and solve for a and 6 One ex- perienced in the selection process may be quite successful in obtaining a good match To illustrate this, we will select two paits of points and obtain two different solutions 258 8 bxciter Voltage, ve, volts 3 & excite Chapter 7 + ’ Id Curent, 1, amperes 3 0 Fig, 7 32. Saturation curve of a separately excited exciter Solution x1 Select i= 3,y, = 90 i 9,v, = 172 Then the equations to solve are 90 = 3a/(3 + b) 172 = 9a/(9 +b) for which the solutions are a = 3159 V b= 753 A U6 164 by Solution 42 Abe = 116 8.0, = 164 4a/(4 + b) 8a/(8 + 6) 2799 Vv 565 A Excitation Systems 289 Both solutions are plotted on Figure 7.32 For solution 1 vp = 315 91/(7.53 +) oF i = 7.530;/G159 ~ vp) (723) and for solution 2 bp = 279.9i/(5.65 + i) oF § = 5650,/(2799 — Uy) (7.24) Example 7.3 Approximate the saturation curve of Figure 7.32 by a modified Frohlich equation Select values of i = 2, 5, and 10 Solution 2 60 = 2a/(2 + b) + 2c i=5 134 = Sa/(S +6) + Se i= 10 179 = 10a/(10 + 6) + 10¢ Solving simultaneously for a, b, and c, a=3599 b= -2195 c= 480 This gives us the modified formula Up = 359i/(i — 21.95) + 48: (725) Equation (7.25) is not plotted on Figure 7.32 but is a better fit than either of the other two solutions, Separately excited buildup by integration. For simplicity, let the saturation curve be represented by the Frohlich equation (722), Then, substituting for the current in (7.16), tebe + bR ola — v;) = v, (7.26) This equation may be solved by separation of variables. Rearranging algebraically, we write dt = (ra — vf) (av, ~ hv,)] dv, (7.27) where we have defined for convenience, h = v, + 6R. Integrating (7 27), (= )/te = (AYO ~ Opn) — (@bR/H?)In|(av, — hvg)/(ar, ~ hvpo)] (728) This equation cannot be solved explicitly for us, so we leave it in this form Example 74 Using the result of formal integration for the separately excited case (7.28), compute the v, versus 1 relationship for values of t from 0 to 1s and find the voltage response ratio by graphical integration of the area under the curve. Assume that the following constants apply and that the saturation curve is the one found in Example 72, solu- tion 2. N= 2500tums = 125 V R=34.9 on 12 k= 12,000 vp = 90 V 260 Chapter 7 Solution First we compute the various constants involved From (7.16) 1, = No/k = (2500)(1.2)/12,000 = 0.25 s Also, from Example 7 2 a= 2799 ~ 280 b = 565 Now, from the given data, the initial voltage Uy, is 90 V. Then from the Frohlich equa- tion (7.22) we compute ig = $.65(90)/(280 — 90) = 2675 A This means that there is initially a total resistance of Ry = 125/2675 = 467 2 of which all but 34 is in the field rheostat. Assume that we completely short out the field rheostat, changing the resistance from 46.7 to 342 att = 0. Since v, is 125 V, we compute the final values of the system variables From the field circuit, i, = v,/R = 125/34 = 3675 A Then, from the Frohlich equation the ceiling voltage is Vee = ai [(b + i) = 280(3,675)/(5.65 + 3.675) = 1103 V Using the above constants we compute the U, versus f relationship shown in Table 74 and illustrated in Figure 7 33 Table'7.4, Buildup of Separately Excited ¥, for Example 7.4 a oF 000 9000 04510955 005 95.85 050 109.79 010 = 10012055109 96 O15 10318 = 0601008, 020 10535 065 110.16 025 10687 = 07041021 030 © 10794 0.75 110.25 035 © 10868 «= 080 110.28 040 109.19 0.85 110.30 From Figure 7.33, by graphical construction we find the triangle acd, which has the same area as that under the v, curve abd Then from (7 5) with cd = 27.9V, as shown in the figure, the response ratio = 27 9/90(0.5) = 0 62. Self-excited buildup by integration. For a self-excited machine whose saturation curve is represented by the Frohlich approximation (7.22), we have Teds + DRUG /(a — Vp) = Us (7.29) Excitation Systems 261 volts Exciter Volto, vy, tou Lt oor 0 Os OF 05 02 OF ime, # Fig 733 Buildup of the separately excited exciter for Example 7 4 This is recognized to be identical to the previous case except that the term on the right side is U, instead of v, Again we rearrange the equation to separate the variables as Tela ~ Up)dvy = SH 7.30) G@- BR, (30) This equation can be integrated ftom 1, to t with the result fo ayy MH _ OR yy AK (731) Ki Um Kp K where K = Example 75 Compute the self-excited buildup for the same exciter studied in Example 74 ‘Change the final resistance (field resistance) so that the self-excited machine will achieve the same ceiling voltage as the separately excited machine Compare the two buildup curves by plotting the results on the same graph and by comparing the computed re- sponse ratios Solution The ceiling voltage is to be 110 3V, at which point the current in the field is 3 68 A (trom the Frohlich equations) Then the resistance must be R = 1103/3.68 = 302 Solving (7 31) with this value of R and using Frohlich parameters from Example 7 4, we have the results in Table 7.5 and the solution curve of Figure 7.34 The response tatio = 15.4/90(0 5) = 0 342 for the self-excited case 262 Chapter 7 . Fesponte ratio citer Voltage, vey volts 4 piu oO FOF 04 OF 08 OF 08097 Tne, 5 Fig. 7.34 Buildup of the self-excited exciter for Example 7 5 Table 75. Buildup of Self-excited b; for Example 7.5 oF oF 0.00 9000 05010338 os 9187 © 055 1041s 010 9361 06010485 015 9523 06510847 020 9673 070 106.03 025 9810 075 106 52 030 9937 080 106.96 035 1005208510736 040 © 10157 090,107.71 045 102.52 : Boost-buck buildup by integration The equation for the boost-buck case is the same as the self-excited case except the amplifier voltage is added to the right side, or Tei + DRUGf(a — vp) = Vp + Ve (7:32) Rearranging, we may separate variables to write dt = r4(a — v,)dv,/(A + Mey — v3) (7:33) where A = avpand M = a — vp - OR Integrating (7.33), we compute fy 2a = My, (M = Q ~ 20M + Q ~ 2p) Te Q (M — Q ~ 2vp)(M + Q — 205) 1, At Me = vB) +3" Le Mog > vip where Q = Viaa ME (7.34) Excitation Systems 263 Example 76 Compute the boost-buck buildup for the exciter of Example 7.4 where the ampli- fier voltage is assumed to be a step function at ¢ = 4 with a magnitude of SOV. Com- pare with previous results by adjusting the resistance until the ceiling voltage is again 1103V. Repeat for an amplifier voltage of 100 V Solution With a ceiling voltage of 110 3V and an amplifier voltage of 50 V, we compute with i, = 0, Ri, = vy + Uy = 1603 This equation applies as long as vz maintains its value of 50V. This requires that /,, again be equal to 3.68 A so that R may be computed as R = 160 3/3.68 = 436 @ This value of R will insure that the ceiling voltage will again be 110.3 V_ Using this & in (7 34) results in the tabulated values given in Table 76 Repeating with vg = 100 V gives a second set of data, also tabulated, in which R= 5720. Table 7.6. Buildup of Boost-Buck v for Example 7.6 t opfor ug = $0 vp for ug = 100 ~ 000 90.00 90.00 005 94.23 96.32 10 9770 100.84 0.15 100 50 103.98 020 10272 106 12 025 104.47 107.56 030 105.84 108.51 035 106.90 109.14 040 107 72 109 56 045 108 34 109.83 0.50 108 82 110.00 055 109.19 110.12 0.60 109.47 110.20 0.65 109 68 110 24 070 109.84 110.27 075 109.96 110.30 080 1100s 110 31 085 110 12 110.32 0.90 110.17 110.33 These results are plotted in Figure 7.35. Note that increasing the amplifier voltage has the effect of increasing the response ratio. In this case changing U, from 50 to 100 V gives a result that closely resembles the separately excited case In each case the re~ sponse ratio (RR) may be calculated as follows: RR (vy = 50) = 2ed/Oa = 2(24.15)/90 = 0537 RR (Up = 100) = 2c'd/Oa = 2(29)/90 ~ 0.645 7.6.2. Linear approximations for de generator exciters Since the Froblich approximation fails to provide a simple ur versus z relationship, other possibilities may be worth investigating. One method that looks attractive be- cause of its simplicity is to assume a linear magnetization curve as shown in Figure 7 36, where 264 Chapter 7 sf ee —e 2 110 ve oow-back s BON te ee oe sh ip, Separately, p ree i 5 ed vg Self-encited oo o OT 02 0.3 0405 08 07 Times + Fig 735 Buildup of boost-buck exciters for Example 7 6. Dp = mit n (7.35) ‘Substituting (7.35) into the excitation equation we have the linear ordinary differential equation TeV = V ~ (R/m\(v, ~ n) (7.36) where 0 =v, separately excited vy self-excited =v + Ug boost-buck excited Exciter Voltage, vge vols Exciter Feld Curren, fy amperes Fig. 7.36 Linear approximation to a magnetization curve 265 This equation may be solved by conventional techniques The question of interest is, What values of m and 1, if any, will give solutions close to the actual nonlinear solu- tions? This can be resolved by solving (7.36) for each case and then systematically trying various values of m and 1 to find the best “fit” This extremely laborious process becomes much less painful, or even fun, if the comparison is made by analog computer In this process, both the linear and nonlinear problems are solved simultaneously and the solutions compared on an oscilloscope A simple manipulation of two potentiom- eters, one controlling the slope and one controlling the intercept, will quickly and easily permit an optimum choice of these parameters. The procedure will be illustrated for the separately excited case Linear approximation of the separately excited case. In the separately excited case we set v = v, so that (7.36) becomes b, = ky — kau, where k= (l/te)(v, + R/m) ky = R/tem (737) Solution of (7.37) gives Ae (= (kif) = Ju(t) + Uppe"u(1) (7.38) Equation (7 38) is solved by the analog computer connection shown in Figure 7.37 and compared with the solution of (7 76) given in Appendix B, shown in Figure B.9 Fig 7.37. Solution of the linear equation Adjusting potentiometers k, and k, quickly provides the “best fit” solution shown in Figure 7.38, which is a graph made directly by the computer. Having adjusted &, and k, for the best fit, the potentiometer settings are read and the factors m and m computed. Ina similar way linear approximations can be found for the self-excited and boost-buck connections Exciter Voltoge, ¥gy volts Time, + Fig 738 Anaiog computer comparison of linear and Frohlich models of the separately excited buildup 266 Chapter 7 7.63 The ac generator exciters As we observed in Chapter 4, there is no simple relationship between the terminal voltage and the field voltage of a synchronous generator, Including all the detail of Chapter 4 in the analysis of the exciter would be extremely tedious and would not be warranted in most cases. We therefore seck a reasonable approximation for the ac exciter voltage, taking into account the major time constants and ignoring other effects Kimbark (16 has observed that the current in the de field winding changes much more slowly than the corresponding change in the ac stator winding Therefore, since the terminal voltage is proportional to i, (neglecting saturation), the ac exciter voltage will change approximately as fast as its field current changes The rate of change of field current depends a great deal on the external impedance of the stator circuit or on the load impedance But, using the response ratio definition (see Def 3.19, Appendix E) we may assume that the ac exciter is open circuited In this case the field current in the exciter changes according to the “direct-axis transient open circuit time constant” ‘rio Where tio = Lelte 8 (739) This will give the most conservative (pessimistic) result since, with a load impedance connected to the stator, the effective inductance seen by the field current is smaller and the time constant is smaller Using relation (7.39) we write, in the Laplace domain, v¢(s) = [K/(1 + rdys)] vals) (7.40) where u,(s) is the Laplace transform of the open circuit field voltage and Ug(s) is the transform of the regulator voltage. If the regulator output experiences a step change of magnitude D att = f, the field voltage may be computed from (7 40) to be. 4y This linearized result does not include saturation or other nonlinearities, but does in- clude the major time delay in the system. An ac excites designed for operation at a few hundred Hz could have a very respectable rp, much lower than that of the large 60-Hz generator that is being controlled 7.64 Solid-state exciters Modern solid-state exciters, such as the SCR exciter of Figure 7 14, can go to ceil- ing without any appreciable delay In systems of this type a small delay may be required for the amplifiers and other circuits involved. The field voltage may then be assumed to depend only on this delay One way to solve this system is to assume that v, changes linearly to ceiling in a given time delay of ¢, s, where (, may be very small, This is nearly the same as per- mitting a step change in vs. For such fast systems the time constants are so much smaller than others involved in the system that assuming a step change in v, should be fairly accurate 7.6.5 Buildup of a loaded de exci Up to this point we have considered the response characteristics of unloaded ex- citers, ie, with ip = 0 If the exciter is loaded, the load current will affect the ter minal voltage of the exciter v, by an amount depending upon the internal impedance of the exciter In modern solid-state circuits this effect will usually be small, amounting to r Excitation Systems 267 essentially a small series iR drop In rotating de machines the effect is greater, since in addition to the i, R drop there is also the brush drop, the drop due to armature reac- tion, and the drop due to armature inductance. (Dahl [35] provides an exhaustive treat- ment of this subject and Kimbark [16] also has an excellent analysis ) We can analyze the effect of load current in a de machine as follows First, we recognize that the armature inductance is small, and at the relatively slow rate of buildup to be experienced this voltage drop is negligible. Furthermore, if the machine has interpoles, we may negiect demagnetizing armature reaction. However, we do have to estimate the effect of cross-magnetizing armature reaction, which causes @ net de- crease in the air gap flux. Thus, the net effect of load is in the resistance drop (including brush drop) and in the decrease in flux due to cross-magnetizing armature reaction To facilitate analysis, we assume the load current i, has a constant value This means the i, drop is constant, and the armature reaction effect depends on the value of current in the field, designated / in our notation. The combined effect is determined most easily by test, a typical result of which is shown in Figure 7.39 To the load Nolo seturation ExeitorValtoge, vp + volt Eaciver Feld Corre, 1, amperer Fig 739 No-load and load sutvration curves. (Reprinted by permission from Power System Stabitirs vol 3, byE W Kimbark © Wiley, 1956 } saturation curve is added the resistance drop to obtain a fictitious curve designated “distortion curve” This curve shows the voltage generated by air gap flux at this value of i, as a function of i, and it differs from the no-load saturation curve by an amount due to armature reaction. The magnitude of this difference is greatest near the knee of the curve Kimbark [16] treats this subject thoroughly and is recommended to the interested reader. We will ignore the loading eflect in our analysis in the interest of finding a reasonable solution that is a fair representation of the physical device As in all en- gineering problems, certain complications must be ignored if the solution is to be manageable jation of E 7.66 Norm The exciter equations in this book are normalized on the basis of rated air gap voltage, ie, exciter voltage that produces rated no-load terminal voltage with no saturation This is the py system designated as C in Figure 723. Thus at no load and with no saturation, B;»°= 1,0 pu corresponds to V, = 10 pu 1 Equations 268 Chapter 7 The slip ring voltage corresponding to 1.0 pu Epp is not the same base voltage as that chosen for the field ciccuit in normalizing the synchronous machine From (4.55) we have Ves = Vela/lew = Se/len V This base voltage is usually a very large number (163 kV in Example 4 1, for example) The base voltage for £,p, on the other hand, would be on the order of 100 V or so. Simply stated, the exciter base voltage and the synchronous machine base for the field voltage differ, and a change of base between the two quantities is required. The re~ quired relationship is given by (4.59), which can be written as Exp = (LyolV317) 0, PA Exp = (ookM;/V3r) ¥¢ V (742) Thus any exciter equation may be divided through by ¥;, to obtain an equation in v,, and then multiplied by L4/W3r, to convert to an equation in Esp, For exam- ple, for the de generator exciter we have an equation of the form r,0, = flue) V__Di viding through by V,, we have the pu equation 7,0,, = fly) Multiplying by Laof V3re, we write the exciter equation 7¢£rp, = f( Esp.) Itis necessary, of course, to always maintain the “gain constant” V37_/L4p be- tween the exciter £,, output and the v, input to the synchronous machine. This con- stant is the change of base needed to connect the pu equations of the two machines. 77 Excitation System Response The response of the exciter alone does not determine the overall excitation system response, As noted in Figure 7 20, the excitation system includes not only the exciter but the voltage regulator as well The purpose of this section is to compute the response of typical systems, including the voltage regulators. This will give us a feel for the equa- tions that describe these systems and will illustrate the way a mathematical model is constructed 7.7.1 Noncontinuously regulated systems Early designs of voltage regulating schemes, many of which are still in service, used an electromechanical means of changing the exciter field theostat to cause the desired change in excitation. A typical scheme is shown in Figure 7 40, which may be explained as follows. Any given level of terminal voltage will, after rectification, result in a given voltage ¥, across the regulating coil and a given coil current {, This current flowing in the regulating coil exerts a pull on the plunger that works against the spring K and dashpot B- Thus, depending on the reference screw setting, the arm attached to the plunger will find a new position x for each voltage V, High values of V, will increase the coil voltage v, and pull the arm to the right, reducing x, ete Note that the ref erence is the mechanical setting of the reference screw Now imagine a gradual increase in V, that pulls the arm slowly to the right, reduc- ing x until the lower contact £ is made, This causes current to flow in the coil L, closing the theostat motor contact and moving the rheostat in the direction to increase Ry This, as we have seen, will reduce V, Note that there is no corrective action at all until a contact is closed. This constitutes an intentional dead zone in which no control action is taken. Once control action is begun, the rheostat setting will change at an assumed con- stant rate until the maximum or minimum setting is reached Mathematically, we can describe this action as follows From (7.16) we have, for the separately excited arrangement, Excitation Systems 269 regulating Quick ee ‘ower contoch: Tine delayed raise Blower conrach —ooperoting calls, Fig 740 A noncontinuous regulator for a separately excited system The scheme illustrated is « simpli fied sketch similar to the Westinghouse type BY system [2] Tee = v, — Ri (7.43) and in this case the regulating is accomplished by a change in R But R changes as a function of time whenever the arm position x is greater than some threshold value K,. This condition is shown in Figure 7 41 where the choice of curve depends on the mag- nitude of x being greater than the dead zone +X,. Note that any change in x from the equilibrium position is a measure of the error in the terminal voltage magnitude This control action is designated the “raise-lower mode” of operation It results in a slow excitation change, responding to a change in ¥, large enough to exceed the threshold K,, where the theostat motor steadily changes the theostat setting. A block diagram of this control action is shown in Figure 7.42 The balanced beam responds to an accelerating force Fy = Klip +2) — Fo = ME + BY + Kx (744) where x is the reference position; { is the unstressed length of the spring; F, is the plunger force; and M, B, and K are the mass, damping, and spring constants respec- tively If the beam mass is negligible, the right side of (7.44) can be simplified In operation the beam position x is changed continuously in response to variations Fig 741 Ry versus for the condition | x] > K, >0 270 Fig 742 Block diagram of the raise-lower control mode in V, Any change in V, large enough to cause |x| 2 XK, results in the rheostat motor changing the setting of Ry. As the theostat is reset, the position x returns to the thres- hold region |x| < K, and the motor stops, leaving Ry, at the value finally reached At any instant the total resistance R is given by R = Ron + Ry = Ron + Ro — Kyt (raise) = Rog + Ro + Kyt (ower) (7.45) Thus the exact R depends on the integration time and on the direction of rotation of the theostat motor. In (7.45) and Figure 7.41, Ry is the value of Ry retained following the last integration, ‘This value is constrained by the physical size of the theostat so that for any time t, Raw < (Ry Kyt) < Row The foregoing discussion pertains to the raise-tower mode only. Referring again (0 Figure 7.40, a second possible mode of operation is recognized, If the x deflection is, large enough to make the QL or QR contacts, the fixed field resistors Rg, or Rog are switched into or out of the field respectively, initiating a quick response in the exciter This control scheme is shown in Figure 7.43 as an added quick control mode to the original controller The quick raise-lower mode is initiated whenever |x| > Ky, with the resulting action described by Retuen leer TERS” gaat % Fig. 743 Block diagram of the combined raise-Jower and quick-raise-lower control modes Excitation Systems 271 R= Ry x > K, (quick raise) = Ry + Ro x < K, (quick lower) (746) If we set K, > K,, this control mode will be initiated only for large changes in V, and will provide a fast response Thus, although the raise-lower mode will also be op- erational when |x| > K,,it will probably not have time to move appreciably before x returns to the deadband The controller of Figure 7.43 operates to adjust the total field resistance R to the desired value. Mathematically, we can describe the complete contro! action by com- ining (7 45)-(7.46) The resulting change in R affects the solution for vy in the exciter equation (7.43). If saturation is added, a more realistic solution results Saturation is often treated as shown in Figure 7 44, where we define the saturation function vgs volts Exciter Field Vollope, tek ict Fold Cote, f) amperes Fig 744 Exciter saturation curve Sem (a - dle ar Then we can show that Te= (+ Selly, Ey = (1 + Se) Eg (7.48) The function Sy is nonlinear and can be approximated by any convenient nonlinear function throughout the operating range (See Appendix D). If the air gap line has slope 1/G, we can write the total (saturated) current as i = Gol + S,) = Go, + GupSe (7.49) Substituting (7,49) into (7.43) the exciter equation is tebp = 0, ~ Ri = U, — RGU, ~ RGYSe (7.50) A block diagram for use in computer simulation of this equation is shown in Fig- ure 7.45, where the exciter voltage is converted to the normalized exciter voltage E» The complete excitation system is the combination of Figures 7.43 and 7.45 7.7.2 Continuously regulated systems Usually it is preferable for a control system to be a continuously acting, propor- tional system, i ¢ , the control signal is always present and exerts an effort proportional 272 Chapter 7 Fig 7.45. Exciter block diagram to the system error (see Def 2.12.1) Most of the excitation control systems in use today are of this type. Here we shall analyze one system, the familiar boost-buck system, since it is typical of this kind of excitation system Consider the system shown in Figure 7 10 where the feedback signal is applied to the rotating amplifier in the exciter field circuit, Reduced to its fundamental compo- nents, this is shown in Figure 7.46, We analyze each block separately Potential transformer and rectifier. One possible connection for this block is that shown in Figure 7.47, where the potential transformer secondaries are connected to bridged rectifiers connected in series Thus the output voltage ¥,, is proportional to the sum or average of the rms values of the three phase voltages. If we let the average rms voltage be represented by the symbol ¥,, we may write Va = Ka¥iJQ + 795) asp where Kg is a proportionality constant and rp is the time constant due to the filtering or first-order smoothing in the transformer-rectifier assembly The actual delay in this system is small, and we may assume that 0 < tT, < 006 s Voltage regulator and reference (comparator). The second block compares the volt- age Vj, against a fixed reference and supplies an output voltage ¥,, called the er- ror voltage, which is proportional to the difference; ie, Ve = KVace ~ Vue) (7.52) This can be accomplished in several ways. One way is to provide an electronic diffe ‘ence amplifier as shown in Figure 7 48, where the time constant of the electronic amp! fier is usually negligible compared to other time delays in the system, There is often an objection, however, to using active circuits containing vacuum tubes, transistors, and the associated electronic power supplies because of reliability and the need for replace- ao Ee Reg bee g amp Fig 746 Simplified diagram of a boost-buck system Excitation Systems 273 ae | ae Fig 747 Potential transformer and rectifier connection ment of aging components. This difficulty could be overcome by having a spare ampli- fier with automatic switching upon the detection of faulty operation Another solution to the problem is to make the error comparison by an entirely passive network such as the nonlinear bridge circuit in Figure 7.49 Here the input current iz, sees parailel paths i, and i, of i, = i, + i, But since the output is con- nected to an amplifier, we assume that the voltage gain is large and that the input cur- rent is negligible, or i, = 0 Under this condition the currents i, and i, are equal Then the output voltage V; is V, = vy - y (7.53) The operation of the bridge is better understood by examination of Figure 7.50 where the v-i characteristics of each resistance are given and the characteristic for the total resistance R, + Ry seen by i, and i, is also given. Since j, = i,, the sum of volt- age drops v, and vy is always equal to V,, the applied voltage. If we choose the non- linear elements carefully, the operation in the neighborhood of Veer is essentially lin- car; i¢., @ deviation v, above or below Veep results in a change ig in the total current, which is also displaced equally above and below igge. Note that the nonlinear resistance shown is quite linear in this critical region. Thus we may write for a voltage devia- tion vs, Uy = Oe + kyy OU, + Aye (7 54) where k, > ky Combining (7 54) and (7 53), we compute Vi = <(ky — ky)Uy = —k vg (1.55) But for a deviation Uy, Vi = Veer + Us, Which may be incorporated into (7.55) to write Ye = kee — Vac) (7:56) We note that (7 56) has the same block diagram representation as the difference ampli- fier shown in Figure 7.48(b), where we set 7 = O for the passive circuit v, Fig 748 Electronic difference amplifier as a comparator: (a) circuit connection, (b) block diagram 274 Chapter 7 Fig 749 Nonlinear bridge comparison circuit, ‘A natural question to ask at this point is, What circuit clement constitutes the volt- age reference? Note that no external reference voltage is applied A closer study of Figure 7.50 will reveal that the linear resistance R, is a convenient reference and that two identical gang-operated potentiometers in the bridge circuit would provide a con- venient means of setting the reference voltage. The nonlinear bridge circuit has the obvious advantage of being simple and entirely passive. If nonlinear resistances of appropriate curvature are readily available, this circuit makes an inexpensive comparator that should have long life without component aging The amplifier. The amplifier portion of the excitation system may be a rotating amplifier, a magnetic amplifier, or conceivably an electronic amplifier In any case we will assume linear voltage amplification K,, with time constant 74, or Va = K,V,/(L + 745) (7.57) As with any amplifier a saturation value must be specified, such a Vewin < Ve < Vem These conditions are both shown in the block diagram of Figure 7 51 The exciter. The exciter output voltage is a function of the regulator voltage as derived in (7.48) and with block diagram representation as shown in Figure 7.44 The major difference between that case and this is in the definition of the constant K,. Since the exciter is a boost-buck system, we can write the normalized equation Exp = Va ~ ExoSe) Ke + ToS) (7.38) where Ke = RG-1 (7.59) The generator, The generator voltage response to a change in v, was examined in ad Fig 750 The v versus / characteristics for the nonlinear bridge Excitation Systems 275 % em Ye wee PP) etn Fig 751 Block diagram of the regulator amplifier Chapter 5 Looking at the problem heuristically, we would expect the generator to respond nearly asa linear amplifier with time constant 7) when unloaded and 7) when shorted, with the actual time constant being load dependent and between these two ex- tremes, Let us designate this value as 7g and the gain as Kg to write, neglecting saturation, KeEroll + tS) (7.60) In the region where linear operation may be assumed, there is no need to consider saturation of the generator since its output is not undergoing large changes. If satura- tion must be included, it could be done by employing the same technique as used for the exciter, where a saturation function 5, would be defined as in Figure 7.44 Example 77 1 Construct the block diagram of the system described in Section 7.7.1 and compute the system transfer function 2. Find the open-loop transfer function for the case where Ol r= 10 Ky = -005 tT, = 005 K, = 40 Ky = 10 3. Sketch a root locus for this system and discuss the problem of making the system stable, Solution } The block diagram for the system is shown in Figure 752. If we designate the feed-forward gain and transfer function as KG and the feedback transfer function as H, the system transfer function is [23] ¥./ Fase = KGG)/UI + KG(S) HO) where, neglecting saturation and limiting, we have Ferg EST] rg , TF Fig. 752 Block diagram of the excitat control system 276 Chapter 7 Ke KG) = Ca + eb a) or Ky Kell + Te8) fer (1 + t4S)(Ke + T98)(1 + r6S)(1 + 125) + KiKoKe and the system is observed to be fourth order Solution 2 The open-loop transfer function is KGH, or KKK; OF aK + tes) + ral + es) Using the values specified and setting K = 400K,K,Kg, we have K (+ 105 - OG + 1) + 20) (amp) (exc) (gen) (ree) KGH KGH Solution 3 Using the open-loop transfer function computed in Solution 2, we have the root- locus plot shown in Figure 7 53, where we compute [22] K=3.21 ot erotsing K=0.05 ot origin Fig 7.53 Root locus for the system of Figure 7.52 (1) Center of gravity = (LP — LZ)M#P - #Z) = -(09 - 00)/4 = -7.75 (2) Breakaway points (by trial and error): left breakaway at 164: 1/36 = 1/64 + 1/154 + 1/165 0.278 = 0.281 right breakaway at -0.43: 1/19 57 + 1/9.57 + 1/0.57 = 1/053 191 189 (3) Gain at jo axis crossing: From the closed-loop transfer function we compute the characteristic equation G(s) = 8 + 3098 4 22695 + 1775 + Kt where K' = 400K - 20 and K = K,K,Kg = 40Kq Excitotion Systems 277 Then by Routh’s criterion we have sat 2269 -K* 8 | 309 7 | 2212 x si] 177 - 014K" 0 e | K For the first column we have: From row s° K'= 400K ~20>0 Kk >005 From row s! K' = 400K — 20 < (177/014) = 1266 K < 321 We may also compute the point of jw axis crossing from the auxiliary poly- nomial in s? with K’ = 1266, o1 224 1266=0 = 573 se 4j24 An examination of the root locus reveals several important system characteristics We note that for any reasonable gain the roots due to the regulator and amplifier excite response modes that die out very fast and will probably be overdamped Thus the response is governed largely by the generator and exciter poles that are very close to the origin. Even modest values of gain are likely to excite unstable modes in the solution. This can be improved by (a) moving the exciter pole into the left half of the s plane, which requires that R in (7.59) have a greater value; (b) moving the generator pole to the left, which would need to be done as part of the generator design rather than afterwards; and (c) adding some kind of compensation that will bend the locus to a more favorable shape in the neighborhood of the je: axis Of these options only (c) is of practical interest Excitation system compensation. Example 77 illustrates the need for compensa- tion in the excitation control system This can take many forms but usually involves some sort of rate or derivative feedback and lead or lead-lag compensation (It is Fig 754 Block diagram of a typical compensated system 278 Chapter 7 interesting to note that Gabriel Kron recognized the need for this kind of compensa- nas early as 1954 when he patented an excitation system incorporating these fea- tures [37].) This can be accomplished by adding the rate feedback loop shown in Figure 7 $4, where time constant 7, and gain K, are introduced. Such a compensation scheme can be adapted to bend the root locus near the jw axis crossing to improve stability substantially, Also notice that provision is made for the introduction of other compensating signals if they should be necessary or desirable. The effect of compensa tion will be demonstrated by an example Example 78 1. Repeat Example 7 7 for the system shown in Figure 7 54. 2. Use a digital computer solution to obtain the “best” values for r, and K; to mit mize the rise time and settling time with minimum overshoot 3. Repeat part 2 using an analog computer solution Yee X; 5 ro [Re oT Tg a THe % | FF @ Veer %] Ye 7 fe | Fe | rg Oe Tg p RF age te Pa & ~— re ® yi Kak, ‘ . =) owe ares ~ TAR TET Fg Kelling) Ky ort be Keira Te © Fig 755 Excitation system with rate feedback neglecting Sy and limiter: (a) original Block diagram, (b) with rate feedback take-off point moved to ¥, (2) with combined feedback Excitation Systems 279 Solution I The system transfer function can be easily computed for S; = 0 and with limiting ignored. Figure 7.55(a) shows a block diagram of the system with $, = 0 and without the limiter By using block diagram reduction, the takeoff point for the rate feedback signal is moved to V,, as shown in Figure 7.55(b), then the two feedback signals are combined in Figure 7.55(c). The forward loop has a transfer function KG(s) given by KK i TateTe (6 + A/T MS + Ke/te)S + 1/6) and the feedback transfer function H() is given by (Krte/Kate)s(s + 1/to)(s + 1/7) + (Ke/te is + 1/te) (s+ V/tes + 1/te) The open loop transfer function is thus given by K,Ky 88 + Afro) + Afte) + KaKote/tetoKs MS + Ure) aTeTs (8 + U]ta MS + Kel tas + 1/to(s + Vt) + 1] e) Substituting the values ry = 0.1, 7, = 05, ty = 0.05, 7 = 10Kz = -005, Kz = 10, and Kx = 10, KG(s) = H(s) = KGH = te (5 + 10s — OLS + I)(s + 1/t,)(s + 20) A given 7; fixes all poles of (7.61). Then the shape of the locus depends on the loca- tion of the zeros, Thus we examine the zeros of (7 61), From the numerator we write s(5 + Is + 20) + 20(7,/K Ms + 1/zp) = 0 KGH = 20K, (761) 2(ro/ Kis + U/9) Keea One eee S29 7 | tia H+ BD) 7) where we let K = 20(r,/K,) anda = 1/1, The locus of the roots of (7.62), which gives the zeros of (7.61), depends upon the value of a= I/r,. There are three cases of interest (note that a > 0): Case I, 0 20 These cases are shown in Fig- ure 7.56 where —mt is the location of the asymptote Case I is sketched in Figure 7 56(a), where a zero falls on the negative real axis at —a, which is between the origin and —1. The /ocus therefore falls between the origin and ~a. This means that (7 61) would have a zero on the real axis near the origin Thus the open loop transfer function of (7.61) will have a pole at 0.1 and a zero on the real axis at -a The locus of the roots for this system will have a branch on the real 2 I Cove Is 1< 0220 ose IN © > 20 W5emenI0 = Weme-08 70.5< Fig 756 Locus of zeros for the open loop transfer function of (7 62) 280 Chapter 7 s | | XO — 0% —— x x—1 x} x 2 SURG IES i 2 ' Com 1 core 18 | I ' x mo-— rex ly x are 20 comet ose t — xno ee he ve i oe tt 8 (Ke freNsls + NG + 20) + 2068 + ad] (5 + 208 + 10V(5 + 1s 0 116 +4) Fig 757. Root loci of KGH = 20K axis near the origin, and the system dynamic performance will be dominated by this root Its dynamic response will be sluggish, Cases II and III are shown in Figures 7.56(b) and (c). In both cases, the root-locus plots of (7 62) have branches that, with the proper choice of the ratio K, give a pair of complex roots near the imaginary axis Again, these are the zeros for the system described by (761). However, in Case II the loci approach the asymptotes to the left of the imaginary axis, while for Case II] the loci approach the asymptotes to the right of the origin The position of the roots of (7.62) and hence the zeros of (7.61), are more likely to be located further to the left of the imaginary axis in Case II than in Case II} A further examination of the possible loci of zeros in Figure 7 56 reveals that for the three zeros, two may appear as a complex pair. Thus there are two situations of interest: (A) all zeros real and (B) one real zero and a complex pair of zeros. Further- more, both conditions can appear in all cases Figure 7.57 provides a pictorial summary of all six possibilities. In all but two cases the system response is dominated by 4 root very near the origin, Only in Cases IIB and IIIB is there any hope of pulling this domi- nant root away from the origin; and of these two, Case ITB is clearly the better choice Thus we will concentrate on Case IIB for further study (Also see (38] for a further study of this subject ) From (7 61) the open loop transfer function is given by _ Ky) + Dis? + 2001 + ty/Ky)s + 20/Ke KOH = 208. SG ONE TNE IMG + Ty ~~} where 1 < I /r, < 20 Excitation Systems 281 he &, 1 . i é 5 Jae ae ae 0.00 0.00, 160 zo Bi nl 2 2 ten 96. K-00 By oe meen 15 $1.20 3 & ge Boe 3 : Fo i ° cep pte df ood % ta ve ia ve ae Sin a fia ; 3 , Foal £ Sea tt eae ae Fig 7.58(2) Elect of variation of Ky on dynamic response: 77 = 0.6, X¢ = 001,002, and 0.03 respectively Type | excitation system Solution 2 The above system is studied for different values of 1, and Ky with the aid of special digital computer programs. The programs used are 2 root-finding subroutine for poly- nomials to obtain the zeros of equation (7 63), a root-locus program, and a time- response program. Two sample cuns to illustrate the effect of 7» and K; are shown in Figure 7 58. In Figure 7.58(a) 17 is held constant at 0.6 while Ky is varied between 0.01 and 003 Plots of the loci of the roots are shown for the three cases, along with the time- response for the “rated” value of K, The most obvious effect of reducing ; is to reduce the settling time In Figure 7.58(b), K; is held constant at 0.02 while 7, is varied between 0.5 and 0.7 The root-locus plots and the time-response for the system are repeated The effect of increasing 7; is to reduce the overshoot 282 Chapter 7 2 0.02 a 0.02 8 15; 3 F g $19 308) - s 5 304 Lae ; 2 oad Oe 2 ° . a 1570, R= 0.02 i §, : Som 2 ew 5 z ° Li, _ we oe we aw fest Ties, 02 00 0 1H 70 30 Tine + a Fig 758(b) Effect of variation of ron dynamic response: Ky = 0.02, re = 05, 0.6, and 07 respectively Type | excitation system From Figures 7 $8(a) and 7.58(b) we can see that the values of 7, and K; signifi- cantly influence the dynamic performance of the system There is, however, a variety of choices of K, and 7;, which gives a reasonably good dynamic response For this par- ticular system, 7; = 0.6 and K, = 002 seem to give the best results Solution 3 ‘An engineer with experience in s plane design may be able to guess a workable location for the zero and estimate the value of K; that will give satisfactory results. For most engineers, the analog computer can be a great help in speeding up the design procedure, and we shalll consider this technique as an alternate design procedure. From Figure 7.54 we write, with ¥, = 0, Veer -M= hh W- Kav Excitation Systems 283 Ve Ingle RCO Vg scale “ner” Ye + Re y Seale Fig 759 Analog computer diagram for a linear excitation system with derivative feedback or Vo = Var -W- (7.64) For the amplifier block of Figure 754 we have Vp = K,¥,/(l + #45), Which may be rearranged as Va = (L/S)(Ka/ta) Ve — fra) Val (7.68) Equation (7.64) may be represented on the analog computer by a summer and (7.65) by an integrator with feedback, All other blocks except the derivative feedback term are similar to (7.65). For the derivative feedback we have V; = sKEsp/(1 + 795), which can be rewritten as Ys = (Kel ta Epo — (U/ 765) % (7 66) Using (7.64)-(7.66), we may construct the analog computer diagram shown in Fig- ure 759. Then we may systematically move the zero from s = 0 to the left and check the response In each case both the forward loop gain and feedback gains may be optimized Table 7 7 shows the results of several typical runs of this kind In all cases Ky has been adjusted to unity, and other gains have been chosen to optimize V, in a qualita- tive sense The constants in these studies may be used to compute the cubic coefficients (7 62), and the equation may then be factored If the roots are known, a root locus Table7.7._ Summary of Analog Computer Studies for Example 7.8 1 Setting Percent 0-90% Run MM Timers overshoot sete, 1 175 0.16 50 135 92 037 2 150 0.16 50 105 80 030 3 125 016 50 105 228 025 4 100 016 50 205 420 0215 5 0.75 0.16 50 very tong 70.0 0.20 284 Chopier 7 Vise Lt Fig 7.60 Analog computer results for Example 7 8, Solution 2 may be plotted and a comparison made between this and the previous uncompensated solution The actual analog computer outputs for run 2 are shown in Figure 760. One- second timing pulses are shown on the chart The plot is made so that 20 such pulses correspond to Is of real time, This system is tuned to optimize the output ¥;, which responds with little overshoot and displays good damping. Note, however, that this re- quires excessive overshoot of Ey» and ¥q, which in physical systems would both be limited by saturation, Inclusion of saturation is a practical necessity, even in linear simulation Examples 7.7 and 78 are intended to give us some feeling for the derivative feed- back of Figure 754 A study of the eigenvalues of a synchronous machine indicates that a first-order approximation to the generator voltage response is only approximately true, Nevertheless, making this simplification helps us to concentrate on the character istics of the excitation system without becoming confused by the added complexity of the generator. Visualizing the root locus of the control is helpful and shows clearly how the compensated system can be operated at much greater gain while still holding a suitable damping ratio, These studies also suggest how further improvements could be realized by adding series compensation, but this is left as an exercise for the interested reader Excitotion Systems 285 7.8 State-Space Description of the Excitation System Refer again to the analog computer diagram of Figure 7 $9 By inspection we write the following equations (including saturation) ¥, % Yr Lew % (Kelta)h — (ita) K = (Ke/tp) Eso — (A /re) Vs = (Kilt ¥e — Of V a Va < Vann Ye > Vario = (ft) Ve - (Se + Kei telEpo Year + - WV a) Since Sg = S,(E;») is a nonlinear function of Esp we linearize at the operating point to write OS, GE gp lo where we define the coefficient S; to describe saturation in the vicinity of the initial operating point Suppose we arbitrarily assign a state to each integrator associated with the excita- tion, Arbitrarily, we set Sta = Ess * StE soa Xt ¥, 5) |h Xo Ve Xu Exo and rewrite (7 67) as ¥, x --t 0 0 0 hs \ . z 6 . kK, KAS; + Ke , ty o -L Ke _ RlSe + Kol) 0 Fn Tie ~ “1K, kK, * %| je) [4-7-2 ° sa] | Oa +H) 2 Hi K, Ewl | Su o 9 6k LS Ke) | 0 TE TE (7 68) In equation (7 68) the term (Kg/»)¥, is a function of the state variables From (4.46) or (6 69) Vi = (1/303 + v5) (7.69) where vy and v, are functions of the state variables: thus (7 69) is nonlinear if the system equations are linearized about a quiescent operating state, a linear relation be- tween the change in the terminal voltage ¥,, and the change in the d and q axis volt- 286 Chapter 7 ages ujy and b, is obtained. Such a relation is given in (6 69) and repeated here: Ya = UP) (2 Vas + FEU) = des + de (7.70 Ho * Y, The linear model is completed by substituting for vj, and v,, in terms of the state variables and from (6 20) and by setting b¢ = (W3%¢/Lao)Eeo 7.8.1 Simplified linear model A simplified linear model can be constructed based on the linear model discussed in Section 6.5 The linearized equations for the synchronous machine are given by (the A subscripts are dropped for convenience) B= ag be my Ti © Kd + KE; ny Vi = Kyo + KE, (1.73) From (771) Ey = -C/Kytig) Ej — (Ki/tin) 8 + i] t4o) Exo (my From the torque equation (6.73) and (7 72) = Taft) — (Ki) 8 — (Raft) Ey - (fr) @ (7.75) and from the definition of w, baw (7.16) The system is now described by (7 68) and (7.72)-(7.76) The state variables are x! = [E{o5¥, V4 Velo]. The driving functions are Vege and T,, assuming that ¥, in (7.68) is zero. The complete state-space description of the system is given by Cr Exo Et ~—bo og -& o 0 0 4 EE 0 . Kyte 79 0 é -& 6 -& 0 0 0 o e | fi o 1 0 0 0 0 ° é ° ye | ~ | Bike g Kiko o 0 0 K 0 Th Tr , KK Ky ia oo 9 ~~ Eb 0 | | heer Si + Ke Eva 0 0 0 0 9 2 -&th |Ip,) | 0 Excitation Systems 287 7.8.2 Complete linear mode! By using the linearized mode] for a synchronous machine connected to an infinite bus developed in Chapter 6, the excitation system equations are added to the system of (620) Before this is done, V, must be expressed in terms of the state variables, using (6.25) and (7.70) These are repeated here (with the A subscript omitted), Vy = dod + 900% Ug = —Ke0s(é) — a) + Reig + Ledy + wolelg + iyphww U_ = —Ksin (6 — a)6 + Rig + Lely ~ welds — theo (7.78) From (7.78) and using V3 Veg & Ke0s(i) - a) V3 Veg & —Ksin (Sy - @) we get By = — V3 (doM ego ~ GeV nao)) + (doRe — qoivol dia + (dyeole + WRedig + UoigoLe ~ doiankaJo + dolcie + Goleig (7.79) Substituting in the first equation in (7.68), Vy = —U fra) Vi ~ Kal t0) V3 (do Venn ~ GV eco) 5 + (Ka/t2) (to Re ~ Fool.) ia + (Kafte) dowel, + GoRedig + (Kai te) dsign ~ doin) Lew + (Ka/ta)doleta + (Kr/te) doLely The remaining equations in (7.68) will be unchanged The equations introduced by the exciter (for V, = 0) will thus become Vy — (Kel te) dol eis ~ (Ke/ta) GoLeiy = (Ka/t2) (doR, ~ Goroke) ia + (Ke/74) (dosiaLe + Q0Re) fy + (Kal t) Uoigo ~ dolse) Leto ~ (Kgl t8) V3 (dV can ~ Ga¥ oan) 5 - (te) Vs Vy = (te) Vs + (Kilts 40) Va = 1K (Si + Ke)/t47e] Exo By = (Kalra) Vi ~ (Kal ta) Va = (Ulta) Va + (Kala) Mase Exp = (Ife) Va ~ (Si + Ke)/tel Ero (7.80) This set of equations is incorporated in the set (6.20) to obtain the complete mathe- matical description The new A matrix for the system is given by A = —M~'K Note that in (7 80) the state variable for the field voltage is E,) and not vu, There- fore, the equation for the field current is adjusted accordingly In this equation the term v, is changed to (W/3r,/L 4p) Exo ‘The matrices M and K are thus given by the defining equation v ~ —Kx — Mi, where ip ip ty ig @ 8 YUN Ve Ex Mis given by 288 Chapter 7 ls et i, ig il be kM, kMy | ip] kM, Lp Mg} o fof KMp My Ly i ° ' Le kMe ig | kMy Le M=o ' 0 i 0 é : Ke | _ Ke Wl Rede, 0 0 | = TRagh, 4 ¥, 0 o 0} 0 0 wei 0 eo oF 0 0 Fy} 0 0 0: 0 0 0 o 0 ‘ ° ° ° o nr) ° ° ° rn) ° where Ky, = —(Kg/te)(doRe ~ QowoL,) Kyu = —(Ke/ta(GoR, + dowol.) Kye = —(KeltaM(oign ~ dotao) Le Ky = —(V3 Kal te)(GoV aan ~ 40M ogo) o ° oo a) o o 0 o 0 10 oot (7.81) Ke KS + Bo) e ™ (i Ste ke (782) Excitation Systems 289 Example 7.9 Expand Example 6 2 to include the excitation system using the mathematical de- scription of (7.80). Assume that the machine is operating initially at the load specified in Example 6 2. The excitation system parameters are given by t, = 001s = 377 pu rp = OSs = 1885 pu Ky = 10 Ky = -0.05 14 = 005s = 18.85 pu 1, = 0.715 = 269.55 pu K, = 40 K, = 0.04 Let the exciter saturation be represented by the nonlinear function Se = Asy exp(BexErp) = 0.0039 exp(1.555 Exp) Solution From the initial conditions Vy = — 1.148 ig = —1.59 VIVeg = —1397 Vp = 1675 i = 070 V3 Viigo = 1.025 Vo = 1.172 Expo = 2.529 dy = (1/3)(G.0/ V9) = ~(1/3)(1.148/1 172) = ~0 3264 9 = (1/3)(Xgo/ Yo) = (1/3){1.675/1 172) = 04762 The linear saturation coefficient at the initial operating point is 1. Se Si” oF, = 1555 [0.0039 exp (1555 x 2.529)] = 0.3095 +0 The exciter time constants should be given in pu time (radians) The new terms in the K matrix are (1.0/3 77)(-0 326 x 002 — 0.476 x 04) = 0.0523 —(1.0/3.77)(—0 326 x 0.4 + 0.476 x 0.02) = 0.0321 (1 0/3.77)(-~0,326 x 0.70 + 0.476 x 159)0.4 = -0.0561 Ky, = (10/3.77)(-0 326 x 1025 + 0476 x 1.397) = 00751 Ire = 0.265 Lt, = 0.0037 Ky [tare = 004/(269 5 x 188 5) = 0.787 x 10° KASE + Ke)/tere = 0787 x 10° x O19 = 0118 x 10-* Ky]tz = 40/1885 = 2.122 = Kioo I/t4 = 1/18.85 = 0.053 I/te = 00053 (Si + Ke)/re = 0.15 x 0.0053 = 0.000796 Vir; /okM, = V3(0.000742)/1 55 = -0.000829 The new K matrix is given by 290 Chapter 7 re ee ° 0 0 oom bo oo | © oo fo ° =0 08 o 0 oo to Ai Ais 1550 o 0 6 © 00s K =| “oo “ios7 —1087 | —4285 2370 o 8 0 oo 0056 007s gun) oon Vaso ° © of 0 o 00 fo om ox 10+ O18 x 10 © 0 fo 0 0 famam 00s 0 o ei o o 0 0 $8 0-003 aooes The new M matrix is given by My, = Kadol./t, = -0.0479 My = Kegol./t, = 00211 2.100 1550 1.550} 1550 1651 1550! 0 e ey 1.550 1550 1605; 0 0 0 0 0 The A matrix is given by 5 z 36062 04s UMD F-34872 25470-24446 pan 4980s 168871 amo stORE | 84545 76 43857 -s6017} 22024 WHE! S40 0 ssur 5000 26197 26097} 26054 soon! TG? =35057 29975-28075! 35218-12532! ae 02011} -07993, we o 0 of 0 of ° 359436019 583014 13850 “35% 0S o 06 of 6 037099 nomen ocmn12 o o oy 0 2221-2122) 53.082 0 ° ° oto ° 53082-07958 Excitation Systems The eigenvalues obtained are A, = 0.0359 + 50.9983 A, = -0.0359 — j0.9983 Dy = 0.2653 dy = —0.0986 As = -0.1217 As = —0.0548, » 00015 + j0.0290 de = -0.0015 - j0 0290 dy = -0.0002 + j0 0064 Yio = ~0.0002 - j0.0064 Au = 0.0037 291 Example 710 Repeat Example 79 for different exciters. Use the same machine loading late the data used and the eigenvalues obtained Solution Tabu- For this example we will use the same machine loading of Example 5.1 and three exciters made by the same manufacturer: W TRA, W Brushless, and W Low rz Brush- less, Data for the exeiters and the appropriate M and K constants are given in Ta- ble 7.8. The eigenvalues obtained are tabulated in Table 7.9. Table 7.8, Exciter Data and Elements of Matrices M and K (Loading of Example 5.1) Constants and matrix elements WIRA Ky 400 % 005 Ke -017 te 09s Kr 004 . 10 Ke 10 Tr oor . dey 9.0027 Bex 1304 Sep 00874 Se 01140 Vou 35 fae 35 Ms 3 862069 Mes 4753316 Ku 4 9464 Kea 3 6244 Keo 6.5741 Ker 10.2754 Kus 26 5252 Koy 0.002653, Koo 0 000112, 0 000006 21 220159 0.053050 0.002792 0.000156 IEEE type | exciter, W Brushless 400 0.02 LO 080 0.03 10 10 oor 0.098 0553 0.4282 0.2368 73 -73 3.862069 -4.753316 4.9464 3.6244 ~6.5741 10.2754 26 5252 0.002653 —0 000099 0.000123 53,050398 0.132626 0.003316 0.004101 W low 7g Brushless 0.02 00+ 00761 0.4475 0.2510 0.1123 696 -6.96 3.862069 -4753316 4.9464 3.6248 ~65741 10.2754 26.5252 0.005305 -0.014147 0.015735 53050398 0.132626 0 176835 6.196693, Where 7g = 0.0 take ty ~ 10-* 292 Chapter 7 Table7.9. Eigenvalues for System of Example 7 10 (Loading of Example 5.1) Exciter type WIRA W Brushless W low rz Brushless 0.03594 + j0.99826 —_-0.03594 + j0.99826 0.03594 + j0 99827 0 03594 — 0.99826 0.03594 097876 0.03594 ~ jo 98827 ~0265 x 10 0265 x I ~0 26525 x 10? 0.09804 0.07300 —0 09763 0.12299 =0 12315 0.12302 =0.02536 + 0.03912 0.07870 + j0.02139 — —0 16664 + j0.86637 002536 — j0.03912 0.07870 — j0.02139 0 16664 — }0.86637 0.00076 + j0.02444 — 0.00071 + j0.02444 — 0.00082 + j0,02468 0.00076 — j002444 0.00071 - joO2444 0 00082 ~ j0.02468 ~0.00340 + 0.00249 — ~0 00447 ~000177 ~0.00340 — j0.00249 0.00447 0.00177 The results tabulated in Table 7.9 are for the same machine and loading condition as used in Example 6 3 except for the addition of the exciter models. Comparing the results of Examples 6 3 and 7 10, we note that two pairs of complex eigenvalues and two real eigenvalues are essentially present in all the results. We can conclude that these eigenvalues are identified with the parameters of the machine and are not de- pendent on the exciter parameters. The additional eigenvalues obtained in Exam- ple7 10 and not previously present are comparable in magnitude except for one com- plex pair associated with the W Low rp Brushless exciter For this exciter a frequency of approximately 50 Hz is obtained, which might be introduced by the extremely low exciter time constant The same example was repeated for the loading of Example 52 and for the same exciters. The results obtained indicate that only one pait of complex eigenvalues change with the machine loading This pair is one of the two complex pairs associated with the machine parameters. The eigenvalues associated with the exciter parameters did not change significantly with the machine loading. 7.9 Computer Representation of Excitation Systems Most of the problems in which the transient behavior of the excitation system is being studied will require the use of computers. It is therefore recognized that the solu- tion of systems can be greatly simplified if a standard set of mathematical models can be chosen Then each manufacturer can specify the constants for the model that will best represent his systems, and the data acquisition problem will be simplified for the user ‘As the use of computers has increased and programs have been developed that represent excitation systems, several models have evolved for such systems Actually, the differences in these representations was more in the form of the data than in the accuracy of the representation Recognizing this fact, the EEE formed a working group in the early 1960s to study standardization. This group, which presented its final report in 1967 [15], standardized the representation of excitation systems in four different types and identified specific commercial systems with each type. These models allow for several degrees of complexity, depending upon the available data or impor- tance of a particular exciter in a large system problem, Thus, anything from a very simple linear model to a more complex nonlinear model may be formulated by follow- ing these generalized descriptions We describe the four [EEE models below Excitation Systems 293 The excitation system models described use a pu system wherein 10 pu generator voltage is the rated generator voltage and 10 pu exciter voltage is that voltage re- quired to produce rated generator voltage on the generator air gap line (see Def 3.20 in Appendix E) This means that at no load and neglecting saturation, Ey) = 1.0 pu gives exactly ¥, = 10pu. Table 7.10 gives a list of symbols used in the four [EEE models, changed slightly to conform to the notation used throughout this chapter Table 7.10. Excitation System Model Symbols Symbol Description Symbol Description Egy ~ exciler output voltage 14 = regulator amplifier time constant Jy = generator field current 14 = exciter time constant ¥, = generator terminal voltage J, = generator terminal current Ty = regulator stabilizing circuit time constant K, = regulator gain Tews Tez = Sameas r¢ for rotating rectifier system Kz = exciter constant related to self- 1 = regulator input filter time constant excited field Kj = regulator stabilizing circuit gain Taq = theostat time constant, Type4 K, = current circuit gain in Type 3 Yq = regulator output voltage system Kp = potential circuit gain in Type 1S or Vennsx = Maximum value of Ve Type 3 system Ky = fast raise/lower constant setting, Vamin ~ minimum value of Vq Typed system Sg = oxciter saturation function Varp = regulator reference voltage setting V,_= auxiliary (stabilizing) input signal Fan = field rheostat setting Note: Voltages and currents are s domain quantities 7.9.1 Type 1 system—eontinuously acting regulator and exciter The block diagram for the Type 1 system is shown in Figure 761 Note that pro- vision is made for first-order smoothing or filtering of the terminal voltage V, with a filter time constant of rz Usually rg is very small and is often approximated as zero. aaa ci d ro Rte F Teg Fig 761 Type | excitation system representation for a continuously acting regulator and exciter (IEEE Reprinted from IEEE Trans vol PAS-87 1968 ) The amplifier has time constant x, and gain K,, and its output is limited by Vemes and Vai. Note that if we have no filter and the rate feedback is zero (K; = 0), the input to the rotating amplifier is the error voltage Yom Vase — % (783) 294 Chapter 7 Fig 762. Exciter saturation curves showing procedure for calculating the saturation function Sg. (@ IEEE Reprinted from IEEE Trans vol PAS-87. 1968.) and this voltage is small, but finite in the steady state. The exciter itself is represented asa first-order linear system with time constant 7~, However, a provision is made to include the effect of saturation in the exciter by the saturation function S, The satura- tion function is defined as shown in Figure 7.62 by the relation Sp = (A - BYB (784) and is thus a function of Ep that is nonlinear This alters the amplifier voltage Vq by an amount S,E;p to give a new effective value of Pq, viz , We = Va - SE sw (785) This altered value V, is operated upon linearly by the exciter transfer function, Note that for sufficiently small Ep the system is nearly linear (S, = 0) Note also that the exciter transfer function contains a constant Ky. This transfer function G(s) = 1/(Ke + 748) (7.86) is not in the usual form for a linear transfer function for a first-order system (usually stated as I/(1 + 75). From the block diagram we write Esp = P/(Ke + 125), and substituting (7.85) for V_ we have taSEgp = —KeE ep + Va ~ SeBso (787) which includes the nonlinear function 5;£;). Equation (7 87) corresponds in the time domain to teh = —KeEro + Ve — SeEs (7.88) ‘Comparing with (7 32), for example, where we computed tail = Up + Ug — BRV;/(a — Up) with the nonlinearity approximated by a Frohlich equation, we can observe the obvious similarity. Reference [15] suggests taking Excitation Systems 295 Kz = Selegpin = SEO) (7.89) which corresponds to the resistance in the exciter field circuit at ¢ = 0. Some engineers approximate the saturation function by an exponential function, ie, Se = f(Eso) = Aexexp(BexE so) (7.90) The coefficients 4gy and By, are computed from saturation data, where S, and Epp are specified at two points, usually the exciter ceiling voltage and 75% of ceiling The function (7.90) is easy to compute and provides a sitnple way to represent exciter satura~ tion with reasonable accuracy. See Appendix D Finally we examine the feedback transfer function of Figure 7.61 H(s) = Kys/(1 + 148) (791) where K; and 7, are respectively the gain constant and the time constant of the regu- lator stabilizing circuit This time constant introduces a zero on the negative real axis. Note that (7 91) introduces both a derivative feedback and a first-order lag Reference [15] points out that the regulator ceiling Vana and the exciter ceiling Expnm ate interrelated through S, and Kz Under steady-state conditions we compute Va = KeE ry + SpE (792) with the constraint Vani < Ve < Vamos then Vawas * (Ke + Sears) Eromax (7.93) Thus there exists a constraint between the maximum (or minimum) values Of Espig. and Vmax (Espnin 204 Vein) 7.9.2. Type 15 system—controlled rectifier system with terminal potential supply only This is a special case of continuously acting systems where excitation is obtained through rectification of the terminal voltage as in Figures 7 17 and 7 18 In this case the maximum regulator voltage is not a constant but is proportional to V,,ie. Vrmix = Ke¥, (7.94) Such systems have almost instantaneous response of their main excitation components such that in Figure 7.61 Kz = 1, 7, © 0, and S; = 0. This system is shown in Fig ure 7.63 A state-space representation of the Type IS system can be derived by referring to (7 67) (written for the Type 1 system), setting Ve = Esp and eliminating (7 65), with Fig 763. Type IS system (@ IEEE Reprinted from /EEE Trans vol PAS-87, 1968.) 296 Chapter 7 the result (Ke/ta) Ve — (fre) M1 = (Kilts) Exp ~ (Ute) % (Kalta) Me — (ra) Exo Exo < Vanave Ero > Venin Vaw + ¥- Kh (7.95) By using (7.79) and substituting for i, and i,, we can express ¥; as a function of the state variables. For the linearized system discussed in Chapter 6 where the state variables x! = ligipiniy ig @ 8) = [xy 2X X4X5 %6%)] wwe can show that A SpE» + fers (7.96) a where the f coefficients are constants, Rearranging, we write wl lay -¢ 0 5 |laf jt oo Lin Wl = la] = -#e}|a}+]o o & a 1 K. Ew he ~= | [te 0 0 tt Once +H) (797) where Vow fpr +2 fete Note that only three states are needed in this case 7.9.3 Type 2 system—rotating rectifier system Another type of system, the rotating rectifier system of Figure 7 13, incorporates damping loops that originate from the regulator output rather than from the excitation voltage [39] since, being brushless, the excitation voltage is not available to feed back. The /EEE description of this system is shown in Figure 7 64, where the damping feed- back loop is seen to be different from that of Figure 7.61 Note that two time constants appear in the damping loop of this new system, 7, and 7,,, one of which approximates etortion ometion | seer 1 gigi Fig 764 Type 2 excitation system representation—rotating rectifier system before 1967 (@ IEEE Re- printed from IEEE Trans , vol PAS:87, 1968 ) Excitotion Systems 297 the exciter time delay {39] and is considered “major damping,” with the second or “minor damping” being present to damp higher frequencies A state-space representation of this system may be derived from the following equations: x 1 (Kalra) ¥, ~ 1/9) Ki = (Kita) Ye = (Ufe9) Ve Exo = (U/t2) Ve ~ (Ke + Sp)/tel Ero =~ " KKy kK, TH -—Kr_y, _ Ga + tr) i, - K Tatars Tata Tete THte Ve = Vere +H - MW (7.98) Rearranging, we may write as a o ° o My jx} |ioe 0 6 An is ° ° 1 o ° ws) [000 0 0 ° i & ° m| [ooo ia ° ° ° mm) [9 00 0 0 ° (799) The Type 2 excitation system representation is intended for use in simulating the Westinghouse Brushless excitation system An alternate representation developed by the manufacturer is reported to represent the physical equipment more accurately. This revised Type 2 representation is shown in Figure 7 65 [40] Regilotor beter Veer servation Lag y femox Sse ee . 7 Pte Tey aay ve me ft min Bette) Le “Setrtion Weak tra ‘ercion Dowsing ores Fig 765. Revised Type 2 excitation system representation (Used with permission trom Stability Pro- gram Data Preparation Manual Advanced Systems Technology Rept 70-736, Westinghouse Electric Corp , Dec 1972) 7.9.4 Type 3 system—static with terminal potential and current supplies Some systems use a combination of current and voltage intelligence as a feedback signal to be compared against the reference, eg, the systems of Figures 7.15 and 7 16 These systems are not properly represented by Type | or 1S and requite special treat- ment, as shown in Figure 7.66 (The reactance x, is the commutating reactance of the transformer and is discussed in (41]) Here the regulator and input smoothing are similar to the Type | system However, the signal denoted V, incorporates information fed forward trom ¥, with added information concerning both J, and J, Thus 298 Chapter 7 ro Fig. 765 Type 3 excitation system representation-—static with terminal potential and current supplies (@IEEE Reprinted from /EEE Trans , vol PAS-87, 1968 ) Vc represents the self-excitation from the generator terminals Constants Kp and K, ate proportionality factors indicating the proportion of the “Thevenin voltage,” Van due to potential and current information, Multiplying Vy, is a signal propor- tional to f,, which accounts for variation of self-excitation with change in the angular relation of field current {J,) and self-excitation voltage (Vr) {15} Obviously, systems of this type are nonlinear To formulate a linearized state-space representation, we may write the self-excitation components as Ve = KV, + Kol, + Kyly (7.100) Then we write for the entire system Va = Ve + Ve Egy = Ve/(Kz + 128) Va = (Kf + ras Vy = KeEros/(1 + tS) Vim Ke V(t + 128) (7.101) But we may write the terminal voltage in the time domain as. 2 Ve = frEsm + D hk = SEs + % (7 102) t where for brevity we let v, be the term on the right. Also, for the terminal current we may write i = Mgig + Myiy = Myx, + Myx (7.103) If we define the states as in (7 68), we reduce (7.101)-(7.103) to the following form: nj fa) toe 0 Be Ws fe | af a) | 0 2 Be Beem alls | Ha, «ely ily, il | ba -& -& -2 ° wml" 4s eer + V0 al lao bet | | Ba Be Oy (7.104) Note that v,, i,, and #, are all linear functions of x4, Excitation Systems 299 Fig 767 Type 4 excitation system representation—noncontinuously setting regulator Note: gy limited between Venn and Vega time constant of rheostst travel = req 7.9.5 Type 4 system—nencontinuous acting The previous systems are similar in the sense that they are all continuous acting with relatively high gain and are usually fast acting. However, a great many systems are of an earlier design similar to the rheostatic system of Section 7.7 1 and are noncon- tinuous acting; ie , they have dead zones in which the system operates essentially open loop. In addition to this, they are generally characterized as slow due to friction and inertia of moving parts Type 4 systems (¢.g , Westinghouse BJ30 or General Electric GFA4 regulated sys- tems) often have two speeds of operation depending upon the magnitude of the voltage error, Thus a large-error voltage may cause several rheostat segments to be shorted out, while a small-error voltage will cause the segments to be shorted one at a time The computer representation of a system is illustrated in Figure 767, where Ky is the raise-lower contact setting, typically set at $%, that controls the fast-change mechanism onthe rheostat. If V, is below this limiting value of K,, the rheostat setting is changed by motor action with an integrating time constant of ray An “auctioneer” circuit sets the output Ve to the higher of the two input quantities Because the Type 4 system is so nonlinear, there is no advantage in representing it in state variable form The equations for the Type 4 system are similar to those derived for the electromechanical system of Section 77.1 A comparison of these two systems is recommended, 7.10 Typical System Constants Reference (15] gives, in addition to the system representations, a table of typical constants of physical systems. These data are given in Table 7 11 and, although typical, represent any physical system accurately, For any real system all quantities should be obtained from the manufacturer Also note that the values in Table 7 11 are for a system with a response ratio of 0.5 which, although common, is certainly not fast by today's standards. The RR of modern fast systems are often in the range of 2 0-35 Note that the values of Vener and Vang given in Table 7 11 are unity in column 1 and higher values in columns 2 and 3 This difference is due to the different choice of base voltage for V, by the different exciter manufacturers and does not necessarily imply any marked difference in the regulator ceilings or performance Changing the base voltage of Vz tO Venux affects all the other constants in the forward loop. There- 300 Chapter 7 Table 7.11. Typical Constants of Excitation Systems in Operation on 3600 r/min Steam Turbine Generators (excitation system voltage response ratio = 0.5) Selfsexcited exciters, Selfexcited Rotating rectifier commutator, oF silicon commutator exciter exciter with ‘symbol diode with amplidyne with Mag-A-Stat static voltage voltage regulators voltage regulator regulator a @ o) Te 0.0-0.06 0.0 0.0 Ky 25-50" 400 400 ™ 0.06-0.20 005 002 Vemax 10 33 73 Varin -10 -35 -13 Ke 001-008 O04 003 tr 035-10 10 Lo ip -005 -017 10 te 0s 095 0.80 Seimsx 0 267 095 0.86 Sersmas 0.074 0.22 0.50 “For generators with open circuit field time constants greater than 4s. fore, caution must be used in comparing gains, time constants, and limits for systems of different manufacture. ‘As experience has accumulated in excitation system modeling, the manufacturer and utility engineers have determined excitation system parameters for many existing units. Since these constants are specified on a normalized basis, they can often be used with reasonable confidence on other simulations where data is unavailable. Tables 7 12-7 15 give examples of excitation system parameters that can be used for estimating new systems or for cases where exact data is unavailable Since the formation of the National Electric Reliability Council (NERC) a set of de- Table 7.12. Westinghouse Excitation System Constants for System Studies (excitation system voltage response ratio = 0.5) Symbol Mag-A-Stat__Rotating-rectiier BJ30 Rototrol _Silverstat_ ‘TRA Excitation system type 1 1 404 1 1 7408) 00 00 0.05 0.02 0.08 K, 400 400 200 200 400 749) 005 0.02 . 025 o1 00 Expos (PY) 45 39 428 45 45 45 Erpain (PU)* 45 0 170 -45 03 02 Ke 017 10 10-017 -017,— 017 Kr 0.04 003 0105 0.028 0.028 7 (8) 10 10 125 os os Ky 0.05 TRH 20 3600 «/min 1800 «/min Vases (BU)* 35 13 82 83 35 35 35 Vena (BU)* 35 <1} -820 17-35 03 02 Seas 095 086 = 110-095 095 095 0.95 Sersmas 022 050 © 050 022 022 022 02 te(8) 0.95 08 1300.76 0.85 0.50 0.50 Source: Used with permission from Stability Program Data Preparation Manual Advanced Systems ‘Technology Rept 70-736, Westinghouse Electric Corp., Dec. 1972 *Values given assume of (full load) = 30 pu If not. multiply * values by vg/3 0 Excitation Systems 301 Table 7.13, Typical Excitation System Constants, Type of regulator te Ka ta ema —Varnin a Mag-A-Stat(Iype!) 0 400 O0St. «35-3 0.04 10 SCPT (Type3) 0 120 O15, 1212 O2frip typ 10.0 BY30 (Type 4) 200 005 0 83 18 0 0 Rototrol (Type 1) 0.05 200° 025° «35-35 0.084 12s Silverstat (Type 1) 0 200 010 35-005 0056 os TRA (Type 1) 0 400 00st 35-004 0056 045 GFA4 (Type 4) 00s 208 10 0 0 10 NAION (Type 1) . ‘Amplidyne 0.06 02 10-10 UNSxe/K_ 035 NA108 (Type 1) . ‘Amplidyee 0 02 10-10 Arg /Ry 10 NA143 (Type 1) Ample 2 saw * 02 10-10 Aref Ky 10 Naletapel) 9 #0081010 Bef 10 Bruen?) goog? 73 18am Brushes Type2) gga 0028282 Source: Used by permission from Power System Stability Program User's Guide Philadelphia Elec trie Co.. 1971 “Data obtained from curves supplied by manufacturer For typical values see Appendix D and Table ns tHigh-speed contact setting if knowa sign criteria has been established specifying the conditions under which power systems must be proven stable. This has caused an enlarged interest and concern in the accuracy of modeling all system components, particularly the generators, governors, exciters, and loads Thus it is becoming common for the manufacturer to specify the exeiter model to be used in system studies and to provide accurate gains and time constants for the system purchased Table 7.14. Typical Excitation System Constants Type of regulator Ke te Agy Bax Mag-A-Stat (Type 1) ~017 095 0.0039 1555 SCPT* (Type 3) 10 005 0 0 BJ30(Type 4) 10 076 0.0052 1555 Rototrol (Type 1) ~0.17 085 0.0039 1555 Silverstat (Type |) -017 0s 0.0039 41555 TRA (Type 1) ~017 05 0.0039 1855 GFAA(Type4) 005It 05 0.00105 1.465 Brushless (Type 2) 5 3600e7min 10 08 012 0855 Brushless (Type 2) 1800 r/min 10 13 0.059 u Sources Used by permiasion Wom Power System Stability Program Users Guide Philadelphia Elee ico” BT tet 19 : dy MVA base ky 19] esintcos"'F,) + Wefan ~ Fy] | ete MYA base] denerator MVA base Vemex = | 4Epoee ‘Highspeed contact sting. if known Chapter 7 ‘uoneamies roreroua8 ou pue sa1939 Ov yum “1's a]duexg 30 Fumpeo} yentUT “A4y pue My ui aseoxoul dows %ol JO Bsuodse1 sorwsoUDs Japowt fing god “FA co - I TONKO TONKO tTONTONKO 2 SOL 8 8 on OL = Fy U1 = 42 ‘00=4y “8Q=% ‘01 = Fy TO = M4 “oor = Fw (or|ysTAg asnou fn 44 ul oswasour aI 2g PUP MZ us osEaxoUr days YAO} 01 astOdsO1 1OWws0U98 fapOUI IIA] 69 CH 0 sud OW See = HOG cm = LG eg = “aunso 4) SoaweLEd IDK {"¢ afdwTErY Jo uIP Excitation Systems TONKO TMNKO TONTO N 304 Chopter 7 Table 7.15. Typical Excitation System Constants, fox Exciters with Amplidyne Voltage Regulators (NA 101, NAIO8, NA143) Exciter nominal = Ke t Ke Kan Bet Kamat ext Bext response 05 00445 0s 2075/3 2520 ty— 50S OOH. 465 1.0 0.0333 0.25 0zo/3_ 25 tO) = 5000058 1.06 1S 00280 0.1428 25ziq/13 25 AT p/3 50-0093 0.898 2.0 0.0171 0.0833 25ziq/22__25_——‘A0ra/3_—50——0.0108 0.79 Source: Used by permission from Power Sustem Stabili, Program User s Guide Philadelphia Elec- tric Co. 1971 *For all NAIOL, NA1O8 and NA 143 5 kW or less For NAL43 over 5 kW. 1See (7 99) 7.11 The Effect of Excitation on Generator Performance Using the models of excitation systems presented in this chapter and the full model of the generator developed in Chapters 4 and 5, we can construct a computer simula- tion of a generator with an excitation system The results of this simulation are inter- esting and instructive and demonstrate clearly the effect of excitation on system per- formance For the purpose of illustration, a Type 1 excitation system similar to Figure 761, has been added to the generator analog simulation of Figure 5.18. Appropriate switch- ing is arranged so the simulation can be operated with the exciter active or with con- stant Epp The results are shown in Figure 7 68 for constant E,p and Figure 7 69 with the exciter operative The exciter modeled for this illustration is similar to the Westing- house Brushless exciter Both Figures 7.68 and 7.69 show the response of the system to a 10% step increase in Ty, beginning with the full-load condition of Example 5.1 For the generator with no exciter, this torque increase causes a monotone decay in both A, and ¥, and an in- crease in 6 that will eventually cause the generator to pull out of step This increase in 5 is most clearly shown in the phase plane plot. Adding the excitation system, as shown in Figure 7.69, improves the system re- sponse dramatically Note that the exciter holds 4, and ¥, nearly constant when T,, is changed As a result, is increased to its new operating level in a damped oscillatory manner. The phase plane plot shows a stable focus at the new 3 Following the increase in torque the system is subjected to an increase in Erp. This is accomplished by switching the unregulated machine Ey from 100% to 110% of the Example 5.1 level In the regulated machine a 5% step increase in Vege is made The results are roughly the same with increases noted in \y and V,, and with a decrease in to just below the initial value We conclude that for the load change observed, the exciter has a stabilizing influ- ence due to its ability to hold the flux linkages and voltage nearly constant. This causes the change in 5 to be more stable In Chapter 8 we will consider further the effects of excitation on stability, both in the transient and dynamic modes of operation Problems 71 Consider the generator of Figure 7 2 as analyzed in Example 7 |. Repeat Example 7.1 but assume that the machine is located at a remote location so that the terminal voltage ¥; increases roughly in proportion to £, Assume, however, that the output power is held constant by the governor Excitation Systems 305 72 Consider the generator of Exemple7 I connected in parallel with an infinite bus and oper- ating with constant excitation By means of a phasor diagram analyze the change in 8, 1, and 6 when the governor setting is changed to increase the power output by 20% Note particularly the change in 6 in both direction and magnitude 73 Following the change described in Problem 7.2, what action would be required, and in what amount, to restore the power factor to its original value? 74 Repeat Example 7 | except that instead of increasing the excitation, decrease , to a mag- nitude less than that of V, Observe the new values of 6 and @ and, in particular, the change in 3 and @ 75 Comparing results of Example7 | and Problems 7 1-7 4, can you make any general state- ‘ment regarding the sensitivity of 6 and 6 to changes in Pand E,? 76 Establish a line of reasoning to show that a heavily cumulative compounded exciter is not desirable. Assume linear variations where necessary to establish your arguments 77 Consider the separately excited exciter shown in Figure P7.7. The initial current in the generator field is » when the exciter voltage U, = ko At time f = aa step function in the voltage U, is introduced; i¢, Us = ky + kyult — a) Fig P77 Compute the current ie Sketch this result for the cases where the time constant Lp frp is both very large and very small Plot the current function in the s plane. 78 Consider the exciter shown in Figure P7 8, where the main exciter M is excited by a pilot exciter P such that the relation up = k'w, ki) holds. What assumptions must be made for the above relation to be approximately valid? Compute the curtent & due to a step change in the pilot exciter voltage, i€. for Up = u(t) Fig P78 79 A solenoid is to be used as the sensing and amplification mechanism for a erude voltage regulator The system is shown in Figure P79 Discuss the operation of this device and comment on the feasibility of the proposed design. Write the differential equations that describe the system Feetton | [CEE Fig P79 306 Chapter 7 710. Anenciter for an ac generator, instead of being driven from the turbine-generator shaft, is driven by a separate motor with a large flywheel Consider the motor to have a constant output torque and write the equations for this system 711 Analyze the system given in Figure P7 11 to determine the effectiveness of the demping transformer in stabilizing the system to sudden changes, Write the equations for this sys- tem and show that, with parameters carefully selected, a degree of stabilization is achieved, particularly for large values of Rp. Assumeno load on the exciter e Fig P71 712. The separately excited exciter shown in Figure P7.12 has a magnetization curve as given in Table? 3 Other constants of interest are NV = 2500 bp = 125¥ 8 Min field winding 120 V (rated) . nna ol. k= 12,000 vy woe Fig P7102 (a) Determine the buildup curve beginning at rated voltage: ie, vp) = 120 What are the initial and final values of resistance in the field circuit? (>) What is the main exciter response ratio? 7.13 Given the same exciter of Problem 7.12, consider a self-excited connection with an ampli- dyne boost-buck regulation system that quickly goes to its saturation voltage of +100V following a command from the voltage regulator If this forcing voltage is held constant, compute the buildup Assume v;, = 40V,0,, » 180V 7.14 Assume that the constants 74, Te, To, Kes Kay and K, are the same as in Example 77 Lot zg take the values of 0001, 001, and 01. Find the effect of » on the branch of the root locus near the imaginary axis Excitation Systems 307 715. Repeat Problem 7 14 with r_ = 005 and for values of 14 = 0.05 and 0.2 716 Obtain the loci of the roots for the polynomial of (7 63) for r, = 0.3 and for values of Ry between 0.02 and 0 10 7.17 Obtain (or sketch) @ root-locus plot for the system of Example 78 for Ky = 0.05 and te 03 718 Complete the analog computer simulation of the system of one machine connected to an infinite bus (given in Chapter 5) by adding the simulation of the excitation system Use a ‘Type | exciter Also include the effect of saturation in the simulation 719. For the excitation system described in Example 7.9 and for the machine model and operat- ing conditions described in Example 6 6, obtain the A matrix of the system and find the eigenvalues 720 Repeat Problem 7 19 for the conditions of Example 6.7 721 Repeat Example 7 9 for the operating condition of Example 6 1 722 Repeat Example 7 9 (with the same operating condition) using a Type 2 excitation system Data for the excitation system is given in Table 7.11 7.23 Show how the choice of base voltage for the voltage regulator output Vg affects other constants in the forward loop. Assume the usual bases for V, and Exp References 1. Concordia C, and Temoshok, M Generator excitation systems and power system performance Paper 31 CP 61-536, presented at the IEEE Summer Power Mecting, Portland, Oreg., 1967. 2 Westinghouse Electric Corp Electrical Transmission and Distribution Reference Book Pittsburgh, Pa , 1950. 3 IEEE Committee Report Proposed excitation system definitions for synchronous machines JEEE Trans PAS-88:1248-58, 1969 4 Chambers, GS, Rubenstein, AS. and Temoshok M. Recent developments in amplidyne cegula- {or excitation sysiems for large generators AJEE Trans PAS-80:1066. 72, 1961 5 Alesanderson, E.F W., Edwards, M.A. and Bowman, KK. The amplidyne generator—A dyna. ‘moeleciric amplifier for power conirol General Elecric Rev 43:106-6, 1940 6 Bobo, PO. Carlson JT. and Horton. F Anew regulator and excitation system, EEE Tram PAS-721175-83, 1953 1. Bares. H.C, Oliver, J A. Rubenstein, A, S. and Temoshok M Ailernator-rectifier exciter for Cardinal Plant EEE Trans PAS-87-1189-98, 1968 & Whitney EC. Hoover D. Band Bobo. P.O Anclectsic utility brushless excitation system AEE Trans PAS-16:1821-24. 1959 9 Myers. E Hand Bobo, PO. Brushless excitation system Proc Southwess IEEE Conf (SWIEEECO) 1966 10 Myers, EH. Rotating rectifier exciters for large turbine-driven ac generators. Proc Am Power Conf., Vol 27. Chicago, 1965 11, Rubenstein AS. and’ Temoshok M_ Excitation systems—Designs and practices ia the United States Presented at Association des Ingénicurs Elecriciens de Mastitute Electrotechnique Montefiore, AIM , Liege, Belgium, 1966 12 Domeraizky, LM. Rubenstein, A. and Temoshok, M. A static excitation system for industrial and utility steam turbine generators AZEE Trans, PAS30:1072-77, 1961 1, Lane, LJ. Rogers, DF. and Vance, P A. Design and tests of a static excitation system for indus- tial and utility steam wrbine-generators. A7EE Trans PAS-80,1077-85, 1961 14 Lee, C Hand Keay, F, WA new excitation system and a method of analyzing voltage response IEEE im Conv Rec. 125-14, 1964 15 IEEE Committee Report Computer representation of excitation systems [EEE Trans PAS.87: 1460-64, 1968, 16 Kimbark, E.W_ Power Ssstem Stability Vol 3 Wiley New York 1986, 17 Cornelius, H. A, Cawson, W. F and Cory, H, W Experience with automatic voitage regulation fon a [1S.megawait(urbogeneretor A/EE Trams, PAS-T1:184-8) 1952 18 Dandeno P, L. and MeClymont K R_ Excitation system response: A utility viewpoint AEE Trans. PAS-16:1499-1501, 1957 19 Temoshok M.. and Rothe. F. S. Excitation voltage response definitions and significance in power systems ATEE Trans PAS-76:1491-96, 1957, 20 Rudenberg, R_ Transient Perfarmance of Electric Power Systems Phenomena th Lumiped Networks MoGraw-Hill New York. 1950 (MIT Press, Cambridge, Mass , 1967) 21 Takahashi J. Rabins M J. and Auslander, DM. Coniol and Dynamic Spstems Addison-Wesley, Reading. Mass 1970, 308 Chapter 7 22 Brown RG . and Nilsson, 1 W Introduction 10 Linear Systems Anaivsis Wiley, New York. 1962 23 Savant, C F Je Basic Feedback Conirol System Design McGraw-Hill, New York, 1958. 24 Hunter W A, and Temoshok, M_ Development of a modern amplidyne voltage regulator for large forbine generators AJEE Trans PAS-71:894-900, 1952 25 Porter, F M. and Kinghorn J H_ The development of modern excitation systems for synchronous condensers and generators A/EE Trans PAS-65:1070-27, 1946, 26 Concordia C, Effect of boost-buck voltage regulator on steady-state power limit 41EE Trans PAS- 69:380-84. 1950 27 McClure, JB. Whittlesley. $1, and Hartman, ME Modern excitation systems for large synchro- nous machines ATE Trans PAS-65.939-45, 1946, 28, General Electric Co. Amplidyne regulator excitation systems for large generators Bull GET-2980. 1966. 29 Harder, F 1, and Valentine CE Static voltage regulator for Rototrol excites Electr Eug 64: 601, 1945 30 Kalienback, GK , Rothe. FS, Storm, HF ,and Dandeno, PL. Performance of new magnetic amplifier type voltage regulator for large hydroelectric generators. AZEE Trans PAS-7|:201-6, 1952 SI Hand FW, McClure FN. Bobo. PO, and Casleron, J. T. Magamp regulator tests and operat ing experience on West Penn Power System AEE Trans PAS-73:486-91, 1954 32 Carleton, J.T, and Horton, W. F The figure of merit of magnetic amplifiers A/EE Trans PAS- 71:239-45, 1953. 33 Ogle HeM_ The ampli Oct. 1950 4 Hanna C_R, Oplinges, K A and Valentine C E Recent developments in generator voitage reg ulation. ALEE Trans 58:838-44, 1939 35 Dahl O GC Blecirte Power Clreults Theory and Application Vol 2 McGraw-Hill, New York 1938, 36, Kimbark E W Power System Stabiliny Vol 1 Elements of Stability Calculations Wiley. New York 1948, 37 Kron G Regulating system for dynamoelectric machines Patent No 2.692967. US Patent Office, 19s 38 Oyetunji, A A Effects of system nonlinearities on synchronous machine control Unpubl PhD. thesis Research Rept ERI-7/130. fowa State Univ Ames. 1971 39 Ferguson. RW, Herbst. R, and Miller. R W_ Analytical studies of the brushless excitation sys- lem. AIEE Trans PAS-78:1813-21, 1959 49 Westinghouse Electric Corp Stability program data preparation manual Advanced Systems Tech- nology Rept 70-736 1972 41 Lane, LJ, Mendel, JE. Ewart DN, Crenshaw. ML, and Todd J. MA statie excitation sys- tem for steam turbine generators Paper CP 65-208, presented at the IEEE Winter Power Mecting New York 1965 42 Philadelphia Electrie Co Power system stability program Power System Planning Div. Users Guide U6008.2, 1971 and its applications General Electric Rev Pt 1, Feb; Pt 2, Aug: Pt 3 chopter 8 The Effect of Excitation on Stability 8.1. Introduction Considerable attention has been given in the literature to the excitation system and its role in improving power system stability. Early investigators realized that the so- called “steady-state” power limits of power networks could be increased by using the then available high-gain continuous-acting voltage regulators (I). It was also recognized that the voltage regulator gain requirement was different at no-load conditions from that needed for good performance under load. In the early 1950s engineers became aware of the instabilities introduced by the (then) modern voltage regulators, and stabi- lizing feedback circuits came into common use [2]. In the 1960s large interconnected systems experienced growing oscillations that disrupted parallel operation of large sys- tems [3-12], It was discovered that the inherently weak natural damping of large and weakly coupled systems was the main cause and that situations of negative damping were further aggravated by the regulator gain (13) Engineers learned that the system damping could be enhanced by artificial signals introduced through the excitation sys- tem. This scheme has been very successful in combating growing oscillation problems experienced in the power systems of North America The success of excitation control in improving power system dynamic performance in certain situations has lead to greater expectations among power system engineers as to the capability of such control Because of the small effective time constants in the excitation system control loop, it was assumed that a large control effort could be expended through excitation control with a relatively small input of control energy While basically sound, this control is limited in its effectiveness. A part of the engi- neer’s job, then, is to determine this limit, ie., to find the exciter design and control parameters that can provide good performance at reasonable cost [14] The subject of excitation control is further complicated by a conflict in control requirements in the period following the initiation of a transient, In the first few cycles these requirements may be significantly different from those needed over a few seconds Furthermore, it has been shown that the best control effort in the shorter period may tend to cause instability later, This suggests the separation of the excitation control studies into two distinct problems, the zransient (short-term) problem and the dynamic (long-term) problem It should be noted that this terminology is not universally used Some authors call the dynamic stability problem by the ambiguous name of “steady- state stability” Other variations are found in the literature, but usually the two prob- lems are treated separately as noted 310 Chapter 8 8.1.1. Transient stability and dynamic stability considerations In transient stability the machine is subjected to a large impact, usually a fault, which is maintained for a short time and causes a significant reduction in the machine terminal voltage and the ability to transfer synchronizing power If we consider the ‘one machine-infinite bus problem, the usual approximation for the power transfer is given by P= (VVo/x) sind @1) where V, is the machine terminal voltage and V. is the infinite bus voltage. Note that if V, is reduced, P is reduced by a corresponding amount, Prevention of this reduction in P requires very fast action by the excitation system in forcing the field to ceiling and thereby holding V, at a reasonable value Indeed, the most beneficial attributes the voltage regulator can have for this situation is speed and a high ceiling voltage, thus improving the chances of holding V, at the needed level Also, when the fault is removed and the reactance x of (8 1) is increased due to switching, another fast change in excitation is required These violent changes affect the machine's ability to release the power it is receiving from the turbine These changes are effectively controlled by very fast excitation changes The dynamic stability problem is different {rom the transient problem in several ways, and the requirements on the excitation system are also different By dynamic stability we mean the ability of all machines in the system to adjust to small load changes or impacts. Consider a multimachine system feeding a constant load (a con- dition never met in practice) Let us assume that at a given instant the load is changed by a small amount, say by the energizing of a very large motor somewhere in the system, Assume further that this change in load is just large enough to be recog- nized as such by a certain group of machines we will call the control group The machines nearest the load electrically will see the largest change, and those farther away will experience smaller and smaller changes until the change is not perceptible at all beyond the boundary of the control group. Now how will this load change manifest itself at the several machines in the control group? Since it is a load increase, there is an immediate increase in the output power requirements from each of the machines Since step changes in power to turbines are not possible, this increased power requirement will come first from stored energy in the control group of machines. Thus energy stored in the magnetic field of the machines is released, then somewhat later, rotating energy [(1/2)mv*] is used to supply the load requirements until the governors have a chance to adjust the power input to the various generators, Let us examine the behavior of the machines in the time interval prior to the governor action. This interval may be on the order of 1s. In this time period the changes in machine voltages, currents, and speeds will be different for each machine in the control group because of differences in unit size, design, and elec- trical location with respect to the load Thus cach unit responds by contributing its share of the load increase, with its share being dictated by the impedance it sees at its terminals (its Thevenin impedance) and the size of the unit, Each unit has its own natural frequency of response and will oscillate for a time until damping forces can decay these oscillations Thus the one change in load, a step change, sets up all kinds of oscillatory responses and the system “rings” for a time with many frequencies present, these induced changes causing their own interaction with neighboring machines (see Section 3 6), Effect of Excitation on Stability 3n Now visualize the excitation system in this situation. In the older electromechani- cal systems there was a substantial deadband in the voltage regulator, and unless the generator was relatively close to the load change, the excitation of these machines would remain unchanged, The machines closer to the load change would recognize a need for increased excitation and this would be accomplished, although somewhat slowly ‘Newer excitation systems present a different kind of problem These systems recognize the change in load immediately, either as a perceptible change in terminal voltage, terminal current, ot both. Thus each oscillation of the unit causes the excitation sys- tem to try to correct accordingly, since as the speed voltage changes, the terminal voltage also changes, Moreover, the oscillating control group machines react with one another, and each action or reaction is accompanied by an excitation change. The excitation system has one major handicap to overcome in following these system oscillations; this is the effective time constant of the main exciter field which is on the order of a few seconds or so. Thus from the time of recognition of a desired excitation change until its partial fulfillment, there is an unavoidable delay. During this delay time the state of the oscillating system will change, causing a new excitation adjustment to be made. This system lag then is a dettiment to stable operation, and several investigators have shown examples wherein systems are less oscillatory with the voltage regulators turned off than with them operating [7, 12] Our approach to this problem must obviously depend upon the type of impact under consideration For the large impact, such as a fault, we are concerned with maximum forcing of the field, and we examine the response in building up from normal excitation to ceiling excitation This is a nonlinear problem, as we have seen, and the shape of the magnetization curve cannot be neglected The small impact or dynamic stability problem is different Here we are concerned with small excursions {tom nor- ‘mal operation, and linearization about this normal or “quiescent” point is possible and desirable Having done this, we may study the response using the tools of linear sys- tems analysis; in this way not only can we analyze but possibly compensate the system for better damping and perhaps faster response 8.2. Effect of Excitation on Generator Power Limits We begin with a simple example, the purpose of which is to show that the excitation system can have an effect upon stability Example 81 Consider the two-machine system of Figure 8 1, where we consider one machine against an infinite bus. (This problem was introduced and analyzed by Concordia [1] ) The power output of the machine is given by P= (EEX, + X2)|siné ith (82) Fig 81 One machine-infnite bus system 312 Chapter 8 & Ix, =T — | yer0 Se] bet E, Fig 82 Phasor diagram for Example 8 1 This equation applies whether or not there is a voltage regulator. Determine the effect of excitation on this equation Solution We now establish the boundary conditions for the problem. First we assume that X, = X; © 10 puand that ¥, = 10 pu, Then for any given load the voltages E, and E, must assume e certain value to hold V, at 1.0 pu. If the power factor is unity, E, and E, have the same magnitude as shown in the phasor diagram of Figure 82. If E, and E, are held constant at these values, the power transferred to the infinite bus varies sinusoidally according to (8,2) and has a maximum when 8 is 90° Now assume that E, and £, are both subject to perfect regulator action and that the key to this action is that V, is to be held at 1.0 pu and the power factor is to be held at unity. We write in phasor notation E,=\4+jlaVi+Fe®? Bele jle Vig Pew Adding these equations we have E, + Ey = 2 = 2V1 4 Peosi/2 n 0/2 erect regulator Anale 8, degrees Fig 83 Comparison of power wransferred at unity power factor with and without excitation control Effect of Excitation on Stability 313 or fi = 8, = (83) cos]? Substituting (8 3) into (8 2) and simplifying, we have for the perfect regulator. at unity power factor, P= tané/2 (84) The result is plotted in Figure 8.3 along with the same result for the case of constant (unregulated) E, and Ey In deriving (8 4), we have tacitly assumed that the regulators acting upon £, and E, do so instantaneously and continuously. The result is interesting for several reasons. First, we observe that with this ideal regulation there is no stability limit. Second, it is indicated that operation in the region where 8 > 90° is possible We should comment that the assumed physical system is not realizable since there is always a lag in the excitation response even if the voltage regulator is ideal Also, excitation control of the infinite bus voltage is not a practical consideration, as this remote bus is probably not infinite and may not be closely regulated Example 82 Consider the more practical problem of holding the voltage Z; constant at 10 pu and letting the power factor vary, other things being the same Solution Under this condition we have the phasor diagram of Figure 84 where we note that the locus of £, is the dashed circular are of length 1.0. Note that the power factor is constrained by the relation 12/2 (85) where B= 24 - Bandd = 3; - 3 Writing phasor equations for the voliages, we have &/ f “ Fig 8 4 Phasor diagram for Example 8 2 314 Chopter 8 3.0) \200 2.54 f2s0 2.9} 1200 1st hs s 1.0} joo § od |» Jo oN, ° * y vena ae Fig 85 System parameters asa function of 6: Ey E 14 jP = 1 - Isin@ + jfeos@ = Ee* 1 jl = 1 + Isin@ — jfeos6 ~ Eye (86) where #, 4,, 6, and 6, are all measured positive as counterclockwise Noting that Ez = I, we can establish that T= 2siné, £,siné = 2sind, sind = 2sind, tan 4, = sin 8; /(2 — cosds) (7) Thus once we establish 62, we also fix 4, /, 6, and 4,, although the relationships among these variables are nonlinear. These results are plotted in Figure 8.5 where equations (8 7) are used to determine the plotted values We also note that P = ViLcosd (88) but from the second of equations (8.6) we can establish that Jcos@ = sind; or P = sind 89) 80 6; also establishes P Thus P does have a maximum in this case, and this occurs when 6: = 90° (E; pointing straight down in Figure 8.4), In this case we have at maximum power BE, = 2+ jl = 2235/266° I= 1414 a= ~45° b= 1166" The important thing to note is that P is again limited, but we see that 5 may go Effect of Excitation on Stability 315 1a Power, pu e 9 Teo Tongue Angle 8, degrees Fig 86 Variation of Pwith 5 beyond 90° to achieve maximum power and that this requires over 2 pu E,. The variation of P with 6 is shown in Figure 8.6 These simple examples show the effect of excitation under certain ideal situations Obviously, these ideal conditions will not be realized in practice However, they provide limiting values of the effect of excitation on changing the effective system parameters A power system is nearly a constant voltage system and is made so because of system component design and close voltage control This means that the Thevenin impedance seen looking into the source is very small, Fast excitation helps keep this impedance small during disturbances and contributes to system stability by allowing the required transfer of power even during disturbances Finally, it should be stated that while the ability of exciters to accomplish this task is limited, other considerations make it undesirable to achieve perfect control and zero Thevenin impedance Among these is the fault-interr upting capability 8.3 Effect of the Excitation System on Transient Stability In the transient stability problem the performance of the power system when sub- jected to severe impacts is studied. The concern is whether the system is able to main- tain synchronism during and following these disturbances The period of interest is relatively short (at most a few seconds), with the first swing being of primary impor- tance In this period the generator is suddenly subjected to an appreciable change in its output power causing its rotor to accelerate (or decelerate) at a rate large enough to threaten loss of synchronism. The important factors influencing the outcome are the machine behavior and the power network dynamic relations For the sake of this dis- cussion it is assumed that the power supplied by the prime movers does not change in the petiod of interest. Therefore the effect of excitation control on this type of transient depends upon its ability to help the generator maintain its output power in the period of interest To place the problem in the proper perspective, we should review the main factors that affect the performance during severe transients These are: 1. The disturbing influence of the impact. This includes the type of disturbance, its location, and its duration 2. The ability of the transmission system to maintain strong synchronizing forces during the transient initiated by a disturbance 3. The turbine-generator parameters The above have traditionally been the main factors affecting the so-called first-swing transients. The system parameters influencing these factors are: 316 Chapter 8 1, The synchronous machine parameters Of these the most important are: (a) the inertia constant, (b) the direct axis transient reactance, (c) the direct axis open ci cuit time constant, and (4) the ability of the excitation system to hold the flux level of the synchronous machine and increase the output power during the transient. 2 The transmission system impedances under normal, fauked, and postfault condi- tions. Here the flexibility of switching out faulted sections is important so that large transfer admittances between synchronous machines are maintained when the fault is isolated 3. The protective relaying scheme and equipment The objective is to detect faults and isolate faulted sections of the transmission network very quickly with minimum. disruption. 8.3.1 The role of the excitation system in classical model studies In the classical model it is assumed that the flux linking the main field winding remains constant during the transient If the transient is initiated by a fault, the arma- ture reaction tends to decrease this flux linkage [15]. This is particularly true for the generators electrically close to the location of the fault’ The voltage regulator tends to force the excitation system to boost the flux level. Thus while the fault is on, the effect of the armature reaction and the action of the voltage regulator tend to counter- act each other These effects, along with the relatively long effective time constant of the main field winding, result in an almost constant flux linkage during the first swing of Tsor less (For the examples in Chapter 6 this time constant Kyrjp is about 2.0s.) Ttis important to recognize what the above reasoning implies. First, it implies the presence of a voltage regulator that tends to hold the flux linkage level constant, Sec- ond, it is significant to note that the armature reaction effects are patticularly pro- nounced during a fault since the reactive power output of the generator is large There~ fore the duration of the fault is important in determining whether a particular type of voltage regulator would be adequate to maintain constant flux linkage A study reported by Crary [2] and discussed by Young [15] illustrates the above. The system studied consists of one machine connected to a larger system through 2 200- mile double circuit transmission line The excitation system for the generator is Type 1 (see Chapter 7) with provision to change the parameters such that the response tatio (RR) varies fom 0.10 to 3.0 pu. The former corresponds to a nearly constant field voltage condition The latter would approximate the response of a modern fast excita- tion system. Data of the system used in the study are shown in Figure 8.7 A transient stability study was made for a three-phase fault near the generator The sending end power limits versus the fault clearing time are shown in Figure 8 8 for different exciter responses (curves 1-5) and for the classical model (curve 6) From Figure 8 8 it appears that the classical model corresponds to a very slow and weak excitation system for very short fault clearing times, while for longer clearing times it approximates a rather fast excitation system. If the nature of the stability study is such that the fault clearing time is large, as in “stuck breaker” studies [15], the actual power limits may be lower than those indicated when using the classical model In another study of excitation system zepresentation (16) the authors report (in a certain stability study they conducted) that a classical representation showed a certain generator to be stable, while detailed representation of the generator indicated that loss of synchronism resulted, The authors conclude that the dominant factor affecting loss Effect of Excitation on Stability 317 Exciter x » . "a0 =o Q & fag 4ST | stotiticer ties af, - %9) Generator: Reguiating system. x4=063 pu ag = 20 xy=942 pu wad x-021 pu r=047 s H=50 8 Eqye = 225 ou tip = 50 8 Ene ~ -030 pu *=010 pu Line: ‘System damping: x=08 O/mifline Fault 72012 2/mifline Fault on__ cleared 252% 10% mbo/mifine — 7 System: Th. 0. 3 ty = 0.2 pu Ta 0) 3 H=500's Tn 15 18 Fig 87 Two-machine system with 200-mite transmission lines of synchronism is the inability of the excitation system of that generator, with response ratio of 0.5, to offset the effects of armature reaction 8.3.2. Increased reliance on excitation control to improve stability Trends in the design of power system components have resulted in lower stability margins. Contributing to this trend are the following: 1. Increased rating of generating units with lower inertia constants and higher pu re- actances 2. Large interconnected system operating practices with increased dependence on the transmission system to carry greater loading These trends have led to the increased reliance on the use of excitation control as a 3 dud : Clearing a Curve Time RR q 1 oo s 30 Sof 2 07s 20 2 3 06% s 10 ons 4 20 5 038 oO ON Oe 0 5 Hos oto Feult Clearing Tie. 6 Classical model Fig 88 Sending-end power versus fault clearing time for different excitation system responses 318 Chapter 8 ¢| 8 A Yo 20 a0 Tino, = @) 1 i gE oo 10 20 30 40 50 Time, « © Fig 89 Results of excitation system studies on a western US. system: (a) Onecline diageam with fault lo- cation, () frequency deviation comparison for a fouéycle faull, () frequency deviation compari- son for a 9.6-cycle fault: 4 = 20 ANST conventional exitition system; B = low time constant ex- citation system with rate feedback; C’~ low time constant exciton system without rate feedback (@IEEE Reprinted from JEEE Trans. vol PAS-90. Sept /Oct: 1971) ‘means of improving stability [17] This has prompted significant technological ad- vances in excitation systems. As an aid to transient stability, the desirable excitation system characteristics are a fast speed of response and a high ceiling voltage With the help of fast transient forcing of excitation and the boost of intetnal machine flux, the electrical output of the machine may be increased during the first swing compared to the results obtainable with a slow exciter This reduces the accelerating power and results in improved transient performance Effect of Excitation on Stability 319 Modern excitation systems can be effective in two ways: in reducing the severity of machine swings when subjected to large impacts by reducing the magnitude of the first swing and by ensuring that the subsequent swings are smaller than the first The latter is an important consideration in present-day large interconnected power systems Situations may be encountered where various modes of oscillations reinforce each other during later swings, which along with the inherent weak system damping can cause transient instability after the first swing, With proper compensation a modern excita- tion system can be very effective in correcting this type of problem. However, except for transient stability studies involving faults with long clearing times (or stuck breakers), the effect of the excitation system on the severity of the first swing is rela- tively small. That is, a very fast, high-response excitation system will usually reduce the first swing by only a few degrees or will increase the generator transient stability power limit (for a given fault) by a few percent In a study reported by Perry et al [18] on part of the Pacific Gas and Electric Company system in northern California, the effect of the excitation system response on the system frequency deviation is studied when a three-phase fault occurs in the network {at the Diablo Canyon site on the Midway circuit adjacent to a S00-kV bus). Some of the results of that study are shown in Figure 8.9 A one-line diagram of the network is shown in Figure 8.9(a) The frequency deviations for 4-cycle and 9 6-cycle faults are shown in Figures 8 9(b) and 8 9(c) respectively. The comparison is made between 2.20 response ratio excitation system (curve 4), a modern, low time constant excita- tion with rate feedback (curve B) and without rate feedback (curve C), The results of this study support the points made above 8.3.3 Parametric study Two recent studies (17, 19] show the effect of the excitation system on “first-swing” transients Figure 8.10 shows the system studied where one machine is connected to an infinite bus through a transformer and a transmission network. The synchronous machine data is given in Table 8 | The transmission network has an equivalent transfer reactance X, as shown in Table 8.1, Machine Data for the Studies of Reference {19}, ays 172 pu te = 63s xy 2045 pu 7H = 0033 xf < 033 pu ty = 043 5 ¥y = 168 pu Tyo = 00338 xy = 0.59 pu H- 40s xg = 033 pu y, x y ria © jos 36 Fault Fig. § 10 System representation used in a parametric study of the effect of excitation on transient stability (IEEE Reprinted from /EEE Trans vol PAS-89, July/Aug. 1970) 320 Chapter 8 Figure 810 A transient is initiated by a three-phase fault on the of the transformer The fault is cleared in a specified time After the fault is cleared, the transfer reactance X, is increased from Xj, (the value before the fault) to Xz. (its value after the fault is cleared). The machine initial operating conditions are sum- marized in Table 8 2 Table82. Prefault Operating Conditions, All Values in pu te MPa P Q 02 10 094 0.90 039 04 10 090 090 045 06 10 oor 090 04a 08 10 097 090 O44 With the machine operating at approximately rated load and power factor, a three- phase fault is applied at the high-voltage side of the step-up transformer for a given length of time When the fault is cleared, the transmission system reactance is changed to the postfault reactance X,,, and the simulation is run until it can be determined if the run is stable or unstable This is repeated for different values of X,, until the maxi: mum value of X,, is found where the system is marginally stable. ‘Two different excitation system representations were used in the stirdy: 1. A0.5 pu response alternator-fed diode system shown in Figure 8 I 2 A 30 pu response alternator-fed SCR system with high initial response shown in Figure 8 12 This system has a steady-state gain of 200 pu and a transient gain of 20 pu. An external stabilizer using a signal V, derived from the shaft speed is also used (see Section 8 7) id Fig 8.L1 Excitation block diagram for a 05 RR alternator-fed diode system (@ IEEE Reprinted from IEEE Trans vol PAS-89, July/Aug, 1970) From the data presented in (19), the effect of excitation on the “first-swing” tran- sients is shown in Figure 8.13, where the critical clearing time is plotted against the transmission line reactance for the case where X= Xq and for the two different types of excitation system used. The critical clearing time is used as a measure of relative stability for the system under the impact of the given fault Figure 8 13 shows that for the conditions considered in this study a change in exciter response ratio from 0.5 to 30 resulted in a gain of approximately one cycle in critical clearing time Effect of Excitation on Stability 321 —— =} eH Fig 8 12 Excitation block diagram for a 30 RR alternator-fed SCR excitation system (© IEEE, Re- printed from IEEE Trans vol PAS-89.July/Aug 1970) 8.3.4 Reactive power demand during system emergencies A situation frequently encountered during system emergencies is a high reactive power demand, The capability of modern generators to meet this demand is reduced by the tendency toward the use of higher generator reactances. Modern exciters with high ceiling voltage improve the generator capability to meet this demand, It should be recognized that excitation systems are not usually designed for continuous operation at ceiling voltage and are usually limited to a few seconds of operation at that level Concordia and Brows [17] recommend that the reactive-power requirement during sys- tem emergencies should be determined for a time of from a few minutes to a quarter- or half-hour and that these requirements should be met by the proper selection of the generator rating 8.4 Effect of Excitation on Dynamic Stability Modern fast excitation systems are usually acknowledged to be beneficial to tran- sient stability following large impacts by driving the field to ceiling without delay However, these fast excitation changes ase not necessarily beneficial in damping the oscillations that follow the first swing, and they sometimes contribute growing oscilla~ tions several seconds after the occurrence of a large disturbance With proper design and compensation, however, a fast exciter can be an effective means of enhancing stability in the dynamic range as well as in the first few cycles after a disturbance Since dynamic stability involves the system response to small disturbances, analysis as a linear system is possible, using the linear generator model derived previously [11] For simplicity we analyze the problem of one machine connected to an infinite bus as ol Cleaning Time, eyeles 0 0s 08 at PE Fig 613 Transient stability studies resulting from studies of [19] A = 05 RR diode excitation system; B= 30 pu RR SCR excitation system (© IEEE. Reprinted from JEEE Trans vol. PAS-89, Huly/Aug 1970) 322 Chopter 8 through a transmission line. The synchronous machine equations, for small perturba- tions about a quiescent operating condition, are given by (the subscript A is omitted for convenience) T= Kio + KE; 10) Eg = [Ks [(l + Ksrdos Ero — (KsKa/(l + Ksrensi6 (8.11) V, = Keb + KeEj 8.12) 105 = Ty — T 8.13) where rio is the direct axis open circuit time constant and the constants K, through Ky depend on the system parameters and on the initial operating condition as defined in Chapter 6 In previous chapters it was pointed out that this model is a substantial improvement over the classical model since it accounts for the demagnetizing effects of the armature reaction through the change in £; due to change in 6 ‘We now add to the generator model a regulator-excitation system that is repre- sented as a first-order lag Thus the change in Ero is related to the change in ¥, (again the subscript 4 is dropped) by Em[¥, = -K.(\ + 13) (814) where Kis the regulator gain and z, is the exciter-regulator time constant 8.4.1 Examination of dynamic stability by Routh’s criterion To obtain the characteristic equation for the system described by (8 10)-(8 14), a procedure similar to that used in Section 3 5 is followed. First, we obtain alae, Keke 5 Ofte + KsK,/Ker)) 5 1) [me Tle #4 s(I/r, + 1/Kyrm) $1 + 0h (8.15) From (8.13) for Tq = 0, 85 = (68/1) Te = on /2HT, (8 16) By combining (8 15) and (8 16) and rearranging, the following cheracteristic equa- tion is obtained: tars Br tysta=0 (817) where a = 1/7, + 1/Ksriy 8 = [+ KsKoK)/Keriot,) + Kiln /2H) = Sfp Kilt + Ki/Kardy ~ KaKe/rio) . [su + KKoK) _ Kok + Kitlote ThoTe Applying Routh’s criterion to the above system, we establish the array Effect of Excitation on Stability 323 where a = (lfaah — y) = B - fa a, = (I1/a)(an ~ 0) = 9 _ U6 = v/a) = an B~ y/e bb =0 G=aan (8.18) by = Uaary — aa) According to Routh’s criterion for stability, the number of changes in sign in the first, column (I, @, 4), 61, and c;) corresponds to the number of roots of (8.17) with positive real parts, Therefore, for stability the terms a, a), b,, and c, must all be greater than zero. Thus the following conditions must be satisfied 1 @ = 1/7, + 1/Kyr¥o > 0, and since 7, and rip ate positive, Tie/t. > —1/Ry (8 19) K; is an impedance factor that is not likely to be negative unless there is an exces- sive series capacitance in the transmission network Even then rép/r, is usually large enough to satisfy the above criterion, 2a =B8-y/e>O L+ KsKeK, ss) — Kato. wn [pe (tot a) _ KoKs ( Korot, S38) ~ Kerw or 3H MR, |? ° or Hip? |_wnKoKsKer. 1 Keo ee ences +7) * Kiran | en) This inequality is easily satisfied for all values of constants normally encountered in power system operation. Note that negative K, is not considered feasible From (8.20) K, is limited to values greater than some negative number, a constraint that is always satisfied in the physical system « Bb=1- FATT >0 [« (¢ * 7) - Sk] (r+ Kitia)/Ks riot x (! + Asksk:) _ KiKa ( + ~4 TE RKR, , oe 1h0KiKe | '\ Ration. Ther. Kstiot. 2H Katw + 7 Rearranging, this expression may be written as enKiKaKy KiKi + KsKoK.) op K}K Kir, 2H tly Kyrdit. 2H tio(Katio + 7.) Kyrly + a) KKy ( Sk) + (Bete) Re) 4 BSR) 9 gar (Gees) Be be way We now recognize the first expression in parentheses in the last term of (8 21) to be the positive constant a defined in (8 17) Making this substitution and rearranging 324 Chopter 8 to isolate K, terms, we have elk, — fak.) 4s (ekg = ker = aks) K, 8 22) 2H (« are) 7 Kitare * @2) The expressions in parentheses are positive for any load condition Equation (8 22) places @ maximum value on the gain K, for stable operation 4qensd KAK Kg ~ K;Ks) > KK, Since K,, — K,K, > 0 for all physical situations, we have Ko > KaKi/(KiKy ~ KoKs) (8.23) This condition puts a lower limit on the value of K, Example 8.3 Fos the machine loading of Examples 5.1 and 52 and for the values of the con- stants K; through K, calculated in Examples 6 6 and 6 7, compute the limitations on the gain constant K,, using the inequality expressions developed above Do this for an ex- citer with time constant 7, = 05s Solution In Table § 3 the values of the constants K; through X, are given together with the maximum value of &, from (8.22) and the minimum value of K, from (8.23). The regu- lator time constant 1, used is 0.58, To = 598, and H = 2378. Case | is discussed in Examples 5 | and 6 6; Case 2, in Examples 5 2 and 6 5 From Table 8.3 itis apparent that the generator operating point plays a significant Table83. Computed Constants for the Linear Regulated Machine Constants Case 1(Ex.5.1) Case 2(Ex. 52) Ky 1076 1448 Ky 1.258 1317 K 0307 0.307 Ky 1712 1805, Ks ~0041 0.029 Ke 0.497 0.526, a 2552 2.552 KiKi Kat. 0331 0365 Kytjo + Fe 2313 2313 Kytiots 0.906 0.906 KK farin 0143 0158 RK O85! 0949 eK rig ~0616 0482 Kergot, 5.051 5325 te 4.000 4.000 Ko ~23 -32 Kes 269.0 1120.2 Effect of Excitation on Stability 325 role in system performance. The loading seems to influence the values of K, and Ks more than the other constants At heavier loads the values of these constants change such that in (8.22) the left side tends to decrease while the right side tends to increase This change is in the direction to lower the permissible maximum value of exciter regulator gain K, For the problem under study, the heavier load condition of Case | allows a lower limit for K, than that for the less severe Case 2 Routh’s criterion is a feasible tool to use to find the limits of stable operation in a physical system As shown in Example 8 3, the results are dependent upon both the sys- tem parameters and the initial operating point The analysis here has been simplified to omit the rate feedback loop that is normally an integral part of excitation systems. Rate feedback could be included in this analysis, but the resulting equations become compli- cated to the point that one is almost forced to find an alternate method of analysis Computer based methods are available to determine the behavior of such systems and are recommended {or the more complex cases (20, 21] ‘One special case of the foregoing analysis has been extensively studied [11]. This analysis assumes high regulator gain (K,K,K, >> 1) and low exciter time constant (7, © Kyrjg). In this special case certain simplifications are possible See Problem 8 4 8.4.2 Further considerations of the regular gain and time constant At no load the angle 6 is zero, and the 6 dependence of (8.10)-{8 23) does not apply. For this condition we can easily show that the machine terminal voltage ¥, is the same as the voltage E;. Changes in this latter voltage follow the changes in Epp with a time lag equal 0 ri A block diagram representing the machine terminal voltage at no load is shown in Figure 8 14. From that figure the transfer function for V,/ Veer can be obtained by inspection Vii Vace = KL + K) + s(t. + Tho) + hots?) (8 24) Equation (8.24) can be put in the standard form for second-order systems as Vi Vaee = KIC? + 2fe,5 + wh) (8 25) where Ko K,/tiots wf = 1 + K)/tatn Wey = (l/r + Uf rb) For good dynamic performance, i.¢ , for good damping characteristics, a reasonable value of ¢ is 1/2 For typical values of the gains and time constants in fast exciters wwe usually have rfp >> 7, and K, > 1. We can show then that for good performance K, % t/2r, This is usually lower than the value of gain required for steady-state performance In [Il] de Mello and Concordia point out that the same dynamic per- formance can be obtained with higher values of K, by introducing a lead-lag network with the proper choice of transfer function. This is left as an exercise (see Problem 8 5) fp, Fig 8 14 Block diagram representing the machine terminal voltage at no lod 326 Chapter 8 84.3. Effect on the electrical torque The electrical torque for the linearized system under discussion was developed in Chapter 3. With use of the linear model, the electrical torque in pu is numerically equal to the three-phase electrical power in pu. Equation (3.13) gives the change in the electrical torque for the unregulated machine as a function of the angle 8. The same relation for the regulated machine is given by (3.40), From (3.13) we compute the torque as a function of angular frequency to be T,[8 = Ky ~ UGK Kf + oR 3738) — jwKa rho) (8 26) The real component in (8 26) is the synchronizing torque component, which is reduced by the demagnetizing effect of the armature reaction At very low frequencies the synchronizing torque 7, is given by T,™ Ky — KK Ky (827) In the regulated machine there is positive damping introduced by the armature reaction, which is given by the imaginary part of (8 26). This cortesponds to the coefficient of the first power of s and is therefore a damping term In the regulated machine we may show the effect of the regulator on the electrical torque as follows From (3 40) the change of the electrical torque with respect to the change in angle is given by Te... Bake 5+ (Ife + KsKi/Ket.) é Too o(t+ wg) (BE) Kirn, Kirior K,- Ki Kl + 168) + KKK. . (8.28) ((1/Ky + eK.) + riot + (re + 7. /K3) It can be shown that the effect of the terms KzK4(I + 7.5) in the numerator is very small compared to the term K2KsK, This point is discussed in greater detail in [11] Using this simplification, we write the expression for T, /6 as Te K- KaKsK, . (8.29) 6 [1 /Ks + eK.) + taares?] + s(ra0 + 7./Ks) which at a frequency w can be separated into a real component that gives the synchro- nizing torque T, and into an imaginary component that gives the damping torque Ty These components are given by KAK KC /Ky + KK) ~ oP ri07, [(U/Ks + KeK,) ~ wt rdore? + @(rdo + 76/Ks)* KKK (tao + 76/Ka)w “TK + Kok) ~ Prior) + (rae + 7. (KP Note that the damping torque T, will have the same sign as K, This latter quantity can be negative at some operating conditions (see Example 6.6). In this case the regula- tor reduces the inherent system damping At very low frequencies (8.30) is approximately given by T, = Ky — K2Ks/Ko (8 32) (830) TK, ~ (8 31) which is higher than the value obtained for the unregulated machine given by (8 27) Effect of Excitation on Stat 327 Sep hv, we * Fig 8 15 Block diagram ofa linearized excitation system model Therefore, whereas the regulator improves the synchronizing forces in the machine at low frequencies of oscillation, it reduces the inherent system damping when Ks is nega- tive, a common condition for synchronous machines operated near rated load 8.5 Root-Locus Analysis of a Regulated Machine Connected to an Infinite Bus We have used linear system analysis techniques to study the dynamic response of one regulated synchronous machine In Section 7.8, while the excites is represented in detail, a very simple model of the generator is used. In Section 8.4 the exciter model used is a very simple one In this section a more detailed representation of the exciter is adopted, along with the simplified linear model of the synchronous machine that takes into account the field effects The excitation system model used hete is similar to that in Figure 7 54 except for the omission of the limiter and the saturation function S,. This model is shown in Figure 8 15. In this figure the function G(s) is the rate feedback signal. The signal V, is the stabilizing signal that can be derived from any convenient signal and processed through a power system stabilizer network to obtain the desired phase relations (see Section 8 7) The system to be studied is that of one machine connected to an infinite bus through a transmission line. This model used for the synchronous machine is essentially that given in Figure 6.3 and is based on the linearized equations (8 10)-(8 13). To simulate the damping effect of the damper windings and other damping torques, a damping torque component — Dw is added to the model as shown in Figure 8.16 The combined block diagram of the synchronous machine and the exciter is given in Figure 8 17 (with the subscript 4 omitted for convenience) K ig 816 Block diagram of the simplified linear model of a synchronous machine connected 10 an infinite ‘bus with damping added Chopter 8 328 sayox® UE supe sNOUOLUSUAS sRaUH] BJO WEATEIP xo0Ig poULGWO LI-R Ble, Ot Effect of Excitation on Stability 329 Nie GY Nt) Fig 8 18 Block diagram with F, as the takeoff point for feedback oops To study the effect of the different feedback loops, we manipulate the block diagram so that all the feedback loops “originate” at the same takeoff point This is done by standard techniques used in feedback control systems [22] The common takeoff point desired is the terminal voltage V,, and feedback loops to be studied are the regulator and the rate feedback G,(s) The resulting block diagram is shown in Figure 8 18 In that figure the transfer function N(s) is given by (P+ Kyrie) QHs? + Ds + Kion) — on K2KiKa NO) (8 33) Note that the expression for N(s) can be simplified if the damping D is neglected o} the term containing K, is omitted (K is usually very small at heavy load conditions) The system of Figure 8 18 is solved by linear system analysis techniques, using the digital computer A number of computer programs are available that are capable of solving very complex linear systems and of displaying the results graphically in several convenient ways or in tabular forms (20, 21} For a given operating point we can obtain the loci of roots of the open loop system and the frequency response to a sinu- soidal input as well as the time response to a small step change in input The results of the linear computer analysis are best illustrated by some examples In the analysis given in this section, the machine discussed in the examples of Chap- ters 4, 5, and 6 is analyzed for the loading condition of Example 6.7. ‘The exciter data ate K, = 400, ry = 0.05, Ke = —017, re = 0.95, Ky = 1.0andr,=0 The machine constants are 2H = 4.748, D = 2.0 puand rjp = 598 The constants Ky through Kg in pu for the operating point to be analyzed are, K, = 14479 Ky = 03072 Ks = 00294 K, 213174 Kg = 18052 Ky = 05257 Example 84 Use a linear systems analysis program to determine the dynamic response of the system of Figure 8 18 with and without the rate feedback The following graphical solutions are to be obtained for the above operating conditions: Root-locus plot Time response of Vig to a step change in Vase Bode diagram of the closed loop transfer function Bode diagram of the open loop transfer function Bene {e) Fig 619 Root focus of the system of Figure 8 17: (a) without Fig 8 21 Bode plots ofthe closed loop transfer function: (a) Ge = 0, (0) Gp = 0 a he abies ( Effect of Excitation on Stability 331 =f ees “act otORNEE ABH So tb) Toimssce Fig 822 Bode plots of the open loop transfer function: (a) Gp = 0.(b) Gy = 0 Compute these graphical displays for two conditions: (a) GG) = 0 (b) Ges) = 5Kp/(1 + T45), with Ke = 0.04, andr, = 1,08 Solution The results of the computer analysis are shown in Figures 8 19-8 22 for the different plots In each figure, part (a) is for the result without the rate feedback and part (b) is with the rate feedback Figures 8,19-8.20 show clearly that the system is unstable for this value of gain without the rate feedback Note the basic problem discussed in Example 77 With Gx(s) = 0, the system dynamic response is dominated by two pairs of complex roots near the imaginary axis Table 8.4, Con (a) Kr = 0 (b) Ky = 004 Root-Locus Poles and Zeros of Example 8.4 j10 72620 j10 72620 j10 72620 Zeros Poles 0.21097 + 1045130 —027324 0.21097 = j10.45130 — -20 00000 —017894 =0 35020 + 0 35020 = 1.19724 + jo 83244 10 00000 —1 19724 — j0.83244 17894 —0.40337 + j10.69170 © 0.27324 ~0.40337 — j10.69170 © -0.35021 + 035021 — 1.00000 j10 72620 The pair that causes instability is determined by the field 332 Chapter 8 winding and exciter parameters The effect of the pair caused by the torque angie loop is noticeable in the Bode plots of Figures 8 21-8 22 These roots occur near the natural frequency w, = (1.4479 x 377/474)'? = 1073 rad/s. The rate feedback modifies the root-locus plot in such a way as to make the system stable even with high amplifier gains The poles and zeros obtained from the computer results are given in Table 8.4 Example 8 5 Repeat part (b) of Example 8 4 with (a) D = Oand(b)Ks = 0 Solution (a) For the case of D = 0 it is found (from the computer output) that the poles and zeros affected are only those determined by the torque angle loop These poles now become —0 13910 4 {10 72550 (instead of —0 35021 =: j10 72620). The net effect is to move the branch of the root locus determined by these poles and zeros to just slightly away from the imaginary axis, (b) Ithas been shown that Ks is oumerically small Except for the situations where K, becomes negative, its main effect is to change w, to the value wy = (@e/2H) (Ki — KaKs/Ks) The computer output for K, = Ois essentially the same as that of Example 8.4 The root-locus plot and the time response to a step change in Vagr for the cases of D = Oand K; = Oare displayed in Figures 8 23-8 24 The examples given in this section substantiate the conclusions teached in Sec- tion 77 concerning the importance of the rate feedback for a stable operation at high values of gain. A very significant point to note about the two pairs of complex roots that dominate the system dynamic response is the nature of the damping associated with them. The damping coefficient D primarily affects the roots caused by the torque angle loop at a frequency near the natural frequency &,. The second pair of roots, determined by the field circuit and exciter parameters, gives a somewhat lower fre- Tnoginary t Li 4 Real Real (0) () Fig 823 Root locus ofthe system of Example 65: (2) D = 0b) Ks = 0 Effect of Excitetion on Stability 333 (0) Fig 824 Time response to a step change in Fen for the system of Example 8 5: (a) D 0,0) Ks = 0 quency and its damping is inherently poor. This is an important consideration in the study of power system stabilizers 86 Approximate System Representation In the previous section it is shown that the dynamic system performance is domi- nated by two pairs of complex roots that are particularly significant at low frequencies In this frequency range the system damping is inherently low, and stabilizing signals are often needed to improve the system damping (Section 8,7), Here we develop an ap- proximate model for the excitation system that is valid for low frequencies We recognize that the effect of the rate feedback G,(s) in Figure 8.17 is such that it can be neglected at low frequencies (s = jw -> 0) or near steady state (¢ > «) We have already pointed out that K; is usually very small and is omitted in this approximate model The feedback path through K, provides a small positive damping component that is usually considered negligible (11) The resulting reduced system is composed of two subsystems: one representing the exciter-field effects and the other repre- senting the inertial effects These effects contribute the electrical torque components designated 7, and T,, respectively 8.6.1 Approximate excitation system representation The approximate system to be analyzed is shown in Figure 8 25 where the exciter and the generator have been approximated by simple first-order lags {I1] A straight- forward analysis of this system gives <6, ) Fig 825 Approximate representation of the excitation system 334 Chopter 8 G(s) = KKK, ( te + Kstéo Kstiot (8.34) TH KKeK, \ 7 V4 KKK. 7 1 + Ki KeK, Since K,KK, >> 1 in all eases of interest, (8 34) can be simplified to Ki/Ke 14 (G, + Korie {KsKoKils + (lore /KoKs? KiKi frit. + [(re + Ky tho)/Ka tier. |S + KeK./ thot KK /TioTe KiK. {tint + yas tar dls) GG) = a (8.35) 5 where w, is the undamped natural frequency and ¢, is the damping ratio: = VER tit. fe = (te + Ki tlo)/2weKa riot, (8 36) We are particularly concerned about the system frequency of oscillation as compared tow,. The damping ¢, is usually small and the system is poorly damped The function G,(s) must be determined either by calculation or by measurement on the physical system, A proven technique for measurement of the parameters of G(s) is to monitor the terminal voltage while injecting a sinusoidal input signal at the voltage regulator summing junction [8, 12, 23,24, 25]. The resulting amplitude and phase (Bode) plot can be used to identity G,(s) in (8 35) Lacking field test data, we must estimate the parameters of G,(s) by calculations derived from a given operating condition, It should be emphasized that this procedure has some serious drawbacks First, the gains and time constants may not be precisely known, and the use of estimated values may give results that are suspect (10, 12, 24]. Second, the theoretical model based on the constants K, through K, is not only load dependent but is also based on a one machine-infinite bus system The use of these constants, then, requires that assumptions be made concerning the proximity of the machine under study with respect to the rest of the system A procedure based on deriving an equivalent infinite bus, connected to the machine under study by a series impedance, is given in Section 8.6.2 86.2 Estimate of G,(s) The purpose of this section is to develop an approximate method for estimating K, through Ke that can be applied to any machine in the system These constants can be used in (8 36) to calculate the approximate parameters for G(s) The one machine-infinite bus system assumes that the generator under study is connected to an equivalent infinite bus of voltage V./a through a transmission line of impedance Z, = R, + jX. This equivalent impedance is assumed to be the The- venin equivalent impedance as “seen” at the generator terminals. Therefore, if the driving-point short circuit admittance ¥,, at the generator terminal node / is known, wwe assume that z W¥u (837) The equivalent infinite bus voltage V, is calculated by subtracting the drop /,Z. from the generator terminal voltage V,, where J, is the generator current. The pro- cedure is illustrated by an example Effect of Excitation on Stability 335 Example 86 Compute the constants K, through KX, for generator 2 of Example 26, using the equivalent infinite bus method outlined above Note that the three-machine system is certainly not considered to have an infinite bus, and the results might be expected to differ from those obtained by a more detailed simulation Solution From Example 2 6 the following data for the machine are known (in pu and s) Naz = 08958 x. = 08645 xz2 = 0.0521 Hy = 64 Xi = 01198 xg = 01969 tien = 6.0 We can establish the terminal conditions from the load-flow study of Figure 2 19: ty lad = Ia + jaa = (Pr - Qa = (1.630 — j0,066)/1.025 = 1,592 /-2.339° pu From Figure 5.6 tan (630 = 62) = xgla/(Va — Xerfer) = 1272 bx — Br = 51818" But from the load flow By = 9 280°, bx = 51.818 + 9.280 = 61.098" Then i: — Bz + 2 = S4 156° and Vi, = VB, = 8, = 1.025 /~51.818" = Viz + j¥in = 0.634 — j0 806 pu Te = tr [=(6z — By + 62) = 1,892 (-54.156" = Io +jle = 0932 — j1.290 pu Neglecting the armature resistance, r = 0, Ea Ex Veo - tala = 1749 pu Via — Xanla = 1789 pu From Table 2.6 the driving-point admittance at the internal node of generator 2 is given by Yn = 0.420 — j2724 pu The terminal voltage node of generator 2 had been eliminated in the reduction process However, since it is connected to the internal node by xj:, Z, can be obtained by using the approximate relation Z, = 1/Yaz — jxia. The exact reduction process gives Z, = 0.0550 + j0.2388 = 0 2450 /77.029" pu Then we compute from (6 56) Ky = WARE + (xq + Xe)(eh + Xe)] = 1/0 39925 = 25084 A/Ky = 1 + Kilxy — x4)(x, + X,) = 31476 Ky = 03177 We can compute the infinite bus voltage 336 Chapter 8 Va ~ 2h = 1.025 {9.280° — (0 2450 /77,029")(1 592 /6.941") = 0.9706 ~ j0 2226 = 0.9958 /- 12.914" The angle required in the computations to follow is Y= bry ~ ce = 61.098 ~ (—12.914) = 74.012" Ky = K VfB goiResin y + (xj + X,) cosy] + Iyo(x, — x2)[(x, + X,)sin-y ~ R,cosy]} = 2.4750 Kz = BARE qa + [y(R? + (xq + XP = 3.0941 Ke = VeKi(xg — Xi)IOrq + ¥,)sin-y — R,cosy} = 2.0265 Ky = (KiVo/ Vo) xi VeolRecosy — (xy + X.)siny] — xViol(xi + X,)cosy + R,siny]} = 0.0640 Ke = (Yeo/ Vo) = Kye, + Xe) = Vin Vo) Kix. Summary: 0.5070 K,= 2475 K;=0318 Ks = 0.064 K, = 3094 Ky =~ 2027 Ky = 0.507 Note that these constants are in pu on 100-MVA base whereas the machine is a 192- MVA generator, The constants K, and K; should be divided by 192 to convert to the machine base Example 8 7 The exciter for generator 2 of the three-machine system has the constants K, = 400 and 7, = 095s. Compute the parameters of G,(s) For the system natural fre- quency (see Example 3 4) calculate the excitation control system phase lag (Here again we emphasize the need for actual measurement of the system parameters. Lacking such measurement, a judgment is made as to which parameters should be used. We use the regulator gain and the exciter time constant It is judged that the latter is impor- tant at the low frequencies of interest This point is a source of some confusion in the literature. It is sometimes assumed, erroneously, that the regulator time constant is to be used when the excitation system is represented by one time constant This is not valid for low frequencies ) Solution From (8 36) we have we, = VOSUT x WHI EO x 093) = 5.967 rad/s $= 095 + 0.318 x 60/2 x 5.967 x 0.318 x 60 x 095) = 0.132 and the excitation system is poorly damped From Example 3.4 the dominant frequency of oscillation is approximately | 4 Hz or Gue & 8 8tad/s At any frequency the characteristic equation of G,(s) is obtained by substituting s = jw in the denominator of the first expression in (8.35): d(ja) = 1 — 0028147 + j0.04436 At the frequency of interest (w = 8 8 rad/s) we have (jee) = = 1.1761 + j0.3898 buy = tan! 0 3898/—1 1761) = 161.661" Effect of Excitation on Stability 337 ee 1 a= v)+ je donning ratio te Y Amp 4 T oak bootie it COT 9.02 0,040.05 0.) 02 0906 1 2 (doping ratio ton @ = 22 dew oor 0,0 0.0601 0.3 06 1 36 1 860 100 by Fig 826 Characteristics of a second-order transfer funetion: {a) amplitude. (b) phase shift The excitation system phase lag in Example 8.7 is rather large, and phase compensa- tion is likely to be required (sce Section 87) The phase lag is large because twa > w, and §, is small. For small damping the phase changes very fast in the neighborhood Of w, (WhELE Gg = 90"). Many textbooks on control systems, such as (22), give curves of phase shift as a function of normalized frequency, u = w/w,, as shown in Figure 8.26. In the above example, with u = 88/5967 = 1.47 and ¢ = 013, it is apparent from Figure 8 26(b) that the phase lag is great 86.3 The The inertial transfer function can be obtained by inspection from Figure 817 For the ease where damping is present, rtial transfer function 338 Chapter 8 _ ox /2H one 6.38) se Dogg Kien 9+ Begs + ol 2H ** 3H Where w, is the natural frequency of the rotating mass and f, is the damping factor, & = VKion 2H fy = D/4He, ~ D/2VTHK oy (8.39) The damping of the inertial system is usually very low Example 8.8 ‘Compute the characteristic equation, the undamped natural frequency, and the damping factor of the inertial system of generator 2 (Example 2.6) Use D = 2 pu Solution From the data of Examples 2.6 and 8.6 we compute d(s) = S? + 0.1565 + 72.894 @, = V12,894 = 8.538 rad/s ty = 2/[2(128 x 2975 x 377)'?] = 0.009 d{jo) = 1 — 0.01370? + j0 002146 At the system frequency of oscillation w = wy, = 88 rad/s, uy = tan" [0.0183/(—0.0222 — 0.0604)] = 163.3° 8&7 Supplementary Stabilizing Signals Equation (8 31) indicates that the voltage regulator introduces a damping torque component proportional to Ks We noted in Section 843 that under heavy loading conditions Ks can be negative. These are the situations in which dynamic stability is of concern We have also shown in Section 8 6.2 that the excitation system intro- duces a large phase lag at low system frequencies just above the natural frequency of the excitation system. Thus it can often be assumed that the voltage regulator intro- duces negative damping. To offset this effect and to improve the system damping in general, artificial means of producing torques in phase with the speed are introduced These are called “supplementary stabilizing signals” and the networks used to generate these signals have come to be known as “power system stabilizer” (PSS) networks Stabilizing signals are introduced in excitation systems at the summing junction where the reference voltage and the signal produced from the terminal voltage are added to obtain the error signal fed to the regulator-exciter system For example, in the excitation system shown in Figure 7.54 the stabilizing signal is indicated as the signal ¥,, To illustrate, the signal usually obtained from speed or a related signal such as the frequency, is processed through a suitable network to obtain the desired phase relationship Such an arrangement is shown schematically in Figure 8.27 8.7.1 Block diagram of the linear system We have previously established the rationale for using linear systems analysis for the study of low-frequency oscillations For any generator in the system the behavior Effect of Excitation on Stability 339 Repulctor= exciter [> Feo Fig 827 Schematic diagram of a stabilizing signal {rom speed deviation, ‘can be conveniently characterized and the unit performance determined, from the linear block diagram of that generator. This block diagram is shown in Figure 8 28 The constants K, through K, are load dependent (see Section 8 6 for an approxi- mate method to determine these constants) but may be considered constant for small deviations about the operating point. The damping constant D is usually in the range of 10-30 pu The system time constants, gains, and inertia constants are obtained from the equipment manufacturers or by measurement The PSS is shown here as a feedback element from the shaft speed and is often given in the form [11] _ Kons [+ ns) + a8) O80) = res (De sl + 705) (840) The first term in (8 40) is a reset term that is used to “wash out” the compensation effect after a time lag 79, with typical values of 4 s [11] to 20 or 30s [12]. The use of reset control will assure no permanent offset in the terminal voltage due to a prolonged error in frequency, such as might occur in an overload or islanding condition. The second term in Gs(s) is a lead compensation pair that can be used to improve the phase lag through the system (rom Veer to ws at the power system frequency of oscillation Qualitatively, we can recognize the existence of a potential control problem in the system of Figure 8 28 due to the cascading of several phase lags in the forward loop. In terms of a Bode or frequency analysis (see [22], for example) the system is likely to have inadequate phase margin. This is difficult to show quantitatively in the com- plete system because of its complexity We therefore take advantage of the simplified representation developed in Section 8 6 and the results obtained in that section 8.7.2 Approximate model of the complete exciter-generator system Having established the complete forward transfer function of the excitation con- rol system and inertia, we may now sketch the complete block diagram as in Fig- ure 8.29 We note that a common takeoff point is used for the feedback loop, requiring a slight modification of the inertial transfer function using standard block diagram manipulation techniques. We also note that the output in Figure 8 29 is the negative of the speed deviation. The parameters {,, w, and {,, « are defined in (836) and (8 39) respectively. Examining Figure 829 we can see that to damp speed oscillations, the power system stabilizer must compensate for much of the inherent forward loop phase lag Thus the PSS network must provide lead compensation Chapter 8 340 “rseqgqmis wo1sAs Jomod pur s9yO¥D UP IDL uo swauy| @ Jo WATE YOOIg| e's Bla Effect of Excitation on Stability 341 Vier ye BEA ore ___ Sou x Fak estae Pieper a sie bee Kg Fig 829 Block diagram of a simplified model of the complete system 8.7.3 Lead compensation ‘One method of providing phase lead is with the passive circuit of Figure 8 30(a) If loaded into a high impedance, the transfer function of this circuit is Zo _ fay + azs) E, leq where a= (Ry + R3)/Re > | = RRCAR, + Ra) (841) The transfer function has the pole ze10 configuration of Figure 8.30(b), where the zero Jies inside the pole to provide phase lead For this simple network the magnitude of the parameter a is usually limited to about 5 ‘Another lead network not so restricted in the parameter range is that shown in Figure 8.31 [26] For this circuit we compute Ey L+ (a + ta)s E, (1 + res)[l + (re + rp)s) where 4 = K,RC, = lead time constant te = R,C, = noise filter time constant < 7, (8.42) t¢ = K,RC, = lag time constant tp = RC, = stabilizing circuit time constant << r¢ Ky = Raf/(Ra + Re) Kz = Ro/(Re + Ro) Approximately, then E/E, = (1 + ras){( + te5) = (+ ars)/(l + 75) (8.43) wherea = K,C,/KiC2 > 1 < ro — $ RE f ey ® Fig 830 Lesd aerwork: (a) passive network, (b) pole zero conliguration 342 Chapter 8 Fig $31 Active lead network For any lead network the Bode diagram is that shown in Figure 8.32, where the asymptotic approximation is illustrated [22) The maximum phase lead %» occurs at the median frequency w,, where w, occurs at the geometric mean of the corner fre- quencies; ie, logioem = (1/2)lloge(t/ar) + logie(1/7)) = (1/2)logw(1/a7?) = logit /7Va) Then Vrve (8.44) The magnitude of the maximum phase lead @, is computed from © dm = arg ((1 + jena7)/C + jen r)] = tan“!w_aT = tanoqr 2x—y 643) From trigonometric identities tan( = 3) = (tan — tany)/( + tan xtan y) (8 46) Therefore, using (8.46) in (8 45) tangy = (WAT — eg TLL + (eae, 7)] = Ota — 1/4 eek?) (847) This expression can be simplified by using (8.44) to compute tangy = (a ~ /2va (8.48) Now, visualizing a right triangle with base 2V/@, height (a — 1) and hypotenuse b, Fig 832 Bode diagram for the lead network (1 + ars)/(1 4 18) wherea > | Effect of Excitation on Stability 343 we compute b? = (a - 1 + 4a = (a + ior sin dy = (a — Ifa + 1) (8.49) This expression can be solved for a to compute a = (1 + sing,)/(1 - sing) (8.50) These last two expressions give the desired constraint between maximum phase lead and the parameter a The procedure then is to determine the desired phase lead ¢,. This fixes the parameter @ from (8.50). Knowing both a and the frequency @, determines the time constant r from (8.44) In many practical cases the phase lead required is greater than that obtainable from a single lead network. In this case two or more cascaded lead stages are used Thus we often write (8.40) as Gs(s) = [Koro5/(1 + tosIC1 + ars)/( + 75)" (51) where mis the number of lead stages (usually n = 2 or 3) Example 89 ‘Compute the parameters of the power system stabilizer required to exactly compen- sate for the excitation control system lag of 161 6° computed in Example 8.7 Solution Assume two cascaded lead stages. Then the phase lead per stage is Gm = 1616/2 = 808° From (8.50) a= (1 + sin 80 8)/(1 — sin 80,8) = 154 48 This is a very large ratio, and it would probably be preferable to design the compen- sator with three lead stages such that dy = 53.9", Then a = (1 + sin53.9)/(1 ~ sin53.9) 942 which is a reasonable ratio to achieve physically ‘The natural frequency of oscillation of the system is @. = w, = 88rad/s Thus from (8.44) 1 = Vtg VG = 0.037 ar = 03488 Thus G(s) = [Koros/( + ros)]IU + 0.3495)/(1 + 00375)? A suitable value for the reset time constant is 7 = 108 The gain Ky is usually modest (26}, say 0.1 < Ky < 100, and is usually field adjusted for good tesponse It is also ‘common to limit the output of the stabilizer, as shown in Figure 8.28, so that the stabi- lizer output will never dominate the terminal voltage feedback Example 8 10 Assume a two-stage lead-compensated stabilizer Prepare a table showing the phase ead and the compensator parameters as a function of a Solution As before, we assume that aq = 8 8rad/s 344 Chapter 8 Table 8.5. Lead Compensator Parameters as a Function of a a on bm 1 VonVvt ma Ur ar 19 = War 5 4181 83-62 0.0508 19.68 (0.2541 3.935 10 549010980 0.0359 2783 0.3593 2783 15 6105 122.10 9.0293 34.08 0.4401 2272 20 647912958 0.0254 3935 05082, 1.968 25 6738_——_134.76 0.0227 44.00 0.5682 1.760 These results show that for a large @ or large the corner frequencies wy, and jo must be spread farther apart than for small g,, See Figure 8.32 and Problem 8.11 lized Generator 88 Linear Analysis of the Stal In previous sections certain simplifying assumptions were made in order to give an approximate analysis of the stabilized generator. In this section the system of Fig- ure 8.28 is solved by linear system analysis techniques using the digital computer (see Section 8.5). The results of the linear computer analysis are best illustrated by an ex- ample Example 8 11 Use a linear systems analysis program to determine the following graphical solu- tions for the system of Figure 8 28: 1. Root-locus plot 2. Time response of w, to a step change in Veer 3. Bode diagram of the closed loop transfer function Bode diagram of the open loop transfer function = Furthermore, compute these graphical displays for two conditions, (2) no power system stabilizer and (b) a two-stage lead stabilizer with a = 25: ' | 8 i 6 6 / > e 2 2 : { a4 fa Ao o _ 8 8 ~2 0 a a 0 “eal Real (@) (b) Fig 833 Root locus of the generator 2 system: (a) no PSS. (b) with the PSS having «wo lead stages with a=25 Effect of Excitation on Stability 345 o a, = = 4 /V 2 27 | = 7 21.21 2.0) a rr) o 2 4 6 @ Times Time,s (a) (b) Fig 8 34 Time response to @ step change in Vagr: (a) no PSS. (b) with the PSS having two lead stages with an 25) G(s) = [(LOs/(1 + 105){(1 + 0.5685) /(1 + 0.02275)F The system constants are the same as Examples 8 7 and 8 8 Solution The system to be solved is that of Figure 8 28 except that the PSS limiter cannot be represented in a linear analysis program and is therefore ignored The results are shown in Figures 8.33-8 36 for the four different plots In each figure, part (a) is the result without the PSS and part (b) is with the PSS In the root-locus plot (Figure & 33} the major effect of the PSS is to separate the torque-angle zeros rom the poles, forcing the locus to loop to the left and downward, thereby increasing the damping The root locus shows clearly the effect of lead com- pensation and has been used as a basis for PSS parameter identification [27]. Note that \ f + | Frdumws sce ~ “AMBE JS OERES ® ” Fig 8.35 Frequency response (Bode diagram) of the closed loop transfer function: (2) no PSS, (b) with the PSS having two lead stages with a = 25, g 2g g B @ / / eoltOnLE 19g PONE Ig f ~ ~ TTR EET TP EY SVT PET TIRE TPIT E 8 = = = = Ee ‘) « Fig 836 Frequency response (Bode diagram) of the open loop transfer function: (a) no PSS (b) with the PSS having two lead stages with a = 25. the locus near the origin is unaffected by the PSS, but the locus breaking away vertically from the negative real axis moves closer to the origin as compensation is added [this locus is off scale in 8.33(a)} From the computer we also obtain the tabulation of poles and zeros given in Table 8 6 From this table we note that the natural radian frequency of oscillation is controlled by the torque-angle poles with a frequency of 8.467 rad/s This agrees closely with w, = 8.538 rad/s computed in Example 8 8 using the approxi- mate model and also checks well with the frequency of é,, in Figure 3 3 Figure 8 34 shows the substantial improvement in damping introduced by the PSS network. Note the slightly decreased frequency of oscillation in the stabilized response Table8.6.Root-Locus Poles and Zeros Condition Poles Zeros No PSS =20 000 + j0.000 =0.944 + j0 955 0.179 + j0.000 0.944 — j0.955 0.102 + j0 000 —0452 + j8.467 0.289 + j8.533 0.452 — 8.467 0.289 — j8 533, 1000 + j0.000 WithPSSa = 25 —20.000 + j0.000 =0 100 + {0.000 0.179 + 30.000 =0.941 + 50.959 0.010 + 30.000 =0.941 = j0.959 0.289 + 8.533, =0.955 + 57439 0.289 — 58 533 0.955 — 57.439 =1.000 + j0 000 45.000 + j24 847 0.100 + 0.000 45 000 — 24.847 45 500 + j0.000 5.500 — j0.000 Effect of Excitation on Stability 347 Figures 8.35 and 8 36 show the frequency response of the closed loop and open loop transfer functions respectively The uncompensated system has a very sharp drop in phase very near the frequency of oscillation Lead compensation improves the phase substantially in this region, thus improving gain and phase margins 8.9 Analog Computer Studies The analog computer offers a valuable tool to arrive at an optimum setting of the adjustable parameters of the excitation system. With a variety of compensating schemes available to the designer and with each having many adjustable components and pa- rameters, comparative studies of the effectiveness of the various schemes of compensa- tion can be conveniently made Furthermore, this can be done using the complete non- linear model of the synchronous machine 8.9.1. Effect of the rate feedback loop in Type 1 exciter As a case study, Example $ 8 is extended to include the effect of the excitation sys- tem, The synchronous machine used is the same as in the examples of Chapter 4 with the loading condition of Example 5.1. Three IEEE Type | exciters (see Section 7.9 1) are used in this study: W TRA, W Brushless, and W Low 7, Brushless The parameters for these exciters are given in Table 7.8 The analog computer representation of the excitation system is shown in Figure 8.37 This system is added to the machine simulation given in Figure $ 18. Nove that the output of amplifier 614 (Figure 8.37) connects to the terminal marked £;» in Figure 5 18, and the terminal marked v, in Figure $ 18 connects with switch 421 in Figure 8.37 The new “free” inputs to the combined diagram are Veep and 7 The potentiometer settings for the analog computer units are given in Tables 8 7, 8 8 and $9 for the three excitation systems described in Table 7.8 Saturation is represented by an analog limiter on F, in this simulation With the generator equipped with a W TRA exciter, the response due to a 10% ir crease in 7,, and 5% change in Veer and the phase plane plot of ws versus 6, for the in tial loading condition of Example 51 are shown in Figure 838. The results with W Brushless and W Low 7; Brushless exciters are shown in Figures 769 and 8.39 respectively Table 8.7. Potentiometer Calculations for a Type | Representation of a W TRA Exciter(a = 20) Pat | Amy iat | Amy ot me] on fm)» [ulin] 4.45 The methods of compensation used are: Rate feedback: sK,/(1 + tes) Bridge-7 filter with transfer function C/R = (8 + mas + @D/(s? + nw,s + oR) w, = 21 rad/s nad r=0l Power system stabilizer: r=30s 4=025 = 0055 A sample of data given in reference [28] is shown in Table 8 10 for the initial operating condition of Tn, = 3.0 pu at 085 PF lagging. Table8.10. Comparison of Compensation Schemes Pea Ka. Case Rise Settling == Over- Rise Over. time time __shoot% time shoot % ‘Uncompensated 005 022 866 020 060 100 Excitation rate 005 0.22 800-098 420 60.0 feedback Bridged-T only 005 023 00 021 056 330 Bridged-T,two-stage 004 = 021 234 028 037 50 lead-lag and speed Power system 00s on 826 023 0a? S10 stabilizer Source: Schroder and Anderson (28) Other valuable information that can be obtained from analog computer studies is the response of the machine to oscillations originating in the system to which the ma- chine is connected. This can be simulated on the analog representation of one machine connected to an infinite bus by modulating the infinite bus voltage with a signal of the desired frequency. This is particularly valuable in studies to improve the system damp- ing. When growing oscillations occur in large interconnected systems, the frequencies of these oscillations are usually on the order of 0.2-0 3 Hz, with other frequencies super- imposed upon them. Thus it is important to know the dynamic response of the synchro- nous machine under these conditions Effect of Excitation on Stability 353 Tas Fig 840 Synchronous machine with PSS operating against an infinite bus whose voltage is being modu- lated at one-tenth the natural frequency of the machine A sample of this type of study, taken from (28), is shown here The same machine discussed above, but operating under the heavy loading condition of Example 5 1, has its bus voltage modulated by a frequency of one-tenth the natural frequency The modulating signal varies the infinite bus voltage between 102 and 098 peak. Fig- ure 8.40 shows the eflect of the PSS under these conditions At time A the modulating signal of 2 Irad/s is added The PSS is removed at B, causing growing oscillations to build up especially on P.,, which would simulate tie-line oscillations Note also that the frequency of these oscillations is near the natural frequency of the machine When the stabilizer is reinstated at point C, the oscillations are quickly damped out At point D the modulation is removed 8.10 Digital Computer Transient Stability Studies To illustrate the effect of the excitation system on transient stability, transient stability studies are made on the nine-bus system used in Section 2.10 The impedance diagram of the system (to 100-MVA base) and the prefault conditions are shown in ures 2 18 and 2.19 respectively. The generator data are given in Table 21 The transient is initiated by a three-phase fault near bus 7 and is cleared by opening the line between bus 5 and bus 7. In this study the loads A, B, and C are represented by 354 Ps Chapter 8 y nee Ke - reno) oo Bee Bao « Yamax Fspmox — Ur ignlt sg) - Wee 7 eo tml Loe] bo] os be Se ae Sy We fo AM ee ‘ a Ko bee Typical values ema * | k= 10 Ky 20/Ky y=10 ae! Wee higs KTO1K» kyo 8-2 eOl ran Vann =445 Ky 7100” Kg=02 zp=08h5 Eppoe = 485 Fig 8 41 The Brown Boveri Co alternator diode exciter (Used with permission of Brown Boveri Co.) constant impedances; generators I and 3 are represented by classical models, i€, con- For generator 2, provision is made for the stant voltage behind transient reactance excitation system representation A modified transient stability program was used in this study (It is based on a pro- gram developed by the Philadelphia Electric Co., with modifications to include the re- quired new features.) When the excitation system is represented in detail, the model used for the synchronous machine is the so-called “one-axis mode!" (see Section 4 15 4) with provision for representing saturation When the machine EMF & (corresponding to the field current) is calculated, an additional value £, is added due to saturation Table 8.11. Excitation Systems Data Parameter Amplidyne —_—Mag-A-Stat_—_SCPT Ke 25 400 120 Ke 0.044 -017 10 Kp 0.0805 0.04 002 Ke 10 10 10 k, Lis Kr 262 Vesan 10 35 12 Vania 10 35 -12 Vawae a 278 "4 020 0.05 Os 1 050 095 005 ” 035 10 060 7, 0.06 0 0 A, 0.0016 0.0039 8, 1.465 1.555 Note: See Figure 8 41 for BBC exciter parameters Effect of Excitation on Stability 355 BC exciter~contont Ey = Type = 0 5a BRC exci Clessicet mode! Torque Angle, ds: (clasica!) a ? z 4 E ° ar 02 5 at 05) Tine, § Fig $42 63; for various exciters with a three-cycle fault effect and based on the voltage behind the leakage reactance Ez, Thisis given by Ey = Ayexp(B,(E, — 0.8)) (8.52) The constants 4, and B, are provided for several exciters [see (4 141)] The types of field representation used with generator 2 are Classical model IEEE Type 1, 0.5 pu response, amplidyne NA 101 exciter (see Figure 7.61) IBEE Type 1, 2.0 pu response, Mag-A-Stat exciter (see Figure 7 61) IEEE Type 3, SCPT fast exciter, 2.0 pu response (see Figure 7.66) Brown Boveri Company (BBC) alternator diode exciter (see Figure 8 41) The excitation system data are given in Table 8 11 8.10.1 Effect of fault duration ‘Two sets of runs were made for the same fault location and removal, but for dif- ferent fault durations, The breaker clearing times used were three cycles and six cycles For a three-cycle fault, the results of generator 2 data are shown in Figures 8.42-8.46, Similar results for a six-cycle fault are shown in Figures 8.47-8 50. 356 Chapter 8 Lay oak L L L L L ° ar 02 05 or 0s 06 Time. 5 Fig $43 ¥; and £6 for various exciters with a three-eycle fault Results with three-cycle fault cleaying. Figure 8.42 shows a plot of the first swing of the angle 65, for different field representations Note that the classical run gives the angle of the voltage behind transient reactance, while all the others give the position of the g axis. A run with constant £,, is also added We conclude from the results shown in Figure 8 42 that for a three-cycle clearing time the classical model gives ap- proximately the same magnitude of 4,, for the first swing as the different exciter repre- sentations. When the exciter model was adjusted to give constant E,p, however, a large swing was obtained From Figure 8 43 we conclude that the slow exciter gives the nearest simulation of a constant flux linkage in the main field winding (and hence constant £{) and minimum variation of the terminal voltage after fault clearing The action of the exciter and the armature reaction effects are clearly displayed in Figure 8.44. It is interesting to note that the actual field current, as seen by the EMF E, is hardly affected by the value of Ey, for most of the duration of the first swing after the fault is cleared. The effect of the armatute reaction is dominant in this period Figure 8.45 shows a time plot of P, for this transient. Again it can be seen that the different models give essentially the same power swing for this generator We note, however, that the minimum swing is obtained with the slow exciter while the maximum swing is obtained with the classical model In Figure 8 46 the rotor angle 5, is plotied for a period of 2.0 s for the classical model, a slow IEEE Type 1 exciter, and @ relatively fast exciter with 2.0pu response The plot shows that the first swing is the largest, with the subsequent swings slightly reduced in magnitude Figures 8.42-8.46 seem to indicate that for this fault the system is well below the stability limit, since the magnitude of the first swing is on the order of 60°. All generator Effect of Excitation on Stability 357 5 OF oz 08 on 0s 08 Tine, + Fig 844 ep and & for various exciters with a three-cycle fault 2 models give approximately the same magnitude of rotor angle and power swing and period of oscillation Results with six-cycle fault clearing. For the case of a six-cycle clearing time, the plot of the angle 6,, is shown in Figure 8 47 for the classical model and for two differen types of exciter models The swing curves indicate that this is a much more severe fault than the previous one, and the system is perhaps close to the transient stability limit Here the swing curves for the generator with different field representations are quite different in both the magnitude of swings and periods of oscillation. The effect of the 2 0pu response exciter is pronounced after the first swing The effect of the power system stabilizer on the response is hardly noticeable until the second swing. The magnitude of the first swing for the cases where the excitation system is represented in detail is significantly larger than for the case of the classical representation The Type 1 exciter gives the highest swing Comparing Figures 8.46 and 847, we note that for this severe fault the rotor oscillation of generator 2 depends a great deal on the type of excitation system used on the generator We also note that the classical model does not accurately represent the generator response for this case Tergue Angle Sy, elecs degrest (clotsical) Chapter 8 209 10> veo woh, ——— Clonee! node BEC exciter = 2,0 RR z Mog-A-Star = Type 1~0.5 88 yp] 100 sok a 1 t 1 1 1 ° a1 02 @ 04 as Time, + Fig 843 Output power P; for various exciters with a three-cyce fault Of 06 08 10 N2 da 26 Tine. « Fig 846 Rotor angle 621 for various exciters with a three-cycle fault S 1 oloe.degroes ( with exciter) by Torque Angle » Effect of Excitation on Stability 359 eo} v4 ——— clantcat Type T= 0.5 RR Bac - 2.0 Re BBC with SS Torque Anale, Bos, elec. degree {closical ° 0204 06 Oe 012 te ie 180 The output power of generator 2 is shown in Figure 8.48 for different exciter repre- sentations. While the general shape of these curves is the same, some significant dif- ferences are noted. The excitation system increases the output power of the generator after the first swing The generator acceleration will thus decrease, causing the rotor swing to decrease appreciably. This effect is not noticed in the classical model. It ‘would appear that for slightly more severe faults the classical model may predict differ- ent results concerning stability than those predicted using the detailed representation of the exciter Figures 8 49 and 8.50 show plots of the various voltages and EMF’s of generator 2 for the case of the 20 pu exciter and the Type | exciter respectively The curves for E; show that although the fault is near the generator terminal, the flux linkage in the main field winding (reflected in the value of £;) drops only slightly (by about 5%); and for the duration of the first swing it is fairly constant. The faster recovery occurs with the 20 pu exciter, and £; reaches a plateau at about 11s and stays fairly constant thereafter For the Type I exciter £{ recovers slowly and continues to increase steadily The oscillations of terminal voltage ¥, are somewhat complex. The first swing after the fault seems to be dominated by the inertial swing of the rotor, with the action of the exciter dominating the subsequent swings in V; Thus alter the first voltage dip, the swings in ¥, follow the changes in the field voltage E-» with a slight time lag Again the recovery of the terminal voltage is faster with the 2 0 pu exciter than with the Type 1 exciter. We also note that the excitation system introduces additional frequencies of oscillation, which appear in the ¥, response 360 Chapter 8 ° 02 04 06 08 10d aS Fig 48 Output power P; for various exciters with a six-cyele fault The plots of E clearly show the effect of the armature reaction In the first 0.75, for example, the changes in Ey» are reflected only in a minor way in the total internal EMF £ The component of E due to the armature reaction seems to be dominant be- cause the field circuit time constant is long. The general shape of the EMF plot, how- ever, is due to the effects of both £,, and the armature reaction From the data presented in this study we conclude that for a less severe fault or for fast fault clearing, the excitation representation is not critical in predicting the system dynamic responses However, for a more’severe fault of for studies involving long transient periods, it is important to represent the excitation system accurately to obtain the correct system dynamic response 8.10.2 Effect of the power system stabilizer For large disturbances the assumption of linear analysis is not valid However, the PSS is helpful in damping oscillations caused by large disturbances and can be effective in restoring normal steady-state conditions. Since the initial rotor swing is largely an Effect of Excitation on Stability Voto, ws LE VF Fig 850 Voltages of generator 2 with Type 105 RR exciter 361 362 Chapter 8 Se — Foul) cored ot three evelee 2 sl é Bol 5 & 7 A \ TA a 8 / Ts aw 1.05 Ts 18 Time, + ° 0.55 0B O7e Fig 8 S1 Torque angle 62, for a three-phase fault near generator 2 PSS with a = 25 wae ~ 89 rad/s inertial response to the accelerating torque in the rotor, the stabilizer has little effect on this first swing. On subsequent oscillations, however, the effect of the stabilizer is quite pronounced To illustrate the effect of the PSS, some transient stability runs are made for a three- phase fault near bus 7 applied at z = 0.01675 (1 cycte) and cleared by opening line 5-7 ° 0m 0.8 075108 12s a0 Vs 2 Tine Fig 852 Exciter voltage Eyp with and without « PSS. Effect of Excitation on Stability 363 al t = 0.10s (6 cycles), Generator 2 is equipped with a Type 1 Mag-A-Stat exciter with constants similar to those given in Table 8 11 The PSS constants are the same as in Example 8 12 (a = 25) with « limiter included such that the PSS output is limited to £010 pu. Stability runs were made with and without the PSS_ From the stability runs, data for the angle 6,, and the voltage E,, are taken with and without the PSS. The re- sults are displayed in Figures 8 51 and 8 52 From the plot of 8,, in Figure 8.51 note that while the change in the first peak (due to the PSS) is very small, the improvement in the peak of the second swing is sig- nificant The comparison in Ep, shown in Figure 8 52, is interesting Note that this exciter is not particularly fast(RR = 0.5), and the response tends to be a ramp up and then down. The phase of £,» changes when the PSS is applied to produce a field voltage that is almost 180° out of phase with 5,,. This results in a delayed E,p ramp as 5 swings downward, which tends to limit the downward 4 excursion by retarding the building in 7, The improvement in the angle 6, defined as 3,4 = 53; (oo rss) — 5: cessys has been investigated for different PSS parameters It is found that this angle improvement is sensitive to both the amount of lead compensation and to the cutoff level of the PSS limiter A comparison of several runs is shown in Table 8 12 Table& 12, 6, Improvement at Peak of Second Swing a Limit~ 40.10 Limit = 40.05 25 38 46 16 Sat aT 8.11 Some General Comments en the Effect of Excitation on Stability In the 1940s it was recognized that excitation control ean increase the stability limits of synchronous generators Another way to look at the same problem is to note that fast excitation systems allow operation with higher system reactances This is felt to be important in view of the trends toward higher capacity generating units with higher reactances. For exciters to perform this function, they need high gain, Series compensation makes it possible to have a high de gain and at the same time have lower “transient gain” for stable performance Modern exciters ate faster and more powerful and hence allow for operation with higher series system reactance Concordia [17], however, warns that “we cannot expect to continue indefinitely to compensate for increases in reactance by more and more powerful excitation systems” A limit may soon be reached when further increases in system reactance should be compensated for by means other than excitation control The above summarizes the situation regarding the so-called steady-state stability or power limits Regarding the dynamic performance, modern excitation systems play an important part in the overall response of large systems to various impacts, both in the so-called transient stability problems and the dynamic stability problems The discussion in Section 8.3 and the studies of Section 8 10 seem to indicate that for less severe transients, the effect of modern fast excitation systems on first swing transients is marginal However, for more severe transients or for transients initiated by faults of longer duration, these modern exciters can have a more pronounced effect In the first place, for faults near the generator terminals it is important that the synchronous machine be modeled accurately. Also, if the transient study extends be- yond the first swing, an accurate representation of the field flux in the machine is needed. If the excitation system is slow and has a low response ratio, optimistic results 364 Chapter 8 Veer K(risi1 + Xl + 138) Nervor haf tpved se] Koy | TTT “To oxi baer Tg Floctic powse_ eel Kag Terminal veltoge Fig 853. Block diagram of the PSS for the BBC exciter witha 20 RR: Kg: = Kgs = Kgs = 0,.Kg2 = 1, r= 10 72 = 08 75 = 008, ry = 05, rs = 005, limit = £008 pu may be obtained if the classical machine representation is used Transient studies are frequently run for 2 few swings to check on situations where circuit breakers may fail to operate properly and where backup protection is used. It should be mentioned that several transients have been encountered in the systems of North America where subse- quent swings were of greater magnitudes than the first, causing eventual loss of synchro- nism. This is not too surprising in large interconnected systems with numerous modes, of oscillations. It is not unlikely that some of the modes may be superimposed at some time after the start of the transient in such a way as to cause increased angle deviation As shown in Section 8 10, the effect of excitation system compensation on subsequent ‘swings (in large transients) is very pronounced This has been repeatedly demonstrated in computer simulation studies and by field tests reported upon in the literature [8, 9, 13, 23, 29, 30, 31] For example, in a stability study conducted by engineers of the Ne- braska Public Power District, the effect of the PSS on damping the subsequent swings was found to be quite pronounced, while the effect on the magnitude of the first swing was hardly noticeable The excitation system used is the Brown Boveri exciter shown in Figure 8 41. The PSS used is shown schematically in Figure 8 53, and the swing curves obtained with and without the PSS (for the same fault) are shown in Figure 8.54 Voltage regulators can and do improve the synchronizing torques Their effect on damping torques are small; but in the cases where the system exhibits negative damping characteristics, the voltage regulator usually aggravates the situation by increasing the negative damping Supplementary signals to introduce artificial damping torques and to reduce intermachine and intersystem oscillations have been used with great success These signals must be introduced with the proper phase relations to compensate for the excessive phase lag (and hence improve the system damping) at the desired frequen- cies [32] Large interconnected power systems experience negative damping at very low fre- quencies of oscillations The parameters of the PSS for a particular generator must be adjusted after careful study of the power system dynamic performance and the gen- crator-exciter dynamic response characteristics. As indicated in Section 8 6, to obtain these characteristics, field measurements are preferred. If such measurements are not possible, approximate methods of analysis can be used to obtain preliminary design data, with provision for the adjustment of the PSS parameters to be made on the site after installation Usually the PSS parameters are optimized over a range of frequen- cies between the natural mode of oscillation of the machine and the dominant fre- quency of oscillation of the interconnected power system Effect of Excitation on Stability 365 ali sp bog Oo 28 8s aoe 8 Tine. cycles Fig 854 Effect of the PSS on transient stability (Obtained by private communication and used with per mission ) Recently many studies have been made on the use of various types of compensating networks to meet different situations and stimuli Most of these studies concentrate on the use of a signal derived from speed or frequency deviation processed through a PSS network to give the proper phase relation to obtain the desired damping charac- teristic. This approach seems to concentrate on alleviating the problem of growing oscillations on tie lines (11, 13, 14, 24, 26, 30, 33-39] However, in a large intercon- nected system it is possible to have a variety of potential problems that can be helped by excitation control Whether the stabilizing signal derived from speed provides the best answer is an open question, It would seem likely that the principle of “optimal control” theory is applicable to this problem Here signals derived from the various “states” of the system are fed back with different gains to optimize the system dynamic performance. This optimization is accomplished by assigning a performance index This index is minimized by a control law described by a set of equations These equa- tions are solved for the gain constants. This subject is under active investigation by many researchers [40-44] Problems 81 Construct a block diagram for the regulated generator given by (8.10)-(8 14) What is the order of the system? 82 Use block diagram algebra to reduce the system of Problem 8 | to @ feed-forward transfer function KG(s) and a feedback transfer function H(3), arranged as in Figure 7.19. 83 Determine the open loop transfer function for the system of Problem 8.2, using the numeri- cal data given in Example 83 Find the upper and lower limits of the gain K, for (a) Case | and (b) Case 2 84 Repeat the determination of stable operating constraints developed in Section 8 4.1, with, the following assumptions (see 1)} KKK > 11 « Kiting

Das könnte Ihnen auch gefallen